0% found this document useful (0 votes)
3K views126 pages

Ejercicios de Matematicas

Problem B1 gives

Uploaded by

Khant Si Thu
Copyright
© © All Rights Reserved
We take content rights seriously. If you suspect this is your content, claim it here.
Available Formats
Download as PDF, TXT or read online on Scribd
0% found this document useful (0 votes)
3K views126 pages

Ejercicios de Matematicas

Problem B1 gives

Uploaded by

Khant Si Thu
Copyright
© © All Rights Reserved
We take content rights seriously. If you suspect this is your content, claim it here.
Available Formats
Download as PDF, TXT or read online on Scribd
You are on page 1/ 126

OCM 1999 7 8 9

l
456
l' Problem Al
123
I A, B, C, O, E each has a unique piece ofnews. They make a series ofphone caUslo each other. In each
Find a winning strategy for one ofthe players.

I
I

I •• caU,the caUertells the other party all the news he knows, but is not told anything by the other party.
What is (he minimum number of caUsneeded for aUfive people to know aU five items of news? What is
the minimum for n people?
Answer

Awins


Answer

8, eg BA, CA, DA, EA, AB, AC, AD, AE.

• 2n-2
Solution

• Solutíon
A plays 9.

l' •
Case (1). IfB plays 8, then A plays 9. B must now play 3, then A wins with 1.
Consider the case of n people. Let N be the smaUestnumber of caUs such that after they have been made

l. • at least one person knows all the news. Then N 2: n-I, because each ofthe other n-I people must make at Case (2). IfB plays 7, then A plays 9. B must now play 3, then A wins with 2.
least one call, otherwise no one but them knows their news. After N caUs only one person can know aU
the news, because otherwise at least one person would have known aUthe news before the Nth caUand
N would not be minimal. So at least a further n-I caUsare needed, one to each ofthe other n-I people.
So at least 2n-2 caUsare needed in a11. But 2n-2 is easily achieved. First everyone else caUsX, then X
Case (3). IfB plays 6, then A plays 5. Now ifB plays x, A can play 10-x and wins.

•.. caUseveryone else.

ProblemA2
Case (4). IfB plays 3, then A plays 6. IfB plays 9, then A wins with 3 and vice versa. IfB plays 5, then
A plays 6 and wins. If B plays 4, then A plays 6 and wins.

Find a positive integer n with 1000 digits, none O, such that we can group the digits into 500 pairs so that
This seems messy. ls there a more elegant approach. For example, 22, 23, 24 are losing positions

the sum ofthe products ofthe numbers in each pair divides n.


'.- Problem BI

• ABCO is a trapezoid with AB paraUel to CO. M is the midpoint of AO, .MCB = 150°,BC = x and MC
= y. Find area ABCO in terms ofx and y.

• Answer

••
A B
11...121122112 ... 2112 (960 Is foUowedby 10 2112s)

Solution
~--y -150-X-

• Suppose we take 980 digits to be 1 and 20 digits to be 2. Then we can take 8 pairs (2,2), 4 pairs (2,1) and

'.
488 pairs (1,1) giving a total of 528 = 16·3·11.The sum of the digits is 1020 which is divisible by 3, so n D e
is certainly divisible by 3. We can arrange that halfthe Is and halfthe 2s are in odd positions, which wiU
ensure that n is divisible by 11. FinaUy,n wiUbe divisible by 16 ifthe number formed by its last 4 digits
is divisible by 16, so we take the last 4 digits to be 2112 (=16'132). So, for example, we can take n to be Answer

•• 11...12112 2Jl2 ... 2Jl2, where we have 960 Is foUowedby lO 2112s .


xy/2

_•.
Problem AJ
Solution
A and B playa game as foUows.Starting with A, they altemately choose a number from 1 to 9. The first
to take the total over 30 loses. After the first choice each choice must be one of the four numbers in the
same row or column as the last number (but not equal to the last number):
.,

!
Extend CM to meet the line AB at N. Then CDM and NAM are congruent and so area ABCD = area
CNB. But CN = 2y, so area CNB = y, 2Y'x'sin 1500 = xy/2.

ProblemA2
Problem B2

The diagram shows two pentominos made from unit squares. For whieh n > 1 can we tile a 15 x n
a> 17 is odd and 3a-2 is a square. Show that there are positive integers b "#e sueh that a+b, a+e, b+e and reetangle with these pentominos?
a+b+e are all squares. iI
SoIution

Let a = 2k+ 1. So we are given that 6k+ 1 is a square. Take b = k2-4k, e = 4k. Then b "#e sinee a"# 17. ....
",J

....
Also b is positive sinee a> 9. Now a+b = (k-I )2, a+e = 6k+ 1 (given to be a square), b+e = k2, a+b+c =
(k+lf.

Problem B3
Answer

S D {1, 2, 3, ... , 1000} is sueh that ifm and n are distinet elements ofS, then m+n does not belong to S. all n except 1,2,4,7
What is the largest possible number of elements in S?

...
Answer

501 eg {500, 501, ... , 1000}


SoIution

....
SoIution

We show by induetion that the largest possible subset of {1, 2, ... , 2n} has n+l elements. It is obvious
for n = l. Now suppose it is true for n. ff we do not inelude 2n-1 or 2n in the subset, then by induetion it
ean have at most n elements. Ifwe inelude 2n, then we ean inelude at most one from eaeh ofthe pairs
(1,2n-I), (2,2n-2), ... .tn-Ln+I), So with n and 2n, that gives at most n+1 in aH. Ifwe inelude 2n-l but
....
not 2n, then we ean inelude at most one from eaeh ofthe pairs (I,2n-2), (2,2n-3), ... , (n-I,n), so at most
n in aH. The diagram shows how to tile a 3 x 5 reetangle. That a1lowsus to tile a 15 x 3 reetangle and a 15 x 5
reetangle. Now we ean express any integer n > 7 as a sum of 3s and Ss, beeause 8 = 3+5, 9 = 3+3+3, 10 ..1
OCM2000

Problem Al
= 5+5 and given a sum for n, we obviously have a sum for n+3. Henee we ean tile 15 x n reetangles for
any n ~ 8. We can also do n = 3, 5, 6.

Obviously n = 1 and n = 2 are impossible, so it remains to consider n = 4 and n = 7.



.1
Find all three digit numbers abe (with a"# O) sueh that a2 + b2 + e2 divides 26. ti
Answer
.1
100,110,101,302,320,230,203,431,413,314,341,134, 143,510,501,150,105
.1
...
SoIution

Possible faetors are 1,2, 13,26. Ignoring order, the possible expressions as a sum ofthree squares are: 1
= 12+02+02,2= 1'+ 1'+02, 13=32+22+02,26=52+ 1'+0'=4'+32+ 12
The diagram shows the are 4 ways of eovering the top left square (we only show the 3 left columns of
eaeh 15 x 4 reetangle). Evidently none ofthem work for n = 4. So n = 4 is impossible.
..






Finally, consider n = 7. As for n = 4, there are only two possibilities for covering the top left square, but Answer
two obviously do not work. Consider the possibility in the diagram above. If we cover x using the U
shaped piece, then we cannot cover y. So we must cover x with the cross. Then we have to cover z with

• the U shaped piece. But that leaves the 6 red squares at the bottom, which cannot be covered.

• If we cover the top left square with a C, the a similar argument shows that we must cover the top three
rows and first 5 columns with cross and two UsoBut now we cannot cover the 4 x 6 rectangle


undemeath. Problem A3

ABCDE is a convex pentagon. Show that the centroids ofthe 4 triangles ABE, BCE, CDE, DAE from a

• parallelogram with whose area is 2/9 area ABCD.

• Problem B2

• ABC is acute-angled. The circle diameter AC meets AB again at F, and the circle diameter AB meets
AC again at E. BE meets the circle diameter AC at P, and CF meets the circle diameter AB at Q. Show
that AP=AQ.



D

• Solution

• Use vectors. Take any origin O. Write the vector OA as a, OB as b etc. Let P, Q, R, S be the centroids of
ABE, BCE, CDE, DAE. Then p = (a+b+e)/3, q = (b+e+e)/3, r = (e+d+e)/3, s = (a+d+e)/3. SOPQ = (a-


e)/3 = SR. Hence PQ and SR are equal and parallel, so PQRS is a parallelogram.


QR = (d-b)/3, so the area ofthe parallelogram is PQ x QR = (a-e) x (d-b)/9. The area of ABCD = area
ABC + area ACD = y, CB X CA + y, CA x CD = y, (b-e) x (a-e) + Y, (a-e) x (d-e) = Y, (a-e) x (-(b-
Solution
c)+d-c) = (a-e) x (d-b)/2. So area PQRS = (2/9) area ABCD.

• Problem BI
Thanks lo José Nelson Ramirez

• Write an integer in each small triangles so that every triangle with at least two neighbors has a number
AB is a diameter, so IJAQB = 90°. Similarly, AC is a diameter, so í ,AFQ = r rAFC = 90°. Hence
triangles AQB, AFQ are similar, so AQ/AB = AF/AQ, or AQ2 = AF·AB. Similarly, Ap2 = AE·AC. But


equal to the difference between the numbers in two of its neighbors.
IiBEC = ! IBFC = 90°, so BFEC is cyclic. Hence AF·AB = AE·AC.

• ~.
Problem B3 p

A nice representation of a positive integer n is a representation of n as sum of powers of 2 with each


power appearing at most twice. For example, 5 = 4 + 1 = 2 + 2 + 1. Which positive integers have an
even number of nice representations?

Answer B
A

n=2mod3

Solution

Let f(n) be the number ofnice representations ofn. We show first that (1) f(2n+l) = f(n), and (2) f(2n) =
f(2n-l) + f(n).
Answer
(1) is almost obvious because n = ¿ a;2b¡iff2n+1 = 1 + ¿ a;2btl. (2) is also fairly obvious. There are f(n)
representations of 2n without a 1 and f(2n-1) with a 1 (because any nice representation of f(2n-1) must 2/..J3
have just one 1).
Solution
We now prove the required result by induction. Let Skbe the statement that for n :'S 6k, f(n) is odd for n =
0,1 mod3 andeven forn=2mod3. It is easyto check that f(I)= 1, f(2)=2, f(3) = 1, f(4)=3, f(5)=2,
'IAQB = , IACB + I :CBQ = 90° + 1jCBQ = 90° + Y,! ICOD = 90° + V.IJAQB.Hence ilAQB = 120°,
f(6) = 3. So SI is true. Suppose Sds true. Then f(6k+l) = f(3k) = odd. f(6k+2) = f(3k+l) + f(6k+1) =
odd + odd = even. f(6k+3) = f(3k+l) = odd. f(6k+4) = f(6k+3) + f(3k+2) = odd + even = odd. f(6k+5) = and i lCOD = 60°. So OP = IIcos 30° = 2/..J3.
f(3k+2) = odd. So Sk+1is true. So the result is true for all k and hence all n.
Problem A3
OCM2001
Problem Al Find all squares which have only two non-zero digits, one ofthem 3.

A and B stand in a circle with 2001 other people. A and B are not adjacent. Starting with A they take Answer
tums in touching one oftheir neighbors. Each person who is touched must immediately leave the circle.
The winner is the player who manages to touch his opponent. Show that one player has a winning
strategy and find it. 36, 3600, 360000, ...

Solution

..
Solution

Ifthere is just one person between A and B, then touching that person loses. There are 1999 people who A square must end in 0,1,4,5,6, or9. So the 3 must be the first digit.lfa square ends in O,then it must
can be touched before that happens, so B is sure to lose provided that A never touches someone who is end in an even number ofOs and removing these must give a square. Thus we need only consider
numbers which do not end in O.The number cannot end in 9, for then it would be divisible by 3 but not 9
the only person between him and B.
(using the sum of digits test). It cannot end in 5, because squares ending in 5 must end in 25. So it

Problem A2
remains to consider 1, 4, 6. .1
36 is a square. But ifthere are one or more Osbetween the 3 and the 6, then the number is divisible by 2
C and D are points on the circle diameter AB such that , jAQB = 2 i :COD. The tangents at C and D but not 4, so 36 is the only solution ending in 6.
meet at P. The circle has radius 1. Find the distance ofP from its center.
Suppose 3·10' + 1 = m', so 3·2"S"= (m-I)( m+ 1). But m+1 and m-I cannot both be divisible by 5, so one
must be a multiple of S". But 5" &t; 3·2"+ 2 for n > 1, so that is impossible for n > 1. For n = 1,we have
31, which is not a square. Thus there are no squares 3·10' + 1.

l\
•• A similar argument works for 3·IOn+ 4, beeause if3·IOn + 4 = m', then 5 eannot divide m-2 and m+2, so OCM2002
Snmust divide one ofthem, whieh is then too big, sinee Sn> 3·2n+ 4 for n > l. For n = 1 we have 34,

• whieh is not a square. Problem Al

• Problem Bl For whieh n > 2 can the numbers 1, 2, ... , n be arranged in a eircle so that eaeh number divides the sum
ofthe next two numbers (in a clockwise direetion)?

• Find the smallest n sueh that the sequenee ofpositive integers al, a" ... , a. has eaeh term z; 15 and ad +
a,! + ... + a,l has last four digits 2001. Answer

• Solution n=3

• We find that the last 4 digits are as follows: I! 1, 2! 2, 3! 6, 4! 24, S! 120, 6! 720, 7! 5040, 8! 320, 9!
2880, lO! 8800, II! 6800, 12! 1600, 13! 800, 14! 1200, IS! 8000.
Solution

• Only 1! is odd, so we must inelude it. None of the others has last 4 digits 2000, so we need at least three
Let the numbers be a), a" a3,.... Where neeessary we use eyclie subseripts (so that a.+1means al etc).
Suppose a¡and a¡+1are both even, then sinee 1I¡ divides a¡+1+ lI¡+"lI¡+,must also be even. Henee a¡+3must

• faetorials. But 13! + 14!+ I! works.


Problem B2
be even and so on. Contradietion, sinee only halfthe numbers are even. Henee if 1I¡ is even, a¡+1must be
odd. But a¡+1+ a¡+,must be even, so lI¡+,must also be odd. In other words, every even number is followed


by two odd numbers. But that mcans there are at least twiee as many odd numbers as even numbers.
That is only possible for n = 3. It is easy to check that n = 3 works.
a, b, e are reals sueh that if'p., p, are the roots of ax" + bx + e = Oand q., q, are the roots ofex' + bx + a


= O,then PI, ql, p" q, is an arithmetie progression of distinet terms. Show that a + e = O.
Problem A2

• ABC is aeute-angled. AD and BE are altitudes. area BDE :s area DEA :s area EAB :s ABD. Show that
the triangle is isoseeles.


Solution

Put PI = h-k, ql = h. so p, = h+k, q, = h+2k. Then h'-k' = e/a, 2h = -b/a, h'+2hk = ale, 2h+2k = -b/e.


A

So h = -b/2a, k = b/2a - b/2e and b'I2ae - b'I4e' = e/a, b'I2ae - b'I4a' = ale. Subtraeting, (b'14)(I/a' - I/c')

• = e/a - ale, so (e'-a')(b'14 - ae)/(a'e') = O.Henee a = e or a + e = Oor b' = 4ae. Ifb' = 4ae, then PI = p"
whereas we are given that PI, P,. q., q, are all distinet. Similarly, ifa = e, then {P),P'} = {q"q,}. Henee a


+ e=O.


Problem B3

10000 points are marked on a eirele and numbered clockwise from 1 to 10000. The points are divided 8 e


D
into 5000 pairs and the points of eaeh pair are joined by a segment, so that eaeh segment interseetsjust
one other segment. Eaeh of the 5000 segments is labeled with the produet of the numbers at its


endpoints. Show that the sum ofthe segment labels is a multiple of 4.


Solution
Solution


Suppose points iand j are joined. Thej-i-I points on the are between i and j are paired with eaeh other,
area BDE :s area DEA implies that the distanee of A from the line DE is no smaller than the distance of

-•
with just one exeeption (the endpoint of the segment that interseets the segment i-j). So we must have j =
B, so ifthe lines AB and DE interseet, then they do so on the B, D side. But area EAB :s area ABO
i+4k+2. Thus the segment i-j is labeled with i(i+4k+2) = i(i+2) mod 4. Ifi is even, this is Omod 4. Ifi is
implies that the distanee of D from the line AB is no smaller than the distanee of E, so ifthe lines AB
odd, then it is -1 mod 4. Sinee odd points arejoined to odd points (4k+2 is always even), there are 2500
and DE interseet, then they do so on the A, E side. Henee they must be parallel. But ABDE is eyelie
segmentsjoining odd points. Each has a lahel = -1 mod 4. So their sum = -2500 = Omod 4. AIIthe
(UAOB = I iAEB = 90°), so it must be an isosceles trapezoid and henee I lA = liB.
segmentsjoining even points have labels = Omod 4, so the sum of all the segment labels is a multiple of
4.
Thanks lo José Miguelfor this.

1-
l.
ProblemA3 Thanks lo José Ne/son Ramirez

Define the sequence al, a2, a-, ... by al = A,!ln+1= a. + d(an), where d(m) is the largest factor ofm which Corísiéer the set of odd powers of 2. Suppose al < a2 < ...< an and odd positive integers. Then 231+ 232
is < m. For which integers A > I is 2002 a member of the sequence? + ... + 23n = 231(1 + 232-81+ ... + 23.-"1).Each term in the bracket except the first is even, so the bracket is
odd. Hence the sum is divisible by an odd power of 2 and cannot be a square.

Answer
Problem B3

None.
A path from (0,0) to (n,n) on the lattice is made up ofunit moves upward or rightward. It is ba/anced if
the sum ofthe x-coordinates ofits 2n+1 vertices equaIs the sum oftheir y-coordinates. Show that a
balanced path divides the square with vertices (0,0), (n,O), (n,n), (O,n) into two parts with equal area.

Solution Solution

Let N have largest proper factor m < N. We show that N + m cannot be 2002. Suppose N + m = 2002. Denote the vertices ofthe path as (0,0) = (Xo,yo),(XI,YI),... , (X2n,Y2n)
= (n,n). Since the path proceeds one
Put N = mp. Then p must be a prime (or N would have a larger proper factor than m). So 2002 = step at a time, we have X;+ y; = i. Hence ¿ (x, + y;) = ¿ i = n(2n+ 1). So if¿ x, = ¿ Ybthen ¿ 2x;n =
m(p+ 1). Also p :S m. Hence p < 44. So k = p+I is a factor of2002 smaller than 45 which is Igreater n(2n+ 1). (Note also that n must be even, although we do not use that.) Thus for a balanced path we have
than a prime. It is easy to check that the only possibility is k = 14. So N = 11'132. But this has largest 2xI + 2X2+ ... + 2X2n= n(2n+l) = (2n+I)X2n. Hence 2xI + 2X2+ ... + 2X2n_1 = (2n-I)X2n. Adding X2+ 2x, +
factor 13\ not 11·13.Contradiction. 3X4+ ... + (2n-2)X2n_1to both sides we get 2xI + 3X2+ 4x, + ... + 2nx2n-1= X2+ 2xJ + 3X4+ ... + (2n-l)X2n
or ¿ iX;_I= ¿ (i-Ijx,
Problem BI
Hence ¿ x;_I(i- x.) = ¿ x;(i-I - X;_I)or ¿ x;_IY;= ¿ x;y;_I.Hence ¿ (x, - x;-t)y; = ¿ x;(y; - y;_I).But it is easy
ABC is a triangle. D is the midpoint of Be. E is a point on the side AC such that BE = 2AD. BE and AD to see that the Ihs is the area under the path and the rhs is the area between the path and the y-axis, in
meet at F and :FAE = 60°. Find .FEA. other words the part ofthe large square that is aboye the path. So we have established that the path
divides the large square into two parts of equal area.

Problem B3

A path from (0,0) to (n,n) on the lattice is made up ofunit moves upward or rightward. It is ba/anced if
the sum ofthe x-coordinates of its 2n+ 1 vertices equals the sum oftheir y-coordinates. Show that a
balanced path divides the square with vertices (0,0), (n,O), (n.n), (O,n) into two parts with equal area.

B O e Solution

Denote the vertices ofthe path as (0,0) = (Xo,Yo),(XI,YI),... , (X2n,Y2n)= (n,n). Since the path proceeds one
step at a time, we have x, + y; = i. Hence ¿ (x, + y;) = ¿ i = n(2n+ 1). So if ¿ x, = ¿ y;, then ¿ 2x;n =
Solution
n(2n+ 1). (Note also that n must be even, although we do not use that.) Thus for a balanced path we have
2xI + 2X2+ ... + 2X2n= n(2n+l) = (2n+I)X2n. Hence 2xI + 2X2+ ... + 2Xln_1= (2n-l)X2n. Adding Xl + 2XJ +
Thanks lo José Ne/son Ramirez 3X4+ ... + (2n-2)X2n_1to both sides we get 2xI + 3X2+ 4x, + ... + 2nx2n_1= X2+ 2x, + 3X4+ ... + (2n-I)X2n
or ¿ i X;_I= ¿ (i-l jx,
Let the line parallel to BE through D meet AC at O. Then DCO, BCE are similar and BC = 2 DC, so BE
=2 DO. Hence AD = DO, so lDOA = IDAO = 60°. FE is parallel to DO, so I IFEA = 60°. Hence ¿ x;_I(i- x.) = ¿ x;(i-I - X;_I)or ¿ x;_IY;= ¿ x;y;_I.Hence ¿ (x, - x;_I)y;= ¿ x;(y; - y;_I).But it is easy
to see that the Ihs is the area under the path and the rhs is the area between the path and the y-axis, in.
other words the part ofthe large square that is aboye the path. So we have established that the path
Problem B2
divides the large square into two parts of equal area.

Find an infinite set of positive integers such that the sum of any finite number of distinct elements of the
set is not a square. OCM 2002
Problem Al

Solution
•• There are 2003 stones in apile. Two players alternately select a positive divisor ofthe number of stones Induction on b. For b=3 we require a' ..... 12':3a + 3, or (a-2)(a+I)2 2':O, which is true, with equality iffa =


currently in the pile and remove that number of stones. The player who removes the last stone loses. 2. Suppose the result is true for b. Then ab+1 + I = a(ab + 1) - a + I 2':ab(a+l) - a + I = a(ab-I) +ab + l
Find a winning strategy for one ofthe players. > a(2b-l) + ab + I > a(2b-l) + b + I > a(b+ 1) + b+1 = (a+ I)(b+I), so the result is true, and a strict
inequality, for b+ l. Hence the result is true for all b>2 and lhe only case of equality is b=3, a=2.

• Solution
Problem DI

• Thanks lo José Nelson Ramirez


Two circles meet at P and Q. A line through P meets the circles again at A and A'. A parallelline

•.. The second player has a winning strategy: he always takes l stone from the pile. One his first move the through Q meets the circles again at B and B'. Show that PBB' and QAA' have equal perimeters.
first player must take an odd number of stones, so leaving an even number. Now the second player
always has an even number of stones in the pile and always leaves an odd number. The first player must Solution
always take an odd number and hence must leave an even number. Since O is not odd, the second player
cannot lose.

..•
Thanks lo José Nelson Ramirez

Problem A2
A'
AB is a diameter of a circle. C and O are points on the tangent at B on opposite sides of B. AC, AO meet

•..
the circle again at E, F respectively. CF, DE meet the circle again at G, H respectively. Show that AG =
AH . A

Solution S'

Thanks fa José Nelson Ramirez

Since AP is parallel to BQ and APQB is inscribed in a circle we must have AQ = PB. Similarly, A'Q =
e PB'.

• Since APQB is cyclic, I-:ABQ = IIA'PQ. Since A'PQB' is cyclic, i IA'PQ = 180° - 'A'B'Q, so AB is
1


parallel to AB. Hence AA'B'B is a parallelogram, so AA' = 88'. So the triangles are in fact congruent.
A a


Problem D2


An 8 x 8 board is divided into unit squares. Each unit square is painted red or blue. Find the number of
D ways of doing this so that each 2 x 2 square (of four unit squares) has two red squares and two blue
squares.

• AEB, ABC are similar (' A common and i IAEB = L1ABC= 90°), so AE·AC = AB'. In the same way, Answer


AFB and ABO are similar, so AF·AO = AB2, so AE·AC = AF·AO. Hence CEFO is cyclic. So ! ¡CEO =
CFO, in other words, . AEH = ·AFG. Hence the corresponding chords are also equal, so AH = AG.
29 - 2.

• Problem A3
Solution

• Given integers a> 1, b > 2, show that ab + l 2':b(a+I). When do we have equality?
Thanks fa José Nelson Ramirez

• Solution
We can choose the colors in the first column arbitrarily.

• Thanks fa José Nelson Ramirez


Ifthe squares in the first column alternate in color, then there are 2 choices for the second column, cither
matching the first column, or opposite colors. Similarly, there are 2 choices for each of the remaining


eolumns. There are 2 ways in whieh the squares in the first eolumn can alternate in color, so we get 28 Answer: "1/129.
ways in all with alternating eolors in eaeh eolumn.

Ifthe squares in the first eolumn do not alternate in color, then there must be two adjaeent squares the A
same color. Henee the two squares adjacent to them in the seeond eolumn are determined. Henee all the
squares in the seeond eolumn are determined. It also has two adjaeent squares ofthe same color, so all
the squares in the third eolumn are determined, and so on. There are 28 ways of eoloring the first
eolumn. 2 ofthese ways have altemating colors, so 28 - 2 have two adjaeent squares the same.

Problem B3

Call a positive integer a tico ifthe sum ofits digits (in base 10) is a multiple of2003. Show that there is
an integer N sueh that N, 2N, 3N, ... , 2003N are all tieos. Does there exist a positive integer sueh that all
its multiples are tieos?
B e

Let the side length be x. Using the eosine formula, we have eos APB = (74 - x')/70, eos APC = (89 -
....
Answer x')/80, eos BPC = (113 - x')l112. But eos BPC = eos APC eos BPC - sin APC sin BPC, so (113 - x')/I12
= (74 - x')/79 (89 - x')/80 -"1/((1 - (74 - x')'/70') (1 - (89 - x')'/80'».

No.
We isolate the square root term, then square. We multiply through by 25.256.49 and, after sorne
simplifieation, we get x6 - 138x4+ 1161x2= O. Henee x = O, ±3, ±"l/129.We diseard the zero and
Solution
negative solutions. x = 3 eorresponds to a point P outside the triangle. So the unique solution for a point
P inside the triangle is x = "1/129.
Thanks fa José Nelson Ramirez

Alternative solution by Johannes Tang


Let A = 10001 0001 0001 ... 0001 (with 2003 Is). Then kA isjust 2003 repeating groups for k s: 9999
and is therefore a tieo.
Rotate the triangle about C through 60°. Let P go to P'. We have AP' = 7, CP' = 8 and angle PCP' = 60°,
so PP'C is equilateral. Henee angle CPP' = 60°. Also PP' = 8. Using the eosine formula on triangle APP'
Note that for any N relatively prime to 10 we have (by Euler) 10<P<9N) = 1 mod 9N, in other words, 9Nk = we find angle APP' = 60°. Henee angle APC = 120°.Now applying eosine formula to triangle APC, we
9...9 (cp(9N)9s) for sorne k. Henee Nk is a repunit divisible by N. Now suppose all multiples ofN are get resulto
tieos. Take k so that Nk is a repunit. Suppose it has h 1s. So it has digit sum h. Then 19Nk = 211..109
with h-2 Is, so its digit sum is h + 9. But h and h+9 eannot both be multiples of2003. Contradietion.
Problem A3

OIM 1985
Find the roots r., r" r" r4of the equation 4x4- ax' + bx' - ex + 5 = O, given that they are positive reals
satisfying rl/2 + ri4 + r,/5 + rJ8 = l.
ProblemAl
Solution
Find all integer solutions to: a + b + e = 24, a2+ b2+ e2 = 210, abe = 440.
We have rlr,r,r4 = 5/4 and henee (rl/2) (r,/4) (r,l5) (rJ8) = 1/44.But AMlGM gives that (rl/2) (r,/4) (r,/5)
Solution (r4/8) s: «rl/2 + ri4 + r,l5 + rJ8)/4 )4= 1/44 with equality iffr¡/2 = r,/4 = r,l5 = r4/8.Henee we must
have r¡ = 1/2, r, = 1, r, = 5/4, r, = 2.
ab + be + ca = «a + b + e)2- (a' + b' + e') )/2 = 183, so a, b, e are roots ofthe eubie x' - 24x' + 183x-
440 = O. But it easily faetorises as (x - 5)(x - 8)(x - 11) = O, so the only solutions are permutations of (5, Problem Bl
8,11).
The reals x, y, z satisfy x f. 1, Y f. 1, x f. y, and (yz - x')/(I - x) = (xz - y')/(I - y). Show that (yx - x')/(1 -
Problem A2 x) = x + y+ z.

Pis a point inside the equilateral triangle ABC sueh that PA = 5, PB = 7, PC = 8. Find AB. Solution

Solution
•• We have yz - x' -Iz + yx' = xz -1- x'z + xy'. Hence z(y - x -1 + x') = _y' + xl- x'y + x'. Hence z = OIM 1987
(x + y - xy)/(x + y - 1). Problem Al

• So yz = x + y + Z - xy - xz, so yz - x' = x + y + z - x' - xy - xz = (x + y + z)(I - x), so (yz - x')/(I - x) = (x Find f(x) such that f(x)'f( (I-x)/(I +x) ) = 64x for x not O,± 1.

• + Y + z).
Solution

• Problem B2
Put x = (I-y)/(I+y), then (I--x)/(I+x) = y, so f( (l-y)/(I+y»' f(y) = 64(I-y)/(I+y). Hence f( (l-x)/(l+X)

• The function f(n) is defined on the positive integers and takes non-negative integer values. lt satisfies (1) f f(x) = 64(I-x)/(1 +x). But f(X)4f( (I-x)/(I +x) )' = 64'x', so f(x)' = 64 x'(l + x)/(l - x). Hence f(x) = 4 (
f(mn) = f(m) + fln), (2) f(n) = Oifthe last digit ofn is 3, (3) f(IO) = o. Find f(1985). x'(J + x)/(I - x) )1/]

• Solution ProblemA2

• Iff(mn) = O,then f(m) + f(n) = O(by (1». But f(m) and f(n) are non-negative, so f(m) = f(n) = o. Thus
f(IO) = Oimplies f(5) = o. Similarly f(3573) = Oby (2), so f(397) = o. Hence f(1985) = f(5) + f(397) = o.
In the triangle ABC, the midpoints of AC and AB are M and N respectively. BM and eN meet at P.
Show that if it is possible to inscribe a circle in the quadrilateral AMPN (touching every side), then ABe

• Problem B3
is isosceles.

• o is the circumcenter ofthe triangle ABe. The lines AO, BO, eo meet the opposite sides at D, E, F
Solution

• respectively. Show that liAD + lIBE + l/Cf' = 2/AO.


A

• Solution

• JJ_'~._-------."_ A


,/.
/
/

'.
/
//

l. ( B e

• B e


Ifthe quadrilateral has an inscribed circle then AM + PN = AN + PM (consider the tangents to the circle
from A, M, P, N). But if AB > Ae, then BM > eN (see below). We have AN = AB/2, PM = BM/3, AM
= Ae/2, PN = CN/3, so it folIows that AM + PN < AN + PM. Similarly, AB < Ae implies AM + PN >

• AN + PM, so the triangle must be isosceles.

• Projecting onto the altitude from A, we have AD cos(e - B) = Ae sin e = 2R sin B sin e, so 2R1AD=
cos/C - B)/(sin B sin e). ," .
To prove the result about the medians, note that BM' = BC' + CM' - 2 Be.CM cos e = (BC - eM cos
e)' + (CM sin e)'. Similarly, CN' = (Be - BN cos B)' + (BN sin B)'. But MN is paralIel to Be, so eM


sin e = BN sin B. But AB > AC, so BN > CM and B < C, so cos B > cos e, hence BN cos B > eM eos
e and Be - eM cos C > BC - BN cos B. So BM > eN.
Hence 2R1AD+ 2R1BE+ 2R1eF =cos(e - B)/(sin B sin e) + cos(A - e)/(sin e sin A) + cos(B - A)/(sin

• A sin B). So 2R sin A sin B sin e (liAD + lIBE + l/Cf') = sin A cos(B - C) + sin B cos(e - A) + sin e
cos(A - B) = 3 sin A sin B sin e + sin A cos B cos e + sin B cos A cos e + sin e cos A cos B = 3 sin A
sin B sin C + sin(A + B) cos C + sin e cos A cos B = 3 sin A sin B sin C + sin C (cos C + cos A cos B)
Problem A3

• = 3 sin A sin B sin C + sin C (-cos(A + B) + cos A cos B) = 4 sin A sin B sin C. Hence liAD + l/BE +
I/CF = 2/R.
Show that if(2 + ...J3l = 1 + m + n...J3, for positive integers m, n, k with k odd, then m is a perfeet square.


11
Solution

We have (2 + ..)3)4= 97 + 56..)3 = 14 (7 + 4..)3) - 1 = 14 (2 + ..)3)2- l. Hence (2 + ")3)k+2= 14 (2 + ")3)k_


(2 + ..)3t2• Thus if(2 + ..)3l = ~ + bk..)3,then ~+2 = 14 ~ - ~-2·

Now suppose the sequence Cksatisfies CI = 1, C2= 5, Ck+1= 4 Ck- Ck-I.We claim that Ck2- Ck_ICk+1 = 6.
Induction on k. We have C, = 19, so C,' - CIC,= 25 - 19 = 6. Thus the result is true for k = 2. Suppose it is
true for k. Then Ck+1= 4 Ck- Ck_l,so Ck+l2= 4 CkCk+1 = 4 CkCk+1
- Ck-ICk+1 - Ck2+ 6 = ck(4 Ck+1- Ck)+ 6 = CkCk+2
+ 6, so the result is true for k+ l.

Now put dk = Ck2+ 1. We show that dk+2= 14 dk+1- dk. Induction on k. We have d, = 2, d2 = 26, d, = 362
= 14 d2 - d., so the result is true for k = 1. Suppose it is true for k. We have Ck+l- 4 Ck+2+ Ck+1= O. Hence
12 + 2 Ck+,Ck+1 - 8 Ck+2Ck+1 + 2 Ck+12 = 12. Hence 2 Ck+,'- 8 Ck+2Ck+1 + 2 Ck+12
= 12. Hence 16 Ck+,'- 8 e
Ck+2Ck+1+ Ck+12 + 1 = 14 Ck+/ + 14 - Ck+12 - 1, or (4 Ck+2- Ck+I)2+ 1 = 14 (Ck+,'+ 1) - (Ck+12
+ 1), or Ck+l2+ 1
= 14 (Ck+,'+ 1) - (Ck+12+ 1), or dk+l= 14 dk+2- dk+l. So the result is true for al! k.
If AB is paral!el to CD,then it is obvious that PQ is paral!el to both. So assume AB and CD meet at O.
But al = 2, a, = 26 and a2k+J= 14 a2k+1- a2k-I,and d¡ = 2, d2 = 26 and dk+1= 14 dk - dk-I. Hence a2k-1= d, = Take o as the origin for vectors. Let e be a unit vector in the direction OA and r a unit vector in the
Ck2+ 1. direction OC. Take the vector OA to be ae, OB to be be, OC to be cf, and 00 to be df. Then OP is ( (d -
c)ae + (a - b)dl)/(d - C+ a - b) and OQ is «d - c)be + (a - b)cl)/(d - e + a - b). Hence PQ is (e - d)(a - b)(e
+ 1)/(d - e + a - b). But e and f are unit veetors, so e+ f makes the same angle with eaeh of them and
Problem BI henee PQ makes the same angle with AB and CD.

Define the sequen ce PI, P2, P" ... as fol!ows. PI = 2, and pn is the largest prime divisor OfpIP2 ... pn-I+ 1. OIM 1988
Prove that 5 does not occur in the sequence. Problem Al

Problem B2 The sides of a triangle form an arithmetie progression. The altitudes also form an arithmetie progression.
Show that the triangle must be equilateral.
Show that the roots r, s, t ofthe equation x(x - 2)(3x - 7) = 2 are real and positivc. Find tan-Ir + tanls +
tan"t, Solution

Solution Let the sides be a, a+d, a+2d with d >= O.Then the altitudes are k/a > k/(a+d)::: k/(a+2d), where k is
twiee the area. We c1aim that k/a + k/(a+2d) > 2k/(a + d) unless d = O.This is equivalent to (a + d)(a +
Put f(x) = x(x - 2)(3x -7) - 2 = 3x' - 13x2 + 14x - 2. Then feO) = -2, f(l) = 2, so there is a root between O 2d) + ala + d) > 2a(a + 2d) or 2d2 > O,whieh is obviously true. So the altitudes ean only form an
and l. f(2) = -2, so there is another root between 1 and 2. f(3) = 4, so the third root is between 2 and 3. arithmetie progression if d = Oand henee the triangle is equilateral.
f(x) = Ohas three roots, so they are al! real and positive.
Problem A2
We have tan(a + b + e) = (tan a + tan b + tan C - tan a tan b tan c)/(I - (tan a tan b + tan b tan c + tan C tan
a». So putting a = tan-Ir, b = tanls, C= tan'], we have, tan(a + b + e) = ( (r + s + t) - rst)/( 1 - (rs + st + tr) The positive integers a, b, e, d, p, q satisfy ad - be = 1 and aIb > p/q > e/d. Show that q >= b + d and that
) = (13/3 - 2/3)1(1 - 14/3) = -1. So a + b + C = -¡¡j4 + kn, But we know that each ofr, s, t is real and if q = b + d, then p = a + c.
positive, so a + b + e lies in the range O lo 3¡¡j2. Hence a + b + e = 31t/4.
Problem B3
Solution
ABCD is a eonvex quadrilateral. P, Q are points on the sides AD, BC respeetively such that AP/PD =
BQ/QC = AB/CD. Show that the angle between the lines PQ and AB equals the angle between the lines p/q > c/d implies pd > cq and hence pd >= cq + 1, so p/q z c/d + l/(qd). Similarly, aIb > p/q implies alb
PQ andCD. :::p/q + I/(bq). So aIb - c/d > l/(qd) + l/(qb) = (b + d)/(qbd). But alb - c/d = l/bd. Hence q > b + d.

Solution Now assume q = b + d. We have ad - be = 1 ::;d, so ad + cd - d::; bc + cd and hence (a+c-I )/(b+d) ::;c/d.
So p :::a + C. Similarly ad - bc ::;b, so be + b + ab :::ad + ab, so (a+c+ 1)/(b+d) :::aIb. So p ::;a + C. Hence
p = a + C.

.J
J
•• Problem A3 Let k3 = 2 and let x, y, z be any rational numbers such that x + y k + Zk' is non-zero. Show that there are


rational numbers u, v, w such that (x + y k + Zk')(u + v k + W k') = l.
Pis a fixed point in the plane. Show that amongst triangles ABC such that PA = 3, PB = 5, PC = 7, those
with the largest perimeter have P as incenter. Solution

• Solution We need xu + 2zv + 2yw = 1, yu + xv + 2zw = O, zu + yv + xw = O. This is just a straightforward set of

• linear equations. Solving, we get: u = (x' - 2yz)/d, v = (2r - xy)/d, w = (y' - xz)/d, were d = x3 + 2y3 +
4z3 - 6xyz.

• This would fail if d = O. But if d = O, then multiplying through by a suitable integer we have 6mnr = m'


+ 2n3 + 4r' for sorne integers m, n, r. But we can divide by any common factor ofm, n, r to get them
without any common factor. But 6mnr, 2n3,4r' are all even, so m must be cven. Put m = 2M. Then
12Mnr = 8M3 + 2n3+ 4r3,so 6Mnr = 4M3 + n' + 2r'. But 6Mnr, 4M3and 2r' are all even, so n must be

• even. Put n = 2N. Then 12MNr = 4M3 + 8N3 + 2r', so 6MNr = 2M3 + 4N3 + r', so r must be even. So m,
n, r had a common factor 2. Contradiction. So d cannot be zero.

• Problem B3

• Let S be the collection of all sets of n distinct positive integers, with no three in arithmetic progression.
Show that there is a member ofS which has the largest sum ofthe inverses ofits elements (you do not

• 8 e have to find it or to show that it is unique).

•.. Given points P, B, C and a fixed circle center P, we show that the point A on the circle which maximises
AB + AC is such that PA bisects angle BAC. Consider a point A' close to A Then the change in AB +
AC as we move A to A' is AA'(sin PAC - sin PAB) + O(AA"). So for a maximal configuration we must
have sin PAC = sin PAB, otherwise we could get a larger sum by taking A' on one side or the other. This
Solution

Induction on n. For n = 1, {I} is obviously maximal. Now suppose a, < a, < ...< a. is a maximal set for
n. Take a.+, to be the smallest integer > a. such that {a" a" ... , a.+d has no three members in AP. Now
consider the sequences b¡ < b, < ...< bnwhich have no three in AP and bn+,:s a.+,. There are onlyfinitely

• applies to each vertex ofthe triangle, so P must be the incenter. many such sequences. So we can find one which is maximal. Suppose it is c, < c, < ...< Cn+,.Now take
whichever of a., C¡ has the larger sum of inverses. It is clearly maximal with respect to sequences whose

• Commenl. This is arguably loo advanced, because il uses calculus concepls Iike O(x). Can anyone largest member is:S a.+,. Suppose we have a sequence x¡ < x, < ...< xn+,with no three in AP and xn+,>
supplya more elementary solution? a.+,. Then we have l/xn+,< l/an+,and, by induction, l/x, + ... + l/xn:S l/a, + ... + l/a., so l/x¡ + ... +
I/xn+,< l/a, + ... + I/a.~" so it is worse than the sequence we have chosen.

•• Problem Bl

Points A" A" ... .A, are equally spaced on the side BC of the triangle ABC (so that BA, = A,A, = .
OIM 1989
Problem Al

An_,An= AnC).Similarly, points B" B" ... , B, are equally spaced on the side CA, and points C" C" , Find all real solutions to: x + y _Z = -1; x' _y' + z' = 1, -x' + y3+ Z3=-1.

• C, are equally spaced on the side AB. Show that (AA,' + AA,' + ... + AAn' + BB,' + BB,' + ... + BBn' +
C,' + ... + CCn') is a rational multiple of(AB' + BC' + CA').


Solution
Solution


Answer: (x, y, z) = (1, -1,1) or (-1, -1, -1).
Using the cosine formula, AAk' = AB' + k'BC'/(n+ 1)' - 2k AB.BC/(n+ 1) cos B. So L AAk' = n AB' +


BC'/(n+ 1)' (1' + 22 + ... + n') - 2 AB.BC cos B (1 + 2 + ... + n)/(n+ 1). Similarly for the other two sides. From the first equation x = z - y - 1. Substituting in the second equation: 2z' - 2yz + 2y - 2z = O, so (z -
1)(z - y) = O. Hence z = I or y = Z. If z = 1, then from the first equation x + y = O, and hence from the
Thus the total sum is n (AB' + BC' + CA') + n(2n+I)/(6(n+I» (AB' + BC' + CA') - n (AB.BC cos B + last equation, x = 1, Y= -l. If Y= z, then x = - 1, and hence from the last equation y = z = - l.

• BC.CA cos C + CAAB cos A). But AB.BC cos B = (AB' + BC' - CA')/2, so AB.BC cos B + BC.CA
cos C + CAAB cos A = (AS' + BC' + CA')/2. Thus the sum is rational multiple of(AB' + BC' + CA'). Many thanks to Allan Lacy Morafor pointing out that the original solution was wrong.

• Problem B2 Problem A2


Given positive real numbers x, y, z eaeh less than x/2, show that x/2 + 2 sin x eos y + 2 sin y eos z > sin Solution
2x + sin 2y + sin 2z.
Put f(a, b, e) = (a - b)/(a + b) + (b - e)/(b + e) + (e - a)/(a + e). Let A, B, C be the permutation of a, b, e,
Solution with A <= B <= C.lf(A, B, C) = (b, a, e), (a, e, b) or (e, b, a), then f(a, b, e) = X, where X = (B - A)/(B
+ A) + (C - B)/(C + B) - (C - A)/(A + C).lf(A, B, C) = (a, b, e), (b, e, a) or (e, a, b), then f(a, b, e) =v-X,
Solution by David Krumm

..
Put B = A + h, C = B + k = A + h + k, where h, k 2: O. Sinee A, B, C are the sides of a triangle, we also
have A + B > e or A > k. So X = hI(2A + h) + k/(2A + 2h + k) - (h + k)/(2A + h + k) = hk(h + k)/( (2A +
We have sin 2x + sin 2y + sin 2z - 2 sin x eos y - 2 sin y eos z = 2 sin x(eos x - eos y) + 2 sin y(eos y- h)(2A + h + k)(2A + 2h + k) ). This is obviously non-negative. We c1aimalso that it is < 1/20. That is
eos z) + 2 sin z eos z, so we wish to show that sin x(eos x - eos y) + sin y(eos y - eos z) + sin z eos z < equivalent to: 20h2k + 20hk2 < (2A + h)(2A + h + k)(2A + 2h + k). Sinee k < A it is suffieient to show
'/[/2 (-). that 20h2k + 20hk2:s (2k + h)(2k + h + k)(2k + 2h + k) = 18k' + 27hk2+ 13h2k+ 2h' or 18k' + 7hk2-
7h2k+ 2hJ 2: O. But 7k2- 7hk + 2h2 = 7(k - hl2)2 + h2/4 2: o and h and k are non-negative, so 18k' + h(7k2
- 7hk + 2h2)2: O.

Thus we have established that O <= X < 1/20, whieh shows that f(a, b, e) < 1120,whieh is slightIy
--... _- ....._--.... stronger than the required result. Problem 81
sm z
The ineircle ofthe triangle ABC touehes AC at M and BC at N and has eenter O. AO meets MN at P and
BO meets MN at Q. Show that MP.OA = BC.OQ.
sin y
Solution

sm x
A

cos z cos X
cos y

We have to eonsider six cases: (1) x y:S z; (2) x:s z:S y; (3)
s x:S z; (4) y:S z:S x; (5) z:S x:S y; (6) z
y s

:Sy :Sx. The first case is obvious from the diagram, beeause the Ihs represents the shaded area, and the
rhs represents the whole quarter eircle. 8 N e
In cases (2) and (5) the seeond term is negative, and - sin y < - sin x, so the sum ofthe first two terms is
less than sin x (eos x - eos y) + sin x (eos y - eos z) = sin x (eos x - eos z). But by the same argument as
The key to getting started is to notiee that angle AQB = 90°.
the first ease the two reetangles represented by sin x( eos x - eos z) and sin z eos z are disjoint and fit
inside the quarter eircle. So we have proved (2) and (5).
Angle BAQ = 90° - B/2, so angle OAQ = 90° - B/2 - A/2 = C/2. So OQ = AO sin C/2. Thus we have to
show that MP = BC sin C/2.
In cases (3) and (4), the first term is negative. The remaining two terms represent disjoint reetangles
Iying inside the quarter eirele, so again the inequality holds.
I.et the ineircle toueh AS at L and let Y be the midpoint ofML (also the interseetion ofML with AO).
Angle NMC = 90° - C/2. It is also A/2 + angle MPY, so angle MPY = 90 - C/2 - A/2 = B/2. Henee MP =
In ease (6) the first two terms are negative. The last term is Yí sin 2z:S Yí < '/[/2, so the inequality
MY/sin B/2. We have MY = MO sin MOA = reos A/2 (where r is the inradius, as usual). So MP = (r
eertainly holds. eos A/2)/sin B/2. We have BC = BN + NC = r (eot B/2 + eot C/2), so MPIBC = (eos Al2)/( sin B/2 (eot
B/2 + eot C/2) ). Henee MP/(BC sin C/2) = ( eos A/2 )/( eos B/2 sin C/2 + sin B/2 eos C/2) = eos A/2
Problem A3 Isin(B/2 + C/2) = l.

Ifa, b, e, are the sides ofa triangle, show that (a - b)/(a + b) + (b - e)/(b + e) + (e - a)/(a + e) < 1116. Problem 82
•• The function f on the positive integers satisfies f(1) = 1, f(2n + 1) = f(2n) + 1 and f(2n) = 3 f(n). Find the
set of all m such that m = f(n) for sorne n.
Solution



Solution

We show that to obtain f(n), one writes n in base 2 and then reads it in base 3. For example 12 = 11002,

• so f(l2) = 11003 = 36. Let gen) be defined in this way. Then certainly g(l) = 1. Now 2n+ 1 has the same
binary expansion as 2n except for a final 1, so g(2n+1) = g(2n) + l. Similarly, 2n has the same binary


expansion as n with the addition of a final zero. Hence g(2n) = 3 gen). So g is the same as f. Hence the
set of all m such that m = f(n) for sorne n is the the set of all m which can be written in base 3 without a
digit2.

• Problem 8J B D e

• Show that there are infinitely many solutions in positive integers to 2a2- 3a + 1= 3b2+ b.

••
IJAEI = IJAME = 90°, so AEI and AME are similar. Hence AM/AE = AE/AI or AM·AI = AE1 AE is
tangent to the incircle, so AE2 = AP·AD. Hence AM·AI = AP'AD, so ifP,M,I,D are not collinear, then
they are cyclic.

Solution Thanks to Johann Peter Gustav Lejeune Dirichlet Problem A3


iI
Put A = a - I and the equation becomes A(2A + 1) = b(3b + 1). Let d be the greatest common divisor of f(x) = (x + bf + e, where b and e are integers. Ifthe prime p divides e, but p2does not divide e, show
A and b. Put A = dx, b = dy. Then x(2dx + 1) = y(3dy + 1). Since x and y are coprime, x must divide 3dy that f(n) is not divisible by p2for any integer n. If an odd prime q does not divide e, but divides f(n) for
+ 1. So put 3dy + 1 = nx. Then 2dx + 1 = ny. Solving for x and y in terms ofn and d we get x = (n +
3d)/(n2- 6d\ y = (n + 2d)/(n2- 6d2).
sorne n, show that for any r, we can find N such that q' divides f(N).

•• So we would certainly be home ifwe could show that there were infinitely many solutions to n2- 6d2 =
1. It is not hard to find the first few: 12- 6.02= 1,52 - 6.22= 1,492 - 6.202= 1. We notice that 492= 2.52_
1, so we wonder whether n = 2.492- 1 might be another solution and indeed we find it gives d = 1960 =
2.49.20. This suggests we try (2 n2- 1)2- 6(2nd)2= 4n' - 4n2+ 1 - 24n2d2= 4n2(n2- 6d2- 1) + 1 = l. So
Solution

The first part is trivial. Ifp does not divide (x+b), then it does not divide (X+b)2,so it does not divide
(X+b)2+C.On the other hand, ifp does divide x+b, then p2divides (X+b)2,so p2does not divide (x+bj'+c.

• there are indeed infinitely many solutions to n2- 6d2 = I and we are done.
For the second part, we use induction on r. For r = 1, we are given that q divides f(n). Now suppose that


q' divides f(N) for sorne N. If s'" divides f(N), then we are done. So suppose q'" does not divide f(N),
so f(N) = q'h where q does not divide h. We have f(N+kq') = f(N) + q'(2N+2b)k = q'h + q'(2N+2b)k.
Now q divides (N+b)2+C, and does not divide e, so it does not divide (N+b)2and hence does not divide
OIM 1990 N+b. It is odd, so it does not divide 2N+2b. Hence we can find k such that k(2N+2b) = -h mod q. Then
Problem Al we have s'" divides f(N+kq'), which completes the induction.

The function fis defined on the non-negative integers. f(2" - 1) = O for n = 0,1,2, .... Ifm is not ofthe Problem BI
form 2" - 1, then f(m) = f(m+l) + 1. Show that f(n) + n = 2k - 1 for sorne k, and find f(21990) .

•• Solution
The circle C has diameter AB. The tangent at B is T. For each point M (not equal to A) on C there is a
circle C' which touches T and touches C at M. Find the point at which C' touches T and find the locus of

• We claim that if Z'"<= n < 2m+l, then f(n) = 2m+1 - n - 1. Put r = 2m+1 - n. Then the claim follows by
the center of C' as M varies. Show that there is a circle orthogonal to all the circles C'.

• induction on r. Hence f(21990) = 21 l.


,!,}() - Answer

• Problem A2 C' touches T at the intersection ofT and the line AM


the locus of the center is a parabola vertex B
the circle center A radius AB is orthogonal to all circles C' Solution

• 1is the incenter ofthe triangle ABC and the incircle touches BC, CA, AB at D, E, F respectively. AD
meets the incircle again at P. M is the midpoint ofEF. Show that PMID is cyclic (or the points are
collinear),


~
Let F be the bottom left-hand vertex. Two sides of the triangle EFG are in the path, so at least one of EF
and FG is. But EF and EG are the only segments with endpoint F, so an even number ofthem must be in
O'
the path, so both are in the path. Hence, again considering EFG, EG is not in the path. Hence,
N considering EHG, EH and HG are in the path.

E has an even number of segments on the path, so CE is not on the path. Hence (considering CEH) CH
is on the path. Similarly, GJ is not on the path and HJ is on the path. An even number of segments at H
are on the path, so DH and HI are either both on the path or neither is on the path. But (considering DH!)
B A at least one must be, so they both are. Hence DI is not, and CD is not.

Since n > 2, C is not the top left vertex. Considering MCD, MC and MD are both on the path.
Considering DLI, DL is on the path. There must be an even number of segments at D, so DP is on the
path. Hence MP is not. Now M cannot be the top left vertex (with n = 3) because then it should have an
Let O be the center ofC. Let the line AM meet T at N. Let the perpendicular to T at N meet the line OM odd number of segments, whereas it would have two (MC and MD). So there must be a vertex N aboye
at O'. Then 10'NM = ¡ IMAB (O'N parallel to AB, because both perpendicular to T) = llOMA (OM = M. Considering NMP, MN must be in the path. But now M has an odd number of segments.
OA) = ~O'MN. So O'M = O'N. Hence O' is the center ofC'. Contradiction.

Take B to be the origin and A to be the point (2a,0), so O is (a.O) and C has radius a. If O' is (x.y), then ProbIem B3
we require that 0'0 = x+a or (x-aj'+y' = (x+a)', or y' = 4ax, which is a parabola with vertex B and axis
the x-axis. f(x) is a polynomial of degree 3 with rational coefficients. If its graph touches the x-axis, show that it has
three rational roots.
Triangles AMB, ABN are similar C_.AMB= •"ABN = 90°), so AM/AB = AB/AN and hence AM·AN =
AB'. Now consider the cirele center A radius AB. It must meet the cirele C', because it contains the
Solution
point M. Suppose it meets at X. Then AX' = AB' = AM'AN, so AX is tangent to C' and hence the cireles
are orthogonal.
Without loss of generality, f(x) = x' - ax' + bx - e, where a, b, e are rational. Since the graph touches the
x-axis, there is a repeated root, so we may take the roots to be h, h, k. Hence 2h + k = a, 2hk + k2 = b,
Problem 82 h'k = c. Hence a' - 3b = (h - k)'. Put r = ±..J(a'- 3b), where the sign is chosen so that h = al3 + r/3, k =
al3 - 2r/3. We need to show that r is rational. If r is zero there is nothing to prove, so assume r is non-
A and B are opposite corners of an n x n board, divided into n' squares by lines parallel to the sides. In zero.
each square the diagonal parallel to AB is drawn, so that the board is divided into 2n' small triangles.
The board has (n + 1)' nodes and large number of line segments, each of length I or ..J2. A piece moves We have 9h' = 2a' - 3b + 2ar. Hence 27h'k = -Za' + 9ab + (6b - 2a')r. But 27h'k = 27c. So r = (27c + 2a3
from Ato B along the line segments. It never moves along the same segment twice and its path ineludes _9ab)/(2(3b - a')). Note that 3b - 2a' is non-zero because r is non-zero. So r is a rational combination of
exactly two sides of every small triangle on the board. For which n is this possible?
a, b, e and hence is rational.

Answer OIM 1991

n=z only Problem I

The number I or the number -1 is assigned to each vertex of a cube. Then each face is given the product
of its four vertices. What are the possible totals for the resulting 14numbers?
Solution
Solution

Answer: 14, 6, 2, -2, -6, -10.

If every vertex is 1, we get 14 and that is elear1ythe highest possible total. The lowest possible total
The diagram aboye shows that n=2 is possible (the path is AHEFGHCDlHB). Now suppose n > 2. cannot be lower than -14, but we cannot even achieve that because if all the vertices are -1, then all the
faces are l.
Note that if X is any vertex except A or B, then an even number of segments with endpoint X must be in
the path.

.J
J
Ifwe change a vertex, then we also change three faces. Ifthe vertex and the three faces are all initially gives a, b, d, e = 3, 4,7,8 or 3,5,6,4 as the only possibilities. Checking, we find the solution aboye and
the sarne, then we make a change of ±8. If three are of one kind and one the opposite, then we make a no others.

• change of ±4. Iftwo are of one kind and two the opposite, then we make no change. Thus any sequence
of changes must take us to 14 + 4n for sorne integer n. But we have already shown that the total is
Problem 5


greater than -14 and at most 14, so the only possibilities are -10, -6, -2, 2, 6,10 and 14.

Let p(m, n) be the polynomial2m' - 6mn + 5n'. The range ofp is the set ofall integers k such that k =


We show that 10 is not possible. Ifmore than 2 vertices are -1, then the vertex total is at most 2, there
are only 6 faces, so the total is less than 10. If all vertices are 1, then the total is 14. If all but one vertex p(m, n) for sorne integers m, n. Find which members of {I, 2, ... , lOO}are in the range ofp. Show that
is 1, then the total is 6. So the only possibility for 10 is just two vertices -1. But however we choose any if h and k are in [he range of p. then so is hk.


two vertices, there is always a face containing only one ofthem, so at least one face is -1, so the face
total is at most 4 and the vertex total is 4, so the total is less than 10. The other totals are possible, for Answer
exarnple:

• 1,2,4,5,8,9,10,13,16,17,18, 20,25,26,29,32,34,36,37, 40,41,45,49,50,52,53,58, 61,64,65,68,72,73,74,


80,81,82,85,89,90,97,98,100

• 14: all vertices 1

Solution


6: one vertex -1, rest 1

2: three vertices of one face -1, rest 1 We have p(m,n) = (m-2n)' + (m-n)', so p(2a-b,a-b) = a' + b2 Hence the range ofp isjust the sums of

• -2: all vertices -1


two squares.

• -6: all vertices but one -1 (a' + b')(c' + d') = (ac - bd)' + (ad + be)', which establishes that ifh and k are in the range, then so is hk.


-10: two opposite comers 1, rest -1 Thanks lo Johann Peter Gustav
Problem 8J
A2. Two perpendicular lines divide a square into four parts, three of which have area l. Show that the

• fourth par! also has area 1.


Given three non-collinear points M, N, H show how to construct a triangle which has H as orthocenter
and M and N as the midpoints oftwo sides.

..• AJ. fis a function defined on all reals in the interval [O, 1] and satisfies f(O)= O, f(xl3)
= 1 - f(x). Find f(18/1991).

Problem 4
= f(x)/2, f(l - x)
Solution

•• Find a number N with five digits, all different and none zero, which equals the sum of all distinct three
digit numbers whose digits are all different and are all digits ofN.
Po

'•.
H'
Solution

Answer: 35964

B e

••
There are 4.3 = 12 numbers with a given digit ofn in the units place. Similarly, there are 12 with it in the
tens place and 12 with it in the hundreds place. So the sum ofthe 3 digit numbers is 12.111 (a + b + C +
d + e), where n = abcde. So 8668a = 332b + 1232c + 1322d + 1331e. We can easily see that a = 1 is too
small and a = 4 is too big, so a = 2 or 3. Obviously e must be even. Ois too small, so e = 2, 4, 6 or 8. Take H' so that M is the midpoint of HH'. The circle diarneter NH' meets the line through H
Working mod 11, we see that O= 2b + 2d, so b + d = 11. Working mod 7, we see that 2a = 3b + 6d + e. perpendicular to MN in two points (in general), either ofwhich we may take as A. Then B is the
Using the mod 11 result, b = 2, d = 9 or b = 3, d = 8 or b = 4, d = 7 or b = 5, d = 6 or b = 6, d = 5 or b = reflection of A in M, and C is the reflection of A in N.

•• 7, d = 4 or b = 8, d = 3 or b = 9, d = 2. Putting each ofthese into the mod 7 result gives 2a - e = 4,1,5,2,


6,3, O,4 mod 7. So putting a = 2 and remembering that e must be 2, 4, 6, 8 and that all digits must be
different gives a, b, d, e = 2,4,7,6 or 2, 7,4, 8 or 2,8,3,4 as the only possibilities. It is then
straightforward but tiresome to check that none of these give a solution for c. Similarly putting a = 4,
To see that this works, note that M is the midpoint of HH' and AB, so AHBH' is a parallelograrn. Hence
AH' is parallel to BH and hence perpendicular to AC. In other words 'NAH' = 90°, so A lies on the
r

circle diarneter NH'. MN is parallel to BC, so A lies on the perpendicular to MN through H.


Thanks lo Johann Peter Gustav Lejeune Dirichlet ABC is an equilateral triangle with side 2. Show that any point P on the incircle satisfies PA2+ PB2+
PC2 = 5. Show also that the triangle with side lengths PA, PB, PC has area ("v3)/4.
OIM 1992
Problem Al Solution

a.,is the last digit of 1 + 2 + ... + n. Find al + a2+ ... + a1992. Take vectors centered at the center O ofthe triangle. Write the vector OA as A etc. Then PA2+ PB2+
PC2 = (P _A)2+ (P _ 8)2 + (P _C)2 = 3P2+ (A2+ B2+ C2)_2P.(A + 8 + C) = 15P2,since A2 = B2 = C2
= 4P2and A + 8 + e = O.Finally the side is 2, so an altitude is "'¡3and the inradius is ("3)13 = 1/"3, so
Solution PA2+ PB2+ PC2= 15/3 = 5.

It is easy to compile the following table, from which we see that a, is periodic with period 20, and Take Q outside the triangle so that BQ = BP and CQ = AP. Then BQC and BPA are congruent, so
indeed the sum for each decade (from Oto 9) is 35. Thus the sum for 1992 is 199·35+ 5 + 6 + 8 = 6984. . IABP = I ICBQ and hence I PBQ = 60°, so PBQ is equilateral. Hence PQ is PB and PQC has sides
equal to PA, PB, Pe. Ifwe construct two similar points outside the other two sides then we get a figure
with total area equal to 2 area ABC and to 3 area PQC + area ofthree equilateral triangles sides PA, PB,
n O 2 3 4 5 6 7 8 9 lO 11 12 13 14 15 16 17 18 1920 PC. Hence 3 area PQC = 2 area ABC - area ABC (PA2+PB2+PC2)/PA2= (3/4) area ABC = (3"3)/4. So
area PQC = ("3)14.
O 360 5 865 568 1 5 063 I O O
Thanks lo Johann Peter Gustav Lejeune Dirichlel
sum O 4 lO 10 15 16 24 30 35 40 46 54 55 60 60 66 69 70 70 70
Problem A2
Problem 81
Let f(x) = a¡/(x + al) + ai(x + a2)+ ... + a.,/(x+ a.,),where a, are unequal positive reals. Find the sum of
the lengths of the intervals in which f{x)2: l. Let a., bn be two sequences of integers such that: (1) a¡, = O,bo = 8; (2) a.,+2= 2 a.,+l- a.,+ 2, bn+2= 2 bn+l-
bn, (3) a.,2+ b/ is a square for n > O. Find at least two possible values for (aI992,bl992).
Answer
Answer

(1992,1996,4'1992+8), (1992·1988, -4,1992+8)


Solution

wlog al > a2> ...> a, The graph of each a¡/(x+ a.) is a rectangular hyberbola with asymptotes x = -a, and
y = O.So it is not hard to see that the graph of f(x) is made up of n+ 1 strictly decreasing parts. For x < - Solution
al, f(x) is negative. For x I (-a" -a,+l),f(x) decreases from 00 to -ai, Finally, for x > -a." f(x) decreases
from 00 to O.Thus f(x) = 1 at n values b, < b2< ...< b., and f(x) 2: 1 on the n intervals (-a¡,b¡), (-a2,b2),... a.,satisfies a standard linear recurrence relation with general solution a, = n2+ An + k. But a.,= O,so k =
, (-a."bn). So the sum ofthe lengths ofthese intervals is ¿ (a, + b.), We show that ¿ b, = O. O.Hence a, = n2+ An. Ifyou are not familiar with the general solution, then you can guess this solution
and prove it by induction.
Multiplying f(x) = 1 by nex + aj) we get a polynomial of degree n:
Similarly, bn = Bn + 8. Hence a.,2+b/ = n' + 2AnJ + (A2+B2)n2+ l6Bn + 64. Ifthis is a square, then
looking at the constant and nJ terms, it must be (rr' + An + 8). Comparing the other terms, A = B = ±4.

Thanks lo Juan Ignacio Restrepo


The coefficient of x" is 1 and the coefficient of x"" is ¿ aJ - ¿ a, = O.Hence the sum ofthe roots, which
is ¿ b., is zero.
82. Construct a cyclic trapezium ABCD with AB parallel to CD, perpendicular distance h
Thanks lo Juan Ignacio Restrepo between AB and CD, and AB + CD = m.

Problem A3
83. Given a triangle ABC, take A' on the ray BA (on the opposite side of Ato B) so that AA' =
BC, and take A" on the ray CA (on the opposite side of Ato C) so that AA" = Be. Similarly take
•• Put y = fez), then f( fez) f(x) ) = x' f(x fez) ) = x'z' f(xz) = f( ftxz) ). But f is (1, 1) so f(xz) = f(x) fez).

••
B', B" on the rays CB, AB respectively with BB' = BB" = CA, and C', C" on the rays AB, CB.
Show that the area ofthe hexagon A"A'B"B'C"C' is at least 13 times the area ofthe triangle ABe. Now suppose f(m) > m' for sorne m. Then by (1), f( f(m)) > f(m' = f(m.m) = f(m)2 But f( f(m)) = m'
f(m), so m' > f(m). Contradietion.

OIM 1993
Problem 1 Similarly, suppose f(m) < rrr'. Then m' f(m) = f( f(m) ) < f(m') = f(m)', so m' < f(m). Contradiction. So

• A palindrome is a positive intcgers which is unchanged ifyou reverse the order of its digits. For
we must have f(m) = m'.

•.. example, 23432. If all palindromes are written in increasing order, what possib1eprime values can the
difference between successive palindromes take?

Solution
Problem Bl

ABC is an equilateral triangle. D is on the side AB and E is on the side AC such that DE touches the
incircle. Show that AD/DB + AE/EC = l.

• Answer: 2, 11. Solution

• Let x be a palindrome and x' the next highest palindrome. If x < 101, then it is easy to see by inspection
that x' - x = 1, 2 or 11, so the only prime differences are 2 and 11.
Put BD = x, CE = y, BC = a. Then since the two tangents from B to the incircle are of equallength, and
similarly the two tangents from D and E, we have ED + BC = BD + CE, or ED = x + y-a. By the cosine

• So assume x > 100. If x and x' have the same final digit, then their difference is divisible by 10 and
law, ED' = AE' + AD' - AE.AD. Substituting and simplifying, we get a = 3xy/(x + y). Hence AD/DB =
(2y - x)/(x + y) and AE/EC = (2x - y)/(x + y) with sum l.

• hence not prime. So they must have different digits. Thus either x = d9...9d and x' = d'O...Od',where d <
9 and d' = d+1, or x' has one more digit than x and d = 9, d' = l. In the first case x' - x = 11. In the second Problem B2
case x' - x = 2. So again the only prime differences are 2 and 11.

Problem 2
IfP and Q are two points in the plane, let m(PQ) be the perpendicular bisector ofPQ. S is a finite set of
n> 1 points such that: (1) ifP and Q belong to S, then sorne point ofm(PQ) belongs to S, (2) ifPQ, P'Q',

..
P"Q" are three distinct segments, whose endpoints are all in S, then ifthere is a point in all ofm(PQ),
m(P'Q'), m(P"Q") it does not belong to S. What are the possible values ofn?
Show that any convex polygon of area 1 is contained in sorne parallelogram of area 2.

Answer
Solution

11 Let the vertices X, Y ofthe polygon be the two which are furthcst apartoThe polygon must lie between
n = 3 (equilateral triangle), 5 (regular pentagon).

"..
the lines through X and y perpendicular to XY (for if a vertex Z lay outside the line through Y, then ZY
> XV). Take two sides of a rectangle along these Iines and the other two sides as close together as Solution
possible. There must be a vertices U and V on each of the other two sides. But now the area 01' the
rectangle is twice the area 01' XUYV, which is at most the area of the polygon. [In the case of a triangle There are n(n-1)/2 pairs ofpoints. Each has a point ofS on its bisector. But each point ofS is on at most
one side ofthe rectangle will be XY.] two bisectors, so 2n::::n(n-I)/2. Hence n:S 5.

• Problem A3 The equilateral triangle and regular pentagon show that n = 3, 5 are possible.

•.. Find all functions f on the positive integers with positive integer values such that (1) if x < y, then f(x) <
f(y), and (2) f(y t{x)) = x'f(xy)
Consider n = 4. There are 6 pairs ofpoints, so at least one point ofS must be on two bisectors. wlog A is
on the bisectors of BC and BD. But then it is also on the bisector of CD. Contradiction.


Solution Many thanks lo Tsimermanfor this.

••
Answer: f(x) = X2 Problem B3

Note that (1) implies f is (1, 1). We say that two non-negative integers are related iftheir sum uses only the digits Oand l. For example
22 and 79 are related. Let A and B be two infinite sets of non-negative integers such that: (1) if a I A
Put y = l. Then f( f(x) ) = x' ftx), and b I B, then a and b are related, (2) if e is related to every member of A, then it belongs to B, (3) if e


r

JI
is related to every member ofB, then it belongs to A. Show that in one ofthe sets A, B we can find an Answer
infinite number of pairs of consecutive numbers.
(h/2 + k/2) ,t(hk/2 - h2/4), where h = ICOI,k = IABI
Solution
Solution
Suppose there is a member of A with last digit d. Then every member ofB must have one oftwo
possible last digits. Suppose there are members ofB with both possibilities. Then every member of A
must have last digit d. So either every member of A has the same last digit or every member of B has the
same last digit (or both). Suppose every member of A has the same last digit d.

But now if n belongs to B and n + d has last digit O, then n+ l + d has last digit l. Moreover, if m is any
member of A, then m+n has last digit O and other digits all O or 1. Hence rn+n+1 last last digit 1 and
other digits all O or 1, so n+ 1 must also belong to B. Similarly, ifn is in B and n+d has last digit 1, then
n-I must also belong to B. So in either case there are infinitely many pairs of consecutive numbers in B. x
A B
OIM 1994
Problem Al

Show that there is a number 1 < b < 1993 such that if 1994 is written in base b then all its digits are the
same. Show that there is no number 1 < b < 1992 such that if 1993 is written in base b then all its digits
are the same. Let the circle have center O on AB and radius r. Let OAO = O, OBC = cp.Since ABCO is cyclic,
l'ADC = 1800-cp,so JODA = 900-cp/2.If AD touches the circle at X, then AO = AX + XO = r cot O+ r
tan(cp/2).Similarly, BC = r cot cp+ r tan(0/2). Put t = tan(0/2). Then cot O = (l-e)/2t, so cot O + tan(0/2) =
Solution
(l+t2)/2t = l/sin O. Similarly for cp,so AO + BC = r/sin O + r/sin cp = AO + OB = AB.

Any even number 2n can be written as 22 in base n-l. In particular 1994 = 22996.
Suppose AO and BC meet at H (we deal below with the case where they are parallel). Then HCO and
HAB are similar, so area HCO = (C02/AB2) area HAB and area ABCO = (1 - C02/AB2) area HAB. Also
We have to show that we cannot write 1993 = aaa ... a¡,. Ifthe number has n digits, then 1993 = a(l + b + AB/CO = HA/HC = HB/HO = (HA+HB)/(HC+HO) = (HA+HB)/(HB-BC+HA-OA) =
... + bn-1) = a(bn - 1l/lb - 1). But 1993 is prime, so a must be l. Hence b"! + ... + b - 1992 = O. So b must (HA+HB)/(HA+HB-AB). Hence HA+HB = AB2/(AB-CO), which is fixed. Now for fixed HA+HB we
divide 1992 = 2'3.83. We cannot have n = 2, for then b = 1992 and we require b < 1992. So n > 2. But maximise the area of HAB by taking HA = HB and hence AO = BC.
832 = 6889 > 1993, so b must divide 24. Hence b = 2,3,4,6,8, 12, or 24. But we can easily check that
none ofthese work:
Put h = CO, k = AB. So k cos O + h = k. Hence cos 0= (I-h/k). Hence sin 0= ,t(2h/k - h2/k'). So area
ABCO = V,(h+k)V,k sin 0= (h/2 + k/2) ,t(hk/2 - h2/4) (*).

If AO and BC are parallel then A and B must he on the circle, so that ílDAB = [ :ABC = 90°. But
ABCO is cyclic, so it must be a rectangle. Hence AB = CO and area ABCO = k2/2. In this case (*) still
1 + 3 + ... + 36= 1093, 1 + ... + )1'7 = 3280
gives the correct answer.
1 + 4 + ... + 45 = 1365, 1 + ... + 46 = 5461
Thanks /0 Vivek Kumar Mehra
1 + 6 + ... 6' = 1555, 1 + ... + 65 = 9331
Problem A3
1 + 8 + 82 + 8' = 585, 1 + ... + 84 = 4681

1 + 12+ 122 + 12'= 1885,1 + ... + 12'=22621 There is a bulb in each cell of an n x n board. Initially all the bulbs are off. If a bulb is touched, that bulb
and all the bulbs in the same row and column change state (those that are 00, tum off, and those that are
1 +24+242=601, 1 + ... +24'= 14425 off, tum on). Show that it is possible by touching m bulbs to tum all the bulbs on. What is the minimum
Problem A2 possible value of m? Answer

ABCO is a cyclic quadrilateral. A circle whose center is on the side AB touches the other three sides. o odd, n is mínimum

..
Show that AB = AO + Be. What is the maximum possible area of ABCO in terms of IABIand ICOI? neven, n2 is minimum

..J

• Solutioo

• If o is odd, touch each bulb in the first column. Then bulbs in the first column are each switched n times,
which is odd and so end up on. AIl other bulbs are switchedjust once, and so end up on. n is obviously

•• minimal, because ifm < n, then there is a bulb which is not switched at all (there must be a column with
no bulb touched and a row with no bulb touched, so the bulb in that column and row is not switched).

In n is even, touch each bulb. Then each bulb is switched 2n-1 times, so ends up on. We show that it is


not possible to do better.
B e


Note first that there is no benefit in touching a bulb more than once, so each must be touched zero of one
times. Thus we can represent the scheme as an array of Osand 1s, where Omeans that the corresponding
bulb is not touched, and l means that it is touched.

• Let A, B, C, D be four values at the comers of a rectangle. We claim that A+B has the same parity as
D

••
C+D. Let LABbe the number of 1s in the row AB are touched, similarly LBe(the number of Is in the
column BC), Leo, LDA.Since bulb A is switched we must have LAB+ LOA+ A odd (note that LAB + LOA
double-counts the no. oftouches of A). Similarly, LBe+ Leo + C is odd, so A + C + (LAB+ LBe+ Leo + PAB and PED are similar, so DE/AB = PD/PB. Similarly, DF/AC = PDIPC, so DE/DF =
(AB/AC)(PC/PB). Thus we need PBIPC = AB/AC. So P must lie on the circle of Apollonius, which is

..
LOA)is even. Similarly, considering B and D, we find that B + D + (LAB+ LBe+ Leo + LOA)is even, so
A+C and B+D have the same parity. Adding B+C to both, we get that A+B and C+D have the same the circle we constructed with center X. Similarly, it must lie on the circle of Apollonius with center Y
parity. lt follows that either A = D and B = e, or A t- D and B t- D. and hence be one of their points of intersection. It also lies on the third circle and hence we choose the
point of intersection inside the triangle.
Problem B2


Keeping A and B fixed, we can now vary C (and hence D). It follows that either the row through B
matches that through A, or it has every cell different (to the corresponding cell in row A). Similarly for
the other rows, So we have k rows of one type and n-k rows which are equal to its "complement". n and r are positive integers. Find the smallest k for which we can construct r subsets A" A" ... , A, of
Suppose first that k = n, so that all rows are the same. Ifwe have a1lls, then we have a solution. Ifwe (O, 1, 2, ... , n-I} each with k elements such that each integer O:s m < n can be written as a sum of one

.. have all Os,we obviously do not have a solution. So suppose there is a Oand a I in each row, Then the element from each of the r subsets.
total count at al is n-I higher than at a O(because ofthe extra n-lis in the same column). So they
cannot both be odd (because n is even). Contradiction. Answer

• Finally suppose there is a row and a complement row, So position A in one is 1, then position B in the
same column in the other has O.If a row has h 1s, then a complement row has n-h I S. The column has z
Is, so A has z+h-I or z+n-h-I 1s, and B has z+h or z+n-h I S. But since n is cvcn, z+h and z+n-h have the
smallest integer such that k' ~ n.

• same parity, so A and B have opposite parity. Contradiction. So the only solution for neven is all Is.


,a
Problem Bl

ABC is an acute-angled triangle. P is a point inside its circumcircle. The rays AP, BP, CP intersect the
circle again at D, E, F. Find P so that DEF is equilateral.
Solution

We can form at most k' distinct sums, so k' must be ~ n.


Now consider A, = {O,1,2, ... , k-I}, A, = {O,k, 2k, ... , (k-I)k}, A, = {O,k', 2k', ... , (k-I)k'}, ... , A, =
Solution {O,k,-I,2k'-I, ... , (k-I )k'-I}.Then for any non-negative integer m < k', we can write m with r digits in
base k (using leading zeros as necessary) and hence as a sum of one element from each A;. This subset

•• Let the angle bisector of A meet BC at A'. Let the perpendicular bisector of AA' meet the line BC at X.
Take the circle center X through A and A'. Similarly, let the angle bisector ofB meet AC at B' and let
the perpendicular bisector ofBB' meet the line AC at Y. Take the circle center Y through B and B'. The
two circles meet at a point P inside the triangle, which is the desired point.
works for (k-I )k,-I< n :s k'. For smaller n aboye (k-l )' we cannot use all the elements given aboye, but
we do not need them, so we just replace the elements which are too large by arbitrary elements under n.

For example, supposen= 17, r=4. Weneedk=3. Sowe form A, = {O,1,2},A, = {O,3, 6}, A, = {O,
9, 18}, A. = {O,27, 54}. Now 18,27,54 are unnecessary, so we pad out Al and A. with other elements.

•• We could take A, = {O,1, 9}, A. = {O,1, 2}.



B3. Show that given any integer o < n :'S21000000 we can find at set S of at most 1100000 positive A3. L and L' are two perpendicular lines not in the sarne plane. AA' is perpendicular to both lines,
integers such that S ineludes 1 and n and every element ofS except 1 is a sum oftwo (possibly equal) where A belongs to L and A' belongs to L'. S is the sphere with diarneter AA'. For which points P 00 S
smaller elements of S. can we find points X on L and X' on L' such that XX' touches S at P?

OIM 1995 Problem BI


Problem Al
ABCO is an n x n board. We call a diagonal row of cells a positive diagonal if it is parallel to AC. How
Find all possible values for the sum ofthe digits of a square. many coins must be placed on an n x n board such that every cell either has a coin or is in the same row,
column or positive diagonal as a coin?

Solutioo
Aoswer

Answer: any non-negative integer = O, 1,4 or 7 mod 9.


smallest integer 2: (2n-l )/3
[so 2m-1 for n = 3m-l, 2m for n = 3m, 2m+1 for n = 3m+l]
o' = O, (±l)' = 1, (±2)' = 4, (±3)' = O, (±4f = 7 mod 9, so the condition is necessary.

We exhibit squares which give these values.

Solution
Omod 9. Obviously O' = O. We have 9' = 81, 99' = 9801 and in general 9 ...9' = (IO" - 1)' = lO'n - 2.IOn +
1 = 9 ...980 ...01, with digit sum 9n.
There must be at least n-k rows without a coin and at least n-k columns without a coin. Let r., r" ... , rn-k
be cells in the top row without a coin which are also in a column without a coin. Let r., c" Cl, ... , Cn-kbe
1 mod 9. Obviously l' = 1 with digit sum 1, and 8' = 64 with digit sum 10. We also have 98' = 9604, cells in the first column without a coin which are also in a row without a coin. Each ofthe 2n-2k-1 r, and
998' = 996004, and in general 9 ...98' = (IOn - 2)2 = lO'n - 4.IOn + 4 = 9 ...960 ...04, with digit sum 9n+1.
e, are on a different positive diagonal, so we must have k 2: 2n-2k-1 and hence k 2: (2n-I)/3.

4 mod 9 Obviously 2' = 4 with digit sum 4, and 7' = 49 with digit sum 13. Also 97' = 9409 with digit
Let (ij) denote the cell in row i, col j. For n = 3m-l, put coins in (m,l), (m-I ,2), (m-2,3), ... , (1.m) and
sum 22, 997' = 994009 with digit sum 31, and in general 9 ...97' = (Ion - 3)' = io= -
6.1 O"+ 9 =
in (2m-l,m+I), (2m-2,m+2), ... , (m+ l ,2m-l). It is easy to check that this works. For n = 3m, put an
9 ...940 ...09, with digit sum 9n+4.
additional coin in (2m,2m), it is easy to check that works. For n = 3m+ l we can use the sarne
arrangement as for 3m+2.
7 mod 9 Obviously 4' = 16, with digit sum 7. Also 95' = 9025, digit sum 16,995' = 990025 with digit
sum 25, and in general 9 ...95' = (Ion - 5)' = lO'n - IOn+l+ 25 = 9 ...90 ...025, with digit sum 9n-2.

Problem A2
'"
'"
'"
Find all solutions in real numbers xi, X" ... , Xn+lall at least 1 such that: (1)
Xn"(n+l)= n Xn+l"'; and (2) (x¡ + X, + ... + xn)/n = Xn+l.
XIII' + X,l" + X/4 + ... +
'"
'"
Aoswer '"
The only solution is the obvious, all x, = l.

Solutioo
B2. The incirele ofthe triangle ABC touches the sides BC, CA, AB at O, E, F respectively. AO meets
By Cauchy-Schwartz, (1Xi"')':'S (I I)(&sum x.), with equality iffall x, equal. In other words, ifwe put the cirele again at X and AX = XO. BX meets the cirele again at Y and CX meets the circle again at Z.
XntI = (x¡ + X, + ... + xn)/n, then I x/12:'S n Xn+11/2 But since all x, 2: 1, we have XII12 + X,"l + X/14 + ... + Show that EY = FZ.
xnl/(ntl):'S I Xil12 with equality iff x, = x, = ...= x, = 1. Hence XI"' + x/" + Xl"4 + ... + xnl/(n+I):'S Xn+I"' Problem B3
with equality iffall x, = 1.
f is a function defined on the positive integers with positive integer values. Use fm(n) to mean f(f( ... f(n)
....» = n where fis taken m times, so that f'(n) = f(f(n», for example. Find the largest possible O < k < l
such that for sorne function f, we have fm(n) i- n for m = 1,2, ... , [kn], but fm(n) = n for sorne m (which
may depend on n).
•• Answer
A
we can get k arbitrarily close to 1

Solution

• The basic idea is to take a block of integers m+ 1, m+2, ... , M and to define f(m+ 1) = m+2, f(m+2) =
m+3, ... , f(M-l) = M, f(M) = m+ l. Then for any integer h in the block we have f''(h) f. h for n = 1, 2, ... ,

• M-m-I and t'-m(h) = h. Note that the ratio (M-m)/h is worst (smallest) for h = M.

• For example, take the first block to be 1,2, ... ,N, the second block to be N+l, ... ,N', the third block,
N'+ 1, ... ,N3 and so on. Then for any integer n we have f"(n) f. n for m < kn where k = l - IIN.

OIM 1996
Problem Al

Find the smallest positive integer n so that a cube with side n can be divided into 1996 cubes each with

• side a positive integer.


Applying Menelaus to the triangle ADC, we have (AM/MD)(BD/DC)(CNINA) = 1, so (CNINA) = 2.

• Solution

Answer: 13.
Hence AN/AC = 1/3. Applying Menelaus to the triangle BNC, we have (BMIMN)(AN/AC)(CD/DB) =
1, so BMIMN = 3. That is true irrespective ofwhether AB is tangent to the circle NBe.

If AB is tangent, then AB' = AN.AC = 1/3 AC'. AIso angle ABN = angle BCN, so triangles ANB and
ABC are similar. Hence BC/BN = AC/AB. Hence (BC/BN)' = 3 = BM/BN.
Divide all the cubes into unit cubes. Then the 1996 cubes must each contain at least one unit cube, so the
large cube contains at least 1996 unit cubes. But 12' = 1728 < 1996 < 2197 = 133, so it is certainly not
possible for n < 13. Conversely, if(BC/BN)' = BMIBN, then (BC/BN)' = 3.

lt can be achieved with 13 by i.s' + 11.2' + 1984.1' = 13' (actually packing the cubes together to form a Now applying the cosine formula to AMN and AMB and using cos AMN + cos AMB = 0, we have
13 x 13 x 13 cube is trivial since there are so many unit cubes). (3AN' - 3AM' - 3MN') + (AB' - AM' - BM') = O, so AB' + AC'/3 = AD' + 3/4 BN2 Similarly from
triangles ADC and ADB we get AB' + AC' = 2AD' + BC'/2. So using BN' = BC'/3 we get 2AB' + 2/3
AC' = AB' + AC' and hence (AC/AB)' = 3 = (BCIBN)2 So AC/AB = BC/BN. Note that is not enough
Problem 2 to conclude that triangles ABC and BNC are similar, because the common angle C is not between AC
and AB. However, we have AN/AB = (1/3) AC/AB = AB/AC, so AB' = AN.AC, so AB is tangent to the
M is the midpoint ofthe median AD ofthe triangle ABe. The ray BM meets AC at N. Show that AB is circle NBC.
tangent to the circumcircle ofNBC iffBM/BN = (BC/BN)'.
Problem A3

'•.
Solution
n = k' - k + 1, where k is a prime plus one. Show that we can color sorne squares of an n x n board black
so that each row and column has exactly k black squares, but there is no rectangle with sides parallel to
the sides of the board which has its four comer squares black.

Solution

• We can regard the rows as lines and the columns as points. Black squares denote incidence. So line 3

••
contains point 4 iff square (3, 4) is black. The condition about rectangles then means that there is at most
one line through two distinct points.

•:. J(
J
Suppose we take the points to be (a, b, e), where a, b, e are residues mod p, not all zero, and the
coordinates are homogeneous, so that we regard (a, b, e), (2a, 2b, 2c), ... , ( (p-I)a, (p-I lb, (p-l )c ) as the
Problem B2 .J
same point. That gives (p3 - 1)/(p-I) = p2+ P + 1 points, which is the correct number.
An equilateral triangle of side n is divided into n2equilateral triangles of side l by lines parallel to the
sides. Initially, all the sides of all the small triangles are painted blue. Three coins A, B, C are placed at .J
We can take lines to be lx + my + nz = O, where the point is (x, y, z). In other words, the lines are also vertices ofthe small triangles. Each coin in tum is moved a distance 1 along a blue side to an adjacent
triples (l, m, n), with 1,m, n residues mod p, not all zero and (l, m, n), (21,2m, 2n), ... , ( (P-l)l, (p-l)m, vertex. The side it moves along is painted red, so once a coin has moved along a side, the side cannot be
(p-l)n) representing the same line. used again. More than one coin is allowed to occupy the same vertex. The coins are moved repeatedly in
the order A, B, C, A, B, C, .... Show that it is possible to paint all the sides red in this way.
One way ofwriting the points is p2ofthe form (a, b, 1), P ofthe form (a, 1, O) and lastly (1, O, O).
Similarly for the lines. We must show that (1) each point is on p+llines (so each column has p+1 black Solution
squares), (2) each line has p+1 points (so each row has p+1 black squares, (3) two lines meet injust one
point (so no rectangles).
8

M
(1): Consider the point P (a, b, 1) with a non-zero. Then for any m, there is a unique 1such that la + mb +
8
1.1 = O, so there are p lines ofthe form (l, m, 1) which contain P. Similarly, there is a unique 1such that
la + 1b + 0.1 = O, so one line ofthe form (1, 1, O) contains P. The line (1, O, O) does not contain P. So P
lies on just p+ 1 lines. Similarly for (a, b, 1) with b non-zero. The point (O, O, 1) does not lie on any lines
(l, m" 1), but lies on (l, 1, O) and (1, O, O), so again it lies on p+1 lines.
L.
A e A e

Consider the point Q (a, 1, O) with a non-zero. For any m, there is a unique 1such that Q lies on (l, m, O).
There is also a unique 1such that Q lies on (1,1, O). Q does not lie on (1, O, O), so it lies onjust p+1 lines. We use induction. It is obvious for n = 1 and 2 - see diagram aboye. Note that A, B, C start and end at
Similarly, the point (O, 1, O) lies on the p lines (l, O, O) and on (1, O, O), but no others. vertices ofthe large triangle.

Finally, the point (1, O, O) lies on the p lines (O, m, 1), the line (0,1, O) and no others. Thus in all cases a B
point lies on just p+ 1 lines. The proof of (2) is identical.
"
(3). Suppose the lines are (l, m, n) and (L, M, N). If 1 and L are non-zero, then we can take the lines as (1,
m', n) and (1, M', N). So any point (x, y, z) on both satisfies x + m'y + n'z = O (*) and x + M'y + N'z = O.
Subtracting, (m' - M)y + (n' - N)z = O. The coefficients cannot both be zero, since the lines are distinct.
So the ratio y : z is fixed. Then (*) gives the ratio x : y. So the point is uniquely determined. If just one
of 1,L is non-zero, then we can take the lines as (O, m', n), (1, M', N). We cannot have both m' and n'
zero, so the ratio y : z is determined, then the other line determines the ratio x : y. So again the point is
uniquely determined. Finally, suppose 1 and L are both zero. Then since the lines are distinct y and z
must both be zero. So the unique point on both lines is (1, O, O). ,
I!

Comment. This is a poor question. Jfyou have not metfinite projective geometries before, then you are
in real difficulties. Jf you have, the proof isjust tiresome verification. A e

Problem Bl
Now assume that for n we can find a solution with A, B, C starting and ending at the vertices of the large
triangle. Take n+ l. We start with the paths shown which bring A, B, C to A', B', C' at the vertices of a
n > 2 is an integer. Consider the pairs (a, b) ofrelatively prime positive integers, such that a < b::; n and triangle side n-l. Now by induction we can continue the paths so that we bring A, B, C, back to the
a + b > n. Show that the sum of I/ab taken over all such pairs is 1/2. vertices ofthat triangle after tracing out all its edges. Finally, note that for each ofthe points A', B', C'
there is a path length 2 over untraced segments to a vertex of the large triangle. So we get a solution for
Solution n+ 1 and hence for all n.

Induction on n. lt is obvious for n = 3, because the only pairs are (1, 3) and (2, 3), and 1/3 + 1/6 = 1/2. Problem 83
Now suppose it is true for n. As we move to n+ 1, we introduce the new pairs (a, n+ 1) with a relatively
prime to n+ 1 and we lose the pairs (a, n+ I-a) with a relatively prime to n+ l-a and hence to n+ l. So for Al, A2, ... , A, are points in the plane. A non-zero real number k¡is assigned to each point, so that the
each a relatively prime to n+ 1 and < (n+ 1)/2 we gain (a, n+ 1) and (n+I-a, n+ 1) and lose (a, n+ I-a). But square ofthe distance between A¡ and A, (for i =f. j) is k¡ + k¡ Show that n is at most 4 and that ifn = 4,
I/a(n+ 1) + 1/( (n+ I-a)(n+ 1) ) = ( n+ I-a + a)/( a(n+ I-a)(n+ 1) ) = 1/( a(n+I-a) ). then I/kl + l/k2 + l/k, + l/k. = O. .J
.J
J

• Solution
A

• Suppose we have four points A, B, C, D with assoeiated numbers a, b, e, d. Then AB2 = a + b, AC2 = a +
e, so AB2 - AC2 = b - e. Similarly, DB2 - DC2 = b - e, so AB2 - AC2 = DB2 - DC2. Let X be the foot ofthe

• perpendicular from A to BC, and y the foot ofthe perpendicular from D to Be. Then AB2 - AC2 = (AX2
+ XB2) _ (AX2 + XC2) = XB2 _ XC2 Similarly for D, so XB2 - XC2 = YB2 - YC2• Henee X = Y, so AD is

••
perpendicular to Be. Similarly, BD is perpendicular to AC, and CD is perpendicular to AB. Henee D is
the (unique) orthoeenter of ABC. So n <= 4.

Suppose n = 4, so we have four points A, B, C, D with assoeiated numbers a, b, e, d. We have AB2 +


AC2 - BC2 = (a + b) + (a + e) - (b + e) = 2a. But by the eosine formula it is also 2 AB AC eos BAe.

•• Henee a = AB AC eos BAC. Similarly for A, B, D etc. Henee ab/ed = (AB AC eos BAC)(BA BD eos
ABD)/( (CA.CD eos ACD)(DB OC eos BDC) ) = (AB2/CD2) (eos BAC/eos BOC) (eos ABD/cos ACD).

Take ABC to be aeute with D inside. Then angle ABD = angle ACD ( = 90° - angle BAC), and angle
BOC = 90° + angle ACD = 180°- angle BAe. So eos BAC/eos BDC = -1. Thus ab/ed = - AB2/CD2 =-
B e

• (a + b)/(e + d). Henee ab(e + d) + ed(a + b) = O,so I/a + I/b + I/e + lId = o.
D and P are the refleetions of Q and E respeetively in the line CI. Henee PQ and DE meet at a point on
CI. So U Iies on CI. So IJPIU = 1/2 I IPIE = i IPDE (1is eenter of eirc1ethrough D, P, E) = I IPDU (same

•• OIM 1997

Problem Al
angle). Henee PDIU is eyelie. In other words, 1Iies on the eireumeirc1eof DUP. Similarly, it Iies on the
eireumeireles of ESQ and FTR.

But the same argument shows that DPT = "DIT, so DPIT is eyelie. So T Iies on the eircle through D,


k >= I is a real number sueh that if m is a multiple of n, then [mk] is a multiple of[ nk]. Show that k is an P and I and henee on the eireumeircle of DUP. Similarly, for the other eircles. So the eireumeireles of
integer. CUP and FTR meet at T and 1. Similarly, the eireumeircles of FTR and ESQ meet at S and 1,and the
eireumeircles of ESQ and DUP meet al U and 1. So the three eireumeircles have just one point in

• Solution cornmon, namely 1.

•• Suppose k is not an integer. Take an integer n sueh that nk > 1, but nk is not an integer. Now take a
positive integer e sueh that I/(e+l) <= nk - [nk] < l/c. Then I <= (e+l)nk - (e+l) [nk] < I + I/e. Henee
[(e+l)nk] = (e+ 1) [nk] + 1. Put m = (e+1)n. Then m is a multiple ofn. But if[mk] is a multiple of[nk],
then [mk] - (e+l) [nk] = I is a multiple of[nk], whieh is impossible sinee nk > 1. So we have a
eontradietion. So k must be an integer.
Problem A3

• Problem 2 n > l is an intcgcr. D, is the set oflattiee points (x, y) with [x], Iyl<= n. Ifthe points of D, are colored
with three colors (one for each point), show that there are always two points with the same color such
that the line containing them does not contain any other points of D; Show that it is possible to color the
1is the ineenter ofthe triangle ABC. A eirele with eenter I meets the side BC at D and P, with D nearer points of D, with four colors (one for each point) so that if any line contains just two points of D, then
l. to B. Similarly, it meets the side CA at E and Q, with E nearer to C, and it meets AB at F and R, with F those two points have different colors.

••
nearer to A. The lines EF and QR meet at S, the Iines FD and RP meet at T, and the Iines DE and PQ
meet at U. Show that the eireumeirc1esofDUP, ESQ and FTR have a single point in eommon.
Solution

Solution

••
••
• J?_
intersection ofBC and HN, R is the intersection ofBC and OM, and S is the intersection ofHR and OP.
(-(n-1), n) Show that AHSO is a parallelogram.

Solution

(-n,-n)

Consider the 4 points shown in the diagram. In each case the segment joining them is the diagonal of an e
m x 1 parallelogram or rectangle, so it cannot contain any other lattice points. The next points along each
line are obviously outside set D; That proves the first parto
We show first that o is the circumcenter of ABC. :JABF = 90° - A. The line BC is the reflection in BD
The second part is the standard parity argument. Color (x, y) with color 1 ifx and y are both even, 2 ifx ofthe line BA and the line BF' is the refection ofBF, so angle CBF' = 90° - A. But ifO' is the
is even and y is odd, 3 ifx is odd and y is even, and 4 ifx and y are both odd. Then iftwo points are the circumcenter, then 1I30'C = 2 :JBAC = 2 A, so ! 10'BC = 90° - A. Hence O' lies on BF'. Similarly, it
lies on AE' (the reflection of AE in the angle bisector of A). Hence O = O'.
same color, that means the first coordinates are the same parity and their second coordinates are the
same parity. Hence the midpoint of the segment joining them is also a lattice point and they are not the
only two points of D, on the line.

Problem Bl

Let o(n) be the number of2n-tuples (a" a" ... , an,b., b" ... , bs) such that each a, b, = O or 1 and a.b. +
a,b, + ... + anb"is odd. Similarly, let e(n) be the number for which the sum is even. Show that o(n)/e(n) =
(2" - 1)/(2n+ 1).

Solution

We prove by induction that o(n) = 2,n,1- 2n-l.For n = 1, this reads 0(1) = 21- 2° = 1, which is obviously
true - the only such 2-tuple is (1, 1). Suppose it is true for n.

If (al, a" ... , a., b., b" ... , b") gives an odd sum, then we can take (an+"b"+I)to be any of (O, O), (O, 1),
(1, O) and still get an odd sum for (al, a" ... , a"+"b., b" ... , bn+I).On the other hand if'(a¿ a" ... , a., bl,
b, ... , b-) is even, then we must have an+1= b"+1= 1 to get an odd sumoThus o(n+ 1) = 3 o(n) + e(n). But IMBC = [lMAC = 90° - C (since AH is an altitude) = 11FBC(since BF is an altitude) = .JHBC (same
o(n) = 2'"_1- 2"_1and e(n) = (o(n) + e(n)) - o(n) = 2'" - 2,n-1+ 2n-1= 2'"-1+ 2"-1 So 0(n+1) = 4.2'"-1- 2.2"-1 angle). So triangles HBE and MBE are congruent and HE = EM. [Note: this should be a familiar result.].
= 2'("+ll-1 _2(n+I)-I, which establishes the result for n+ 1 and hence for all n.
AN is a diameter, so angle AMN = 90° = angle AEC, so BC and MN are parallel. Hence P is the
Hence e(n) = 2'" - o(n) = 2'"-1+ 2"-1and o(n)/e(n) = (2" - 1)/(2n+ 1). midpoint ofHN and ofBC. So OP is perpendicular to Be. So AH and OS are parallel.

Problem B2 Since R lies on BC, triangles HER and MER are congruent, so r :EHR = iEMR = [¡AMO (same angle)
= ! [MAO. Hence HS and AO are parallel. So AHSO is a parallelogram.

ABC is an acute-angled triangle with orthocenter H. AE and BF are altitudes. AE is reflected in the
angle bisector of angle A and BF is reflected in the angle bisector of angle B. The two reflections Problem B3
intersect at O. The rays AE and AO meet the circumcircle of ABC at M and N respectively. P is the

•• Given 1997 points inside a circle ofradius 1, one ofthem the center ofthe circle. For each point take the
distance to the closest (distinct) point. Show that the sum ofthe squares ofthe resulting distances is at
most9.
rlAQM = , :EQD (opposite angle)
[1B/2 (*).
= IJEDC (CD tangent to circle EQD) = (1800 - [ JC)/2 = UN2 +


MF2 = MQ.ME (MF tangent to circle FQE). So AM = AF is equivalent to AM2 = MQ.ME or AMIMQ =
Solution ME/ AM. But since triangles AMQ and EMA have a common angle M, AMIMQ = ME/ AM iff they are
similar, and hence iff ilAQM = [ .A. Using (*) AM = AF iff lA = [ .B.

• Let the points be Pi for i = 1,2, ... , 1997. Take PI to be the center ofthe given unit circle. Let x, be the
distance from Pi to the closest ofthe other 1996 points. Let C, be the circle center Pi radius Xi/2. Then C, Problem AJ


and C, cannot overlap by more than one point because x, and Xj::::PiPj. Also Xi::::1, since PIPi:::: l. Thus
C¡ is entirely contained in the circle center PI radius 3/2. Since the circles C, do not overlap, their total
Find the smallest number n such that given any n distinct numbers from {I, 2, 3, ... , 999}, one can
area cannot exceed the area ofthe circle radius 3/2. Hence (X12+ xl + ...+ XI99l)/4:::: 9/4.


choose four different numbers a, b, e, d such that a + 2b + 3c = d.

OIM 1998
Solution

..•
Problem Al

Answer: n = 835.
There are 98 points on a circle. Two players play altemately as follows. Each player joins two points
which are not already joined. The game ends when every point has been joined to at least one other. The
Consider the set S = {I66, 167, ... , 999}. The smallest possible value for a + 2b + 3c, for distinct a, b, e

••
winner is the last player to play. Does the first or second p1ayer have a winning strategy?
in S is 168 + 2.167 + 3.166 = 1000. So we cannot find distinct a, b, e, d in S with a + 2b + 3c = d. So the
smallest n > 834.
Solution

Answer: the first player has a winning strategy. Now suppose S is any subset of 835 elements which satisfies the condition. Take it elements to be m =


al < a2 < ...< 3s35 = M. Obviously M 2: m + 834 2: 835, so -3m 2: 3.834 - 3M and hence M - 3m 2: 2502 -
2M 2: 2502 - 2.999 = 504. Put k = M - 3m.
Assume there are n points. The first to play so that n-2 points each have at least one segment loses,


because the other player simply joins the last two points and the game ends. But there are N = (n-3)(n-
There are at least 167 disjoint pairs (a, b) ofnumbers taken from {I, 2, ... , 999} with a + 2b = k, namely
4)/2 possible plays amongst the first n-3 points to get a segment. For n = 1 or 2 mod 4, N is odd and for
n = O or 3 it is even. So the first player wins for n = 1 or 2 mod 4 (and in particular for n = 98) and the

• second player for n = O or 3 mod 4.


(k - 2, 1)

•• Problem A2

The incircle ofthe triangle ABC touches BC, CA, AB at D, E, F respectively. AD meets the circle again
at Q. Show that the line EQ passes through the midpoint of AF iff AC = Be.
(k - 4,2)

(k - 6, 3)

• Solution
(k-334, 167) - note that in the extreme case k = 504 this is (170, 167)

• A
At least one number from each pair must either (1) be M or m or (2) not belong to S - or otherwise we
would have a + 2b + 3m = M for distinct elements a, b, m and M in S. None ofthe numbers can be M

• M
and at most one ofthem can be m, so we have at least 166 numbers which are not in S. That mean s S
contains at most 999 - 166 = 833 numbers. Contradiction. So S cannot have 835 elements. Nor can it


have more than 835 elements (or we just take a subset of 835 elements, which must also satisfy the
condition, and get a contradiction).
Problem Bl
,,

• ,
,
,
Representatives from n > I different countries sit around atable. If two peop1e are from the same

• ,,-./
I
./
country then their respective right hand neighbors are from different countries. Find the maximum
number of people who can sit at the table for each n.

•• B D e Solution
Put p(x) = x2 - 1998x - 1. Then p(l998) = -1, p(I999) = 1998, so 1998 < k < 1999. Also k is irrational
(using the formula for the root of a quadratic). We have x, = [k Xn-l],so x, < k Xn.land > k Xn.l- l. Hence
xn/k < Xn-l< x,/k + l/k, so [xnIk] = Xn.l- l.
Obviously there cannot be more than n2 people, For ifthere were, then at least one country would have
more than n representatives. But there are only n different countries to choose their right-hand
neighbours from. Contradiction. k = (1998k + I)/k = 1998 + l/k. Hence kx, = 1998xn + x/k. Hence Xn+l= [kx.] = 1998xn + [xn/k] =
I998xn + Xn-l- l. Hence Xn+l= Xn-l- I mod 1998. So Xl99S= 1 - 999 = 1000 mod 1998.
Represent someone from country i by i. Then for n = 2, the arrangement 1122 works. [It wraps round, so
that the second 2 is adjacent to the first l.] Suppose we have an arrangement for n. Then each of 11, 22, OIM 1999
... , nn must occur just once in the arrangement. Replace 11 by I (n+ 1)11, 22 by 2( n+ 1)22, ... , and (n-
I)(n-I) by (n-I )(n+ I)(n-I )(n-I). Finally replace nn by n(n+ 1)(n+ I)nn. lt is easy to check that we now
Problem Al

have an arrangement for n+ l. We have added one additional representative for each of the countries 1 to Find all positive integers n < 1000 such that the cube ofthe sum ofthe digits ofn equals n2.
n and n+1 representatives for country n+l, so we have indeed got (n+1)2 people in all. We have also
given a representative of each country I to n a neighbour from country n+ 1 on his right and we have
given the (n+l) representatives from country n+1 neighbours (on their right) from each ofthe other Solution
..
countries. Otherwise we have left the seating unchanged.
n < 1000, so the sum ofthe digits is at most 27, so n2 is a cube not exceeding 27'. So we are looking for
Problem 82 m' which is also a square. That implies m is a square. So the only possibilities are m = 1, 4, 9, 16, 25.
Giving n = 1, 8,27,64,125. The corresponding cubes ofthe digit sums are 1,512,729,1000,512,
whereas the corresponding squares are 1,64,729,4096, 15625. Thus the only solutions are n = 1,27.
PI, P2, ... , P, are points in the plane and ri, r2, ... , rn are real numbers such that the distance between P;
and P, is r, + rJ(for i not equal to j). Find the largest n for which this is possible.
ProblemA2
Solution
Given two circles C and e we say that C bisects C' iftheir common chord is a diameter of C', Show that
for any two circles which are not concentric, there are infinitely many circles which bisect them both.
Answer: n = 4. Find the locus ofthe centers ofthe bisecting circles.

Draw a circle radius r; at P;. Then each pair of circles must touch. But that is possible iff n :s 4. Solution

»<:"">....... Let C, C' have center O, O' respectively and radius r, r' respectively. Let a circle center P bisect C.
/. "- Suppose it meets C at A and B. Then AB is perpendicular to OP and is a diameter ofC. Hence PA2 =
;" -, r.
op2 + Converse1y, the circ1e center P, radius v'(OP2 + r') bisects C. So P will bisect C and C' iffOP2 +
r
:,! " = Op,2+ r'2

\
• j .......
1,,"
-----.""-. <,
......
\" ;.1,,, <,
It is well-known that the locus ofpoints P' with equal tangents to C and C' is the radical axis. Call the

.'
I
,/_ ...
"...::.......__.-:'"';}i~{
", "-~"'.
"J
\

'...."
,
\

j I
r
radical axis R. For a point P' on the radical axis we have p'02 - = p'0,2 - r". Ifwe reflect P' in the
r
perpendicular bisector of 00' to get P, then PO = P'O' and PO' = P'O, so PO,2 - = P02 -~2and hence
P02 + r', Call the reflection ofthe R in the perpendicular bisector ofOO' the line R'. We have
established that points on R' form par! ofthe locus. Conversely, ifP' is such that there is a circ1e center P'
[,.. • X
.. ,
,<
,/
.' r r
bisecting both circ1es, then OP02 + = 0'p,2 + r'2, so ifP is the reflection ofP' then Op2 - = Op,2 - r,2
\.... .,....
,/ \. / /'
and hence P lies on the radical axis R. Hence P' must lie on R'.
<,__ ...r,.;,/ ....

--_--
,.......... "."./

Radical axis

Problem 83

k is the positive root ofthe equation x2 - 1998x - 1 = O. Define the sequence Xo,x., X2,.. by Xo= 1, Xn+l=
[k xn]. Find the remainder when XI998is divided by 1998.

Solution
•• There are n-I circles with diameter P¡P¡+I.Obviously, each pair has a common tangent. If n-I > k, then
two ofthem must have the same color.

•• Ifn-I ::s k, then color all circles with diameter p¡Pj and i <j with color i. Then iftwo circles have the
same color, then both have a tangent at one ofthe points. Hence one lies inside the other and they do not
have a common external tangent.


Problem BI

••
Show that any integer greater than 10 whose digits are all members of {I, 3, 7, 9) has a prime factor >
11.

•• r r
We have PT2= P02 - = PX2+ OX2- ,and similarly PT'2= PX2+ 0'X2 - r'2. So PT = PT' iffOX2 -
= 0,x2 - r'2.There is evidently a unique point X for which that is true, so the locus of such P is the line
through X perpendicular to 00'
r
Solution

Such a number cannot be divisible by 2 (or its last digit would be even) or by 5 (or its last digit would be
Oor 5). So ifthe result is false then the number must be ofthe form 3m7n for non-negative integers m, n.

••
But we c1aimthat a number ofthis form must have even lOs digit.

It is easy to prove the c1aimby induction. Jt is true for 3 and 7 (the digit is Oin both cases). But ifwe
multiply such a number by 3 or 7, then the new lOs digit has the same parity as the carry from the units
digit. But multiplying 1,3,7,9 by 3 gives a carry ofO, O,2, 6 respectively, which is always even, and

• multiplying by 7 gives a carry ofO, 2, 4, 6, which is also always even. So the new number also has an
even JOsdigit.

• Problem B2

•• o is the circumcenter ofthe acute-angled triangle ABC. The altitudes are AD, BE and CF. The line EF
cuts the circumcircle al P and Q. Show that OA is perpendicular to PQ. IfM is the midpoint ofBC, show
that Ap2 = 2 AD·OM.

• Solution

• If the circles intersect, then the point X evidently lies on the linejoining the two common points,

•• r
because OX2- = _Xy2 = 0,x2 - r,2.In any case the midpoint of each common tangent evidently lies on
the line, so that provides a way of constructing it.

Problem AJ

• Given points PI, P2,... , P, on a line we construct a circle on diameter p¡Pj for each pair i, j and we color
the circle with one ofk colors. For each k, find all n for which we can always find two circles ofthe
same color with a common extemal tangent.

Solution

Answer: n > k+ l.

r'-l
Conversely, suppose XI= 180 tls, with r and s relatively prime and s not a power of2. Then Xn+1 = 180
2nrls cannot be O mod 180, so the sequence does not terminate. Put s = 2bc, with e odd. Let d = cp(c),
where cp(m)is Euler's phi function, so that 2d = I mod c. Then Xb+1
= 180 r/c mod 180 and 2b+d= 2b mod
= 180 r/c mod 180. Hence the sequence is periodic.
e, so Xb+d+1

OIM 2000
Problem Al

Label the vertices of a regular n-gon from I to n > 3. Draw all the diagonals. Show that if n is odd then
p we can label each side and diagonal with a number from I to n different from the labels of its endpoints
so that at each vertex the sides and diagonals all have different labels.

\ -.,
'.<,
\ "
~,
..,......
e Solution

Labeling the diagonal/side between i andj as i+j (reduced ifnecessary mod n) almost works. The labels
,.-.,"-.,-._ ••••••••• J •••• _••• , •• ;
for all the lines at a given vertex will be different. But the line between i and n will have labe! i, the same
as one endpoint. However, we are not using the label 2i for the lines from vertex i. So for the line
between i and n we use 2i instead of i+n. The only points that need checking are (1) whether a line from
i to n has a label different from n, and (2) whether all the Iines at n have different labels. Both points are
Let OA and PQ meet at T. I !AEH = I IAFH = 90°, so AEHF is cyclic, so lJAFT = [ :AFE (same angle) = ok because n is odd.
"AHE = 90°_ UHAE = 90° - IIDAC (same angle) = oc. But LITAF= ! 10AF (same anglc) = 90° - (1/2) Problem A2
':AOB = 90° - 1 .C, Hence ,AFT = 90°, which establishes that OA and PQ are perpendicular.
Two circles C and e have centers o and O' and meet at M and N. The common tangent closer to M
Let the circumradius be R and let AA' be a diameter. We have AF = AC cos A = 2R sin B cos A. Hence touches C at A and e at B. The line through B perpendicular to AM meets the line 00' at D. BO'B' is a
AT = AF cos OAB = AF sin C = 2R cos A sin B sin C. Now PT2 = PT-TQ = AT.A'T = AT(2R - AT). diameter of C', Show that M, D and B' are collinear.
Hence Ap2 = 2R·AT = 4R2 cos A sin B sin C.
Solution
We have AD = AC sin C = 2R sin B sin C, and OM = OC cos COM = R cos A. Hence 2 AD·OM = AP2.

,,
-',.,.,.,.---- ...._._._ v-,

Problem 83 / .....
//
,, ,,
Given two points A and B, take C on the perpendicular bisector of AS. Define the sequence CI, C2, CJ, : \
... as follows. CI = C. If C, is not on AB, then Cn+1is the circumcenter ofthe triangle ABCn. If C, Iies on /~-~---..I
,I
,,
..
AB, then Cn+1is not defined and the sequence terminates. Find all points C such that the sequence is / ~ \
periodic from sorne point on.
/ °
I
~
(-m ,(~2 \ 1)12 .:
\
'\ ;
I
(n, (r? + 1}~12)
l
I

Solution ~\, 'X', //


;
"" /M~ 1) ,/
Answer: any C such that I IACB = 180° tts, with r and s re!atively prime integers and s not a power of2.
............ .~,. ...... .,....... ,.,."'"

A(-m ,O) B (n, O)


Let I ACnB = x., where the angle is measured clockwise, so that x, is positive on one side of AB and
negative on the other side. Then x, uniquely identifies C, on the perpendicular bisector.
A neat coordinate solution by Massaki Yamamoto (a competitor) is asfollows.

We have Xn+1 = 2xo. To make this work in all cases we have to take it mod 180°(so that if ACnBis
Take AB as the x-axis and the perpendicular line through M as the y-axis. Choose the unit oflength so
= Xn+l.for sorne n >
obtuse, then Cn+1Iies on the other side of AB). lf x, is eventually periodic then Xm+1
that M has coordinates (O, 1). Let A be (-m, O) and B be (n, O). Then considering the right-angled
m, so (Z" - 2m)XI= O mod 180. Hence XI= 180 rls for sorne relative!y prime integers r, s. Also s cannot
triangle O'MK, where K is (n, 1) we find that O' is (n, (n2+1)/2 ). Similarly, O is (-m, (m2+1)/2 ) ).
be a power of 2 for then we would have Xk= 180r for sorne k, in which case the sequence would
terminate rather than be periodic.
The gradient ofthe Iie AM is 11m,so the gradient ofthe line BD is -m and hence its equation is mx + y
= mn. The gradient ofthe line 00' is (n-m)/2, so its equation is 2y - x(n-m) = mn+l. These intersect at (


'.•
(mn-I )/(m+n), (mn2+m)/(m+n) ). B' is (n, n2+ 1). It is now easy to check that the lines MB' and MD both Let d = x - 1, so all terms ofthe AP have the form 1+nd for sorne positive integer n. Suppose a = 1 + md,
have gradient n, so M, D, B' are collinear. a2 = 1 + nd, then (1 + md)" = 1 + nd, so d = (n - 2m)/m2, which is rational. Hence a is rational. Suppose a
= b/e, where b and e are relatively prime positive integers and e> l. Then the denominator ofthe nth
term ofthe GP is en, which beeomes arbitrarily large as n inereases. But if d = h/k, then all terms ofthe
This looks easy. but choosing the right coordinale system is critical. AP have denominator at most k. So we eannot have e > l. So a must be a positive integer.


Problem A3 On the other hand, it is easy to see that any positive integer works. Take x = 2, then the AP includes all
positive integers and henee ineludes any GP with positive integer terms.


Find all solutions to (m + 1)" = rn'' + 1 in integers greater than l.
Problem 82
Answer
Given apile of2000 stones, two players take tums in taking stones from the pile. Eaeh pi ayer must
11 (m, a, b) = (2, 2, 3). remove 1,2,3,4, or 5 stones from the pile at eaeh tum, but may not take the same number as his


opponent took on his last move. The player who takes the last stone wins. Does the first or seeond player
have a winning strategy?
Solution

• Thanks fa José Nelson Ramirez


Solution

• Taking equation mod m+ 1 we get (_I)b = -1, so b is odd. Henee we can divide the rhs by m+ 1 to get m"!
The first player has a winning strategy. He takes 4 on his first move leaving 7 mod 13 (2000 = 153.13 +
_ mb-2+ ... _ m + 1. This has an odd number ofterms. Ifm is odd, then eaeh term is odd and so the total is 7 + 4). Now we e1aim that the first pi ayer can always Icave: (1) O mod 13, (2) 3 mod 13 by taking away
odd, but (m+ 1)"., is even (note that a> 1). Contradicton, so m is even. 3, (3) 5 mod 13 by taking away 5, or (4) 7 mod 13, and that the second player can never leave O mod 13.

" We have m'' = (m+ l )" - l. Expanding the rhs by the binomial theorem, and using b > 1, we see that m
must divide a. So a is even also. Put a = 2A, m = 2M. We can factorise (m+ l )" - 1 as ( (m+ I)A + 1) (
(m+l ]" - 1). The two factors have difference 2, so their gcd divides 2, but both factors are even, so their
Let us look at each ofthese in turno [fthe first player leaves O mod 13, then the second player can take 3
and leave 10. In that case the first player takes 5 (a type (3) move). Ifthe second player takes 1,2,4 or 5,
leaving 12, 11,9 or 8 mod 13, then the first pi ayer takes 5, 4, 2, l (respectively) and leaves 7 mod 13 (a
type (4) move).
gcd is exactly 2.

[f the first player leaves 3 mod 13 by taking away 3, then the second player cannot leave O mod 13,
[fM = l or a power of2, then the smaller factor 3A - 1 must be 2, so A = 1 and we have 3A + 1 = 4, so because he cannot take 3 stones. If he takes 1, 2 leaving 2, 1 mod 13 respectively, then the first player
(2M)b = 8. Hence M = 1 and b = 3 and we have the solution (m, a, b) = (2, 2, 3). takes 2, 1 leaving O mod 13 (a type (1) move). Ifthe second player takes 4,5 leaving 12, 11 mod 13,
then the first pi ayer takes 5, 41eaving 7 mod 13 (a type (4) move).
[f M is not a power of 2, then Mb > 2b, so we must have the larger factor 2·Mb and the smaller factor 2b-'.
But the larger factor is now > 2b", so the difference between the factors is at least 3·2b.' > 2. Ifthe first player leaves 5 mod 13 by taking 5, then the second player cannot leave O mod 13, because he
Contradiction. cannot take 5 stones. Ifhe takes 1,2,3,4 stones, leaving 4,3,2,1 mod 13, then the first player takes 4,

• Problem 81
3,2, 1 stones leaving O mod I3 (a type (1) move).


Finally, ifthe first player leaves 7 mod 13, and the second player takes 1 stone, then the first player takes
Sorne terms are deleted from an infinite arithmetic progression 1, x, y, .. ofreal numbers to leave an 3 stones leaving 3 mod 13 (a type (2) move). If the second player takes 2, 3, 4, or 5 stones leaving 5, 4,
infinite geometric progression 1, a, b, .... Find all possible values of a. 3,2 mod 13, then the first player takes 5,4,3,2 stones leaving O mod 13 (a type (1) move).

So the second player can never leave O mod 13 and hence, in particular, can never take the last stone.


Solution
But we have shown that the first player can always make a move of one ofthe four types, so can always
move and henee must win (sinee after less than 2000 moves there will be no stones left).


Answer: the positive integers.

Problem 83
[f a is negative, then the terms in the G P are altemately positive and negative, whereas either all terms in

• the AP from a certain point on are positive or all terms from a certain point on are negative. So a eannot
be negative. [f a is zero, then all terms in the GP exeept the first are zero, but at most one term ofthe AP A convex hexagon is ealled a unit if it has four diagonals ofIength 1, whose endpoints inelude all the
vertices of the hexagon. Show that there is a unit of area k for any O < k:s 1. What is (he largest possible

••
is zero, so a cannot be zero. Thus a must be positive, so the AP must have infinitely many positive terms
and hence x 2: l. area for a unit?

15
Solution

Answer: We can get arbitrarily close to (but not achieve) (3v'3)/4 (approx 1.3) by:

E D

The area ofthe hexagon is area AEDC + area AFE + area BAC. The part ofthe segment BF that lies
2 inside AEDC is wasted. The rest goes to provide height for the triangles on bases AE and AC. So area
AFE + area BAC can be maximised by taking F close to A and UBAC as close to a right angle as
possible, so that the height ofthe triangle BAC (on the base AC) is as large as possible. We can then get
To prove the first part, consider the diagram below. Take AB = AC = 1 and angle BAC = 29. Take DE = arbitrarily close to the area of:
DF = 1 and take the points of intersection X and Y such that AX = DX = AY = DY = 2/3. It is easy to
check that the area ofthe hexagon is sin 29. So by taking 9 in the interval (O,7[/4]we can get any area O
< k ~ 1.

A F
Ac7
(~,,~~~~J.
B

e
D

E
We obviously make AEB a straight line. Now area ADE + area ADC = area ACE + area CDE. So ifwe
e regard every point except D as fixed, then we maximise the area by taking .IEAD = r !CAD, so that D is
the maximum distance from CE. Thus a maximal configuration must have UAED = [lCAD. Similarly, it
must have I ICAD = , ICAB, so all three angles must be equal. That disposes of case l.
D
In cases 2 and 6 we find by a similar (but more tedious argument) the same maximum, although in one
case we have to use the argument at the end for the final optimisation. In the other cases the maximum is
It is easy to check that there are six possible configurations for the unit diagonals, as shown in the smaller.
diagram below.
..
...___
.. -- ""''''\'...... -- -....
)

~if715t
~, 5y-~"
2 "
.~

.- _,-_'7~~"""--,,_
., ..
1J.' /' ~~~, 4

'w'
,/' \
.)

. "\~
..
__ /
./ ..
i- __ J
./

'1..
__ ~
./ I ,

l'
\" ,/
Consider case l. -.~--=-i~
However, all these details would take an already long solution way over length. Does anyone have a
better approach?

• No. 6 (second case) can be made arbitrarily close to the figure below (with AB = AC = BD = 1). To
optimise it, suppose :JACB = e. Area ABDC = area ABC + area BCD. Ifwe fix e, then BC is fixed, so

• to maximise area BCD we must take IICBD = 90°. But e cannot be optimal unless also I ICAD = 90°.
We have BA = BD and hence ImAD = [IBDA = 45° - el2. Hence 90° = fJCAD = lJBAC - liBAD =

••
(180° - 26) - (45° - 6/2). Hence 6 = 30°. So [JACO = ilBDC = 60° and I !CAB = [JABO = 120°.It is easy
to check that this has area (3..J3)/4.

A 8

• B D e



~ Solution
e D

• 0lM2001
AF = AE, so I ¡AFE = 90° - N2. Hence I !BFP = 90° + N2. But I IFBP = B/2, so UFPB = C/2. But BFP
and BDP are congruent (BF = BO, BP cornmon, I :FBP = OFDP), so I IDPB = C/2 and rmPQ = C.


Similarly, I IDQP = B. Hence i 1PDQ = A. SODQP and ABC are similar. So if one is isosceles, so is the
Problem Al
other.


Show that there are arbitrarily large numbers n such that: (1) all its digits are 2 or more; and (2) the
Thanks to Johann Peter Gustav Lejeune Dirich/et
product of any four of its digits divides n.

• Solution
A3. Let X be a set with n elements. Given k > 2 subsets of'X, each with at least r elernents, show that
we can always find two of them whose intersection has at least r - nkl(4k - 4) elements.

• 3232 = 16 x 202 and 10000 = 16 x 625. So any number with 3232 as its last 4 digits is divisible by 16.
So consider N = 22223232. Its sum of digits is 18, so it is divisible by 9. Hence it is divisible by 9.16 =
Problem BI

• 144. But any four digits have at most four 2s and at most two 3s, so the product of any four digits
divides 144 and hence N. But now we can extend N by inserting an additional9m 2s at the front. Its Call a set of 3 distinct elements which are in arithmetic progression a trio. What is the largest number of
digit sum is increased by 18m, so it remains divisible by 144 and it is still divisible by the product of any trios that can be subsets of a set of n distinct real numbers?

..• four digits.

A/ternative so/ution
Answer

(m-I)m for n = 2m

• The number IIIIIIIII


equals IIIIIIIII
with nine Is is divisible by 9. Hence the number with twenty-seven Is which
x 1000000001000000001 is divisible by 27. So N, the number with twcnty-scven 3s,
m' for n = 2m+ I

••
is divisible by 34• Now the number with 27n 3s is divisible by N and hence by 34.
Solution

Problem A2
Let X be one ofthe elements. What is the largest number oftrios that can have X as middle element?
Obviously, at most max(b,a), where b is the number of elements smaller than X and a is the number


ABC is a triangle. The incircle has center I and touches the sides BC, CA, AB at D, E, F respectively. larger. Thus ifn = 2m, the no. oftrios is at most 0+ I + 2 + ... + m-I + m-I + m-2 + ... + I + O= (m-
The rays BI and CI meet the line EF at P and Q respectively. Show that ifOPQ is isosceles, then ABC is I)m. Ifn = 2m+l, then the no. is at most 0+1 + 2 + ... + m-I + m + m-I + ... + I + O= m2•
isosceles.

• These maxima can be achieved by taking the numbers 1,2,3, ... , n.

• B2. Two players playa game on a 2000 x 2001 board. Each has one piece and the players move


their pieces alteroately. A short move is one square in any direction (including diagonally) or no
move at all. On his first turn each player makes a short move. On subsequent turns a player must


Join each pair of points, thus dividing the plane into polygonal regions. lf a point P moves around within
make the same move as on his previous tum followed by a short move. This is treated as a single one of the regions then the number of triangles it belongs to does not change. But if it crosses one of the
move. The board is assumed to wrap in both directions so a player on the edge ofthe board can lines then it leaves sorne triangles and enters others. Suppose the line is part ofthe segmentjoining the
move to the opposite edge. The first pi ayer wins ifhe can move his piece onto the same square as points Q and R ofthe set. Then it can only enter or leave a triangle QRX for sorne X in the se!. Suppose
his opponent's piece. For example, suppose we label the squares from (O, O) to (1999, 2000), and x points in the set lie on the same side ofthe line QR as P. Then there are 6 - x points on the other side of
the first player's piece is initially at (O, O) and the second player's at (1996, 3). The first player the line QR. SOP leaves x triangles and enters 6-x. Thus the net change is even. Thus ifwe move P until
could move to (1999, 2000), then the second player to (1996, 2). Then the first player could move it is in the outer infinite region (outside the convex hull ofthe other 8 points), then we change the
to (1998, 1998), then the second player to (1995, 1). Can the first player always win irrespective number oftriangles by an even number. But in the outside region it belongs to no triangles.
of the initial positions of the two pieces?
Note that the same argument worksfor any odd number ofpoints.
Problem A3
B3. Show that a square with side 1 cannot be covered by five squares with side less than 1/2.

ABC is an equilateral triangle. Pis a variable interior point such that [,APC = 120°.The ray CP meets
AB at M, and the ray AP meets BC at N. What is the locus ofthe circumcenter ofthe triangle MBN as P
OIM 2002 varies?

Problem Al Solution

The numbers 1, 2, ... , 2002 are written in order on a blackboard. Then the 1st, 4th, 7th, ... , 3k+ Ith, ... Answer: the segment ofthe perpendicular bisector ofBG (where Gis the center ofthe triangle) which
numbers in the list are erased. Then the 1st, 4th, 7th, ... 3k+ Ith numbers in the remaining list are erased forrns a rectangle with AC.
(Ieaving 3, 5, 8, 9, 12, ... ). This process is carried out repeatedly until there are no numbers left. What is
the last number to be erased?

Solution

Answer: 1598.

Let a" be the first number remaining after n iterations, so lI(, = 1, a, = 2, a, = 3, !I.¡ = 5 etc. We claim that:

a,,+1 = 3/2 a" if a" is even, and

3/2 (a" + 1)-1 íf a, is odd.

We use induction on n. Suppose a" = 2N. Consider the number 3N. There are initially N smaller numbers
= 1 mod 3. So after the first iteration, it willlie in 2Nth place. Henee, it willlie in first place after n+ 1
iterations. Similarly, suppose a" = 2N+ l. Consider 3N+2. There are initially N+I smaller numbers = 1
mod 3. So after the first iteration, it willlie in 2N+ 1st place. Hence, it willlie in first place after n+ 1
iterations. That completes the induction.

We may now calculate successively the members ofthe sequence: 1,2,3,5,8,12,18,27,41,62,93,


140,210,315,473,710, 1065, 1598,2397. Hence 1598 is the last surviving number from 1,2, ... , 2002. UMPN =, APC = 120°and IMBN = 60°, so MBNP is cyclic, in other words, P lies on the circumcircle ..J
ofBMN.
Problem A2
P also lies on the circle AGC. so CPG = CAG (ifP is on the same side of AG as A) = 30° = MBG.
So PMBG is cyclic. In other words, G also lies on the circumcircle ofBMN. IfP lies on the other side,
Given a set of9 points in the plane, no three collinear, show that for each point P in the set, the number
the same conclusion follows from considering APG.
oftriangles containing P forrned from the other 8 points in the set must be even.

Since B and G lie on the circumcircle, the center O must lie on the perpendicular bisector of BG. But it
Solution
is clear that the extreme positions ofO occur when P is at A and B and that these are the feet ofthe
perpendiculars from A and B to the perpendicular bisector.
Problem Bl
•• ABC is a triangle. BD is the an angle bisector. E, F are the feet ofthe perpendiculars from A, C
respectively to the line BO. M is the foot ofthe perpendicular from D to the line BC. Show that 1'DME
Using Maple, wefind that a; isfirst negativefor n = 1572.

• =UDMF.
Problem B3

• Solution
A game is played on a 2001 x 2001 board as follows. The first player's piece is the policeman, the
second player's piece is the robber. Each piece can move one square south, one square east or one square

• A
northwest. In addition, the policernan (but not the robber) can move from the bottom right to the top left
square in a single move. The policernan starts in the central square, and the robber starts one square

••
diagonally northeast ofthe policeman. If the policeman moves onto the same square as the robber, then
the robber is captured and the first player wins. However, the robber may move onto the same square as
the policeman without being captured (and play continues). Show that the robber can avoid capture for

..
at least 10000 moves, but that the policeman can ultimately capture the robber.

Solution

••
Color the squares with three colors as follows:

.. B M e

Let H be the foot ofthe perpendicular from D to AB. IJAHD = [.]AED= 90°, so AHED is cyclic. Hence
IIOAE = IDHE. But M is the reflection ofH is the line BO, so [lOME = I .oAE.
O l2 O l2 O

120

2012012
12 O l

O l 2 O l 2 O ... 2
2

AE is parallel to CD, so ilDAE = ílDCF. ,OFC = I !DMC, so DMCF is cyclic. Hence . IDCF =
!lDMF. Hence I1DME= LlDMF.
120 l 2 O 1 O

Problem 82

2 O 1 2 O 1 2 ... l
The sequence a" is defined as follows: al = 56, an+l= an - lIan. Show that an< O for sorne n such that O <
n <2002. The middle square is color 2 (moving 999+1 squares E from the top left increases the color by 1, then
moving 999+ l S increases it by another 1) and the square immediately NE of it is also 2. So both P and
Solution R start on color 2. Note that any move increases the color by l mod 3, except for P's special move which
changes the color from 1 to O.

.. Note that whilst a" remains positive we have al > a, > a, > ...> a". Hence if amand am+nare in this part of
the sequence, then am+l= am- lIam,am+2= am+l- lIam+1< am+l- I/am= am- 21am.By a trivial induction
am+n < am- n/a.;
Until P has made this move, afier each move of P, P's color is always 1 more than R's color (mod 3), so
P cannot win (irrespective of the moves made by either player). Immediately after he makes the special
move for the first time, P is on color O and R is on color 1, so immediately afier his move P's color is

..
now l less than R's color mod 3. Again P cannot win. But after P has made the special move for the
Ir we use one step then we need 56' = 3136 terms to get a1+3l36 < 56 - 56'/56 = O, which is not good second time, P's color is the same as R's (mod 3) immediately after P's move.
enough. So we try several steps.

Note that it takes P at least 2001 moves to complete his special move for the first time and at least 6002
Thus suppose that a" > O for all n<= 2002. Then we get successively: moves (in total) to complete his special move for the second time. This solves the first part ofthe

•..
am < 56 - 336/56 = 50 question. Suppose Rjust moves down to the bottom right and then moves in small circles (one move
am < 50 - 500/50 = 40 NW, one move S, one move E) waiting for P. It takes P at least 6002 + 3999 (moving from top lefi to the
am7 < 40 - 400/40 = 30

..
capture square, one square short ofthe bottom right) = 10001 to capture him, so R makes at least 10000
alm < 30 - 300/30 = 20 moves before being captured .
am7 < 20 - 200/20 = 10
alm < 10 - 100/10 = O.
We c1aimthat P wins ifhe can get into any ofthe positions shown below relative to R, with R to move
(*):
Contradiction. So we must have an< O for sorne n < 2002.
We start with the easiest case: in the two following positions. R cannot move to z, so he must move east
x P x x x or south on each move. Hence he will (after at most 4000 moves) reach the bottom right comer. He then
loses moving out of it.
P x x P x"

x x R x x
x P x
x P x x P
Pz x
x x x P x
x x R
Iffollows that P can also win from the four positions below (**):
The other cases of (*) are slightly more complicated. Starting from either of the two positions below, we
x x x P x x x show that R must eventually reach the extreme left column.

xxxxxxx w x Px

xxxxxxx x R z x

PxxRxxP x y x P

xxxxxxx R cannot move to z, so he can only make NW and S moves. But his total number ofS moves can never
exceed his total number ofNW moves by more than 2000 because he cannot move offthe bottom ofthe
xxxxxxx board, so he must eventually reach the extreme left column. [If he reaches the bottom row at y, then P
can always move to z to preserve the configuration. If R reaches the top row by moving to w, then P can
x x x P x x x always move to z to preserve the configuration.]

For in each case at least one of R's possible moves allow P to move immediately into one of the winning Having reached the extreme left column he is forced to move south. Eventually moving to y will take
positions at (*). But R can only make the other moves a limited number oftimes before running into the him to the comer. P then moves to z and R is captured on his next move.
border. [That is obvious ifthe other two moves are E and S. Ifthey are NW and E, then every NW move
takes R closer to the top border, but his total number of E moves can never exceed his total number of
NW moves by more than 2000 because ofthe right border. Similarly, for NW and S.] The final case to consider is the two positions below. R cannot move to z; so must move E or NW. A
similar argument to the previous case shows that he must eventually reach the top row. Having reached
it at w, P moves to z. So R is forced to move right along the top row. When he reaches the comer at y, P
Now let d be the number of rows plus the number of columns that R and Pare aparto It is easy to check moves to z and R is captured when he moves out ofthe comer.
that the positions in (*) and (**) represent the only possibilitics for d = 2 and 3. We show that P can
always get to d = 2 or 3. For P can always copy R's move, so he can certainly move so that d never
increases. But one ofR's moves will always a1low P to decrease d by I or 2. There are three cases to
consider: wxx
Case l. If P is east of R and R moves E, then P moving NW will decrease d by l or 2. That is not
possible ifP is in the top row, but then moving S will decrease d by 2 unless R is also in the top row. If x R Y
both are in the top row, then P moves S. Now after R's next move, P moves NW which reduces d by 2.
Case 2. If P is south of R and R moves S, then a similar argument, shows that P can always decrease d P z x
by I or 2 in one or two moves.
Case 3. If Pis not south or east or R, and R moves NW, then P can always decrease d by I or 2 by x x P
moving S or E.
OIM 2003
But repeated decreases by I or 2 must bring d ultimately to 2 or 3 and hence to one of (*) or (**). So P
can always win. Problem A1

It remains to prove the claim that (*) are winning positions. The reason is that in each case R has one Let A, B be two sets ofN consecutive integers. IfN = 2003, can we form N pairs (a, b) with a A, b 1
move blocked off, so must make one ofthe other two. P then copies R's move, so next tum R has the B such that the sums ofthe pairs are N consecutive integers? What about N = 2004?
same move blocked off. Repeated use ofthe other two moves will bring him ultimalely lo one of the
sides.
Answer
•• Yes, no. Problem Bl

• Solution
A ¡ I {I, 2, 3, ... , 49) does not contain six consecutive integers. Find the largest possible value of'[A]
How many such subsets are there (ofthe maximum size)?

•• wlog A = B = {I, 2, ... , N} - ifwe have a solution for A = {a+l, a+2, ... , a+N} and B = {b+l , b+2, ... ,
b+N}, then subtracting a from every element of A and b from every element ofb gives a solution for A
= B = { 1, 2, ... , N}. Suppose the sum set is (m+1), (m+2), ... , (m+N). It has sum N(2m+N+ 1)/2 and A
and B each have sum N(N+1)/2, so we must have 2m = N+ 1, hence N must be odd. So we cannot do it
Answer

max = 41; no. ways 495


for N = 2004.
Solution


Suppose N = 2M+I, take the pairs (I,M+I), (3,M), (S,M-I), ... , (2M+I,I), (2,2M+I), (4, 2M), ... , (2M,
M+2).
We must exclude at least one element of each ofthe 8 sets {I, 2, ... , 6), {7, ... , 12}, {13, ... , 18}, ... ,


{43, ... , 48). SOIAI~ 41. But a value of 41 is certainly possible, for example, exclude 2,8,14, ... ,44.
Problem A2
The largest excluded element must be at least 44 (or we have the 6 consecutive elements 44, 45, 46, 47,

• C is a point on the semicircle with diameter AB. D is a point on the are Be. M, P, N are the midpoints of 48,49). The smallest excluded element must be at most 6. Ifwe exclude 2 and 44, then the difference
AC, CD and BD. The circumcenters of ACP and BDP are O, O'. Show that MN and 00' are parallel. between them is 7·6 and so the other 6 excluded elements are fixed. But ifwe exclude 3 and 44, for


example, then there are several possible choices for the other elements.
Solution


There are 5 ways of choosing the smallest and largest excluded element to get a difference of 7·6
between them (2 and 44,3 and 45, 4 and 46,5 and 47, 6 and 48). There are 4 ways to get a difference of
7·6 - 1 (3 and 44, 4 and 45,5 and 46, 6 and 47). There are 3 ways to get a difference 0[7·6 - 2 (4 and 44,
5 and 45,6 and 46), 2 ways to get a difference of 7·6 - 3 (5 and 44,6 and 45), and 1 way to get a
difference 0[7·6 - 4 (6 and 44).

Ifthe difference is 7·6 - 1, then we can shorten any ofthe 7 gaps, so there are 7 possibilities. For
example, with 3 and 44, we could shorten the first gap, so excluding 3,8, 14,20,26,32,38 and 44, or
x B the second gap, so excluding 3, 9, 14,20,26,32,38 and 44, and so on.
A

Ifthe difference is 7·6 - 2, then we can shorten one gap by two (7 possibilities) or two gaps by one (21

l.
Lct the center ofthe circle be X and the radius r. Let AXM = e, .BXN = (j). Note that O is the possibilities), total 28. Ifthe difference is 7·6 - 3, then we can shorten on gap by three (7), one by two
intersection ofXM and the perpendicular to CD at Q, the midpoint of Cf'. We have XM = r cos e. Let and one by one (42) or three by one (35), total 84. Finally, ifthe difference is 7·6 - 4, we can shorten one
CD and XM meet at Y. Then I !PYX = 90° - I IPXY = 90° - IPXC - ICXM = e + (j) - (j) = e. Hence OX
l. by four (7), one by three and one by 1 (42), two by two (21), one by two and two by one (105), or four
= PQ sec (j), so OXlXM = PQ/(r cos ecos (j». Similarly, O'XlON = PQ/(r cos ecos (j», so 00' and MN by one (35), total 210.
are parallel.
So the total number ofpossibilities is 5·1 + 4·7 + 3·28 + 2·84 + 1·210 = 495.

•• ProblemA3

..
Problem B2
Pablo was trying to solve the following problem: find the sequence Xo, x, X2,... , X200J
which satisfies Xo
= 1, O ~ X¡~ 2 X¡.,for 1 ~ i ~ 2003 and which maximises S. Unfortunately he could not remember the
ABCD is a square. P, Q are points on the sides BC, CD respectively, distinct from the endpoints such
expression for S, but he knew that it had the form S = ± x¡ ± X2± ... ± X2IK12
+ X200J.Show that he can still that BP = CQ. X, y are points on AP, AQ respectively. Show that there is a triangle with side lengths
solve the problem.
/ BX,XY, YD.

• Solution
Solution



For any combination of signs the maximum is obtained by taking all X¡as large as possible. Suppose we
have a different set of x, Then for sorne k we must have Xk< 2Xk.'and X¡= 2x¡., for all i > k. Suppose
2xk.' _Xk= h > O. Then we can increase x, by h, Xk+'by 2h, Xk+2 by 4h, .... So the sum will be increased
by h(± 1 ± 2 ± ... ± 2m.' + 2'") for sorne m ~ O. But ± 1 ± 2 ± ... ± 2m.' ~ -(1 + 2 + ... + 2m.') = -Z'"+ 1, so
the overall sum will be increascd by at least l. So the set of x, was not maximal.

...• (i
The easiest way to do that is to use the law of quadratic reciprocity, but that is almost certainly outside
the syllabus. We note that 4'502 = 5 mod 2003, so 502 is a square iff5 is a square. It is sufficient to
•..
show that 51001= -1 mod 2003, for then ifwe had X2= 5, we would have X2OO2 = -1 mod 2003, whereas
we know that X2OO2 = 1 mod 2003. We note that 1001 = 7·11·13. We start by showing that 57 = 8 mod
2003. We have 55 = 3125 = 1122 mod 2003, so 56 = 5610 = 1604 mod 2003, so 57 = 8020 = 8 mod 2003.
A

We calculate successively 211 = 2048 = 45 mod 2003, so 222 = 2025 = 22 mod 2003. Multiplying by 22
is relatively easy, so 244= 484, 266 = 10648 = 633, 288 = 13926 = -95, 2110 = -2090 = -87, 2132 = -1914 =
89,2141 = 4005 = -1 all mod 2003. Hence 81111= -1 mod 2003, so 51001= -1 mod 2003, as required, and
we are done.

D Q e

We have DY < BY :s BX + XY (this is almost obvious, but to prove formally use the cosine formula for
....
BA Y and DA Y and notice that IJBA Y> rIDA Y). Similarly, BX < DX:S DY + YX. So it remains to
show that XY < BX + DY. ....
Take Q' on the extension of'Bf. so that BQ' = DQ, as shown in the diagram. Take Y' on AQ' so that AY'
= AY. Then XY':s BX + BY' = BX + DY. Now we claim that I IPAQ' > i I PAQ, so it follows by the
same observation as aboye that XY' > XY. But the c1aim is almost obvious. Note that PQ' = AS

A'

So take P' on AD with ~IP'PQ' = 90°. Then A lies inside the circle P'PQ', so extend PA to meet it again at
A'. Then i:PA'Q' = I IPP'Q' = 45°, so I IPAQ' = UPA'Q' + IIAQ'Q' > 45°. But UPAQ' + ! IPAQ = 90°, so
! I PAQ' > PAQ as c1aimed.
Problem 83

+ b., bn+1= bn2OO1


The sequences 110,al, a2, ... and bo, br, b., ... are defined by 110= 1, bo = 4,110+1= 1102001 +
110.Show that no member of either sequence is divisible by 2003.

Solution

2003 is prime, so a2'X'2= 1 mod 2003 for any a not divisible by 2003. Thus 110+1 + b, mod 2003, bn+1
= 110,1
= b," + 110mod 2003. Put e, = aobn. Then Cn+1= cn + l/c, + 2 = (e, + 1)l/Cn mod 2003. So if'c, fe Omod
2003, then Cn+1 i'" O mod 2003 unless Cn = -1 mod 2003. Then if cn+) = -1 mod 2003, we must have (Cn 2 +
3cn + 1)/cn = O mod 2003, so cn2 + 3cn + 1 = Omod 2003. Note that Co = 4. So it is sufficient to show that
there are no solutions to X2+ 3x + 1 = O mod 2003, or equivalently to (x - 1000)2 = 10002 - 1 = 502 mod
2003. In other words, we have to show that 502 is a quadratic non-residue mod 2003.


•• IMO 1959
fonn a quadratic equation in cos 2x whose roots are the same values of x. Compare the


equations in cos x and cos 2x for a=4, b=2, c=-1.
Problem Al



Prove that (2In+4)/(14n+3) is irreducible for every natural number n.
SolutioD

You need that cos 2x = 2 cos'x - 1. Some easy manipulation then gives:

• SolutioD

• 3(14n+3) - 2(2 1n+4) = 1. The equations are the same for the values ofa, b, e given. The angles are 27r.1S (or 87r.1S) and
47r.15 (or 61ti5).

• Problem Al Problem Bl

• Prove that (2In+4)1(14n+3) is irreducible for every natural number n.

ProblemAl
Given the length IAC¡, construct a triangle ABC with LABC = 90', and the median BM
satisfying BM2 = AB·BC.

• For wbat real values oh is ..J(x+ ..J(2x-I)) + ..J(x- ..J(2x-I)) = A, given (a) A = ..J2, (b) A = SolutioD

----~

1, (e) A = 2, where only non-negative real numbers are allowed in square roots and the root
a1ways denotes the non-negative root?
// ...... .........-,

Answer
.--/~~---
A \ M le
(a) any x in the interval [1/2,1]; (b) no solutions; (e) x=3/2.
\',.,.------- ...///
• Area = AB'BC/2 (because LABC = 90°= BM2/2 (required) = AC2/8 (because BM = AM =


SolutioD
MC), so B lies a distance AC/4 from AC. Take B as the intersection of a circle diameter AC
Note that we require x ~ 1/2 to avoid a negative sign under the inner square roots. Since (x- with a line parallel to AC distance AC/4.
Ii ~ o, we have x ~ ..J(2x-I), so there is no difficulty with ..J(x- ..J(2x-I)), provided that x ~

• 1/2.


Squaring gives 2x + 2..J(x2-2x+1) = A2. Note that the square root is Ix-II, not simply (x-I), Problem B2
So we get finally 2x + 21x-11= A2. It is now easy to see that we get the solutions aboye.
An arbitrary point M is taken in the interior of the segment AB. Squares AMCD and MBEF

••• ProblemA3

Let a, b, e be real numbers. Given the equation for cos x:


are constructed on the sarne side of AB. The circles circumscribed about these squares, with
centers P and Q, intersect at M and N.
(a) prove that AF and BC intersect at N;
(b) prove tbat the lines MN pass through a fixed point S (independent ofM);
(e) find the locus of the midpoints of the segments PQ as M varies.

• a cos'x + b cos x + C = O,




Solution
,, ,
Hf.--------- I
s : e
/" ,
,/
//
A 1....,.- _ .
/

(\
E

:0
,,
(a) LANM = LACM = 45°. But LFNM = LFEM = 45°, so A, F, N are collinear.
Similarly, LBNM = LBEM = 45°and LCNM = 180° - LCAM = 135°, so B, N, C are
collinear. Let the planes meet in the line L. Then AB and CO must be parallel to L. Let H be the foot
ofthe perpendicular from C to AS. The faet that a circle can be inscribed implies AB + CO
(b) Since LANM = LBNM = 45°, LANB = 90°, so N Hes on the semicircle diameter
= BC + AO (equal tangents from A, B, C, O 10 the circle). Also CO = AB ± 2BH. This
leads 10 AH = AO = BC.
AB. Let NM meet the circle diameter AB again at S. L ANS = L BNS implies AS = BS
and hence S is a fixed point.
The construction is now easy. First construct the point H. Then using the circle center C
radius AH, construct B. Using the circle center A radius AH construct O.
(e) Clearly the distance of the midpoint ofPQ from AB is AB/4. Since it varies
continuously witb M, it must be the interval between the two extreme positions, so the
Note that if CH > AH then no construction is possible. If CH < AH, then there are two
locus is a segment length AB/2 centered over AS.
solutions, one with AB > CO, the other with AB < CO. If CH = AH, then there is a single
solution, which is a square.
Problem Bl
IMO 1960
The planes P and Q are not parallel. The point A Hes in P but not Q, and the point C Hes in
Q but not P. Construct points B in P and O in Q such that the quadrilateral ABCO satisfies ProblemAl
the following conditions: (1) it Hes in aplane, (2) the vertices are in the order A, B, C, O,
(3) it is an isosceles trapezeoid with AB is parallel 10 CO (meaning that AD = BC, but AD
Determine a11 3 digit numbers N which are divisible by II and where N/II is equal 10 the
is not parallel 10BC unless it is a square), and (4) a circle can be inscribed in ABCO
sum ofthe squares of the digits ofN.
touching the sides.

Answer
Solution
550,803.

Solution

So, put N/II = lOa + b. Ifa + b:s 9, we have 2a2 + 2ab + 2b2 = lOa + b (.), so b is even.
Put b = 2B, then B = a(a-5) + 2aS + 4B2, which is even. So b must be a multiple of 4, so b



• = o, 4 or 8. If b = O,then (.) gives a = 5 and we get the solution 550. Ifb = 4, then (.) gives the left ofH). So tan a = (QH - PH)/(AH2 + QH-PH) = AH·PQ/(AH2 + (MH - a/2nXMH +
a/2n» = (ahlny(a2/4 - a2/(4n2» = 4nh1(an2 - a).


a2 _ a + 14 = O,which has no integral solutions. Ifb = 8, then (since a + b:s 9 and a> O) a
must be 1, but that does not satisfy (.).
Problem Bl
Ifa + b > 9, we have (a+I)2 + (a+b-IOi + b2 = lOa + b, or 2a2 + 2ab + 2b2 - 28a- 2Ib +
101 = O(•• ), so b is odd. Put b = 2B+1. Then a2 + 2aB + 4B2 - 13a - 17B + 41 = O. But a(a- Construct a triangle ABC given the lengths of the altitudes from A and B and the length of
13) is even, so B is odd. Hence b = 3 or 7. Ifb = 3, then (•• ) gives a2 - Ila + 28 = O, so a = the median from A.

• 4 or 7. But a + b > 9, so a = 7. That gives the solution 803. Ifb = 7, then (•• ) gives a2 -7a +
26 = O, which has no integral solutions.


Solution
ProblemA2


For what real values ofx does the following inequality hold:

• Answer

• - 1/2 :s x < 45/8.


e

•.. Solution

We require the first inequality to avoid imaginary numbers. Hence we may set x = -1/2 +
a212,where a 2: O.The inequality now gives immediately a < 7/2 and hence x < 45/8. It is a
Let M be the midpoint of BC, AH the a1titude from A, and BI the altitude from B. Start by
constructing AHM. Take X on the circle diameter AM with MX = B1/2. Let the Iines AX,


HM meet at C and take B so that BM = MC. [This works because CMX and CBI are
matter of'taste whether lo avoid x = o. I would allow it because the limit as x tends to O of
similar with MX = BI/2 and hence CM = CBI2.]
the Ihs is 4, and the inequality holds.


Problem B2
Thanks lo Dave Arthur for the idea ofputting x = -J/2 + a, which saves some tedious
algebra. The cube ABCOA'B'C'O' has A aboye A', B aboye B' and so on. X is any point ofthe face

•.- ProblemA3

In a given right triangle ABC, the hypoteneuse BC, length a, is divided into n equal parts
with n an odd integer. The central part subtends an angle a at A. h is the perpendicular
distance from A to BC. Prove that:
diagonal AC and y is any point of B'O'.
(a) find the locus ofthe midpoint ofXY;
(b) find the locus ofthe point Z which Iies one-third ofthe way along XY, so that
ZY=2XZ.


11
tan a = 4nh1(an2- a).

Solution
Solution

... Let M be the midpoint of BC, and P and Q the two points a/2n either side of it, with P
nearer B. Then a = LPAQ = LQAH - LPAH (taking angles as negative ifP (or Q) Iies to





SolutioD

Let the vertex ofthe cone be V, the center of the sphere be o and the center oflhe base be
X. Let the radius ofthe sphere be r and the half-angle of the cone a.

Then the the cone's height is VO + OX = r(1 + IIsin a), and the radius ofits base is r(l +
IIsin a) tan a. Hence VIN2= (1/6) (1 + IIsin a)3 tan2a = (1 + s¡J(6s(1 - S2», where s = sin a.

We c1aim that (1 + s)3(6s(1 - S2» 2:: 4/3. This is equivalent to 1 + 3s + 3s2 + S32:: 8s - 3s3 or l
The key idea is that the midpoint must lie in the plane half-way between ABCD and - 5s + 3s2 + 9s3 >= O. But we can factorise the cubic as (1 - 3s)2(1 + s). So we have VIN2 2::
A'B'C'D'. Similarly, Z must lie in the plane one-third oflhe way from ABCD to A'B'C'D'. 4/3 with equality iff S = 1/3.

(a) Regard ABCD as horizontal. Then the locus is the square with vertices the midpoints of Problem 84
the vertical faces (shown shaded in the diagram).
In the isosceles trapezoid ABCD (AB parallel lo OC, and BC = AD), let AB = a. CD = e
Take Yat B' and let X vary, then we trace out MN. Similarly, we can get the other sides. and let the perpendicular distance frorn A to CD be h. Show how lo construct all points X
Now with Y at B', take X in general position, so the midpoint ofXY is on MN. Now move on the axis of symmetry such that L BXC = L AXD = 90°. Find the distance of each such
y lo D', the midpoint traces out a line parallel to the other two sides ofthe square, so we X from AB and from CD. What is the condition for such points to exist?
can get any point inside the square. But equally, it is clear that any point inside the triangle
LMN corresponds to a point y on the ray D'B' not between B' and D', so it does not lie in
the locus. Similarly for the other Ihree triangles. So the locus is the square.
SolutioD
(b) A similar argument shows that the locus is the rectangle shown in the diagram below
which is -./2/3x 2-./2/3.

o• c'

m
1(
,,
,
.
....
e
A B
Since angle BXC = 90°, X lies on the circle diameter Be. In general this will intersect Ihe

....
axis of symmetry in O, 1 or 2 points. By syrnmetry any points of intersection X will also lie
Problem B3
on the circle diameter AD and so will have angle AXD = 90° also.
A cone ofrevolution has an inscribed sphere tangent to the base of the cone (and to the
Let L be the midpoint of AB, and M the midpoint of CD. Let X lie on LM a distance x from
sloping surface of the cone). A cylinder is circumscribed about Ihe sphere so that its base
L. We have LB = a/2, MC = c/2, and XM = h - x, The triangles LBX and MXC are similar,
lies in the base ofthe cone. The volume ofthe cone is VI and the volume of the cylinder is
so 2x1a = c/(2(h-x». Hence 4x2 - 4xh + ac = O, so x = h/2 ± (-./(h2- ac) )/2.
V2•
(a) Prove that VI f. V2;
There are O, 1,2 points according as h2 <, =, > ac.
(b) Find the smallest possible value ofVIN2. For this case construct the half angle of the
cone.
IMO 1960
'.1
ProblemAl



• Solve the following equations for x, y and z: equilateral. Now 2 sin(X + 30·) = .J3 sin X + cos X. so using the cosine rule a2 = b2 + c2 -

•..
2be cos X, we get Ihe required inequality.

ProblemAl

..
What conditions must a and b satisfy for x, y and z to be distinct positive numbers?
Solve the equation cos"x - sin"x = 1, where n is a natural number.

Solution


Solution

Since cos'x + sin2x = 1, we cannot have solutions with n not 2 and O < leos x], [sin x] < 1.

• A little care is needed with the conditions. Clearly x, y, z positive implies a > O, and then z Nor can we bave solutions with n=2, because the sign is wrong. So the only solutions bave
positive implies Ibl < a The expression under the root must be positive. It helps ifyou sin x = O or cos x = O, and these are: x = multiple of It, and neven; x even multiple of It and
notice Ihat it factorizes as (3a2 - b2X3b2 - a2). The second factor is positive because Ibl< a, n odd; x = even multiple of It + 3:n:12and n odd.

• so the first factor must also be positive and hence a < .J3 [b].These conditions are also
sufficient to ensure that x and y are distinct, but then z must also be distinct because Z2 =


xy.
Problem Bl
ProblemA2

•..
Pis inside the triangle ABC. PA intersects BC in D, PB intersects AC in E, and PC
Let a, b, c be the sides of a triangle and A its area. Prove that: intersects AB in F. Prove that at least one of APIPD, BPIPE, CPIPF does not exceed 2, and
at least one is not less than 2.

When do we have equality?


Solution

A
Solution


One approach is a routine slog from Heron's formula. The inequality is quickly shown to be
equivalent to a2b2 + bV + c2a2 ~ a4 + b4 + c4, which is true since a2b2 ~ (a4 + b4)12. We get
equality iff the triangle is equilateral.

• Ancther approach is to take an altitude Iying inside the triangle. If it has length h and
divides the base into lengths r and s, then we quickly find that the inequality is equivalent to
B e

• (h - (r + s).J312)2 + (r - st 2: O, which is true. We have equality iffr= s and h = (r + s).J312,


which means the triangle is equilateral. Take lines through the centroid parallel to the sides of the triangle. The result is then

•...
obvious.
A third solution is due to Jonathan Mizrahi (somewhat adapted):
Problem B2
We have b2 + c2 >= 2be with equality iff b = c. Also for any angle x in the range O· to 180·
we have 2be 2: 2be sin(X + 30·) with equality iff X = 60·. So taking X to be the angle Construct the triangle ABC, given the lengths AC = b, AS = e and the acute LAMB = a,
between the sides b and e (we cannot call it A because A is already used to mean the area in where M is the midpoint of BC. Prove that the construction is possible if and only if

• this question!) we have tbat b2 + c2 2: be sin(X + 30") with equality iff the triangle is



2}
b tan(al2) ~ e < b. Given 3 non-collinear points A, B, e and aplane p not parallel to ABe and such that A, B,
e are all on the same side of p. Take three arbitrary points A', B', e' in p. Let A", B", e" be
the midpoints of AA', BB', ee' respectively, and let o be the centroid of A", B", e". What ....
When does equality hold? is the locus ofO as A', B', e' vary?

...
Answer

Equality holds if L BAe = 90· and L AeB = al2


Solution

Tbe key is to notice that o is the midpoint of the segrnent joining the centroids of ABe and
...
A'B'C'. The centroid of ABe is fixed, so the locus is just the plane parallel to p and midway
between p and the centro id of ABe.

IMO 1961
Solution
ProblemAl

Find the smallest natural number with 6 as the last digit, such that if the final 6 is moved to
the front ofthe number it is multiplied by 4.

Solution

We have 4(IOn+6) = 6. 10m + n, where n has m digits. So 13n + 8 = 2·10m. Hence n =2n'
and 13n' = 10m - 4. Dividing, we quickly find that the smallest n', m satisfying this are: n' =
7692, m = 5. Hence the answer is 153846.

ProblemAl
....
e
Find all real x satisfying: ,JO - x) - ,J(x + 1) > 1/2.
....
The key is to take N so that A is the midpoint ofNB, then LNeB = a.

The construction is as follows: take BN length 2AB. Take circle through B and N such that
the L BPN = a for points P on the are BN. Take A as the midpoint of BN and let the circle Solution
center A, radius Ae cut the are BN at e. In general there are two possibilities for e.
It is easy to show that the inequality implies Ix-II > ,J3118, so x > 1 + ,J31/8, or x < 1 -
Let X be the intersection of the are BN and the perpendicular to the segrnent BN through A. ,J31/8. But the converse is not true,
For the construction to be possible we require AX :::Ae > AB. But ABIAX = tan a/2, so
Indeed, we easily see that x > 1 implies the Ihs < o. Also care is needed to ensure that the

..
we get the condition in the question.
expressions under the root signs are not negative, which implies -1 ~ x ~ 3. Putting this
Equality corresponds to e = X and hence to LBAe = 90· and LAeB = al2. together, suggests the solution is -1 ~ x < 1 - ,J31/8, which we can easily check.

Problem B3 ProblemAl .
..



• The cube ABCDA'B'C'D' has upper face ABCD and lower face A'B'C'D' with A directly SolutioD

• aboye A' and so on. The point x moves at constant speed along the perimeter of ABCD, and
the point Y moves at the same speed along the perimeter of B'C'CB. X leaves A towards B
at the same moment as Y leaves B' towards C'. What is the locus ofthe midpoint ofXY?
Put e = cos x, and use cos3x = 4c3 - 3c, cos 2x = 2 c2 - 1. We find the equation given is
equivalent to e = O, c2 = 1/2 or c2 = 3/4. Hence x = n/2, 3n/2, n/4, 3n/4, n/6, 5n/6 or any

• multiple of It plus one of these.

Problem 82

• SolutioD
Given three distinct points A, B, C on a circle K, construct a point D on K, such that a


circle can be inscribed in ABCD.
D' C'

• SolutioD

• 8 o

..•
..
C

Answer: the rhombus CUVW, where U is the center of ABCD, V is the center of ABB'A,
and W is the center of BCC'B'.

Take rectangular coordinates with A as (O, O,O) and C' as (1, 1, 1). Let M be the midpoint
ofXY. Whilst X is on AB and y on B'C', X is (x, O,O) and y is (1, x, 1), so M is (x/2 + 1/2,
x/2, 1/2) = x (1,1/2,1/2) + (I-x) (1/2, 0,1/2) = x W + (l-x) V, so M traces out the line VW.

Whilst X is on BC and Y is on C'C, X is (1, x, O) and y is ( 1, 1, l-x), so M is (1, x/2+l/2,


1/2 - x/2) = x (1,1, O)+ (I-x) (1,1/2,1/2) = x C + (I-x) W, so M traces out the line WC. I be the centerofthe inscribed circle. Considerthe quadrilateral ABCI. LBAI = 1/2
LBAD and LBCI = 1/2 LBCD, so LBAI + LBCI = 90°, since ABCD is cyclic. Hence
Whilst X is on CD and y is on CB, X is (l-x, 1, O) and y is (1, l-x, O), so M is (1-x/2, 1-
L AIC = 270° - L ABC. So if we draw a circle through A and C such that for X points on

• x/2, O) = x (1, 1, O)+ (l-x) (1/2, 1/2, O) = x C + (I-x) U, so M traces out the line CU.

Whilst X is on DA and y is on BB', X is (O, l-x, O) and y is (1, O, x), so M is (1/2,1/2-


the are AC L AXC = 90° + L ABC, then the intersection of the circle with the angle
bisector of LABC gives the point 1.


11
x/2, x/2) = x (1/2, 0,1/2) + (I-x) (1/2,1/2, O) = x V + (I-x) U, so M traces out the line UV.

Problem 81
To draw this circle take the diameter AE. Then LCAE =
180° - LACE - LAEC 90°- =
L ABC. So we want AE to be tangent to the circle. Thus the center of the circle is on the
perpendicular to AE through A and on the perpendicular bisector of AC.

Find all real solutions to cos/x + cos22x + cos23x = l. To prove the construction possible we use the faet that a quadrilateral ABCD has an
.l. inscribed circle iff AB + CD = BC + AD. For D near C on the circumcircle of ABC we
have AB + CD < BC + AD, whilst for D near A we have AB + CD > BC + AD, so as D


moves continuously along the circumcircle there must be a point with equality. [Proofthat




..J
.J

the condition is sufficient: it is clearly necessary (use fact that tangents from a point are of
equallength). So take a circle touching AB, BC and AD and let the other tangent from C
(not BC) meet AD in D'. Then CD' - CD = AD' - AD, hence D'= D.]

Problem B3

The radius ofthe circurncircle of an isosceles triangle is R and the radius of its inscribed
circle is r. Prove that the distance between the two centers is ,f(R(R - 2r)).

Solution
This result is known as Euler's formula and is true for any triangle. Suppose two chords PQ
and ST of a circle intersect at l. Then PIS and TIQ are similar, so PUQ = SUT. Take the
special case when ST is perpendicular to al, where o is the center ofthe circle, then SI·fT
= st' = ¡f - or, where R is the radius of the circle, so PUQ = R2 - or.

Let the triangle be ABC with AB = AC, let the incenter be I and the circumcenter O. Let D
the distance 10 be d, taking d positive if O is closer to A than 1,negative if I is closer. Let
the L OAB be O.
Now let O be the circumcenter, I the incenter of an arbitrary triangle ABC. Extend Al to
meet the circumcircle again at D. Then by the aboye to' = ¡f -AI·ID.lfE is thefootofthe
Then r = (R + d) sin O, and r + d = R cos 26. It helps to draw a figure to check that this
perpendicular from I to AC, then Al = r/sin(Al2). We show that DI = DB. LDBI = LDBC
remains true for the various possible configurations. Using cos 20 = 1 - 2 sin20, we find that
(d + R + rXd2 - R(R - 2r)) = O. But OI < OA, so d is not - R - r. Hence result. + LCBI = LDAC + LDBI = Al2 + B/2. LDIB = LIAB + LIBA = Al2 + B/2. Hence
LDBI = LDIB, so DI = DB, as claimed Take F on the circle so that DF is a diameter,
Comment. then LDFB = LDAB = Al2, so DB = 2R sin Al2. Thus to' = R2 - r/sin(Al2) 2R sin(Al2) =
¡f -2Rr.

Problem B4

Prove that a regular tetrahedron has five distinct spheres each tangent to its six extended
edges. Conversely, prove that ifa tetrahedron has five such spheres then it is regular.

Solution

11


• First part is obvious. The wrong way to do the second part is to start looking for the locus
of the center of a sphere which touches three edges. The key is to notice that the tangents to
a sphere from a given point have the same length.
consider the plane through P perpendicular to AP and the parallel planes through the other
two points ofintersection of AP with the two spheres (apart from A) which pass through B
andC.

....
Let the tetrahedron be AIA2A3A¡. Let S be the sphere inside the tetrahedron, SI the ProblemA3
tetrahedron opposite Al, and so on. Let the tangents to S from A¡ have length II¡. Then the
side AAj has length a¡+a¡.Now consider the tangents to SI from Al. Their lengths are al + An n-gon has all angles equal and the lengths of consecutive sides satisfy al 2: az 2: ... 2: lIn·
2a2, al + 2a3, and al + 214. Hence a2 = a, = 14. Similarly, considering S2, we have that al = Prove that all the sides are equal.
a3 = 14.

IMO 1962
Solution
ProblemAI
For n odd consider the perpendicular distance of the shortest side from the opposite vertex.
This is a sum of terms a¡ x cosine of sorne angle. We can go either way round. The angles

•..
For which real values of p does the equation
are the sarne in both cases, so theinequalities give that al = an-I, and hence al = II¡ for all i <
..J(x2- p) + 2 ..J(x2- 1) = x have real roots? What are the roots? n. We get al = an by repeating the argument for the next shortest side. The case neven is
easier, because we take a line through the vertex with sides al and an making equal angles

....
with them and look at the perpendicular distance to the opposite vertex. This gives
immediately that al = lIn·
Solution
Problem BI
1 must admit to having formed rather a dislike for this type of question which carne up in
almost every one ofthe early IMOs. Its sole purpose seems to be to teach you to be careful Find all solutions XI, ... , Xsto the five equations X¡+ X¡+2= y X¡+Ifor i = 1, ... ,5, where
with one-way implications: the faet that a2 = b2 does not imply a = b. subscripts are reduced by 5 if necessary.

The Ihs is non-negative, so X must be non-negative. Moreover 2..J(x2- I):S x, so X :s 2/..J3.


Also ..J(x2- p):S x, so p 2: O.
Solution
s 2/..J3

,.
Squaring etc gives that any solution must satisfy X2= (p - 4)'/(16 - 8p). We require x
and hence (3p - 4)(p + 4):S O, so p:S 4/3. Successively eliminate variables to get xI(y - 2)(y2 + Y- 1)' = O. We have the trivial
solution X¡= O for any y. For y = 2, we find X¡= s for all i (where s is arbitrary). Care is
Substituting baek in the original equality we get 13p-41+ 21pl= Ip - 41,which is indeed true needed for the case y2 + Y - 1 = O, because after eliminating three variables the two
for any p satisfying O :s p :s 4/3. remaining equations have a factor .¡ + y - 1, and so they are autornatically satisfied. In this


case, we can take any two X¡arbitrary and still get a solution. For example, XI = s, X2= 1, X3
ProblemA2 = - S + yt, X. = - ys - yt, Xs = ys - t.

• Given a point A and a segment BC, determine the locus of all points P in space for which
LAPX = 90° for sorne X on the segment BC.
Problem B2

Prove that cos xl7 - cos 2x17 + cos 3x17 = 1/2.

Solution
Solution
Take the solid sphere on diarneter AB, and the sol id sphere on diarneter AC. Then the locus
is the points in one sphere but not the other (or on the surfaee of either sphere). Given P,
ji

..•

Consider the roots ofx7 + 1 = O. They are ei"Kf7, ei3"Kf7, ••• ,eil3"Kf7 and must have sum zero Solution
since there is no x6 termo Hence, in particular, their real parts sum to zero. But cos7xJ7 = - 1
and the others are equal in pairs, because cos(2lt - x) = cos x. So we get cos 7rl7+ cos 37r17 The condition that a, b, e be the sides of a triangle, together with the appearance of
+ cos 57r17 = 1/2. Finally since cos(lt - x) = - cos x, cos 5xJ7 = - cos 27r17. quantities Iike a + b - e is misleading. The inequality holds for any a, b, e 2: O.

ProblemB3 At most one of (b+c-a), (c+a-b), (a+b-c) can be negative. If one ofthem is negative, then
certainly:
Five students A, B, e, D, E were placed 1 to 5 in a contest with no tieso One prediction was
that the result would be the order A, B, e, D, E. But no student finished in the position abe 2: (b + e - aXc + a - bXa + b - c)(*)
predicted and to two students predicted to finish consecutively did so. For example, the
outcome for e and D was not 1,2 (respectively), or 2,3, or 3, 4 or4, 5. Another prediction since the Ihs is non-negative and the rhs is non-positive.
was the order D, A, E, e, B. Exactly two students finished in the places predicted and two
disjoint pairs predicted to finish consecutively did so. Determine the outcome. (*) is also true ifnone ofthem is negative. For then the arithmetic/geometric mean on b + C
- a, c + a - b gives:
c2 2: (b + e - aXc + a - b).

Solution Similarly for a2 and b2. Multiplying and taking the square root gives (*). Multiplying out
easily gives the required result.
Start from the second prediction. The disjoint pairs can only be: DA, Ee; OC, eB; or AE,
eB. The additional requirement of just two correct places means that the only possibilities ProblemAJ
(in the light ofthe information about the second prediction) are: DABEe, DAeBE,
EDAeB, AEDCB. The first is ruled out because AB are consecutive. The second is ruled Triangle ABe has sides a, b, c. Tangents to the inscribed circle are constructed parallel to
out because e is in the correct place. The fourth is ruled out because A is in the correct the sides. Each tangent forms a triangle with the other two sides of the triangle and a circle
place. This leaves EDAeB, which is indeed a solution. is inscribed in each of these three triangles. Find the total area of all four inscribed circles.

lMO 1964

ProblemAl Solution

(a) Find all natural numbers n for which 7 divides 2" - l.


(b) Prove that there is no natural number n for which 7 divides 2" + l.

Solution

23 = I (mod 7). Hence 23m = I (mod 7), 23m+1 = 2 (mod 7), and 23m+2 = 4 (mod 7). Hence we
never have 7 dividing 2° + 1, and 7 divides 2° - I iff 3 divides n.

ProblemAl
This is easy once you realize that the answer is not nice and the derivation a slog. Use T =
Suppose that a, b, e are the sides oí a triangle. Prove that: 2·area/perimeter and Heron's formula: area k is given by 16k2 = (a + b + cXb + e - aXc + a-
bXa+b-c).

The small triangles at the vertices are similar to the main triangle and smaller by a factor (h
- 2r)/h, where h is the relevant altitude. For the triangle opposite side a: (h - 2rYh = I -
2(2k/p)/(2k/a) = I - 2a/p = (b + c - a)/(a + b + c).



• Hence the total area is «a + b + C)2+ (b + e - al + (e + a - b)2 + (a + b - c)2)/(a + b + cl pi perpendicular to AC, and the through C perpendicular to AB. So we only get one point of

• r= intersection instead ofthree, thus losing another 10.2 = 20 points. These coincidences are
clearly all distinct (the categories do not overlap), so they bring us down to a maximum of
(a2 + b2 + c2).pi.(b + c - aXc + a - bXa + b - c)/(a + b + ci. 435 - 120 =315.

• Problem Bl There is no obvious reason why there should be any further coincidences. But that is not


quite the sarne as proving that there are no more. Indeed, for particular positions of the
Each pair from 17 people exchange letters on one of three topics. Prove that there are at points A, B, C, D, E we can certainly arrange for additional coincidences (the constraints
least 3 people who write to each other on the same topic. [In other words, if we color the given in the problem are not sufficient to prevent additional coincidences). So we have 10

• edges of the complete graph K17with three colors, then we can find a triangle all the sarne prove that it is possible to arrange the points so that there are no additional coincidences. I
color.] cannot see how 10do this, short of exhibiting a particular set of points, which would be
extremely tiresome. Apparently the contestants were instructed verbally that they did not


have to do it,

Solutioo

• Take any persono He writes to 16 people, so he must write to at least 6 people on the same Problem B3


topic. If any of the 6 write to each other on that topic, then we have a group of three writing
10each other on the same topic. So assume they all write 10each other on the other two ABCD is a tetrahedron and Do is the centroid of ABC. Lines parallel to DDo are drawn
topics. Take any of them, B. He must write to at least 3 of the other 5 on the same topic. If through A, B and C and meet the planes BCD, CAD and ABD in Ao, Bo, and Co


two of these write to each other on this topic, then they form a group of three with B. respectively. Prove that the volume of ABCD is one-third ofthe volume of AoBoCoDo.Is
Otherwise, they must all write to each other on the third topic and so from a group of three. the result true if Do is an arbitrary point inside ABC?

Problem B2

5 points in a plane are situated so that no two of the lines joining a pair of points are Solutioo
coincident, parallel or perpendicular. Through each point lines are drawn perpendicular to
each ofthe lines through two ofthe other 4 points. Determine the maximum number of Yes, indeed it is true for an arbitrary point in the plane of ABC not on any of the lines AB,
intersections these perpendiculars can have. BC,CA

Take D as the origino Let A, B, C be the poínts a, b, e respectively. Then Do is pa + qb + re


with p + q + r = 1 and p, q, r> O. So a point on the line parallel to DDo through A is • +

• Solutioo s(p. + qb + re. It is also in the plane DBC if 5= -l/p, so Ao is the point - q/p b - r/p e.
Similarly, Bo is - p/q • - r/q e, and Co is - p/r • - q/r b.

• It is not hard to see that the required number is at most 315. But it is not at all obvious how
you prove it actually is 315, short of calculating the 315 points intersection for a specific
example.
The volume of ABCD is 1/61. x b.e] and the volume of AoBoCoDois 1/61(p. + (q + q/p)b +
(r + r/p)c) x «p + p/q)a + qb + (r + r/q)c).«p + p/r). + (q + q/r)b + re)1

• Call the points A, B, e, D, E. Given one ofthe points, the other 4 points determine 61ines, Thus vol AoBoCoDo/volABCD = abs value ofthe determinant:


so there are 6 perpendiculars through the given point and hence 30 perpendiculars in all,
These determine at most 30.29/2 = 435 points of intersection. But sorne of these necessarily
p q + q/p r + r/p
coincide. There are three groups of coincidences. The first is that the 6 perpendiculars
through A meet in one point (namely A), not the expected 15. So we lose 5.14 = 70 points. p + p/q q r + r/q
JI' Second, the lines through C, D and E perpendicular 10AB are all parallel, and do not give
the expected 3 points of intersection, so we lose another 10.3 = 30 points. Third, the line p + p/r q + q/r r

• through A perpendicular 10BC is an altitude of the triangle ABC, as are the lines through B
which is easily found 10be 2 + P + q + r = 3.




.JI
--------------------

.J
.J

The tetrahedron ABCD is divided into two parts by aplane parallel to AB and CD. The
.J
IMO 1965
distanee ofthe plane from AB is k times its distanee from CD. Find the ratio ofthe volumes
ProblemAl of the two parts.

Find all x in the interval [O, 2nj whieh satisfy:

2 cos x:S 1"(1 + sin 2x) - "(1 - sin 2x)1 :s "2. Solution

Let the plane meet AD at X, BD at Y, BC at Z and AC at W. Take plane parallel to BCD


through WX and let it meet AB in P.
Solution
Since the distance of AB from WXYZ is k times the distanee of CD, we have that AX =
Let y "(1- sin 2x)l. Then y2 = 2 - 21eos Zx]. Ifx belongs to [O, nl4j or
= 1"(1 + sin 2x) -
[3n14, 5n14j or [7n14j, then eos 2x is non-negative, so l = 2 - 2 eos 2x = 4 sin2 x, so y =
k·XD and henee that AXI AD = k/(k+ 1). Similarly AP/ AB = A W/AC = AXI AD. XY is
parallel to AB, so also AXlAD = BYIBD = BZlBC.
21sin x], We have eos x <= [sin x] exeept for x in [O, nl4j and [7n14,2n). So that leaves
{3n14, 5n14j in whieh we certainly have [sin xl S 11"2. vol ABWXYZ = vol APWX + vol WXPBYZ. APWX is similar to the tetrahedron ABCD.
The sides are k/(k+ 1) times smaller, so vol APWX = k'(k+ 1i vol ABCD. The base of the
Ifx belongs (nl4, 3n14) or (5n15,7n14), then eos 2x is negative, so y2 = 2 + 2 cos 2x = 4 prism WXPBYZ is BYZ whieh is similar to BCD with sides k/(k+ 1) times smaller and
cos/x. So y = 2 leos x], So tbe first inequality eertainly holds. The seeond also holds. henee area k2(k+1)2times smaller. Its height is I/(k+ 1) times tbe height of A aboye ABCD,
so vol prism = 3 k2(k+ 1i vol ABCD. Thus vol ABWXYZ = (k3 + 3k2)1(k+1)3vol ABCD.
Thus the inequalities hold for all x in [nl4, 7n14j. We get the vol ofthe other pieee as vol ABCD - vol ABWXYZ and henee the ratio is (after
a little manipulation) k\k+ 3)/(3k+ 1).
ProblemA2

The eoeffieients a;j of the following equations


Problem Bl
al!xl + al2 X2+al3 x, = O
a21XI+ a22X2+ a23X3= O Find all sets of four real numbers sueh that the sum of any one and the product of the other
a31XI+ a32X2+ a33x3= O three is 2.

satisfy the following: (a) al!, a22,a33are positive, (b) other a;j are negative, (e) the sum of
the eoefficients in eaeh equation is positive. Prove that the only solution is XI = X2= X3= O.
Answer

1,1,1,1 or 3,-1,-1,-1.
Solution

The slog solution is to multiply out the determinant and show it is non-zero. A slieker
solution is to take the x, with the largest absolute value. Say IXII2: IX21,IX31.Then looking at Solution
the first equation we have an immediate eontradietion, since the first term has larger
absolute value than the sum ofthe absolute values ofthe seeond two terms. Let the numbers be XI, ... , x,. Let t = XIX2X3X,.
Then XI+ t/x¡ = 2. So all the x, are roots of
the quadratic X2- 2x + t = O. This has two roots, whose produet is t.
ProblemA3
Ifall x, are equal to X,then x3 + X = 2, and we must have X = 1. If not, then if x¡ and X2are
unequal roots, we have XIX2= t and XIX2X3X, = t, so X3X,= l. But X3and x, are still roots of
X2_ 2x + t = O. They cannot be unequal, otherwise X3X,= t, whieh gives t = 1 and hence all



• x¡ = l. Hence they are equal, and hence both I or both -I. Both I gives t = I and al1 Xi = l. The key is that if two segments length d do not intersect then we can find an endpoint of

..•
Both -1 gives t = -3 and hence x, = 3, -1, -1, -1 (in sorne order). one which is a distance > d from an endpoinl of the other.

Problem B2 Given this, the result follows easily by induction, If false for n, then there is a point A in
three pairs AB, AC and AD oflength d (the maximum distance). Take AC lo lie between
The triangle OAB has LO acute. M is an arbitrary poinl on AB. P and Q are the feet of the AB and AD. Now C cannot be in another pairo Suppose it was in cx. Then CX would have


perpendiculars from M lo DA and OB respectively. What is the locus of H, the orthocenter lo cut both AB and AD, which is impossible.
of the triangle OPQ (the point where its altitudes meet)? Whal is the locus if M is allowed
lo vary of the interior of OAB?

Solution


o

• To prove Ihe result about the segments, suppose Ihey are PQ and RS. We must have angle

•..
PQR less than 90°, otherwise PR > PQ = d. Similarly, the other angles ofthe quadrilateral
musl al1 be less than 90°. Contradiction,
A B
IMO 1966
ProblemAl
Let X be the foot of the perpendicular from B lo DA, and y the foot of the perpendicular


from A lo 08. We show that the orthocenter ofOPQ Hes on XY.
Problems A, B and C were posed in a mathematical contesto 25 competitors solved al least
one of the three. Amongst those who did not solve A, twice as many solved B as C. The
MP is parallel lo BX, so AMlMB = AP/Px. Let H be the intersection of XY and Ihe
number solving only A was one more than the number solving A and al least one other. The


perpendicular from P lo OB. PH is paral1el lo A Y, so AP/PX = YHlHX. MQ is parallel lo
number solvingjusI A equalled the number solvingjusI B plus the number solvingjusI C.
A Y, so AMlMB = YQIBQ. Hence YQIBQ = YHlHX and so QH is paral1el lo BX and
How many solved just B?
hence perpendicular lo AO, so H is the orthocenter of OPQ as claimed.

• If we restrict M lo lie on a line A'B' paral1el lo AB (with A' on DA, B' on OB) then the
locus is a line X'Y' parallel to XY, so as M moves over the whole interior, the locus is the


Answer
interior ofthe triangle OXY.
6.


ProblemB3

Given n > 2 points in the plane, prove that at most n pairs of points are the maximum


distance apart(ofany two points in the set),
Solution

Let a solve just A, b solve just B, e solve just C, and d solve B and C but not A. Then 25 - a
- b - e - d solve A and al least one of B or C. The conditions give:
JI' Solution


b+d=2(c+d);a= I +25-a-b-c-d;a=b+c.



•.. 25
Eliminating a and d, we get: 4b + e = 26. But d = b - 2c 2:: O, so b = 6, e = 2. Prove that l/sin 2x + l/sin 4x + ...+ l/sin 2"x = cot x - cot 2"x for any natural number n and
any real x (with sin 2"x non-zero).

ProblemA2
Solution
Prove that if BC + AC = tan C/2 (BC tan A + AC tan B), then the triangle ABC is isosceles.
cot y - cot 2y = cos y/sin y - (2 cos'y - 1)/(2 sin y cos y) = 1/(2 sin y cos y) = IIsin 2y. The
result is now easy. Use induction. True for n = l (just take y = x). Suppose true for n, then
taking y = 2"x, we have l/sin 2n+lx = cot 2"x - cot 2n+lxand result follows for n + 1.
Solution
Problem B2
A straight slog works. Multiply up to get (a + b) cos A cos B cos C/2 = a sin A cos B sin
C/2 + b cos A sin B sin C/2 (where a = BC, b = AC, as usual). Now use cosCA + C/2) = cos Solve the equations:
A cos C/2 - sin A sin C/2 and similar relation for cos (B + C/2) to get: a cos B cosCA + C/2)
+ b cos A cos (B + C/2) = O. Using C/2 = 90° - N2 - B/2, we find that cosCA + C/2) = - la¡- all XI + la¡- a21X2+ la¡- a31X3+ la¡ -141'4 = 1, i = 1,2,3,4, where a¡ are distinct
cos(B + C/2) (and = O only if A = B). Result fOllOWS. reals.

ProblemA3 Answer

Prove that a point in space has the smallest sum of the distances lo the vertices of a regular XI = I/(al -14), X2= X3= O, '4 = I/(al -14).
tetrahedron iff it is the center of the tetrahedron.

Solution
Solution
Take al > a2 > a, > 14. Subtracting the equation for i=2 from that for i=I and dividing by (al
Let the tetrahedron be ABCD and let P be a general point. Let X be the midpoint ofCD. - a2) we get:
Let P' be the foot ofthe perpendicular from P lo the plane ABX. We show that ifP does not
coincide with P', then PA + PB + PC + PD > P'A + P'B + P'C + P'D.

PA > P'A (because angle PP'A = 90°) and PB > P'B, P'CD is isosceles and PCD is not but P Subtracting the equation for i=4 from that for i=3 and dividing by (a3 - 14) we get:
is the sarne perpendicular distance from the line CD as P'. It follows that PC + PD > P'C +
P'D. The easiest way to see this is to reflect e and D in the line PP' to give C' and D'. Then
PC = PC', and PC' + PD > C'D = P'C' + P'D = P'C + P'D.
Hence XI= '4. Subtracting the equation for i=3 from that for i=2 and dividing by (a2 - a3)
So if P has the smallest sum, it must lie in the plane ABX and similarly in the plane CDY, weget:
where Y is the midpoint of AB, and hence on the line XY. Similarly, it must lie on the line
joining the midpoints of another pair of opposite sides and hence must be the center. - XI - X2+ X3+ '4 = o.

Hence X2= X3= O, and XI = '4 = l/(al -14).

ProblemBl Problem B3



• Take any points K, L, M on the sides BC, CA, AB ofthe triangle ABC. Prove that at least

•.. one ofthe triangles AML, BKM, CLK has area:S 1/4 area ABC.
)
--- ,
....
......
Solution
~--++-~c )
A

.. &
" ........__/"/

Evidently the parallelogram is a red herring, since the circles cover it iff and only ir the
three circles center A, B, o cover the triangle ABO .
B K C
The three circles radius X and centers the three vertices cover an acute-angled triangle ABO

•.. If not, then considering ALM we have 4·AL·AM·sin A > AB·AC·sin A, so 4·AL·AM >
AB'AC = (AM + BMXAL + CL), so 3·AL·AM > AM'CL + BM·AL + BM·CL. Set k =
BKlCK, I = CUAL, m = AMIBM, and this inequality becomes:

3 > 1+ l/m + 11m.


iffx is at least R, the circurnradius. The circurncenter O is a distance R from each vertex so
the condition is clearly necessary. Ifthe midpoints ofBO, OA, AB are P, Q, R, then the '
circle center A, radius R covers the quadrilateral AQOR, the circle center B, radius R
covers the quadrilateral BROP, and the circle center O radius R covers the quadrilateral
OPOQ, so the condition is also sufficient.

• Similarly, considering the other two triangles we get: 3> k + 111+ kll, and 3 > m + l/k +
mlk.

Adding gives: 9> k + I + m + Ilk + 111+ l/m + kll + 11m+ mlk, which is false by the
We need an expression for R in terms ofaand A. We can express BO two ways: 2R sin A,
and ...J(a2+ I - 2a cos A). So a necessary and sufficient condition for the covering is 4 sin2 A
2: (a2 + I - 2a cos A), which reduces to a:s cos A + ...J3 sin A, since cos A:S a (the foot ofthe
perpendicular from O onto AB must lie between A and B).
arithmetic/geometric mean inequality.
Problem2
IMO 1967
Prove that a tetrahedron with just one edge length greater than I has volurne at most 1/8.

• ProblemAl

•.. The parallelogram ABCO has AB = a, AO = 1, angle BAO = A, and the triangle ABO has
all angles acute. Prove that circles radius I and center A, B, C, O cover the parallelogram
iff

a:s cos A + ...J3 sin A.


Solution


•...
Solution
D

Let the tetrahedron be ABCO and assume that all edges except AB have length at most l.
The volurne is the 1/3 x area BCO x height of A aboye BCO. The height is at most the
height of A aboye CO, so we maximise the volurne by taking the planes ACO and BCO to

• be perpendicular. If AC or AO is less than 1, then we can increase the altitude from A to



•..
CD whilst keeping BCD fixed by taking AC ; AD ; l. A similar argument shows that we
must have BC ; BD; l.
Solutioo
But the volume is also the 1/3 x area ABC x height of D aboye ABC, so we must adjust CD
to maximise this height. We want the angle between planes ABC and ABD to be as close as
e.._----- __.
possible to 90°. The angle increases with increasing CD until it becomes 90°. CMD is then
a right-angled triangle. Now the angle ACB must be less than the angle between the planes /'
z "
"". \.....,',
\
"- ,\
ACD and BCD and hence < 90°, so angle ACM < 45°, so CM > IfV2. Similarly DM. Hence '-, '¡.oD
when CMD ; 90° we have CD > l. Thus we maximise the height of D aboye ABC by
taking CD ; l.

So BCD is equilateral with area (V3)/4. ACD is also equilateral with altitude (V3)f2. Since
the planes ACD and BCD are perpendicular, that is also the height of A aboye BCD. So the
volume is 1/3 x(V3)/4 x (V3)/2 ; 1/8.

ProblemA3

Let k, m, n be natural numbers such that m + k + 1 is a prime greater than n + l. Let e, ;


s(s+ 1). Prove that:

Take any triangle similar 10AIBICI and circumscribing AoBoCo.For example, take an
arbitrary line through Ao and then lines through Bo and Co at the appropriate angles to the
is divisible by the product ClC2... Cn. first line. Label the triangle's vertices X, Y, Z so that Ao lies on YZ, Bo on ZX, and Co on
XY. Now any circumscribed ABC (Iabeled with the same convention) must have C on the
circle through Ao, Bo and Z, because it has LC; LZ; LCI. Similarly it must have B on

...
the circle through Co, Ao and Y, and it must have A on the circle through Bo, Co and X.
Solutioo
Consider the side AB. It passes through Co. Its length is twice the projection ofthe line
The key is that Ca- es= (a - bXa + b + 1). Hence the product (Cm+l- CkXCm+2 - Ck)... (Cm+n-
Ck) is the product ofthe n consecutive numbers (m - k + 1), ... ,(m - k + n), times the
product ofthe n consecutive numbers (m + k + 2), ... , (m + k + n + 1). The first product is
just the binomial coefficient (m-k+n)Cn times ni, so it is divisible by n!. The second
product is If(m + k + 1) x (m + k + 1Xm + k + 2) ... (m + k + n + 1) ; I/(m + k + 1) x
joining the centers of the two circles onto AB (because each center projects onto the
midpoint of the part of AB that is a chord of its circle). But this projection is maximum
when it is parallel to the line joining the two centers. The area is maximised when AB is
maximised (because al! the triangles are similar), so we take AB parallel to the line joining
...
the centers. [Note, in passing, that this proves that the other sides must also be parallel to
(m+k+n+I)C(n+I) x (n+I)!. But m + k + 1 is a prime greaterthan n + 1, so it has no factors
the lines joining the respective centers and hence that the three centers form a triangle
in common with (n+ 1)!, hence the second product is divisible by (n+ 1)!. Finally note that
similar to AIBICt-)
ClC2... Cn; n! (n+ 1)!.
ProblemB2

al, ... ,ag are reals, not al! zero. Let cn; a¡" + a2n+ ... + agnfor n= 1,2,3, .... Given that an
Problem Bl
infinite number of Cnare zero, find al! n for which Cnis zero.
AoBoCoand AIBICI are acute-angled triangles. Construct the triangle ABC with the largest
possible area which is circumscribed about AoBoCo(BC contains Ao, CA contains Bo, and
AB contains Col and similar to AIBICI.
Solutioo



•• Take lall ~ la21~ ... ~ lasl.Suppose that lall, ... , la.! are all equal and greater than 1a,...11.
for sufficiently large n, we can ensure that 111,,1"
Then
< 1/81all" for s > r, and hence the sum of 111,,1"
for all s > r is less than lalln.Hence r must be even with half of al, ... , a. positive and half
Using the cosine rule, we find cos A = (a+5)1(2a+4), cos B = (a+I)I2a, eos e = (a-3)12a.
FinaHy, using cos 2x = 2 cos/x - 1, we find solutions a = 4 for e = 2A, a = 1 for B = 2A,
and no solutions for e = 2B.

••
negative,
a = 1 is a degenerate solution (the triangle has the three vertiees collinear), The other
If that does not exhaust the ar, then in a similar way there must be an even number of a¡ with solution is 4, 5, 6.
the next largest value ofla¡l, with halfpositive and halfnegative, and so on, Thus we find
that en = Ofor all odd n.
ProblemAl

• Problem B3

In a sports eontest a total of m medals were awarded over n days. On the first day one Find all natural numbers n the produet of whose decimal digits is n2 - IOn - 22.

• medal and 117 ofthe remaining medals were awarded. On the second day two medals and
1/7 of the remaining medals were awarded, and so on. On the last day, the remaining n
medals were awarded. How many medals were awarded, and over how many days?

• Solution


Suppose n has m > 1 digits. Let the first digit be d. Then the produet of the digits is at most
d.9m-1< d.lOm-1<= n. But (n2 - IOn - 22) - n = n(n - 11) - 22 > Ofor n >= 13. So there are no
Solutian
solutions for n ~ 13. But n2 - IOn - 22 < Ofor n::: 11, so the only possible solution is n = 12

•... Let the number of medals remaining at the start of day r be m¿ Then mi = m, and 6(mk - and indeed that is a solution.
k)17 = mk+1for k < n with mn = n.

After a little rearrangement, we find that m = 1 + 2(7/6) + 3(7/6i + ... + n(7/6)"-I .


Summing, we ~et m = 36(1 - (n + IX7/6)· + n (716)·+1) = 36 + (n - 6)7n/61>-1.6 and 7 are ProblemAJ

••
eoprime, so 6n- must divide n - 6. But 61>-1> n - 6, so n = 6 and m = 36.
a, b, e are real with a non-zero. XI,X2,... , Xnsatisfy the n equations:

ax? + bx, + e = Xi+t.for I :::i < n


IMO 1968
ax.2 + bx, + e = XI

iI Prove that the system has zero, 1 or > 1 real solutions according as (b - 1)2- 4ac is <O, =O or
>O.


ProblemAl

Find all triangles whose side lengths are conseeutive integers, and one of whose angles is


twiee another.
Solution

Let ~x) = ax2+ bx + e - x. Then f(x)la = (X+ (b-I)I2ai + (4ac - (b-li)l4a2. Hence if 4ac-
(b-l ) > O,then f(x) has the same sign for all x, But f(x) > Omeans ax2+ bx + C > X, so if
Solutian
{x.} is a solution, then either XI< X2< ... < X"< XI,or XI> X2> ... > X. > XI.Either way we
Let the sides be a, a+ 1, a+2, the angle oppose a be A, the angle opposite a+ 1 be B, and the have a contradiction. So if 4ac - (b-I i > Othere cannot be any solutions.
angle opposite a+2 be C.

••
'

'la
.••
If 4ac - (b-I i = 0, then we can argue in the same way that either XI ~ X2~ ... ~ Xn~ XI, or XI
°
2: X22: ... 2: Xn2: XI·SOwe must have all x, = the single root off(x) = (which c1early is a
solution).
So f(x+2a) = 112+ -.J( f(x+a) - f(x+a)2 ) = 112+ -.Jf(x+a)-.J(I- flx+a)
f\X)2) = 1/2 + (f\x) - 1/2) = f(x). So f is periodic with period 2a
= 112+ -.J(1I4 - f\x) +

i °
If 4ac - (b-I < 0, then f\x) = has two distinct real roots y aod z and so we have at least
We may take f(x) to be arbitrary in the interval [0,1). For example, let f(x) = 1 for °~
X < 1,

....
f\x) = 112for 1 ~ X < 2. Then use f(x+2) = f(x) to define f(x) for all other values ofx.
two solutions to the equations: all x, =y, and all x, = z. We may, however, have additional
solutions. For example, if a = 1, b = 0, e = -1 and n is even, then we have the additional
ProblemB3
solution XI = X) = Xs = ... = 0, X2= l4 = ... = -l.
For every natural number n evaluate the sum

[(n+I)I2] + [(n+2)/4] + [(n+4)/8] + ... + [(n+2k)l2k+l] + ... ,where [x] denotes the greatest
integer ~ x.
Problem 81

Prove that every tetrahedron has a vertex whose three edges have Ihe right lengths to form a
triangle.
Solutioo

Solutioo and °~
For any real X we have [x] = [x/2] + [(x+I]/2]. For ifx = 2n + 1 + k, where n is an integer
k < 1, then Ihs = 2n + 1, and rhs = n + n + 1. Similarly, ifx = 2n + k.

The trick is to consider the longest side. That avoids getting into lots of different possible
1
Hence for any integer n, we have: [n/2k]_ [n/2k+1] = [(nI2k + 1)12]= [en + 2k)/2k+1 Hence
cases for which edge is longer than the sum of the other two. summing over k, and using the fact that n < 2k for sufficiently large k, so that [n/2 ] = 0,
we have: n = [en + 1)/2] + [en + 2)/4] + [en + 4)/8] + ...
So assume the result is false and let AB be the longest side. Then we have AS > AC + AD IMO 1969
and SA > SC + SO. So 2AS > AC + AD + SC + SC. But by the triangle inequality, AS <
AC + CS, AS < AO + OS, so 2AS < AC + CS + AO + Os. Contradiction.
ProblemAI

..
Prove that there are infinitely many positive integers m, such that n4 + m is not prime for
Problem 82 any positive integer n.

Let fbe a real-valued function defined for all real numbers, such that for sorne a > we
have
°
Solutioo
f(x + a) = 1/2 + -.J(f(x)- f\xi) for all x.
n4 + 4 r4 = (n2 + 2m + 2~Xn2 - 2m + 2~). Clearly the first factor is greater!han 1, the
Prove that f is periodic, and give an example of such a non-constant f for a = l. second factor is (n - r)2 + ~, which is also greater than 1 for r greater than l. So we may
take m = 4 r4 for any r greater!han 1.


ProblemAl
Solutioo

..
Let f(x) = cos(al + x) + 1/2 cos(a2 + x) + 1/4 costa, + x) + ... + 1120-1cosra, + x), where a¡
Oirectly from the equality given: f(x+a) 2: 1/2 for all x, and hence f(x) 2: 112for all x. are real constants and X is a real variable. If f(XI) = f(X2)= 0, prove that XI - X2is a multiple
of x.
-..1
••

• Problem Bl


SolutioD

fis not identieally zero, because f(-al) = 1 + 1/2 eos(a2 - a,) + ... > 1 - 1/2 - 1/4 - ... - 1/20-' > C is a point on the semieircle diameter AB, between A and S. D is the foot ofthe
perpendicular from C to AB. The eircle K, is the in-eircle of ABC, the eircle K2 touehes


O.
CD, DA and the semieircle, the eircle K3 touches CD, DB and the semicircle. Prove that
Using the expression for eos(x + y) we obtain f(x) = b eos x + e sin x, where b = cos a, + K" K2 and K3 have another eommon tangent apart from AB.


1/2 eos a2 + ...+ 1/2"-' eos a." and e = - sin a, - 1/2 sin a2 - ... - 1/2"-' sin anob and e are not
both zero, since fis not identieally zero, so f(x) = -.I(b2+ e2) eos(d + x), where eos d =
b/-.l(b2+ e2), and sin d = e/-.l(b2 + e2). Henee the roots of f(x) = O are just mn: + n:l2- d.

•..
SolutioD
ProblemAJ

For each ofk = 1,2,3,4,5 find necessary and suffieient eonditions on a > O sueh that there
exists a tetrahedron with k edges length a and the remainder length l.



SolutioD xo P o v le
I

A plodding question. Take the tetrahedron to be ABCD. //


Take k = 1 and AB to have length a, the other edges length l. Then we can hinge triangles
ACD and BCD about CD 10vary AS. The extreme values evidendy oceur with A, B, C, D
eoplanar. The least value, O, when A coincides with B, and the greatest value -.13,when A
and B are on opposite sides of CD. We rule out the extreme values on the grounds that the Let the three eenters be O" 02 and 03. We show that O, is the midpoint Of0203. In faet it
tetrahedron is degenerate, thus obtaining O < a < -.13. is suffieient 10 show that O, Iies on 0203, because then we can refleet the known tangent
AB in the line 0203.
For k = 5, the same argument shows that O < 1 < -.13a, and henee a> 1/-.13.
As usual, let AB = e, BC = a, CA = b. Let the in-circle toueh AB at P, AC at Q and BC at

.. For k = 2, there are two possible eonfigurations: the sides length a adjacent, or not,
Consider first the adjacent case. Take the sides length a to be AC and AD. As before, the
two extreme cases gave A, B, C, D eoplanar. If A and B are on opposite sides of CD, then a
= -.1(2- -.13).If they are on the same side, then a = -.1(2+ -.13).So this eonfiguration allows
any a satisfying -.1(2- -.13)< a < -.1(2+ -.13).
R. Then since angle ACB = 90, O,QCR is a square. Also AQ = AP and BP = BR. so r¡ = b
- AP, and r¡ = a - BP = a - (e - AP). Adding: r¡ = (a + b - e)l2, and AP = (b + e - a)/2.

Let the eircle eenter 02 toueh AB at X, and the eircle eenter 03 toueh AB at Y. Let O be the
midpoint of AS. Now eonsider the right-angled triangle OX02. Since the eircle eenter O2


touehes the semieircle, 002 = cl2 - r2. OX = OD + DX = (e/2 - AD) + r2. Also, by similar
The other eonfiguration has AB = CD = a. One extreme case has a = O. We can inerease a triangles, AD = b2/e. So, using Pythagoras: (e/2 - rd = rl + (cl2 - b2/e + rd. Multiplring
out and rearran~ing: r22- 2r2(e - b2/e) - (b2 - b'/e2). But ABC is right-angled, so e2 = a + b2,


until we reach the other extreme case with ADBC a square side 1, giving a = -.12.So this
eonfiguration allows any a satisfying O < a < -.12.Together, the two eonfigurations allow and henee e - b le = a2/e and b2 - b'/e2 = a2b2/e2.So rl + 2r2 a2/e - a2b2/e2= O, whieh has
roots r2 = a - a2/e (positive) and - a + a2/e (negative). So r2 = a - a2/e. Similarly, r3 = b - b2/e.
any a satisfying: O < a < "(2 + -.13).
So 02X + 03 Y = XY = rl + r3 = a + b - e = 2 r,.

••• This also solves the case k = 4, and allows any a satisfying: a> 11"(2 + -.13)= -.1(2- -.13).

For k = 3, any value of a> O is allowed. For a <= 1, we may take the edges length a to form
a triangle. For a 2: 1 we may take a triangle with unit edges and the edges joining the
vertices to the fourth vertex to have length a,
XP = AP - AX = AP - (AD - DX) = (b + e - a)l2 - (b2/e - r2) = (b + e - a)l2 - (e - a)
e)/2 = r,. We now have all we need: XP = PY = PO" and X02 + Y03 = 2 PO,.

ProblemB2
= (a + b-





..J

Given n > 4 points in the plane, no three collinear. Prove that there are at least (n-3Xn-4 )/2
convex quadrilaterals with vertices amongst the n points.
We also use (z,/y, - ZzlY2)2 ~ O. Now x,y, > Z,2 ~ O, and x¡ > O, so y, > O. Similarly, Y2> O.
So:

Sclution

(n-3Xn-4)f2 is a poor lower bound.

Observe first that any 5 points inelude 4 forming a convex quadrilateral. For take the
convex hull. If it consists of more than 3 points, we are done. If not, it must consist of 3
points, A, B and C, with the other 2 points, D and E, inside the triangle ABC. Two vertices Adding (1) and 2a,a2 gives: 8a,a2:S (a, + ad + (a, + a2Xx,Y2+ X2Y,- 2Z,Z2) = a(a, + a2),
of the triangle must lie on the sarne side of the line DE and they form convex quadrilateral where a = (x¡ + X2Xy,+ Y2)- (z¡ + zd. Dividing by a,a2a gives the required inequality.
wilhDand E.
Equality requires a, = a2 from {I), y, = Y2from (2), z, = Z2from (3), and hence x¡ = X2.
Given n points, we can choose 5 in n(n-IXn-2Xn-3)(n-4)/120 different ways. Each choice Conversely, it is easy to see that these conditions are sufficient for equality.
gives us a convex quadrilateraJ, but any given convex quadrilateraJ may arise from n-4
different sets of 5 points, so we have at least n(n-I Xn-2Xn-3)/120 different convex
quadrilateraIs. We now show that n(n-IXn-2Xn-3)f120 ~ (n-3Xn-4)f2 for aIl n ~ 5.
IMO 1970
We wish to prove that n(n-I)(n-2) ~ 60(n-4), or n(n-IXn-2) - 60(n-4) ~ O. Trial shows
equality for n = 5 and 6, so we can factorise and get (n-5Xn-6Xn+8), which is elearly at ProblemAl
least O for n at least 5.
M is any point on lhe side AB of the triangle ABC. r, r" r2are the radii of the cireles
inscribed in ABC, AMC, BMC. q is the radius of the circle on the opposite side of AB to C,
touching the three sides of AB and the extensions of CA and CB. Similarly, qi and qi,
Problem B3 Prove that r,r2q = rqiqi.

Given real numbers xi, X2,y" Y2,Z" Z2,satisfying x¡ > O, X2> O, x,y, > Z,2, and X2Y2> zl,
prove that:
Solutíon

Give necessary and sufficient conditions for equality.

We need an expression for r/q. There are two expressions, one in terms of angles and the
other in terms of sides. The latter is a poor choice, because it is both harder to derive and
Selution less useful. So we derive the angle expression.

Let a, = xiyr - Z,2 and a2 = X2Y2- zl We apply the arithmetic/geometric mean result 3 Let 1 be the center of the in-cirele for ABC and X the center of the extemal cirele for ABC.
times: lis the intersection of the two angle bisectors from A and B, so e = r (cot N2 + cot B/2).
The X lies on the bisector ofthe external angle, so angle XAB is 90° - N2. Similarly, angle
XBA is 90° - B/2, so e = q (tan N2 + tan B/2). Hence r/q = (tan N2 + tan B/2)/(cot A/2 +
cot B/2) = tan N2 tan B/2.
-..J
Applying this to the olher two triangles, we get rdq, = tan N2 tan CMN2, r2/q2 = tan B/2
tan CMB/2. But CMB/2 = 90° - CMN2, so tan CMB/2 = litan CMN2. Hence result. .J
..J
..J
~.



• ProblemAl SolutioD

• We have O:s x, < b for i = O, 1, ... ,n and Xo> O, Xo_1> O. Ifa > b, and XnXn-I
the number A base a and B base b, whilst Xo-IXn-2
...Xo represents
...Xo represents the number A' base a and
The only primes dividing numbers in the set can be 2, 3 or 5, because if any larger prime
was a factor, then it would only divide one number in the set and hence only one product.


B' base b, prove that A'B < AB'. Three of the numbers must be odd. At most one of the odd numbers can be a multiple of3
and at most one can be a multiple of 5. The other odd number cannot have any prime
factors. The only such number is 1, so the set must be {1, 2, 3, 4, 5, 6}, but that does not
work because only one of the numbers is a multiple of5. So there are no such sets.
Selutíon

• We have a'b" > bOamfor n > m. Hence aOB'> bOA'.Adding a"b° to both sides gives aOB>
bOA.Hence x.a"B > xob°A. But x.a° = A - A' and xobo = B - B', so (A - A')B > (B - B')A. Problem B2


Hence result.
In the tetrahedron ABCD, angle BOC = 90° and the foot ofthe perpendicular from D to
Note that the only purpose ofrequiring Xo_1> O is to prevent A' and B' being zero. ABC is the intersection ofthe altitudes of ABC. Prove that:

• ProblemA3

•.. The real numbers 30,a" a2, ... satisfy I = 30 <= al :s a2 <= .... b" b2, i>J, ... are defined by bo
= sum for k = I 10n of (1 -1It.-I/IIt.)I.JI4..

(a) Prove that O:s bo < 2.


When do we have equality?

SoIutioD
(b) Given e satisfying O:Se < 2, prove that we can find a. so that bo > e for a11sufficiently
large n. The first step is to show that angles ADB and AOC are a1so 90°. Let H be the intersection
ofthe altitudes of ABC and let CH meet AB at X. Planes CED and ABC are perpendicular
and AB is perpendicular to the line of intersection CE. Hence AB is perpendicular 10the
ji plane CDE and hence to ED. So BD2 = DE2 + BE2. Also CB2 = CE2 + BEl Subtracting:
CB2 _ BD2 = CE2 _ DE2. But CB2 _ BD2 = CD2, so CE2 = CD2 + DE2, so angle CDE = 90°.
Solution


But angle CDB = 90°, so CD is perpendicular to the plane DAB, and hence angle CDA =
(a) Each term of tbe sum is non-negative, so b, is non-negative. Let ct = .J1It.. Then the kth 90°. Similarly, angle ADB = 90°.
term = (1 - 14.-1/I4.)I.J1It. = Ck_121ck - 1/1It.) = Ck_12/ct(1/ck_1+ I/ckXl/ck_1- IICk).But Ck_


(1I1It._1
12/ck(I/Ck-1+ IICk):S2, so the kth term:S 2(I/ck_1- I/ck). Hence bo <= 2 - 2/co < 2. Hence AB2 + BC2 + CA2 = 2(DA2 + DB2 + OC2). But now we are done, because Cauchy's
inequality gives (AB + BC + CA)2:s 3(AB2 + BC2 + CA2).


(b) Let Ck= dk, where d is a constant > 1, which we will choose later. Then the kth term is
(1 - I/d2)l/dk, so bu = (1 - IId2XI - IIdo+I)I(1 - lId) = (1 + IIdXI - I/dn+I). Now take d We have equality iffwe have equality in Cauchy's inequality, which means AB = BC =
sufficiently close 10 I that I + lId> e, and then take n sufficiently large so that (I + I/dXI - CA.

•.-
IIdn+l) > C.

Problem Bl
ProbIem B3
Find all positive integers n such that the set {n, n+ 1, n+2, n+3, n+4, n+5} can be partitioned
Given 100 coplanar points, no 3 collinear, prove that at most 70% of the triangles formed


into two subsets so that the product of the numbers in each subset is equal.
by the points have a11angles acute.




the last two tenns is: (14 - as){(a¡ - aSXa2- aSXa3- as) - (a¡ -14Xa2 -14Xa3 -14)} ;:::O. Hence
Es;:::O.
Solution

Improved and corrected by Gerhard Woginger, Technical University Graz


ProblemA2
At most 3 of the triangles fonned by 4 points can be acute. It follows that at most 7 out of
the lO triangles fonned by any 5 points can be acute. For given lO points, the maximum no. Let P¡ be a convex polyhedron with vertices Al, A2, ... , A9. Let P, be the polyhedron
of acute triangles is: the no. of subsets of 4 points x 3/the no. of subsets of 4 points obtained from P¡ by a translation that moves A¡ to A¡. Prove that at least two ofthe
containing 3 given points. The total no. oftriangles is the same expression with the first 3 polyhedra PI, P2, ... , P9 have an interior point in common.
replaced by 4. Hence at most 3/4 ofthe lO, or 7.5, can be acute, and hence at most 7 can be
acute.

The same argument now extends the result to 100 points. The maximum number of acute Solution
triangles fonned by 100 points is: the no. of subsets of 5 points x 7/the no. of subsets of 5
points containing 3 given points. The total no. of triangles is the same expression with 7 The result is false for 8 vertices - for example, the cube. We get 8 cubes, with only faces in
replaced by lO. Hence at most 7/10 ofthe triangles are acute. common, fonning a cube 8 times as large.

This suggests a trick. Each p¡ is contained in O, the polyhedron fonned from P¡ by doubling
the scale. Take A¡ as the origin and take the vertex B¡ to have twice the coordinates of A¡.
IMOl971 Given a point X inside PI. the midpoint of P¡X must lie in P¡ by convexity. Hence the point
with doubled coordinates, which is obtained by adding the coordinates of A¡ to the
ProblemAl coordinates ofX, lies in o. In other words every point of'P, lies in O. But the volume of D
is 8 times the volume of Pi, which is less than the sum ofthe volumes of'Pj, ... ,P9.
Let En = (al - a2Xa¡ - a3) ... (a¡ - a.) + (a2 - a¡Xa2 - a3) ... (a2 - a,) + ... + (ao - a¡Xao - a2) ... (ao
- 8n-¡).Let So be the proposition that En;::: O for all real a, ProblemAJ

Prove that So is true for n = 3 and 5, but for no other n > 2. Prove that we can find an infinite set ofpositive integers ofthe fonn 20 - 3 (where n is a .1

Solution
positive integer) every pair ofwhich are relatively prime.
...
Solution
Take a¡ < O,and the remaining a¡ = O.Then En = a¡n-¡ < O for neven, so the proposition is
false for even n. We show how to enlarge a set ofr such integers to a set ofr+ l. So suppose 2°¡ - 3, ... , 2°r-
3 are all relatively prime. The idea is to find 2' - 1 divisible by m = (2'¡ - 3) ... (2'r - 3),
Suppose n;:::7 and odd. Take anl e > a > b, and let a¡ = a, a2 = a, = 14= b, and as = lI6= because then 2' - 3 must be relatively prime to all ofthe factors ofm. At least two of2°, 2¡,
proposition is false for odd n > 7.
a. = c. Then En = (a - b)3(a - c)' < O. So the ... ,2m must be congruent mod m. So suppose m¡ > m2 and 2m¡ = 2m2 (mod m), then we
must have 2m¡ -m2 - 1 = O (mod m), since m is odd. So we may take nr+¡to be m¡ - m2.
Assume a¡ ;:::a2;:::a3. Then in E3 the sum ofthe first two tenns is non-negative, because (a¡ -
a3);:::(a2 - a3). The last tenn is also non-negative. Hence E3 ;:::O, and the proposition is true Problem Bl
for n = 3.
AII faces ofthe tetrahedron ABCO are acute-angled. Take a point X in the interior ofthe
It remains to prove Ss. Suppose a¡ ;:::a2;:::a3;:::14;:::as. Then the sum ofthe first two tenns in segment AB, and similarly Y in BC, Z in CO and T in AD.
Es is (a¡ - a2){(a¡ - a3Xa¡ -14Xa¡ - as) - (a2 - a3Xa2-14Xa2 - as));::: O. The third tenn is non-
negatíve (the first two factors are non-positive and the last two non-negative). The sum of



• *
(a) If L DAB + L BCD L CDA + L ABC, then prove that none of the closed paths constraints on 1Im+1are that we do not have IClal + C2a2+ ... + Cmllm+ 211m+11 equal to O or I

• XYZTX has minimal length;

(b) If L DAB + L BCD = L CDA + LABC, then there are infinitely many shortest paths
XYZTX, each with length 2 AC sin k, where 2k = L BAC + L CAD + L DAB.
for any C¡= O,2 or -2, apart from the trivial cases of all C¡= O. Each II = O rules out a single
pomt and each II = I rules out a circle which intersects the circle radius 1/2 about the origin
at 2 points and hence rules out two points. So the effect of the constraints is to rule out a
finite number of points, whereas we have uncountably many to choose from.

Problem B3

Let A = (lI¡j),where i, j = 1, 2, ... , n, be a square matrix with all aij non-negative integers.
Solution For each i, j such that lI¡j= O,the sum of the elements in the ith row and the jth column is at
least n. Prove that the sum of all the elements in the matrix is at least n212.
The key is to pretend the tetrahedron is made of cardboard, cut it along three edges and


unfold it. Suppose we do this to get the hexagon CAC'BDB'. Now the path is a line joining
y on B'C to Y' on the opposite side BC' of the hexagon. Clearly this line must be straight
for a minimal path. If B'C and BC' are parallel, then we can take y anywhere on the side

•..
Solution
and the mini mal path length is the expression given.
Let x be the smallest row or column sumo Ifx >= n/2, then we are done, so assume x < n/2.
But ifthey are not parallel, then the minimal path will come from an extreme position.
Suppose it is a row. (If not, interchange rows and columns.) The number of non-zero
Suppose CC' < BB'. Ifthe interior angle CAC' is less than 180·, then the minimal path is
elements in the row, y, must also satisfy y < n/2, since each non-zero element is at least 1.
obtained by taking y at C. But this does not meet the requirement that y be an interior
Now move across this row summing the columns. The y columns with a non-zero element
point of the edge, so there is no minimal path in the permitted set. If the interior angle CAC'
have sum at least x (by the definition ofx). The n-y columns with a zero have sum at least
is greater than 180, then the minimal path is obtained by taking X and T at A. Again this is
n-x. Hence the total sum is at least xy + (n - xXn - y) = n2/2 + (n - 2xXn - 2yy2 > n2/2.
not permitted.
The result is evidently best possible, because we can fill the matrix altemately with zeros
The problem therefore reduces to finding the condition for B'C and BC' to be parallel. This
and ones (so that a¡j = I ifi andj are both odd or both even, O otherwise). For neven, every
is evidently angles BCD + DCA + CAD + BAD + BAC + ACB = 360·. But DCA + CAD =
row and column has nI2 I s, so the condition is certainly satisfied and the total sum is n2/2.

..
180· - AOC, and BAC + ACB = 180· - ABC, so we obtain the condition given.
Forn odd, odd numbered rows have (n+lY2 Is and even numbered one less. But the only
zeros are in positions which have either the row or the column odd-numbered, so the sum in
such cases is n as required. The total sum is n2/2 + 1/2. Altematively, for neven, we could
place n/2 down the main diagonal.
Problem 82

•..
IMO 1972
Prove that for every positive integer m we can find a finite set S of points in the plane, such
that given any point A of S, there are exactly m points in S at unit distance from A.
ProblemAl

.. Solution

Take al. a2, ... ,am to be points a distance 1/2 from the origin O. Form the set of2m points
Given any set often distinct numbers in the range lO, 11, ... ,99, prove that we can always
find two disjoint subsets with the same sumo

••• ±al ±a2 ± ... ±1Im.Given such a point, it is at unit distance from the m points with just one
coefficient different. So we are home, provided that we can choose the lI¡to avoid any other
pairs ofpoints being at unit distance, and to avoid any degeneracy (where sorne ofthe 2m
points coincide) .
Solution

The number of non-empty subsets is 210- I = 1023. The sum of each subset is at most 90 +
... + 99 = 945, so there must be two distinct subsets A and B with the same sumo A \ B and
B \ A are disjoint subsets, also with the same sumo


The distance between two points in the set is IClal + C2a2+ ... + cmllml,where C¡= O, 2 or -2.
So let us choose the lI¡inductively. Suppose we have already chosen up to m. The



ProblemAl Find all positive real solutions to:

Given n > 4, prove that every cyclic quadrilateral can be dissected into n cyclic (X12- x3xsXxl- X3XS):SO
quadrilaterals. (xl- '4xIXxl- '4XI):S O
(xl- XSX2X'42- XSX2):SO
('42 - xlx3Xxl- XIX3):SO

Solution
(xl- X2'4XX/ - X2X4):SO
...
A little tinkering soon shows that it is easy to divide a cyclic quadrilateral ABCD into 4
cyclic quadrilaterals. Take a point P inside the quadrilateral and take an arbitrary line PK
joining it to AB. Now take L on BC so that L KPL = 180·- LB (thus ensuring that KPLB
is cyclic), then M on CD so that LLPM = 180·- LC, then N on AD so that LMPN = 180·
Solution

Answer: XI = X2= X3= '4 = Xs.


...
- L D. Then L NPK = 180·- LA. We may need to impose sorne restrictions on P and K to
ensure that we can obtain the necessary angles. It is not clear, however, what to do next. The difficulty with this problem is that it has more information than we need. There is a
neat solution in Greitzer which shows that all we need is the sum of the 5 inequalities,
The trick is to notice that the problem is easy iftwo sides are parallel. For then we may take because one can rewrite that as (XIX2- XI'4)2 + (X2X3- x2xd + ... + (XsXI- XSX3)2 + (XIX3-
arbitrarily many lines parallel to the parallel sides and divide the original quadrilateral into xlxsi + ... + (XSX2- xs'4i :s O.The difficulty is how one ever dreams up such an idea!
any number of parts.
The more plodding solution is to break the symmetry by taking XI as the largest. Ifthe
So we need to arrange our choice of P and K so that one of the new quadrilaterals has second largest is X2,then the first inequality tells us that XI2or X22= X3XS.But if X3and Xs
parallel sides. But that is easy, since K is arbitrary. So take PK parallel to AD, then we must are unequal, then the larger would exceed XI or X2.Contradiction. Hence X3= Xsand aIso
equals X2or XI. Ifthey equal xr, then they would also equal X2(by definition OfX2), so in
also have PL parallel to CD.
any case they must equal X2.Now the second inequality gives X2= XI'4. SOeither all the
It remains to consider how we ensure that the points lie on the correct sides. Consider first numbers are equal, or XI > X2= X3= Xs> '4. But in the second case the last inequality is
K and L. K cannot lie on AD since PK is parallel to AD, and we can avoid it Iying on BC violated. So the only solution is all numbers equal.
by taking P sufficiently close to D. Similarly, taking P sufficiently close to D ensures that L
lies on BC. Now suppose that M and N are both on AD. Then ifwe keep K fixed and move If the second largest is Xs,then we can use the last inequality to deduce that X2= '4 = xs and
P closer to CD, N will move on to CD, leaving M on AD. proceed as before.

If the second largest is X3,then the fourth inequality gives that XI = X3= Xsor XI = X3= '4.
In the first case, xs is the second largest and we are horne already. In the second case, the
third inequality gives xl = X2XSand hence X3= X2= Xs(or one of X2,Xswould be larger
ProblemA3
than the second largest). So Xsis the second largest and we are home.
Prove that (2m)!(2n)! is a multiple of m!n!(m+n)! for any non-negative integers m and n.
Finally, ifthe second largest is '4, then the second inequality gives XI = X2= X4or XI = X3=
X4.Either way, we have a case already covered and so the numbers are all equal.

Solution

..
The trick is to find a recurrence relation for f(m,n) = (2m)!(2n)!f(m!n!(m+n)!). In fact,
Problem B2
f(m,n) = 4 f(m,n-l) - f(m+l,n-l), which is sufficient to generate all the f(m,n), given that
f(m,O) = (2m)!f(m!m!), which we know to be integeral. fand g are real-valued functions defined on the realline. For all Xand y, f(x + y) + f(x - y)
= 2f(x)g(y). fis not identically zero and lf(x)l:S I for all x. Prove that 19(x)l:S 1 for al! x. .
Problem Bl





Solution ProblemAl

Let k be the least upper bound for If(x)l.Suppose 19(y)1> 1. Take any x with If(x)1> O, then Can we find a finite set of non-coplanar points, such that given any two points, A and B,


2k::: If(x+y)1+ lf(x-y)1:::If(x+y) + f(x-y)1= 2Ig(y)lIf(x)I,so lf(x)1< k/lg(y)l· In other words, there are two others, C and D, with the Iines AB and CD parallel and distinct?
k/lg(y)1is an upper bound for lf(x)1which is less than k. Contradiction.

• Problem ro
Given four distinct parallel planes, prove that there exists a regular tetrahedron with a
Solution

• vertex on each planeo To warm up, we may notice that a regular hexagon is aplanar set satisfying the condition.

Take two regular hexagons with a common long diagonal and their planes perpendicular.

• Solution
Now ifwe take A, B in the same hexagon, then we can find C, D in the same hexagon. If
we take A in one and B in the other, then we may take C at the opposite end of a long


diagonal from A, and D at the opposite end of a long diagonal from B.
Intuitively, we can place A and B on the two outer planes with AB perpendicular to the
planes. Then tilt AB in one direction until we bring C onto one of the middle planes ProblemA3

• (keeping A and B on the outer planes), then tilt AB the other way (keeping A, B, C on their
respective planes) until D gets onto the last planeo a and b are real numbers for which the equation x' + axJ + bx2 + ax + l
real solution. Find the least possible value of a2 + bl
= O has at least one

..
Take A as the origino Let the vectors AB, AC, AD be b, e, d. Take p as one of the outer
planes. Let the distances to the other planes be e, f, g. Now we find a vector n satisfying:
n.b = e, n,c = f, n.d = g. This is a system ofthree equations in three unknowns with non-
zero determinant (because b.c 1d is non-zero), so it has a solution n. Scale the tetraiJedron Solution
by Inl,orient p perpendicular to n/lnl, then B, C, D will be on the other planes as required.
Put y = x + l/x and the equation becomes y2 + ay + b - 2 = O, which has solutions y = -aJ2

..
.ti IMO 1973

ProblemAI

OP" OP2, ... , OP2n+1are unit vectors in a planeoPI, P2, ... , P2a•1alllie on the same side of a
±v(a2 + 8 - 2b)l2. We require IYI::::2 for the original equation to have a real root and hence
we need lal + v(a2 + 8 - 4b)::: 4. Squaring gíves 21aj- b::: 2. Hence a2 + b2 :::a2 + (2 - 21aj)2=
5a2_ 81al+ 4 = 5(1aj- 4/5i + 4/5. So the least possible value of a2 + b2 is 4/5, achieved
when a = 4/5, b = -2/5. In this case, the original equation is x' + 4/5 xJ - 2/5 X2+ 4/5 x + 1 =
(x + 1)2(x2- 6/5 x + 1).

• line through O. Prove that IOPI + ... + OP2n+11::: 1.


ProbIem 81

• Solution
A soldier needs to sweep a region with the shape of an equilateral triangle for mines. The
detector has an effective radius equal to halfthe altitude ofthe triangle. He starts at a vertex


of the triangle. What path should he fo11owin order to travel the least distance and sti11
We proceed by induction on n. It is clearly true for n = l. Assume it is true for 2n-1. Given sweep the whole regíon?
OP¡ for 2n+ 1, reorder them so that a11OP¡ lie between OP2Dand OP2a+l. Then u = OP2D+

..•
OP2n+1lies along the angle bisector of angle P2DOP2n+1 and hence makes an angle less than
90· with v = OPI + OP1 + ... + OP1n-1(which must lie between OPI and OPln-1and hence
between OP2Dand OP2n+1.By induction lvl ::: 1. But [u+ vi ::: lvl (use the cosine formula). Solution
Hence the result is true for 2n+ l .
- In particular he must sweep the other two vertices. Let us take the triangle to be ABC, with
It is clearly best possible: take OPI = ... = OPn = -OPn+1= ... = -OP2n,and OP1n+1in an side 1 and assume the soldier starts at A. So the path must intersect the circles radius -.J3/4

• arbitrary direction. centered on the other two vertices. Let us look for the shortest path of this type. Suppose it



3)
intersects the circle center B at X and the circle center C at Y, and goes first to X and then Solution
to Y. Clearly the path from A to X must be a straight line and the path from X to Y must be
a straight lineo Moreover the shortest path from X to the circle center C follows the line XC We notice that the constraints are linear, in the sense that if'b, is a solution for ar, q, and b¡'
and has length AX + XC - ..J3/4. So we are looking for the point X which minimises AX + is a solution for a,', q, tben for any k, k' > O a solution for ka¡ + k'a,', q is kb, + k'b,', AIso a
XC. "near" solution for a.. = 1, otber a¡ = O is b¡ = qh-I, b2 = qh-2,... , hIt-1= q, b¡, = 1, bh+l = q, ... ,
b. = q.-b. "Near" because the inequalities in (a) and (b) are not strict,
Consider the point P where the altitude intersects the circle. By the usual reflection
argument the distance AP + PC is shorter!han the distance AP' + P'C for any other point P' However, we might reasonably hope tbat the inequalities would become strict in the linear
on the line perpendicular to the altitude tbrough P. Moreover for any point X on the circle, combination, and indeed that is true. Define b, = q,-Ial + q,-2a2+ ... + qa,-I + a, + qa,..1+ ... +
take AX to cut the line at P'. Then AX + XC > AP' + P'C > AP + Pe. qO-'a..Then we may easily verify tbat (a) - (e) hold.

It remains to check that the three circles center A, X, Y cover the triangle. In fact the circle
center X covers the whole triangle except for a small portion near A and a small portion
near C, which are covered by the triangles center A and y. IMO 1974

Problem B2 ProblemAl

G is a set of non-constant functions f. Each f is defined on tbe realline and has the form Three players play the following garne. There are three cards each with a different positive
f{x) = ax + b for some real a, b. Iffand g are in G, then so is fg, where fg is defined by integer. In each round the cards are randomly dealt 10 the players and each receives the
fg(x) = f{g(x)). If f is in G, then so is the inverse r'. If f(x) = ax + b, then fl(X) = xla - b/a. number of counters on his cardo Alter two or more rounds, one player has received 20,
Every fin G has a fixed point (in other words we can find xr such that f(xr) = Xr.Prove tbat anotber 10 and the tbird 9 counters. In the last round the player with 10 received the largest
a11 the functions in G have a common fixed point, number of counters. Who received the middle number on the first round?

Solution Solution

f{x) = ax + b has fixed point b/(l-a). If a = 1, then b must be O,and any point is a fixed The player witb 9 counters.

......
point. So suppose ft.x) = ax + b and g(x) = ax + b' are in G. Then h tbe inverse of f is given
by h(x) = xla - b/a, and hg(x) = x + b'/a - b/a. This is in G, so we must have b' = b. The total oftbe scores, 39, must equal the number ofrounds times the total oftbe cards.
But 39 has no factors except 1, 3,13 and 39, the total oftbe cards must be at least 1 + 2 + 3
Suppose ft.x) = ax + b, and g(x) = ex + d are in G. Then fg(x) = acx + (ad + b), and gf(x) = = 6, and the number of rounds is at least 2. Hence there were 3 rounds and the eards total
acx + (be + d). We must have ad + b = be + d and hence b/(l-a) = e/(l-d), in other words f 13.
and g have the same fixed point,
The highest score was 20, so the highest eard is at least 7. The score of lO inc1uded at least
Problem B3 one highest card, so the highest eard is at most 8. The lowest eard is at most 2, beeause if it
was higher then the highest card would be at most 13 - 3 - 4 = 6, whereas we know it is at
al, a2, ... , a. are positive reals, and q satisfies O < q < 1. Find bj, b2, ... , b. sueh that: least 7. Thus tbe possibilities for the eards are: 2, 3, 8; 2, 4, 7; 1,4,8; 1,5,7. But the only
one oftbese tbat allows a score of20 is 1,4,8. Thus the seores were made up: 8 + 8 + 4 =
(a) a¡ < b, for i = 1,2, ... , n, 20,8+ 1 + 1 = 10,4 + 4 + 1 = 9. The last round must have been 4 to tbe player witb 20,8
to the player witb 10 and 1 10the player witb 9. Hence on each of tbe other two rounds tbe
(b) q < b¡+I/b¡< I/q for i = 1,2, ... , n-I,

(c) bl + b2 + ... + b. < (al + a2 + ... + a.XI + q)/(1 - q).


eards musl have been 8 lo the player with 20, 1 lo lhe player witb 10 and 4 10the player
wilh9.

ProblemAl
. ...

..J
.J
'•.
• Prove that Ihere is a poinl D on Ihe side AB of the tríangle ABC, such that CD is Ihe Solution

• geometric mean of AD and DB if and only if sin A sin B ~ sin2(C/2)


The requirement that the number of black and while squares be equal is equivalent lo


requiring that the each rectangle has an even number of squares. 2 + 4 + 6 + S + 10 + 12 +
14 + 16 = 72 > 64, so p < S. There are S possible divisions of 64 into 7 unequal even
Solution numbers: 2 + 4 + 6 + S + 10 + 12 + 22; 2 + 4 + 6 + S + 10 + 16 + IS; 2 + 4 + 6 + S + 12 +


14 + IS; 2 + 4 + 6 + 10 + 12 + 14 + 16. The first is ruled out because a rectangle wilh 22
Extend CD lo meet Ihe circumcircle of ABC al E. Then CD·DE = AD'DB, so CD is Ihe squares would have more than S squares on its longest side. The others are a11possible.
geometric mean of AD and DB iff CD = DE. So we can find such a poinl iff the distance of

• C from AB is less Ihan Ihe distance of AB from the furthest point of the are AB on the
opposile side of AB lo C. The furthesl poinl F is evidenlly the midpoinl of Ihe arc AB. F
lies on the angle bisector of C. So L FAB = L FAC = L C/2. Hence distance of F from AB
2 2 2 2 4 2 2 2


2 2 2 2 222 4 2 2 2 2 2 2 2
is c/2 tan C/2 (as usual we set e = AB, b = CA, a = BC). The distance ofC from AB is a sin
B. So a necessary and sufficienl condition is c/2 tan C/2 2: a sin B. But by the sine rule, a = 1 5 5 4 5 5
e sin Nsin C, so Ihe condition becomes (sin C/2 sin C)/(2 cos C/2) 2: sin A sin B. But sin C

• = 2 sin C/2 cos C/2, so we obtain Ihe condilion quoted in Ihe question.
111 115
5 5 4

5 4
1 1 1 1

1 1 1 1 1 1 5 5
5

• ProblemAJ
6 4 3 3 7 6 6

•..
3 3 3 3 6 6 4 3 3 3 7

Prove that the sum from k = O to n of (2n+ 1)C(2k+ 1) 2Jk is not divisible by 5 for any non- 3 3 3 3 7 7 4 4 4 4 4 4 4 4 4
negative integer n. [res denoles the binomial coefficient r!l(s!(r-s)!) .]

2 2 2 2 2 2 7 1 1

• Solution 2 2 2 2 2 2 2 7 1 1 1 1 1 1 1 1


4 4 2 2 2 2 2 2 7
Lel k = ..,fS.Then (1 + kin+1 = a + bk, where b is Ihe sum given in Ihe questíon. Similarly,
(1 - kiD+1 = a - bk. This looks like a dead end, because eliminating a fves an unhelpful 111 4 4 2 2 2 2 2 2 2 7
expression for b. The trick is to multiply Ihe two expressions lo gel 7 0+1= Sb2 - a2. This

• still looks unhelpful, but happens lo work, because we soon find Ihat 728+1 1- ±2 (mod S). So 4 4 3 3 3 3 6 6
if b was a multiple of S then we would have a square congruent to ±2 (mod S) which is
3 3 3 3 3 6
impossible.

• 3

5 5 5
3 3

5 5
6

6
4 4 4 4 4 5

4 4 4 4 4 5 5 5
5

• Problem81
Prob1em 82

•••
An S x S chessboard is divided into p disjoint rectangles (a1ong the lines between squares),
Determine a11possible values of a/(a+b+d) + b/(a+b+c) + cI(b+c+d) + d/(a+c+d) for
so that each rectangle has Ihe same number of white squares as black squares, and each
posítive real s a, b, c, d.
rectangle has a different number of squares. Find Ihe maximum possible value of p and a11
possible sets of rectangle sizes .


Solution


••
..
We show first that the sum must lie between I and 2. If we replace each denominator by Solution
a+b+c+d then we reduce each term and get l. Hence the sum is more than l. Suppose a is
the largest of the four reals. Then the first term is less than l. The second and fourth terms If x > x' and y > y', then (x - y)2 + (x' - y,)2:s (x - y,)2 + (x' - y)'. Hence ifi <j, but z¡:S Zj,
have denominators greater than b+c+d, so the terms are increased if we replace the then swapping Z¡and z¡ reduces the sum of the squares. But we can retum !he order of the Z¡
denominators by b+c+d. But then the last three terms sum to l. Thus tbe sum of the last to y¡ by a sequence of swaps ofthis type: first swap I to the 1st place, then 2 to the 2nd
three terms is less than l. Hence the sum is less than 2. place and so on.

If we set a = e = I and make b and d small, then the first and third terms can be made ProblemAl
arbitarily close to I and the other two terms arbitarily close to O, so we can make the sum
arbitarily close to 2. Ifwe set a = 1, e = d and make b and clb arbitarily small, then the first Let al < a2 < a3 < ...be positive integers. Prove that for every i >= 1, there are infinitely
term is arbitarily close to 1 and the last three terms are all arbitarily small, so we can make many a" that can be written in the form a" = ra, + saj, wilh r, s positive integers and j > i.
the sum arbitarily close to l. Hence, by continuity, we can achieve any value in the open
interval (1,2).

Solution

Problem B3 We must be able to find a set S of infinitely many a" in sorne residue class mod a¡. Take lIj
to be a member of S. Then for any a" in S satisfying a" > lIj, we have a" = lIj + a multiple of
Let P(x) be a polynomial with integer coefficients of degree d > O. Let n be the number of a,
distinct integer roots to P(x) = 1 or -1. Prove that n :s d + 2.
ProblemA3

Given any triangle ABC, construct external triangles ABR, BCP, CAQ on the sides, so that
Solution LPBC =4So, LPCB = 30°, LQAC =4So, LQCA =30°, LRAB = ISO, LRBA = ISO.
Prove that LQRP = 90° and QR = RP.
Suppose that A(x) and B(x) are two polynomials with integer coefficients which are
identical except for their constant terms, which differ by 2. Suppose A(r) = O, and B(s) =O
with r and s integers. Then subtracting we get 2 plus a sum ofterms a(¡-i - Si). Each of these
terms is divisible by (r - s), so 2 must be divisible by (r - s). Hence r and s differ by O, I or

.....
Solution
2.
Trigonometry provides a routine solution. Let BC = a, CA = b, AB = c. Then, by the sine
Now let r be the smallest root of P(x) = I and P(x) = -I. The polynomial with r as a root can rule applied to AQC, AQ = b/(2 sin lOSO)= b/(2 cos ISO).Similarlr PB = a/(2 cos IS).
have at most d distinct roots and hence at most d distinct integer roots. If s is a root of tbe Also AR = RB = c/(2 cos ISO).So by the cosine rule RP' = (a2 + e - 2ac cos(B+60°)Y(4
other equation then s must differ from r by O, 1, or 2. But s::: r, so s = r, r+ I or r+2. Hence cos2IS0), and RQ2 = (b2 + c2 _ 2bc cos(A+600))/(4 cos2IS0). So RP = RQ is equivalent to: a2
the other equation adds at most 2 distinct integer roots. - 2ac cos(600+B) = b2 - 2be cos(600+A) and hence to a2 - ac cos B + "3 ac sin B = b2- bc
cos A + "3 be sin A. By the sine rule, the sine terms cancel. Also b - b cos A = a cos C, and
IMO 1975 a - e cos B = b cos e, so the last equality is true and hence RP = RQ. We get an exactly
similar expression for PQ2 and show that it equals 2 RP2 in the same way.
ProblemAl
A more elegant solution is to construct S on the outside of AB so that ABS is equilateral.
Let x¡ :::X2:::... :::xn, and Yl:::Y2:::... :::Yn be real numbers. Prove that if Z¡is any Then we find that CAS and QAR are similar and that CBS and PBR are similar. So QRlCS
permutation of the y¡, then: = PRlCS. The ratio of the sides is the same in each case (CAlQA = CBIPB since CQA and
CPB are similar), so QR = PRoAlso there is a 4So rotation between QAR and CAS and
another 4So rotation between CBS and PBR, hence QR and PR are at 90°.

..
J



• Problem 81 (2) for all real x, y, z: P(y + z, x) + pez + x, y) + P(x + y, z) = O;

• Let A be the sum ofthe decimal digits of 444444", and B be the sum of the decimal digits (3) p(I, O) = 1.


of A. Find the sum of the decimal digits of B.

Solution
Solution
3
(1) means that P is homogeneous of degree n for sorne n. Experimentin with low n, shows
Let X = 444444".Then X has less than 4.4444 = 17776 digits, so A is at most 9.17776 = that the only solutions for n = 1, 2, 3 are (x - 2y), (x + yXx - 2y), (x + y) (x - 2y). It then
159984. Hence B is at most 6.9 = 54. But all these numbers are congruent mod 9.4444 =-2 obvious by inspection that (x + y¡O(x- 2y) is a solution for any n. Taking x = y = z in (2)
(mod 9), so X = (_2)4444(mod 9). But (-2¡J = 1 (mod 9), and 4444 = 1 (mod 3), so X = -2 = shows that P(2x,x) = O, so (x - 2y) is always a factor. Taking x = y = 1, z = -2 gives P(I,-I)

• 7 (mod 9). But any number less than 55 and congruent to 7 has digit sum 7 (possibilities are
7, 16,25,34,43,52). Hence tbe answer is 7.
(2" - 2) = O,so (x + y) is a factor for n > 1. AH this suggests (but does not prove) that the
general solution is (x + y¡O(x- 2y).

• Take y = 1 - x, z = O in (2) and we get: P(x, l-x) = -1 - P(I-x, x). In particular, P(O,I) = -2.
Now take z = 1 - x - y and we get: P(I-x, x) + P(I-y, y) + p(x+y, I-x-y) = O and hence

..•
Problem 82 f(x+y) = f(x) + f(y), where f(x) = P(I-x, x) - 1. By induction we conclude that, for any
integer m and real x, f(mx) = mf(x). Hence f(1/s) = lis f(1) and f(r/s) = r/s f(l) for any
Find 1975 points on the circumference of a unit circle such that the distance between each integers r, s. But P(O,I) = -2, so f(1) = -3. So f(x) = -3x for all rational x. But fis

..
pair is rational, or prove it impossible . continuous, so f(x) = -3x for all x. So set x = b/(a+b), where a and b are arbitrary reals (with
a+b non-zero). Then P(a,b) = (a+b)"p(I-x, x) = (a+b)"(-3b/(a+b) + 1) = (a+b)"-I(a-2b), as
claimed. [For a+b = O, we appeal to continuity, or use the already derived faet that for n >
1, P(a,b) = O.]
Solution

Let x be the angle cos-14/5, so that cos x = 4/5, sin x = 3/5. Take points on the unit circle at
angles 2nx for n integral. Then the distance between the points at angles 2nx and 2mx is 2 IMO 1976


sin(n - m)x. The usual formula, giving sin(n - m)x in terms of sin x and cos x, shows that
sin(n - m)x is rational. So it only remains to show that this process generates arbitarily ProblemAI
many distinct points, in other words that x is not a rational multiple of lt.


Aplane convex quadrilateral has area 32, and the sum of two opposite sides and a diagonal
This is quite hard. There is an elegant argument in sections 5 and 8 of Hadwiger et al, is 16. Determine all possible lengths for the other diagonal.
Combinatorial geometry in the Plane. But we can avoid it by observing that there are only

• finitely many numbers with are nth roots ofunity for n:;; 2 x 1975, whereas there are
infinitely many Pythagorean triples, so we simply pick a triple which is not such a root of
Solution


unity.

At first sight, the length of the other diagonal appears unlikely to be significantly

•.-
constrained. However, a Iittle experimentation shows that it is hard to get such a low value
Problem 83 as 16. This suggests that 16 may be the smallest possible value.

Find all polynomials P(x, y) in two variables such that: If the diagonal which is part of the 16 has length x, then the area is the sum of the areas of
two triangles base x, which is xyl2, where y is the sum of'the altitudes of the two triangles.
(1) P(tx, ty) = t"P(x, y) for sorne positive integer n and all real t, x, y; y must be at most (16 - x), with equality only if the two triangles are right-angled. BU!x(16

• - x)/2 = (64 - (x - 8¡2)/2 :;;32 with equality only iff x = 8. Thus the only way we can achieve



the values given is with one diagonallength 8 and two sides perpendicular to this diagonal
with lengths totalling 8. But in this case the other diagonal has length 8'./2.
This is somewhat messy. The basic idea is that the sides cannot be too long, because then
the ratio becomes too big. Let k denote the (real) cube root of 2. Given any integer n, let n'
denote the least integer such that n'k <= n. Let the sides of the box be a S b S c. So we
require 5a'b'c' = abc (.).
...
ProblemA2 It is useful to derive n' for small n: l' = O,2' = 1, 3' = 2, 4' = 3,5' = 3, 6' = 4, 7' = 5, 8' = 6, 9'
=7,10'=7.
Let PI(X) = X2- 2, and Pi+1= PI(Pi(X» for i = 1,2,3, .... Show that the roots ofP..(x) = x are
real and distinct for all n. Clearly n'k ~ n-2. But 63 > 0.483, and hence (n'k)3 ~ (n - 2)3 > 0.4 n3 for all n ~ 8. We can
check directly that (n'k)' > 0.4 n3 for n = 3, 4, 5, 6, 7. So we must have a = 2 (we cannot
have a = 1, because l ' = O).

Solution From (.) we require b or e to be divisible by 5. Suppose we take it to be 5. Then since 5' =
3, the third side n must satisfy: n' = 2/3 n. We can easily check that 2k13 < 6/7 and hence
We show that the graph of P, can be divided into 2· lines eachjoining the top and bottom (2/3 nk + 1 ) < n for n ~ 7, so n' > 2/3 n for n ~ 7. This just leaves the values n = 3 and n = 6
edges of the square side 4 centered on the origin (vertices (2,2), (-2,2),(-2,-2), (-2,2». We to check (since n' = 2/3 n is integral so n must be a multiple of3). Referring to the values
are then home because the upward sloping diagonal of the square, which represents the above, both these work. So this gives us two possible boxes: 2 x 3 x 5 and 2 x 5 x 6.

....
graph ofy = x, must cut each of these lines and hence give 2· distinct real roots ofP..(x) = x
in the range [-2,2]. But p. is a polynomial of degree 2·, so it has exactly 2· roots. Hence all The only remaining possibility is that the multiple of5 is at least 10. But then it is easy to
its roots are real and distinct. check that ifit is m then m'/m ~ 7110. It follows from (.) that the third side r must satisfy
r'/r <= 4/7. But using the limit above and referring to the small values above, this implies
We prove the result about the graph by induction. It is true for n = 1: the first line is the that r must be 2. So a = b = 2. But now e must satisfy c' = 4/5 c. However, that is
graph from x = -2 to O, and the second line is the graph from O to 2. So suppose it is true for impossible because 4/5 k > l.

..
n. Then PI tums each ofthe 2· lines for p. into two lines for Pn+I,so the result is true for
n+l. Problem Bl

Alternative solutionfrom Arthur Engel, Problem-Solving Strategies, Springer 1998 Determine the largest number which is the product ofpositive integers with sum 1976.
[Problem boolcs in mathematics series), ISBN 0387982191. A rather good training book.

Put x = 2 cos t (so we are restricting attention to -2 S x S 2). Then we find p.(x) = 2 cos 2"1,
so the equation p.(x) = x becomes cos 2"1= cos t. By inspection, has the 2· solutions t = Solution

..
2kxl(2· - 1) and t = 2kxl(2· + 1), giving 2· distinct solutions in x.
Answer: 2·36S8.

There cannot be any integers larger than 4 in the maximal product, because for n > 4, we
ProblemA3 can replace n by 3 and n - 3 10 get a larger product. There cannot be any 1s, because there
must be an integer r > 1 (otherwise the product would be 1) and r + l > I.r. We can also
A rectangular box can be completely filled with unit cubes. If one places as many cubes as replace any 4s by two 2s leaving the product unchanged. Finally, there cannot be more than
possible, each with volume 2, in the box, wilh their edges parallel to the edges of the box, two 2s, because we can replace three 2s by two 3s lo get a larger product. Thus the product
one can fill exactly 40% of the box. Determine the possible dimensions of the box. must consist of 3s, and either zero, one or two 2s. The number of2s is determined by the

Solution

Answer: 2 x 3 x 5 or 2 x 5 x 6.
remainder on dividing the number 1976 by 3.

1976 = 3,658 + 2, so there must be just one 2, giving the product 2,36S8•
. ....
...
..



• Problem B2 Construct equilateral triangles ABK, BCL, CDM, DAN on the inside ofthe square ABCD.

• n is a positive integer and m = 2n. ai¡ = 0, I or -1 for 1 ~ i ~ n, 1 ~j ~ m. The m unknowns


XI. X2,... , Xm satisfy the n equations:
Show that the midpoints of KL, LM, MN, NK and the midpoints of AK, BK, BL, CL, CM,
DM, DN, AN form a regular dodecahedron.



Solution
for i = 1,2, ... , n. Prove that the system has a solution in integers of absolute value at most
m, not all zero. The most straightforward approach is to use coordinates. Take A, B, C, D to be (1,1), (-
1,1), (-1,-1), (1,-1). Then K, L, M, N are (O, -2k), (2k, O), (O, 2k), (-2k, O), where k = ('./3-
1)/2. The midpoints ofKL, LM, MN, NK are (k, -k), (le, k), (-k, k), (-k, -k). These are a11a
distance b/2 from the origin, at angles 315, 45,135,225 respectively. The midpoints of
Solution AK, BK, BL, CL, CM, DM, DN, AN are (h,j), (-h,j), (-j, h), (-j, -h), (-h, -j), (h, -j), (j, -h),
(j, h), where h = 1/2, j = (1 - 1/2 -./3).These are a1so at a distance k-./2from the origin, at
angles 15, 165, 105,255, 195,345,285,75 respectively. For this we need to consider the


We use a counting argument. Ifthe modulus of each Xiis at most n, then each ofthe linear
combinations has a value between _2n2and 2n2, so there are at most(4n2 + 1) possible right-angled triangle sides k, h,j. The angle x between h and k has sin x = j/k and cos x =
values for each linear combination and at most (2n2 + l)"possible sets ofvalues. But there h/k. So sin 2x = 2 sin x cos x = 2hj/k2 = 1/2. Hence x = 15.


are 2n+ 1 values for each x, witb modulus at most n, and hence (2n+ l in = (4n2+4n+ l)" sets
of values. So two distinct sets must give the sarne set of values for the linear combinations. So the 12 points are all at the same distance from the origin and at angles 15 + 30n, for n =
But now ifthese sets are x¡ and Xi',then the values Xi-X¡'give zero for each linear 0, 1, 2, ... , 11. Hence they form a regular dodecagon.
combination, and have modulus at most 20. Moreover they are not a11zero, since the two
sets ofvalues were distinct.

ProblemA2

Problem B3 In a finite sequence of real numbers tbe sum of any seven successive terms is negative, and
the sum of any eleven successive terms is positive. Determine the maximum number of
The sequence 110,uj, U2,... is defined by: 110=2, u¡ = 5/2, Un+l= Un(Un-12- 2) - u¡ for n = 1,2, terms in the sequence .
. Prove that [un] = 2(2n-(-I)n)l3,
where [xl denotes the greatest integerless than orequal to
X.

Solution

Solution Answer: 16.

• Experience with recurrence relations suggests that the solution is probably the value given
for [unl plus its inverse. It is straightforward to verify this guess by induction.
x¡ + .. + X7< 0, XI + ... + Xl. < 0, so xr + ... + Xl. < O. But X. + ... + Xl. > 0, so x¡ + Xl + X3
°
< O. Also Xs+ ... + Xll < and x¡ + ... + Xll > 0, so x. > O. lf there are 17 or more elements

• Squaring Un-lgives the sum of positive power of 2, its inverse and 2. So Un-l- 2 = the sum
then the same argument shows that xs, x¡;, X7> O. But x¡ + ... + X7< 0, and Xs+ ... + xu < 0,
whereas x¡ + ... + xu > 0, so xs + X¡; + X7<O. Contradiction.

•.-
of a positive power of 2 and its inverse. Multiplying this by Ungives a positive power of 2 +
its inverse + 2 + 112, and we can check tbat tbe power of2 is correct for Un+l. If we assume tbat tbere is a solution for n = 16 and that the sum of 7 consecutive terms is -1
and tbat the sum of 1I consecutive terms is 1, then we can easily solve tbe equations to gel:
IMO 1977 5,5, -13, 5, 5, 5, -13, 5, 5, -13,5,5,5, -13, 5, 5 and we can check tbat Ibis works for 16.

ProblemAI




ProblemAl Problem 82

Given an integer n > 2, let V n be the set of integers I + kn for k a positive integer. A Let a and b be positive integers. When a2 + b2 is divided by a + b, the quotient is q and the
number m in V n is called indecomposable if it cannot be expressed as the product of two remainder is r. Find all pairs a, b such that q2 + r = 1977.
members of V a- Prove that there is a number in Vo which can be expressed as the product of
indecomposable members of V n in more than one way (decompositions which differ solely

...
..
in the order of factors are not regarded as different).
SolutioD

a2 + b2 >= (a + b)'/2, so q::: (a + b)/2. Henee r < 2q. The largest square less than 1977 is
SolutioD 1936 = 442. 1977 = 442 + 41. The next lar~est gives 1977 = 432 + 128. But 128> 2.43. So
we must have q = 44, r = 41. Henee a2 + b = 44(a + b) + 41, so (a - 22)2 + (b - 22)2 = 1009.
Take a, b, e, d = -1 (mod n). The idea is to take abcd whieh factorizes as ab.ed or ac.bd. The By trial, we find that the only squares with sum 1009 are 282 and IS2. This gives two
hope is that ab, cd, ac, bd will not faetorize in Vo. But a little eare is needed, sinee this is solutions SO,37 or SO,7.
not necessarily true.
Problem B3
Try taking a = b = n - 1, e = d = 2n -l. a2 must be indecomposable beeause it is less than the
square ofthe smallest element in V n- Ifae = 2n2 - 3n + 1 is deeomposable, then we have The funetion f is defined on the set of positive integers and its values are positive integers.
kk'n + k + k' = 2n - 3 for sorne k, k' >= l. But neitber of k or k' can be 2 or more, because Given that ftn+ 1) > ftftn» for a11n, prove tbat ftn) = n for a11n.
then the Ihs is too big, and k = k' = 1 does not work unless n = S. Similarly, if e2 is
deeomposable, then we have kk'n + k + k' = 4n - 4. k = k' = 1 only works for n = 2, but we
are told n > 2. k = 1, k' = 2 does not work (it would require n = 712). k = 1, k' = 3 only
works for n = 8. Other possibilities make the Ihs too big. SolutioD

So ifn is not S or 8, then we can take tbe numberto be (n - 1)'(2n - 1)', whieh factors as (n The first step is to show that f(l) < ft2) < ft3) < .... We do tbis by induetion on n. We take
- 1)' x(2n _1)2 oras (n - lX2n - 1) x (n - IX2n - 1). This does not work for 5 or 8: 16·81 = So to be the statement that ftn) is the unique smallest element of ( ftn), f{n+1), t{n+2), ... l·
36'36, but 36 deeomposes as 6'6; 49·225 = lOS,lOS, but 22S deeomposes as 9·25.
For m> 1, ftm) > fes) where s = ftm-l), so ftm) is notthe smallest member ofthe set (ftl),
For n = 5, we can use 3136 = 16·196 = S6·S6. For n = 8, we can use 2S921 = 49·S29 = ft2), f(3), ... l. But the set is bounded below by zero, so it must have a smallest rnember.
161·161. Hence the unique smallest member is f{1). So SI is true.

Problem 81 Suppose So is true. Take m> n+ 1. Then m-I > n, so by SIbftm-I) > ftn). But So a1sotells
us that ftn) > ftn-I) > ... > ftl), so ftn)::: n - 1 + ftl)::: n. Henee ftm-l)::: n+1. So ftm-l)
Define ftx) = 1 - a eos x - b sin x - A cos 2x - B sin 2x, where a, b, A, B are real eonstants. belongs to ( n+ 1, n+2, n+3, .. l. But we are given that f{m) > ftftm-I», so f{m) is not tbe
Suppose that ftx)::: Ofor a11real X. Prove that a2 + b2::; 2 and A2 + B2::; 1. smallest element of { f{n+1), ftn+2), ftn+ 3), ... l. But there must be a smallest element, so
i\n+ 1) must be the unique smallest member, which establishes Su+ I. So, So is true for all n.

So n::; m implies f(n) <= ftm). Suppose for sorne m, f{m)::: m+ 1, then fti\m»::: ftm+ 1).
SolutioD Contradietion. Henee f{m)::; m for all m. But since ftl):::1 and ftm) > ftm-l) > ... > f{1),
we also have ftm)::: m. Hence f{m) = m for a11m.
Take y so that ~os y = aI..J(a2+ b2), sin y = b/..JCa2+ b2), and z so that cos 2z = N..JCA2 + B2),
sin 2z = B/..J(A + B2). Then f(x) = 1 - e eos(x - y) - C cos2(x - z), where e = ..J(a2+ b2), C = IMO 1978
..J(A2 + B2).
ProblemAl
f{z) + f{lt + z)::: O gives C ::; 1. fty + Jt!4) + f(y - Jt!4)::: O gives e::; ..J2.

~.

• m and n are positive integers with m < n. The last three decimal digits of 1978m are the ProblemA3


ji
same as the last three decimal digits of 1978". Find m and n such that m + n has the least
possible value. The set ofall positive integers is the union oftwo disjoint subsets {f(I), 1(2), 1(3), ... },
{g(I), g(2), g(3), ... }, where f(l) < 1(2) < 1(3) < ..., and g(l) < g(2) < g(3) < ..., and g(n) =
I(l(n» + 1 for n = 1,2,3, .... Determine 1(240).


Solution

We require 1978m(1978"·m- 1) to be a multiple of 1000=8,125. So we must have 8 divides Solution

• 1978m, and hence m 2 3, and 125 divides 1978"-111

By Euler's theorem, 197Sop(12S)


- 1.

= 1 (mod 125). CP(125)= 125 - 25 = lOO,so 1978100= 1 (mod


Let F = {I(I), f(2), f(3), ... }, G = {g(I), g(2), g(3), ... }, No = {I, 2, 3, ... , n}. f(1)21, so
1(f(1» ::: 1 and hence g(I)::: 2. So 1 is not in G, and hence must be in F. It must be the

• 125). Hence the smallest r such that 1978' = 1 (mod 125) must be a divisor of lOO(because
if it was not, then the remainder on dividing it into 100 would give a smaller r). That leaves
smallest element ofF and so 1(1) = 1. Hence g(1) = 2. We can never have two successive
integers n and n+ 1 in G, because if g(m) = n+ 1, then f(something) = n and so n is in F and


9 possibilities to check: 1,2,4,5, 10,20,25,50, lOO.To reduce the work we quickly find G. Contradiction. In particular, 3 must be in F, and so f(2) = 3.
that the smallest s such that 1978' = 1 (mod 5) is 4 and hence r must be a multiple of 4.
That leaves 4, 20, lOOto examine. Suppose f(n) = k. Then g(n) = f(k) + l. SOINf(k)+1n GI = n. But INf(k)+1n FI = k, so n + k =

..•
I(k)+ 1,0rf(k)=n+k-1. Henceg(n)=n+k. Son+k+ 1 mustbeinFandhencef(k+l)
We find 9782 = 109 (mod 125), and hence 9784 = 6 (mod 125). Hence 97820 = 6s = 36·91 = = n + k + 1. This so given the value offfor n we can find it for k and k+ 1.
26 (mod 125). So the smallest r is lOOand hence the solution to the problem is 3,103.

....
Using k+1 each time, we get, successively, f(2) = 3, 1(4) = 6, f(7) = 11, 1(12) = 19,1(20) =
ProblemAl 32, f(33) = 53, 1(54) = 87, f(88) = 142, f(143) = 231, f(232) = 375, which is not much help.
Trying again with k, we gel: f(3) = 4, f(4) = 6,1(6) = 9, f(9) = 14, f(14) = 22, f(22) = 35,
P is a point inside a sphere. Three mutually perpendicular rays from P intersect the sphere 1(35) = 56, f(56) = 90, f(90) = 145, f(145) = 234. Still not right, but we can try backing up
at points U, Yand W. Q denotes the vertex diagonally opposite P in the parallelepiped slightly and using k+ 1: f(146) = 236. Still not right, we need to back up further: 1(91) =
determined by PU, PY, PW. Find the locus ofQ for all possible sets ofsuch rays from P. 147,1(148) = 239, f(240) = 388.

.. Solution Problem Bl

• Suppose ABCD is a rectangle and X any point inside, then XA2 + XC2 = XB2 + X02. This
is most easily proved using coordinates. Take the origin O as the center of the rectangle and
take OA to be the vector ª, and OB to be h. Since it is a rectangle, IªI = Ihl. Then OC is -ª
In the triangle ABC, AB = AC. A circle is tangent intemally to the circurncircle of the
triangle and also to AB, AC at P, Q respectively. Prove that the midpoint ofPQ is the
center ofthe incircle of the triangle.

• and OD is -h. Let OX be g. Then XA2 + Xc2 = ~ _!<)2+ ~ +!<i = 2ª2 + 2!<2= 2h2 + 2!<2=
xa' + xn'.

• Let us fix U. Then the plane k perpendicular to PU through P cuts the sphere in a circle
center C. y and W must lie on this circle. Take R so that PVRW is a rectangle. By the
SoIution

..

resultjust proved CR2 = 2Cy2 - cp2. OC is also perpendicular to the plane k. Extend it to It is not a good idea to get bogged down in complicated formulae for the various radii. The
X, so that CX = PU. Then extend XV to Y so that YR is perpendicular to k. Now OY2 = solution is actuaIly simple.
OX2 + Xy2 = OX2 + CR2 = OX2 + 2Cy2 _ cp2 = OU2 _ UXt + 2Cy2 _ Cp2 = OU2 - cp2 +
2(Oy2 - OC2) - cp2 = 30U2 - 20p2. Thus the locus of Y is a sphere. By symmetry the midpoint, M, is already on the angle bisector of A, so it is sufficient to
show it is on the angle bisector of B. Let tbe angle bisector of A meet the circurncircle
again at R. AP is a tangent to the circle touching AS at P, so LPRQ = LAPQ = LABC.




Now the quadrilateral PBRM is cyclic because the angles PBR, PMR are both 90°. Hence are home. But each difference equals the difference oftwo of the original as, so ifit is in
L PBM = L PRM = ( L PRQ)/2, so BM does indeed bisect angle B as claimed. C 1 we are also home.

So suppose they are all in the other four countries. At least 17 must come from the same
country, call it C3. Write the 17 as e¡ < C2< ...< C17.Now form the 16 differences C2- ci, C3
Problem B2 - Cl, ... , Cl7- ci. If any of them are in C3, we are home. Each difference equals the
difference oftwo b.s, so ifany ofthem are inC2 we are home. [Forexample, consider cr-
{llIt}is a sequence of distinct positive integers. Prove that for all positive integers n, ¿In ci. Suppose c¡ = bn - b¡ and e¡ = bm- bl, then C¡- e¡ = b, - bm,as claimed.]. Each difference
llItlk22: ¿In 11k. also equals the difference oftwo as, so ifany of them are in CI, we are also home. [For
example, consider C¡- cr, as before. Suppose bn = a¡ - al, bm = llIt- al, then C¡- e¡ = b. - bm =
a¡ - llIt,as claimed.]

Solution So suppose they are all in the oiher three countries. At least 6 must come from the sarne
country, call it C4. We look at the 5 differences and conclude in the same way that at least
We use the general rearrangement result: given b¡ 2: b2 2: ... 2: b.. and e, :::;C2:::;... :::;Cn,if 3 must come from C5. Now the 2 differences must both be in C6 and their difference must
{a;) is a permutation of {c¡}. then ¿ a;b¡ 2: ¿ c¡b¡.To prove it, suppose that i < j, but a; > a¡. be in one of the Cl, ... , C6 giving us the required sumo
Then interchanging II¡ and a¡ does not increase the sum, because (a¡ - a¡Xb¡- bj) 2: O, and
hence a;b¡ + ajbj 2: a¡b¡+ a;b¡. By a series of such interchanges we transform {a;} into {c¡}
(for example, first swap e¡ into first place, then C2into second place and so on).
IMO 1979
Hence we do not increase the sum by permuting {a;} so that it is in increasing order. But
now we have a; > i, so we do not increase the sum by replacing a; by i and that gives the ProblemAl
sum from 1 to n of 11k.
Let m and n be positive integers such that:
...
Problem B3

An intemational society has its members from six different countries. The list of members
mlm = 1 - 1/2 + 1/3 - 1/4 + ... - 1/1318 + 1/1319.

Prove!hat m is divisible by 1979. ...


has 1978 names, numbered 1, 2, ... , 1978. Prove that there is at least one member whose
number is the sum of the numbers of two members from his own country, or twice the
number of a member from his own country. Solution

This is difficult.

Solution The obvious step of combining adjacent terms to give 1/(n(n+ 1) is unhelpful. The trick is to
separate out the negative terms:
The trick is to use differences.
1 - 1/2 + 1/3 - 1/4 + ... -111318 + 1/1319 = 1 + 1/2 + 1/3 + ... + 1/1319 - 2(1/2 + 1/4 + ...
At least 6.329 = 1974, so at least 330 members come from the same country, call it CI. Let + 111318)= 11660+ 1/661 + ... + 111319.
their numbers be al < a2 < ...< a330.Now take the 329 differences a2 - al, a3 - al, ... , a330-
ai. If any ofthem are in CI, Ihen we are home, so suppose they are all in Ihe other five and to notice that 660 + 1319 = 1979. Combine terms in pairs from Ihe outside:
countries.
11660 + 1/1319 = 1979/(660.1319); 1/661 + 1/1318 = 1979/(661.1318) etc. ..1
At least 66 must come from the same country, call it C2. Write Ihe 66 as bl < b2 < ...< b66.
Now form Ihe 65 differences b2 - bl, b3 - bl, ... ,b66 - bl. Ifany ofthem are in C2, then we
\.

• There are an even number of terrns, so this gives us a sum of terrns 1979/m with m not

..•
divisible by 1979 (since 1979 is prime and so does not divide any product of smaller
numbers). Hence the sum of the l/m gives a rational number with denominator not divisible
by 1979 and we are done .

. ProblemAl

... A prism with pentagons AIA2A3~S and BIB2B3B.Bs as the top and bottom faces is
given. Each side of the two pentagons and each of the 25 segments A;Bj is colored red or

... green. Every triangle whose vertices are vertices of the prism and whose sides have all been
colored has two sides of a different color. Prove that all 10 sides of the top and bottom
faces have the same color.

• Let the circles have centers O, O' and let the moving points by X. X. Let P be the reflection
of A in the perpendicular bisector ofOO'. We show that triangles POX, X'O'P are

• Solution congruent. We have OX = OA (pts on circle) = O'P (reflection). Also OP = O'A (reflection)
= O'X' (pts on circle). AIso LAOX = LAO'X' (X and X' circle at same rate), and LAOP =


We show first that the A; are all the same color. lfnot then, there is a vertex, call it Al, with LAO'P (reflection), so LPOX = LPO'X'. So the triangles are congruent. Hence PX = PX'.
edges AIA2, AlAs of opposite color. Now consider the five edges AIB;. At least three of
them must be the same color. Suppose it is green and that AIA2 is also green. Take the three Another approach is lo show that XX' passes through the other point 01 intersection of the

....
edges to be AIB;, AIBj, AIBk. Then considering the triangles AIA2B;, AIA2Bj, AIA2Bk, the two circles, bu: that involves looking al many differenl cases depending on the relative
three edges A2B;, A2Bj, A2Bkmust all be red. Two of B¿ Bj, Bk must be adjacent, but if the positions of the moving points.
resulting edge is red then we have an all red triangle with A2, whilst if it is green we have
an all green triangle with Al. Contradiction. So the A; are all the same color. Similarly, the
B; are all the same color. Jt remains to show that they are the same color. Suppose
otherwise, so that the A; are green and the B; are red. Problem Bl

.. Now we argue as before that 3 ofthe 5 edges AIB; must be the same color. lfit is red, then
as before 2 ofthe 3 B; must be adjacent and that gives an all red triangle with Al. So 3 of
the 5 edges AIB; must be green. Similarly, 3 ofthe 5 edges A2B; must be green. But there
must be a B; featuring in both sets and it forrns an all green triangle with Al and A2.
Given a plane k, a point P in the plane and a point Q not in the plane, find all points R in k
such that the ratio (QP + PR)/QR is a maximum.


Contradiction. So the A; and the B; are all the same color.
Solution

• ProblemA3
Consider the points R on a circle center P. Let X be the foot ofthe perpendicular from Q to
k. Assume P is distinct from X. then we minimise QR (and hence maximise (QP + PR)/QR)
for points R on the circle by taking R on the line px. Moreover, R must lie on the same side

• Two circles in aplane intersect. A is one of the points of intersection. Starting


simultaneously from A two points move with constant speed, each traveling along its own
ofP as X. Hence ifwe allow R to vary over k, the points maximising (QP + PR)/QR must
lie on the ray px. Take S on the line PX on the opposite side ofP from X so that PS = PQ.

..
Then for points R on the ray PX we have (QP + PR)/QR = SRlQR = sin RQS/sin QSR. But


circle in the same sense. The two points return to A simultaneously after one revolution.
Prove that there is a fixed point P in the plane such that the two points are always sin QSR is fixed for points on the ray, so we maximise the ratio by taking L RQS = 90°.
equidistant from P. Thus there is a single point maximising the ratio .

lf P = X, then we still require L RQS = 90°, but R is no longer restricted to a line, so it can
be anywhere on a circle center P.

• SoJution



relation with the general solution: a2. = a(2 + .J2)O-I+ b(2 - .J2)n-1.But we easily see directly
that a. = 2, a.; = 8 and we can now solve for the coefficients to get the solution given.
Problem B2

Find all real numbers a for which there exist non-negative real numbers XI, X2,X3,x.., Xs
satisfying: IMO 1981

XI + 2X2+ 3X3+ 4x.. + 5xs = a, ProblemAl


XI + 23X2+ 33X3+ 43x..+ 53xs = a2,
XI + 2SX2+ 3SX3+ 4sx.. + 5sxs = a3. P is a point inside the triangle ABC. D, E, F are the feet of the perpendiculars from P to the
lines BC, CA, AB respectively. Find all P which minimise:

BCIPD + CAlPE + ABIPF.


Solution

Take a2 X 1st equ - 2a X 2nd equ + 3rd equ. The rhs is O.On the Ihs the coefficient of x, is
a2n _ 2an3 + nS = n(a - n2f So the Ihs is a sum ofnon-negative terms. Hence each term Solution
must be zero separately, so for each n either X. = O or a = n2. So there are just 5 solutions,
corresponding to a = 1,4,9,16,25. We can check that each ofthese gives a solution. [For a We have PD.BC + PE.CA + PF.AB = 2 area oftriangle. Now use Cauchy's inequality with
= n2, Xn= n and the other X¡are zero.] XI = .J(PD·BC), X2= .J(PE·CA), X3= .J(PF·AB), and YI = .J(BCIPD), Y2= .J(CAlPE), Y3=
.J(ABIPF). We get that (BC + CA + AB)2 < 2 Xarea oftriangle X(BCIPD + CAlPE +
ProblemB3 ABIPF) with equality only if x/y¡ = const, ie PD = PE = PF. So the unique minimum
position for P is the incenter.

...
..
Let A and E be opposite vertices of an octagon. A frog starts at vertex A. From any vertex
except E itjumps to one ofthe two adjacent vertices. When it reaches E it stops. Let a. be
the number of distinct paths of exactly n jumps ending at E. Prove that: ProblemA2

a2o-1= O Take r such that I :s r:S n, and consider all subsets of r elements of the set {I, 2, ... , n}.
a2n = (2 + .J2)O-I¡.J2- (2 _ .J2)n-1¡.J2. Each subset has a smallest element. Let F(n,r) be the arithmetic mean ofthese smallest
elements. Prove that:

Solution

Eachjump changes the parity ofthe shortest distance to E. The parity is initially even, so an
F(n,r) = (n+ I )/(r+ 1).
...
odd number of jumps cannot end at E. Hence a2n-1= O. Solution

We derive a recurrence relation for a2n- This is not easy to do directly, so we introduce b. Denote the binomial coefficient n!l(r!(n-r)!) by nCr.
which is the number of paths length n from C to E. Then we have immediately:
Evidently nCr F(n,r) = 1 (n-I)C(r-l) + 2 (n-2)C(r-l) + ... + (n-r+ l)(r-l)C(r-I). [The first
a2n = 2a2n-2+ 2b2n_2for n > 1 term denotes the contribution from subsets with smallest element 1, the second term
b2n = 2b2n_2+ a2n-2for n > 1 smallest element 2 and so on.]

Hence, using the first equation: a2n- 2a2n-2= 2a2n-2- 4a2n4 + 2b2n-2- 4b2n4 for n > 2. Using Let the rhs be g(n,r). Then, using the relation (n-i)C(r-l) - (n-i-I)C(r-2) = (n-i-l)C(r-I), we
the second equation, this leads to: a2n = 4a2n-2- 2a2.4 for n > 2. This is a linear recurrence find that g(n,r) - g(n-I,r-I) = g(n-I,r), and we can extend this relation to r=1 by taking
g(n,O) = n+l = (n+I)C1. But g(n,l) = 1 + 2 + ... + n = o(n+l)12 = (n+I)C2. So it now



• follows by an easy induction that g(n,r) = (n+ 1)C(r+ 1) = nCr (n+ 1Y(r+ 1). Hence F(n,r) =

• (n+I)/(r+I).
SolutioD

•..
A more elegam solution by Oliver Nash is as follows
(a) n = 3 is not possible. For suppose x was the largest number in the set. Then x cannot be
Let k be the smallest element. We want to evaluate g(n, r) = Ll.-11O_1 k (n-k)C(r-I). divisible by 3 or any larger prime, so it must be a power of2. But it cannot be a power of2,

..
Considerthe subsets with r+l elements taken from 1,2,3, ... , n+I. Suppose k+1 is the because 2m - 1 is odd and 2m - 2 is not a positive integer divisible by 2m.
" second smallest element. Then there are k (n-k)C(r-l) possible subsets. So g(n, r) =
(n+ 1)C(r+ 1). Hence F(n, r) = (n+ 1)C(r+ 1) / nCr = (n+ 1y(r+ 1), as required. For k 2: 2, the set 2k-l, 2k , ... , 4k-2 gives n = 2k. For k 2: 3, so does the set 2k-5, 2k-4, ... ,
4k-6. For k 2: 2, the set 2k-2, 2k-3, ... , 4k-2 gives n = 2k+ l. For k 2: 4 so does the set 2k-
6,2k-5, ... , 4k-6. So we have at least one set for every n 2: 4, which answers (a).

•.. ProblemAl

Determine the maximum value of m2 + n2, where m and n are integers in the range 1, 2, ... ,
1981 satisfying (n2 - mn _ m2)2 = 1.
(b) We also have at least two sets for every n 2: 4 except possibly n = 4,5,7. For 5 we may
take as a second set: 8,9, 10, 11, 12,andfor7wemaytake6, 7,8,9,10,11,12. That
leaves n = 4. Suppose x is the largest number in a set with n =4. x cannot be divisible by 5
or any larger prime, because x-I, x-2, x-3 will not be. Moreover, x cannot be divisible by 4,
because then x-I and x-3 will be odd, and x-2 only divisible by 2 (not 4). Similarly, it


cannot be divisible by 9. So the only possibilities are 1, 2, 3, 6. But we also require x 2: 4,
which eliminates the first three. So the only solution for n = 4 is the one we have already
SolutioD found: 3, 4, 5,6.

Experimenting with small values suggests that the solutions ofn2 - mn - m2 = 1 or -1 are
successive Fibonacci numbers. So suppose n > m is a solution. This suggests trying m+n, n:
(m+n)" _(m+n)n - n2 = m2 + mn - n2 = -(n2 - mn - rrr') = 1 or -1. So ifn > m is a solution, Problem B2
then m+n, n is another solution. Running this forward from 2,1 gives 3,2; 5,3; 8,5; 13,8;
21,13; 34,21; 55,34; 89,55; 144,89; 233,144; 377,233; 610,377; 987,610; 1597,987; Three circles of equal radius have a common point O and lie inside a given triangle. Each
2584,1597. circle touches a pair of sides of the triangle. Prove that the incenter and the circumcenter of
the triangle are collinear with the point O.
But how do we know that there are no other solutions? The trick is to run the recurrence the
other way. For suppose n > m is a solution, then try m, n-m: m2 - m(n-m) - (n-mi = m2 +
mn - n2 = _(n2 - mn - m2) = 1 or -1, so that also satisfies the equation. Also ifm > 1, then m
> n-m (for ifnot, then n >= 2m, so n(n - m) >= 2m2, so n2 - nm - m2 >= m2 > 1). So given a SolutioD
solution n > m with m > 1, we have a smaller solution m > n-m. This process must
eventually terminate, so it must finish at a solution n, 1 wilh n > l. But the only such Let the triangle be ABC. Let the center of the circle touching AB and AC be D, the center

• solution is 2, 1. Hence the starting solution must have been in the forward sequence from 2,
1.
of the circle touching AB and BC be E, and the center of the circle touching AC and BC be
F. Because the circles center D and E have the same radius the perpendiculars from D and


E to AB have the same length, so DE is parallel to AB. Similarly EF is parallel to BC and
Hence Ihe solution to Ihe problem stated is 15972 + 9872. FD is parallel to CA. Hence DEF is similar and similarly oriented to ABC. Moreover D
must lie on the angle bisector of A since the circle center D touches AB and AC. Similarly

•...
ProbJem B1 E lies on Ihe angle bisector of B and F lies on Ihe angle bisector of C. Hence the incenter 1
of ABC is also Ihe incenter of DEF and acts as a center of symmetry so that corresponding
(a) For which n > 2 is there a set of n consecutive positive integers such that the largest points P of ABC and P' of DEF lie on a line Ihrough 1 wilh PIIP'I having a fixed ratio. But
number in the set is a divisor of the least common multiple of Ihe remaining n - 1 numbers? OD = OE = OF since the three circles have equal radii, so O is the circumcenter ofDEF.
Hence it lies on a line with 1 and Ihe circumcenter of ABC.
(b) For which n > 2 is there exactly one set having Ihis property?

• •



Problem BJ

The function f(x,y) satisfies: f(O,y) = y + 1, f(x+I,O) = f(x,I), f(x+ l,y+l) = f(x,f(x+ l.y) for ProblemAl
all non-negative integers x, y. Find f(4, 1981).
A non-isosceles triangle AIAzA3 has sides al, az, a3 with a¡ opposite A¡. M¡ is the midpoint
of side a¡ and T¡ is the point where the incircle touches side a¡. Denote by S¡ the reflection of
T¡ in the interior bisector of LA¡. Prove that the lines MIS!, MzSz and M3S3are concurren!.
Solution

I\I,n) = 1\0,f(1,n-I» = I + I\I,n-I). So f(1,n) = n + 1\1,0) = n + f(O,I) = n + 2.


Solution
Q2,n) = 1\1,1\2,n-I» = 1\2,0-1) + 2. So Q2,n) = 2n + 1\2,0) = 2n + f(I,I) = 2n + 3.
Let B¡ be the point of intersection of the interior angle bisector of the angle al A¡ with the
1\3,n) = 1\2,1\3,n-I» = 21\3,n-l) + 3. Let Un= 1\3,n) + 3, then Un= 211n-1.
Also ue = f(3,0) + 3 opposite side. The first step is to figure out which side of B¡T¡ lies. Let Al be the largest
= 1\2,1) + 3 = 8. So Un= 2n+3,and 1\3,n) = 20+3- 3. angle, followed by Az. Then Tzlies between Al and Bz, Tj Iies between Al and B3, and TI
lies between Az and BI' For LOBzAI = 180· - Al - Azl2 = A3 + Az/2. But A3 + Az/2 < Al +
1\4,n) = f{3,1\4,n-I» = 2f(4,n-1}t3- 3.1\4,0) = 1\3,1) = 24 - 3 = 13, We calculate two more Ay2 and their surn is 180·, so A3 + Ay2 < 90·. Hence Ts lies between Al and Bz. Similarly
terms 10see the pattern: 1\4,1) = 2Z4- 3, 1\4,2) = 2224 - 3, In fact it looks neater ifwe replace for the others,
4 by 2z, so that 1\4,n) is a tower ofn+3 2s less 3.
Let O be the center of the incircle. Then LTIOSz = LTIOTz - 2 LTzOBz = 180· - A3-
IMO 1982 2(90· - LOBzTz) = 2(A3 + Azl2) - A3 = Az + A3, A similar argument shows LTIOS3 = Az
ProblemAl + A3. Hence SZS3is parallel 10AzA3.
The function I\n) is defined on the positive integers and takes non-negative integer values.
Now LT30SZ = 360· - LT30TI - LTIOSz = 360· - (180· - Az) - (Az + A3) = 180· - A3 =
1\2) = O,1\3) > O, 1\9999) = 3333 and for all m, n:
Al + Az. LT30S1 = LT30TI + 2 LTIOBI = (180· - Az) + 2(90· - LOBITI) = 360· - Az-
I\m+n) - I\m) - I\n) = O or 1. 2(A3 + AI/2) = 2(AI + Az + A3) - Az - 2A3 - Al = Al + Az = LT 30SZ, So SISZ is parallel to
AIAz. Similarly we can show that SIS3 is parallel 10AIA3.
Determine 1\1982).
...
Solution

We show that 1\n) = [nl3 J for n <= 9999, where [ J denotes the integral part.
So SISZS3is similar to AIAzA3 and turned through 180·. But MIMzM3 is also similar 10
AIA2A3 and turned through 180·, So SIS2S3 and MIM2M3 are similar and similarly
oriented. Hence the lines through corresponding vertices are concurren!.

ProblemAl
..
We show first that 1\3) = 1. 1\1) must be O,otherwise 1\2) - 1\1) - 1\1) would be negative.
Hence 1\3) = 1\2) + 1\1) + Oor 1 = O or 1. But we are told 1\3) > O, so 1\3) = 1. It follows by Consider infinite sequences (xnl of positive reals such !hat xe = I and Xo 2':=XI 2':Xz2':....
induction that 1\3n) 2':n. For 1\3n+3) = 1\3) + 1\3n) + O or 1 = 1\3n) + 1 or 2. Moreoverifwe
ever get 1\3n) > n, then the sarne argument shows that 1\3m) > m for all m> n. But (a) Prove that for every such sequence there is an n 2':1 such that:
1\3.3333) = 3333, so 1\3n) = n for all n <= 3333.

Now 1\3n+l) = 1\3n) + 1\1) + Oor 1 = n om + 1. Bu! 3n+1 = 1\9n+3)2': 1\6n+2) + f(3n+l)2':
31\3n+ 1), so 1\3n+ 1) < n+ 1. Hence 1\3n+ 1) = n. Similarly, f{3n+2) = n. In particular (b) Find such a sequence for which:
1\1982) = 660.



• AMlAC = CN/CE = r.

• Solution Determine r if B, M and N are collinear.

• (a) It is sufficient to show that the surn ofthe (infinite) sequence is at least 4. Let k be the
greatest lower bound of the limits of all such sequences. Clearly k 2:: l. Given any E > O, we


can find a sequence {Xn} with surn less than k + E. But we may write the sum as: Solution

For an inelegant solution one can use coordinates. The advantage of this type of approach is


that it is quick and guaranteed to work! Take A as (O,-.J3), B as (I,-.J3), C as (3/2,-.J3/2, O as
The tenn in brackets is another surn ofthe sarne type, so it is at least k. Hence k + E > l/x¡ (1,0). Take the point X. coordinates (x,O), on ED. We find where the Jine BX cuts AC and
+ xjk, This holds for a11E > O, and so k 2:: I/x, + xjk. But l/x¡ + xjk 2:: 2-.Jk, so k 2:: 4. CE. The general point 00 BX is (k + (I-k)x,k-.J3). Ifthis is a1so the point M with AMlAC =

• (b) Let Xn= 1/2'. Then X<J2/Xl


+ X,2/X2+ ... + Xn_.2/xn= 2 + 1 + 1/2 + ... + 1/20-2= 4 _ 1120-2<
r then we have: k + (I -k)x = 3r/2, k-.J3 = (I -r)-.J3 + r-.J3/2. Hence k = 1 - rl2, r = 2/( 4-x).
Similarly, ifit is the point N with CN/CE = r, then k + (I-k)x = 3(I-r)l2, k-.J3 = (l-r)-.J312.
Hence k = (I -r)/2 and r = (2-xY(2+x). Hence for the ratios to be equal we require 2/(4-x) =


4.
(2-x)/(2+x), so X2- 8x + 4 = O. We a1so have x < 1, so x = 4 - -.J12.This gives r = 1/-.J3.

..•
A more elegant solution uses the ratio theorem for the triangle EBC. We have CMlMX
Problem Bl XBIBE ENINC = -1. Hence (I-r)/(r - 1/2) (-1/4) (I-r)/r = -1. So r = 1/-.J3.

Prove that if n is a positive integer such that the equation

...
ProblemB3

.. has a solution in integers x, y, then it has at least three such solutions. Show that the
equation has no solutions in integers for n = 2891.
Let S be a square with sides length 100. Let L be a path within S which does not meet itself
and which is composed of Jine segments ~" AlA2, A2A3, ... , An-,An with ~ = A n-
Suppose that for every point P on the boundary of S there is a point of L at a distance from
P no greater than 112.Prove that there are two points X and Y of L such that the distance


between X and y is not greater than 1 and the length of the part of L which lies between X
Solution and y is not smaller than 198.

Ifx, y is a solution then so is y-x, -x. Hence also -y, x-y. Ifthe first two are the sarne, then y
= -X, and X = y-x = -2x, so X = Y = O, which is impossible, since n > O. Similarly, if any
other pair are the same. Solution

• 2891 = 2 (mod 9) and there is no solution to x3 - 3xy2 + y3 = 2 (mod 9). The two cubes are Let the square be A'B'C'D'. The idea is to find points of L close to a particular point of A'D'
but either side of an excursion to B'.


each -1, O or 1, and the other tenn is O, 3 or 6, so the only solution is to have the cubes
congruent to I and -1 and the other tenn congruent to O. But the other term cannot be
congruent to O, unless one ofx, y is a multiple of3, in which case its cube is congruent to O, We say L approaches a point P' on the boundary ofthe square if there is a point P on L with


not 1 or-1. PP':S 112.We say L approaches P' before Q' ifthere is a point P 00 L which is nearer to ~
(the starting point ofL) than any point Q with QQ':S 112.

• Problem B2 Let A' be the first vertex of the square approached by L. L must subsequently approach
both B' and O'. Suppose it approaches B' first. Let B be the first point on L with BB':S 112.
The diagonals AC and CE ofthe regular hexagon ABCDEF are divided by inner points M We can now divide L into two parts Lj, the path from ~ to B, and L2, the path from B to

• and N respectively, so that: s;




,,3
Take X' to be the point on A'D' closest to D' which is approached by LI. Let X be the Let X be the other point of intersection of the two circles. Tbe key is to show that Az, Mz
corresponding point on LI. Now every point on X'D' must be approached by Lz (and X'D' is and X are col1inear, for then L MzAOz = L MzXOz (by reflection) and OzAzX is isosceles.
non-empty, because we know that D' is approached by L but not by LI). So by compactness
X' itself must be approached by Lz. Take Y to be the corresponding point on Lz. XY :5XX' But since O is the center of similitude, MzAz is parallel to MIA, and by reflection LXMzO
+ X'Y:5 1/2 + 1/2 = 1. Also BB':5 1/2, so XB 2: X'B' - XX' - BB' 2: X'B' - I 2: A'B' - I = 99. = L AMzO, so we need to show that triangle AMIMz is isosceles. Extend XA to meet PIPZ
Similarly YB 2: 99, so the path XY 2: 198. at Y. Then YPlz = YA. YX = yp/, so YX is the perpendicular bisector of MIMz, and hence
AMI = AMz as required.
IMO 1983

ProblemAI
ProblemA3
Find all functions f defined on the set ofpositive reals which take positive real values and
satisfy: Let a, b and e be positive integers, no two of which have a common divisor greater than l.
Show that 2abe - ab - be - ca is the largest integer which cannot be expressed in the form
Qx(f(y» = yQx) for all x, y; and Qx) --+ O as x --+ oo.
xbe + yca + zab, where x, y, z are non-negative integers.

Solution Solution
If flk) = 1, then f(x) = QxQk» = kf(x), so k =1. Let y = I/Qx) and set k = xf(y), then f(k) = We start with the lemma that be - b - c is the largest number which cannot be written as mb
QxQy» = yQx) = 1. Hence f(l) = I and QI/f(x» = l/x. Also QQy» = QIQy» = y. Hence + nc with m and n non-negative. [Proof: O, e, 2c, ... , (b-I)c is a complete set of residues
QI/x) = 1/Qx). Finally, let z = f(y), so that Qz) = y. Then Qxy) = QxQz» = zflx) = Qx)try). mod b. Ifr> (b-I)c - b, then r= nc (mod b) for sorne 0:5 n:5 b-I. But r> nc - b, so r= nc +
mb for sorne m 2: O. That proves that every number larger than be - b - e can be written as
Now notice that QxQx» = xftx), Let k = xffx), We show that k = 1. f(kz) = Qk)f(k) = kZand mb + nc with m and n non-negative. Now consider be - b - c. It is (b-I)c (mod b), and not
by a simple induction QkD) = kD, so we cannot have k > 1, or f(x) would not tend to O as x congruent 10 any nc with O :5n < b-I. So if be - b - e = mb + nc, then n 2: b-I. Hence mb +
tends 10 infinity. But QIIk) = IIk and the same argument shows that we cannot have IIk > nc 2: nc 2: (b-I)c > be - b - c. Contradiction.]
1. Hence k = 1.
O, be, 2be, ... , (a-I)be is a complete set of residues mod a. So given N > 2abe - ab - be - ca
So the only such function f is Qx) = l/x. we may take xbe = N (mod a) with O <= x < a. But N - xbe > 2abe - ab - be - ca - (a-I)be =
abe - ab - ca = a(bc - b - e), So N - xbc = ka, with k > be - b - c. Hence we can find non-
ProblemAl negative y, z so that k = zb + yc. Hence N = xbe + yca + zab.

Let A be one of the two distinct points of intersection of two unequal coplanar circles el Final1y, we show that for N = 2abe - ab - be - ca we cannot find non-negative x, y, z so that
and ez with centers 01 and Oz respectively. One ofthe common tangents to the circles N = xbc + yca + zab. N = -be (mod a), so we must have x = -1 (mod a) and hence x 2: a-1.
touches el at PI and ez at Pz, while the other touches el at QI and el at Qz. Let MI be the Similarly, y 2: b-I, and z 2: c-1. Hence xbe + yca + zab 2: 3abe - ab - be - ca > N.
midpoint ofPIQI and Mz the midpoint OfP1QZ. Prove that LOIAOz = LMIAMz. Contradiction.

Solution Problem BI
Let PIPZ and 0101 meet at O. Let OA meet ez again at Az. O is the center of similitude for Let ABC be an equilateral triangle and E the set of all points contained in the three
el and ez so LMIAOI = LMzAzOz. Hence ifwe can show that LMzAOz = LMzA10l, segments AB, Be and eA (including A, B and e). Determine whether, for every partition
then we are home. of E into two disjoint subsets, at least one of the two subsets contains the vertices of a right-
angled triangle.

• Let a, b and e be the lengths of the sides of a triangle. Prove that

• Solution a2b(a - b) + b2c(b- c) + c2a(c - a)::>:O.

• It does. Determine when equality occurs.


Suppose otherwise, that E is the disjoint union of e and e' with no right-angled triangles in
either set. Take points X, Y, Z two-thirds of the way along BC, CA, AB respectively (so
that BXlBC = 2/3 etc). Then two ofX, Y, Z must be in the same set. Suppose X and Y are Solution


in e. Now YX is perpendicularto BC, so all points ofBC apart from X must be in e'. Take
W to be the foot ofthe perpendicular from Z to BC. Then B and W are in e', so Z must be Put a = y + z, b = Z+ x, e = x + y. Then the triangle condition becomes simply x, y, Z> O.
in e. ZY is perpendicular to AC, so all points of AC apart from Y must be in e'. e' is now far The inequality becomes (after sorne manipulation):

• too big. For example let M be the midpoint of BC, then AMC is in e' and right-angled,
xy3 + yz3 + ZX3::>:
xyz(x + y + z).

Applying Cauchy's inequality we get (xy3 + yz3 + ZX3)(Z+ X+ y)::>:xyz(y + z + X)2with


equality iffxy3/z = yz3/X= zx3/y. So the inequality holds with equality iffx = y = Z. Thus


Problem 82 the original inequality holds with equality iffthe triangle is equilateral.

Is it possible to choose 1983 distinct positive integers, allless than or equal lo io', no three
of which are consecutive terms of an arithmetic progression?
IIIIO 1984
ProblemAl

Solution Prove that O :s yz + zx + xy - 2xyz :s 7/27, where x, y and z are non-negative real numbers
satisfying x + y + Z = 1.
We may notice that an efficient way to build up a set with no APs length 3 is as follows.
Having found 2" numbers in {I, 2, ... , un} we add the same pattem starting at 2un, thus
giving 2n+1numbers in {I, 2, ... , 3un-I}. Ifx is in the first part and y, z in the second part,
then 2y is at least 4un, whereas x + Z is less than 4un, so x, y, Zcanoot be an AP length 3. If Solution
x and y are in the first part, and z in the second part, then 2y is at most 2u., but x + z is


more than 2un, so x, y, z canoot be an AP length 3. To start the process off, we have the 4 (1-2x)(I-2y)(1 -2z)= 1 -2(x+y+z)+4(yz+zx+xy)-8xyz=4(yz+zx+xy)-8xyz-
numbers 1,2,4,5 in (I, 2, 3, 4, 5). SOU2= 5. This gives UII = 88574, in other words we 1. Hence yz + zx + xy - 2xyz = 1/4 (1 - 2x)(1 - 2y)(1 - 2z) + 1/4. By the
can find 2048 numbers in the first 88574 with no AP length 3. arithmetic/geometric mean theorem (I - 2x)(1 - 2y)(1 - 2z):S «1 - 2x + 1 - 2y + 1 - 2z)/3)3

• Ifwe are lucky, we may notice that ifwe reduce each number in the set we have
constructed by 1 we get the numbers which have no 2 when written base 3. This provides a
= 1/27. So yz + zx + xy - 2xyz :s 1/4 28/27 = 7/27.

ProblemAl

• neater approach. Take x, y, z with no 2 when written in base 3. Then 2y has only Osand 2s
when written base 3. But x + z only has no Is ifx = Z. SOx, y, z cannot form an AP length Find one pair ofpositive integers a, b such that ab(a+b) is not divisible by 7, but (a+b)" - a7


3. Also there are 211 = 2048 numbers ofthis type with 11 digits or less and hence:S - b7 is divisible by 77.
J1111111 1113= 88573.

Solution
Problem 83





,~
We find that (a + b)7 - a7 - b7 = 7ab(a + bXa2 + ab + b2i. So we must find a, b such that a2 + If AB and CD are parallel, then AB is tangent to the cirele on diameter CD if and only if
ab + b2 is divisible by 73 AB = CD and hence if and only if ABCD is a parallelogram. So the result is true,

At this point I found a = 18, b = I by tri al and error. Suppose then that AB and OC meet at O. Let M be the midpoint of AB and N the midpoint
ofCD. Let S be the foot ofthe perpendicular from N to AB, and T the foot ofthe
A more systematic argument tums on notieing that a2 + ab + b2 = (a3 - b3)/(a - b), so we are perpendicular fromM to CD. We are given that MT = MA. OMT, ONS are similar, so
looking for a, b with a3 = b3 (mod 73). We now remember that al'(m)= I (mod m). But q>(73) OMIMT = ON/NS and hence OB/OA = (ON - NS)/(ON + NS). So AB is tangent to the
= 2·3·49, so a3 = I (mod 343) ifa = n98. We find 298 = 18 (343), which gives the solution eirele on diameter CD ir and only if OB/OA = OC/OD whieh is the condition for BC lo be
18, l. parallel to AD.

This approach does not give a flood of solutions. n98 = O, 1, 18, or 324. So the only
solutions we get are 1,18; 18,324; 1,324.
Problem B2

Let d be the surn of the lengths of all the diagonals of aplane convex polygon with n > 3
ProblemAl vertices, Let p be its perimeter, Prove that:

Given points O and A in the plane. Every point in the plane is eolored with one of a finite n - 3 < 2d/p < [n/2) [(n+ 1)/2)- 2, where [x) denotes the greatest integer not exceeding x.
number of colors. Given a point X in the plane, the eirele C(X) has center O and radius OX
+ (LAOX)/OX, where LAOX is measured in radians in the range [O, 21t). Prove that we
can find a point X, not on OA, such that its color appears on the eireumference of the cirele
C(X). Solutioo

Given any diagonal AX, let B be the next vertex countereloekwise from A, and Y the next
vertex counterclockwise from X. Then the diagonals AX and BY interseet at K. AK + KB
Solutioo > AB and XK + KY > XY, so AX + BY > AB + XY. Keeping A fixed and surnming over
X gives n - 3 cases. So if we then sum over A we get every diagonal appearing four times
Suppose the result is false, Let CI be any cirele eenter O. Then the locus of points X such on the Ihs and every side appearing 2(n-3) times on the rhs, giving 4d > 2(n-3)p.
that C(X) = CI is a spiral from O to the point of intersection ofOA and CI. Every point of
this spiraI must be a different color from all points ofthe circle CI. Hence every cirele Denote the vertices as Ao, ... , An-I and take subscripts mod n. The ends of a diagonal AX
center O with radius smaller than C l must inelude a point of different color to those on C l. are connected by r sides and n-r sides, The idea of the upper limit is that its length is less
Suppose there are n colors, Then by taking successively smaller circles e2, C3, ... , Cn+1we than the surn of the shorter number of sides. Eval uating it is slightly awkward.
reach a contradietion, sinee each eirele ineludes a point of different color to those on any of
the larger circles, We considern odd and neven separately. Let n = 2m+1. Forthe diagonal A¡Ai+rwith r s m,
we have AiA¡+r~ A¡A¡+2+ ... + A¡A¡+r.Summing from r = 2 to m gives for the rhs (m-
I)A¡Ai+1+ (m-I)A¡+IA¡+2+ (m-2)A¡+2A¡+3+ (m-3)A¡+3Ai+4+ ... + I.A¡+m_IAi+m. Now
surnming over i gives d for the Ihs and p( (m-I) + (1 + 2 + ... + m-I» = p( (m2 + m - 2)/2 )
Problem Bl forthe rhs. So we get 2d/p ~ m2 + m - 2 = [n/2) [(n+I)/2)- 2.

Let ABCD be a convex quadrilateral with the line CD tangent to the eircle on diameter AS. Let n = 2m. As before we have A¡A¡+r<= A¡A¡+2+ ... + A¡A¡+rfor 2 ~ r ~ m-l. We may also
Prove that the line AB is tangent to the circle on diameter CD if and only if BC and AD are take A¡A¡+rn~ p/2. Surnming as in the even case we get 2d/p = m2 - 2 = [n/2) [(n+I)/2)- 2.
parallel.

Problem B3
Solution

..J



• Let a, b, e, d be odd integers such that o < a < b < e < d and ad = be. Prove that if a + d = 2k Solution

• and b + C = 2m for sorne integers k and m, then a = 1.


n and k are relatively prime. so O. k. 2k. ...• (n-l jk form a complete set ofresidues mod n.
So k. 2k, ...• (n-l jk are congruent to the numbers 1.2, ...• n-I in sorne order. Suppose ik is

• Solution
congruent to r and (i+l)k is congruent to s. Then either s = r + k, or s = r + k - n. lfs = r +
k. then we have immediately that r = s - k and s have the same color. lf s = r + k - n, then r


= n - (k - s), so r has the same color as k-s. and k - s has the same color as s. So in any case
a < e, so a(d - c) < e (d - e) and hence be - ac < c(d - c). So b - a < d - c. or a + d > b + C. so r and s have the same color. By giving i values from 1 to n-2 this establishes that all the
numbers have the same color.
k>m.

• be = ad, so b(2m - b) = a(2k - a). Hence b2 - a2 = 2~b - 2k-ma).But b2 - a2 = (b + a)(b - a).


and (b + a) and (b - a) cannot both be divisible by 4 (since a and b are odd), so 2m-1must

• divide b + a or b - a. But ifit divides b - a, then b - a 2': 2m-1.so b and e > 2m-1 and b + e >
2m.Contradiction. Hence 2m-1divides b + a. If b + a 2': 2m = b + e, then a 2': c. Contradiction.
ProblemAl

For any polynomial P(x) = ao + aix + ... + atXkwith integer coefficients, the number of odd


Hence b + a = 2m-l.
coefficients is denoted by o(P). For i = O. 1,2, ... let Qi(X)= (1 + xi. Prove that ifi .. h•...•
So we have b = 2m-1- a, C = 2m-1+ a, d = 2k - a. Now using bc = ad gives: 2ka = 22m-1 But a in are integers satisfying O :s i I < i2 < ... < in. then:


is odd, so a= 1.
O(Qil + Qi2+ ... + Qin) 2': o(Qn).
IMO 1985

•.. ProblemAl
Solution

.. A circ1e has center on the side AB of the cyc1ic quadrilateral ABCD. The other three sides
are tangent to the circ1e. Prove that AD + BC = AB.
If i is a power of 2, then all coefficients of Qi are even except the first and last. [There are
various ways to prove this. Let iCr denote the rth coefficient, so iCr = i!l(r!(i-r)!). Suppose
O < r < i. Then iCr = i-I Cr-I ut; but i-I Cr-I is an integer and i is divisible by a higher
power of 2 than r, hence iCr is even.]


Solution
Let Q = Qu + ... + Qin. We use induction on in. If i,= 1, then we must have n = 2. ii = O, and
i2 = 1, SO Q = 2 + x, which has the same number of odd coefficients as Qu = l. So suppose
Let the circle touch AD, CD, BC at L, M, N respectively. Take X on the line AD on the
it is true for smaller values of in. Take m a power of2 so that m s in < 2m. We consider two

• same side of A as D, so that AX = AO, where O is the center ofthe circle. Now the
triangles OLX and OMC are congruent: OL = OM = radius of circle, L OLX = L OMC = cases il 2': m and il < m.

••
90·, and LOXL = 90· - N2 = (1800 - A)/2 = C/2 (since ABCD is cyclic) = LOCM. Hence
Consider first il 2': m. Then Qu = (1 + xtA, Q = (1 + x)mB, where A and B have degree less
LX = MC. So OA = AL + MC. Similarly, OB = BN + MD. But MC = CN and MD = DL than m. Moreover, A and B are ofthe same form as Qil and Q, (all the ijs are reduced by ID,
(tangents have equallength), so AB = OA + OB = AL + LD + CN + NB = AD + BC. so we have o(A):S o(B) by induction. Also o(Qn) = 0«(1 + xtA) = oCA+ xmA) = 20(A):S
20(B) = o(B + xmB) = 0«1 + x)ms) = o(Q), which establishes the result for in.

ProblemAl lt remains to consider the case i I < m. Take r so that ir < m, ir+1> m. Set A = Qu + ... + Qir,

•.- Let n and k be relatively prime positive integers with k < n. Each number in the set M = (l.
2,3, ... , n-I) is colored either blue or white. For each i in M, both i and n-i have the same
color. For each i in M not equal to k, both i and li-kl have the same color. Prove that a11
numbers in M must have the same color.
(1 + x)mB = Qir+1+ ... + Qin. so that A and B have degree < m. Then o(Q) = oCA+ (1 +
x)mB) = oCA+ B + xmB) = oCA+ B) + o(B). Now oCA- B) + o(B) >= oCA- B + B) = oCA),
because a coefficient of A is only odd if just one of the corresponding coefficients of A - B
and B is odd. But oCA- B) = oCA+ B). because corresponding coefficients of A - B and A
+ B are either equal or ofthe same parity. Hence oCA+ B) + o(B) 2': oCA). But oCA) 2': O(Qii)


by induction. So we have established the result for in.

••
•..
Problem Bl LACN= 180° - LXCN, or we may have LXMN = 180° - LBMN= 180° - LBKN =
LAKN = 180° - LACN = 180° - LXCN.
Given a set M of 1985 distinct positive integers, none of which has a prime divisor greater
than 23, prove that M contains a subset of 4 elements whose product is the 4th power of an Now XM.XB = XK.xN = X02 - ON2. BM·BX = BN·BC = B02 - ON2, so XM·XB-
integer.
BM·BX = X02 - B02. But XM·XB - BM·BX = XB(XM - BM) = (XM + BMXXM - BM) =
XM2 - BM2. So X02 - B02 = XM2 - BM2. Hence OM is perpendicular to XB, or L OMB =
90°.
SolutioD

Suppose we have a set of at least 3.2·+ 1 numbers whose prime divisors are all taken from a
Problem BJ
set ofn. So each number can be written as PI'I ...pn'nfor sorne non-negative integers ri,
where Pi is the set of prime factors common to all the numbers. We classify each ri as even
For every real number XI, construct the sequence x.. x2, ... by setting:
or odd. That gives 2· possibilities. But there are more than 2· + I numbers, so two numbers
have the same classification and hence their product is a square. Remove those two and
Xn+1= xn(xn + l/n).
look at the remaining numbers. There are still more than 2n + 1, so we can find another pairo
We may repeat to find 2· + 1 pairs with a square product. [After removing 2· pairs, there
are still2· + 1 numbers left, which is just enough to find the final pair.] But we may now Prove that there exists exactly one value of x¡ which gives O< X. < Xn+1< 1 for all n.
classify these pairs according to whether each exponent in the square root oftheir product
is odd or even. We must find two pairs with the same classification. The product of these
four numbers is now a fourth power.
SolutioD
Applying this to the case given, there are 9 primes less than or equal to 23 (2, 3, 5, 7, 11,
13,17,19,23), so we need at least 3.512 + 1 = 1537 numbers for the argument to work Define So(x) = Xj. S.(x) = Sn.I(X)(S n-I(X)+ I/n). The motivation for this is that X. = So-I(XI).
(and we have 1985).
S.(O) = O and Sn(l) > 1 for all n> 1. Also S.(x) has non-negative coefficients, so it is
The key is to find the 4th power in two stages, by first finding lots of squares. If we try to strictly increasing in the range [0,1]. Hence we can find (unique) solutions a., b, to S,,(a.) =
1 - I/n, S.(bn) = 1.
go directly to a 4th power, this type of argument does not work (we certainly need more
than 5 numbers to be sure of finding four which sum to O mod 4, and 59 is far too big).
S.+I(a.) = S.(a.) (Sn(a.) + I/n) = l - l/n > 1 - I/(n+ 1), so a. < ~I. Similarly, Sn+I(b.) =
ProbIem B2 S.(b.) (S.(b.) + I/n) = 1 + I/n > 1, so b. > bn+l. Thus a. is an increasing sequence and hu is
a decreasing sequence with all a. less than all b n- So we can certainly find at least one point
XI which is greater than all the a. and less than all the bn- Hence 1 - I/n < S.(XI) < 1 for all
A circle center O passes through the vertices A and C of the triangle ABC and intersects the
n. But Sn(XI) = Xn+l.SOXn+1< 1 for all n. AIso X. > 1 - I/n implies that Xn+1= x.(xn + l/n) >
segrnents AB and BC again at distinct points K and N respectively. The circumcircles of
ABC and KBN intersect at exactly two distinct points B and M. Prove that angle OMB is a Xn·Finally, we obviously have Xn> O. So the resulting series X. satisfies all the required
right angle. conditions.

It remains to consider uniqueness. Suppose that there is an XI satisfying the conditions


given. Then we must have S.(XI) Iying in the range l - l/n, 1 for all n. [The lower limit
SolutioD follows from Xn+1= x.(x. + I/n).] Hence we must have a. < XI < b. for all n. We show
uniqueness by showing that b. - a. tends to zero as n tends to infinity. Since all the
coefficients of S.(x) are non-negative, it is has increasing derivative. S.(O) = Oand S.(b.) =
The three radical axes of the three circles taken in pairs, BM, NK and AC are concurrent.
Let X be the point of intersection. [They cannot all be para1lel or B and M would coincide.] 1, so for any X in the range O, b. we have S.(x):S xIb n- In particular, 1 - I/n < anlbu- Hence
bn - a.:s b, - b.(1 - l/n) = b.,ln < I/n, which tends to zero.
The first step is to show that XMNC is cyclic. The argument depends slightly on how the
points are arranged. We may have: LXMN = 180° - LBMN = LBKN = 180° - LAKN =
IMO 1986

..J
.J



• ProblemAl It remains to show that Ihe three rotations give a translation. Define reetangular coordinates

• Let d be any positive integer not equal to 2, 5 or 13. Show that one ean find distinet a, b in
the set {2, 5, 13, d} sueh that ab - 1 is not a perfeet square.
(x, y) by taking Al to be the origin and A, to be (a, b). Let A3 be (e, d). A cloekwise
rotation through 120 degrees about Ihe origin takes (x, y) to (-x/2 + y"3/2, -x"3/2 - y/2). A
clockwise rotation through 120 degrees about sorne oiher point (e, f) is obtained by

• subtracting (e, 1)to get (x - e, y - 1), the coordinates relative to (e, 1), Ihen rotating, then
adding (e, 1) to get the eoordinates relative to (O, O). Thus after the three rotations we wiH
end up with a linear eombination of x's, y's, a's, b's, c's and d's for eaeh eoordinate. But the

• SolutioD linear eombination of x's and y's must be just x for Ihe x-coordinate and y for the y-
eoordinate, sinee three sueeessive 120 degree rotations about Ihe same point is Ihe identity.


Consider residues mod 16. A perfect square must be O, 1,4 or 9 (mod 16). d must be 1, 5, 9, Hence we end up wilh simply (x + constant, y + eonstant), in other words, a translation.
or 13 for 2d - 1 to have one of these values. However, if d is 1 or 13, then 13d - 1 is not one
of Ihese values. If d is 5 or 9, then 5d - 1 is not one of these values. So we cannot have all [Of course, there is nolhing to stop you actually earrying out Ihe eomputation. It makes

• Ihree of 2d - 1, 5d - 1, 13d - 1 perfect squares. Ihings slightly easier to take the triangle to be (O, O), (1, O), (a, b). The net resuit turns out to
be (x, y) goes to (x + 3a12 - b"3/2, y - "3 + a"3/2 + 3bl2). For this to be the identity
Altemative solution from Mareo Dalai requires a = 112,b = "312. So Ihe third vertex must make the triangle equilateral (and it

• Suppose 2d-l, 5d-l, I3d-1 are all squares. Squares mod 4 must be Oor 1, eonsidering 2d-1,
must be on Ihe eorreet side of Ihe line joining the other two). This approach avoids Ihe need
for the argument in Ihe first paragraph aboye, but is rather harder work.]


so d must be odd. Put d = 2k+ l. Then IOk+4 = b'. So b must be even, so k must be even.
Put k = 2h, Ihen 5k+1 is a square. Similarly, 52h+ 12 is a square, so 13h+3 is a square. ProblemA3
Henee (13h+3)-(5h+1) = 8h+2 is a differenee oftwo squares, whieh is impossible (a

• difTerenee oftwo squares must be O, 1, or 3 mod 4). To each vertex of a regular pentagon an integer is assigned, so Ihat Ihe sum of all five
numbers is positive. Iflhree eonseeutive vertiees are assigned Ihe numbers x, y, z
ProblemAl respectively, and y < O, then Ihe foHowing operation is allowed: x, y, z are replaced by x +


y, -y, z + y respeetively. Sueh an operation is performed repeatedly as long as at least one
Given a point Po in Ihe plane ofthe triangle AIA,A3. Define As = As-3for all s ~ 4. ofthe five numbers is negative. Determine whether Ihis proeedure neeessarily eomes to an
Construet a set of points PI, P" P3, ... sueh Ihat Pk+1is the image of Pk under a rotation end after a finite number of steps.

• eenter Ak+1through 120· clockwise for k = 0,1,2, .... Prove Ihat ifP1986= Po, Ihen the
triangle AIA,A3 is equilateral.

• Solution


Solution Let S be Ihe sum of the absolute value of eaeh set of adjacent vertiees, so if Ihe integers are
a,~~~~Ih~S=~+~+~+~+~+~+~+~+~+~+~+~+~+~+~+~
The produet of three sueeessive rotations about the three vertiees of a triangle must be a +b+~+~+e+~+~+d+~+~+e+~+~+a+~+~+b+e+~+~+e+d+~

• translation (see below). But Ihat means that P1986(whieh is the result of662 sueh + le + d + e + al + Id + e + a + bl + le + a + b + el + la + b + e + d + el. Then Ihe operation
operations, sinee 1986 = 3 x 662) can only be Po if it is Ihe identity, for a translation by a reduces S, but S is a greater than zero, so Ihe proeess must terminate in a finite number of
non-zero amount would keep moving the point further away. It is now easy to show that it steps. So see that S is redueed, we can simply write out all Ihe terms. Suppose the integers

• can only be the identity if the triangle is equilateral. Take a eircle eenter Al, radius AIA,
and take P on the eircle so that a 120· elockwise rotation about Al brings P to A,. Take a
eircle eenter A3, radius A3A, and take Q on the eirele so that a 120· elockwise rotation
are a, b, e, d, e before the operation, and a+b, -b, b+c, d, e after it. We find that we mostly
get Ihe same terms before and after (although not in the same order), so that the sum S' after
Ihe operation is S -la + e + d + el + la + 2b + e + d + el. Certainly a + e + d + e > a + 2b + e

••• about A, takes A, to Q. Then successive 120· clockwise rotations about Al, A2, A3 take P
to Q. SO iflhese three are equivalent to the identity we must have P = Q. Henee LAIA,A3
= LAIA,P + LA3A,P = 30· + 30· = 60·. Also A,P = 2AIA,eos 30· and = 2A,A3eos 30· .
Henee AIA, = A,A3. So AIA,A, is equilateral. Note in passing Ihat it is not suffieient for
+ d + e sinee b is negative, and a + e + d + e > -(a + 2b+ e + d + e) beeause a + b + e + d + e
>0.

S is not lhe only expression we ean use. Jf we take T = (a - e/ + (b - d/ + (e - e/ + (d - o/


+(e - bl. then after replaeing a, b, e by a+b, -b, b+e, we gel T' = T + 2b(a + b + e + d +


Ihe triangle to be equilateral. We also have to take Ihe rotations in Ihe right order. If we
e) < T. Thanks lo Demetres Chrisofides for T
move around Ihe vertiees Ihe opposite way, then we get a net translation.




.J

Problem 81 O. But it also foHows that x + y ~ 2, and so f(x + y) = O and hence f{xf(y» f(y) = f(x + y) as
required. ¡f2x/(2 - y) < 2, then f(xf(y» f(y) = 2/(2 - 2x/(2-y)) 2/(2 - y) = 21{2- x - y) = f{x +
Let A, B be adjacent vertices of a regular n-gon (n ~ 5) with center O. A triangle XYZ, y). So the unique function satisfying the conditions is:
which is congruent to and initially coincides with OAB, moves in the plane in such a way
that Y and Z each trace out the whole boundary of the polygon, with X remaining inside the f{x) = O for x ~ 2, and 2/(2 - x) for O:S x < 2.
polygon. Find the locus of X.
Problem 83

Given a finite set of points in the plane, each with integer coordinates, is it always possible
Solutien to color the points red or white so that for any straight line L parallel to one of the
coordinate axes the difference (in absolute value) between the numbers ofwhite and red
Take AB = 2 and let M be the midpoint of AS. Take coordinates wíth origin at A, x-axis as points on L is not greater!han I?
AB and y-axis directed inside the n-gon. Let Z move along AB from B towards A. Let
L YZA be t. Let the coordinates ofX be (x, y). L YZX = 1fI2 -1fIn, so XZ = IIsin 1fInand y
= XZ sin(t + 1fI2 -1fIn) = sin t + cot 1fIn cos t.
Solution
BY sin 21f1n= YZ sin t = 2 sin t. MX = cot 1fIn. So x = MY cos t - BY cos 21f1n+ MX sin t
= cos t + (cot 1fIn- 2 cot 21f1n) sin t = cos t + tan 1fInsin t = Y tan 1fIn. Thus the locus ofX is Answer: yeso
a star formed of n lines segments emanating from O. X moves out from O to the tip of a
line segement and then back to O, then out along the next segment and so on. X2+ / = We prove the result by induction on the number n of points. It is clearly true for n = 1.
(lIsin21f1n + l/cos21f1n)cos'(t + 1fIn). Thus the length of each segment is (1 - cos 1fIn)/(sin Suppose it is true for all numbers less than n. Pick an arbitrary point P and color it red. Now
1fIncos 1fIn). take a point in the same row and color it white. Take a point in the same column as the new
point and color it red. Continue until either you run out of eligible points or you pick a
point in the same colurnn as P. The process must terminate because there are only finitely
many points. Suppose the last point picked is Q. Let S be the set of points picked.
Problem82
If Q is in the same column as P, then it is colored white (because the "same row" points are
all white, and the "same column" points are all red). Now every row and column contains
*
Find all functions f defined on the non-negative reals and taking non-negative real values
such that: f(2) = O, f{x) O for O:S x < 2, and f{xf{y» f(y) = f{x + y) for all x, y. an equal number of red points of S and of white points of S. By induction we can color the
points excluding those in S, then the difference between the numbers of red and white
points in each row and colurnn will be unaffected by adding the points in S and so we will
have a coloring for the whole set. This completes the induction for the case where Q is in
Solution the same column as P.

f(x+2) = f(xf(2» f(2) = O. So f(x) = O for all x ~ 2. If it is not, then continue the path backwards from P. In other words, pick a point in the
same column as P and color it white. Then pick a point in the same row as the new point
f(y) f(2-y)f(y» = f(2) = O. So ify < 2, then f(2-y) f(y» = O and hence (2 - y) f(y) ~ 2, or and color it red and so on. Continue until either you run out of eligible points or you pick a
f(y) ~ 2/(2 - y). point to pair with Q. If Q was picked as being in the same row as its predecessor, this
means a point in the same colurnn as Q; if Q was picked as being in the sarne column as its
Suppose that for some Yowe have f(yo) > 2/(2 - Yo),then we can find YI > yo (and YI < 2) so predecessor, this means a point in !he same row as Q. Again the process must terminate.
that f(yo) = 2/(2 - YI). Now let XI = 2 - YI. Then f(xlf(yo» = f(2) = O, so f(XI + Yo)= O. But XI Suppose the last point picked is R. Let S be the set of all points picked.
+ yo < 2. Contradiction. So we must have f(x) = 2/(2 - x) for all x < 2.

We have thus established Ihat if a function f meets the conditions then it must be defined as
aboye. It remains to prove that with Ihis definition f does meet Ihe conditions. CIearly f(2) =
If R pairs with Q, then we can complete Ihe coloring by induction as before. Suppose S
does not pair wilh Q. Then Ihere is a line (meaning a row or column) containing Q and no
uncolored points. There is also a line containing R and no uncolored points. These two lines
have an excess of one red or one white. AH other lines contain equal number of red and
....
O and f(x) is non-zero for O:s x < 2. f(xf(y» = f(2x/(2 - y». If2x/(2 - y) ~ 2, Ihen f(xf(y» =

..J
.J



• white points of S. Now color the points outside S by induction. This gives a coloring for the

• whole set, because no line with a color excess in S has any points outside S. So we have ProblemAl
completed the induction.
In an acute-angled triangle ABC the interior bisector of angle A meets BC at L and meets


the circumcircle of ABC again at N. From L perpendiculars are drawn to AB and AC, with
IMO 1987
feet K and M respectively. Prove that the quadrilateral AKNM and the triangle ABC have
ProblemAI
eq ual areas.

• l. Let pn(k) be the number ofpermutations


fixed points. Prove Lo" (k pn(k» = n!.
ofthe set {I, 2, 3, ..., n} which have exactly k

• Solution


by Gerhard W6ginger
Fint Solution
AKL and AML are congruent, so KM is perpendicular to AN and area AKNM =
We show first that the number of permutations of n objects with no fixed points is n!(I/O! -

• 1/1! + 1/2! - ... + (-I)"/n!). This follows immediately from the law ofinclusion and
exclusion: let N¡ be the number which fix i, N¡j the number which fix i and j, and so on.
KM.AN/2.
AKLM is cyclic (2 opposite right angles), so angle AKM = angle ALM and hence KMlsin
BAC = AMlsin AKM (sine rule) = AMlsin ALM = AL.


Then No, the number with no fixed points, is n! - all N¡ + all N¡j - ... + (-I)"NI..". But Ni =
ABL and ANC are similar, so AB.AC = AN.AL. Hence area ABC = 1/2 AB.AC sin BAC =
(n-I)!, Nij = (n-2)! and so on. So No = n! ( I - I/I! + ... + (-I)'(n-r)!/(r! (n-r)!) + ... + (-
1/2 AN.AL sin BAC = 1/2 AN.KM = area AKNM.
I)"/n!) = n! (l/O! - I/I! + ... + (-1)"/n!).

• Hence the number of permutations of n objects with exactly r fixed points = no. of ways of
choosing the r fixed points x no. of perms of the remaining n - r points with no fixed points
ProblemA3


Let XI, X2,... , Xnbe real numbers satisfying Xl2+ x/ + ... + xn2= 1. Prove that for every
= n!/(r! (n-r)!) x (n-r)! (l/O! - I/I! + ... + (-1 )";/(n-r)! ). Thus we wish to prove that the sum
integer k 2:: 2 there are integers al, a2, ... , a., not all zero, such that 1a¡1 ::sk - 1 for all i, and
fromr= 1 tonofl/(r-I)! (l/O! -I/I! + ... + (-I)""'/(n-r)! )is 1. We useinductiononn. Itis
true for n = 1. Supp,0se it is true for n. Then the sum for n+ 1 less the sum for n is: l/O! (- [ajx¡ + a2x2+ ... + a.xnl::S(k - 1)~nI(k· - 1).

• I)"/n! + I/I! (-I)""/(n-I)! + ... + I/n! l/O! = I/n! (1-1)"=0.


hence for all n.
Henceitistrueforn+ I,and

• Comment Solution

This is an application of the pigeon-hole principie.


This is a plodding solution. Ifyou happen to know the result for no fixed points (which
many people do), then it is essentially a routine induction.
Assume first that all x, are non-negative. Observe that the sum of the X¡is at most ~n. [This


is a well-known variant, (í..ISiSnxl::s n LI~iSnXi2,ofthe AM-GM result. See, for example,
Arthur Engel, Problem Solving Strategies, Springer 1998, p163, ISBN 0387982191].
Second solution


Consider the k· possible values of LI~ b;Xi,where each b; is an integer in the range [O,k-
Count all pairs (x, s) where s is a permutation with x a fixed point ofx. Clearly, ifwe fix x,
1]. Each value must lie in the interval [O, k-I ~n]. Divide this into kn_1equal subintervals.
then there are (n-I)! possible permutations s. So the total count is n!. But ifwe count the
Two values must lie in the same subinterval. Take their difference. Its coefficients are the

•••
number of permutations s with exactly k fixed points, then we get the surn in the question.
required a, Finally, if any x, are negative, solve for the absolute values and then flip signs
in the a¡ .
Comment

This much more elegant solution is due to Gerhard WOginger (email24 Aug 99). Comment


This solution is due to Gerhard Woeginger, email24 Aug 99.




Problem Bl Let n be an integer greater than or equal to 3. Prove that there is a set of n points in the
plane such that the distance between any two points is irrational and each set of 3 points
Prove that there is no function f from the set of non-negative integers into itself such that
f{f{n)) = n + 1987 for all n.
determines a non-degenerate triangle with rational area. ..1
..1
Solution
Solution

We prove that iff{f(n)) = n + k for all n, where k is a fixed positive integer, then k must be
even. Ifk = 2h, then we may take f{n) = n + h.

Suppose f(m) = n with m = n (mod k). Then by an easy induction on r we find f{m + kr) = n
+ kr, f{n + kr) = m + k(r+ 1). We show this leads to a contradiction. Suppose m < n, so n =
Let x. be the point with coordinates (n, n2) for n = 1,2,3, .... We show that the distance
between any two points is irrational and that the triangle determined by any 3 points has
non-zero rational area.

Take n > m. Ix. - xmlis the hypoteneuse of a triangle with sides n-m and n2 - m2 = (n -
mXn + m). So Ix. - xml= (n - m)"(1 + (n+mj'), Now(n + mi< (n + mi
+ 1 <rn + m + 1)2
..
..1
m + ks for sorne s > O. Then f{n) = flm + ks) = n + ks. But f{n) = m + k, so m = n + k(s - 1) = (n + m)2 + 1 + 2(n + m), so (n + m)2 + 1 is not a perfect square. Hence its square root is

.....
:::n. Contradiction. So we must have m z n, so m = n + ks for sorne s :::O. But now f{m + k) irrational. [For this we may use the classical argument. Let N' be a non-square and sugpose
= f{n + k(s+I)) = m + k(s + 2). But f(m + k) = n + k, so n = m + k(s + 1) > n. Contradiction. .JN' is rational. Since N' is a non-square we must be able to find a prime p such that p +1 ..1
divides N' but p2a+2does not divide N' for sorne a::: O. Define N = N'/p2a. Then "N =
So iff(m) = n, then m and n have ditTerent residues mod k. Suppose they have r¡ and r2 (VN')/p', which is also rational. So we have a prime p such that p divides N, but p2 does not
respectively. Then the same induction shows that al! sufficiently large s = r¡ (mod k) have r.
divide N. Take "N = r/s with r and s relatively prime. So s2N = Now p must divide r,
f{s) = r2 (mod k), and that all sufficiently large s = r2 (mod k) have f(s) = r¡ (mod k). Hence hence p2 divides r and so p divides S2 Hence p divides s. So r and s have a common factor.
if m has a ditTerent residue r mod k, then f{m) cannot have residue r¡ or r2. For if f(m) had Contradiction. Hence non-squares have irrational square roots.]
residue r¡, then the same argument would show that all sufficiently large numbers with
residue r¡ had f(m) = r (mod k). Thus the residues form pairs, so that if a number is
congruent to a particular residue, then f of the number is congruent to the pair of the
residue. But this is impossible for k odd.
Now take a < b < c. Let B be the point (b, a2), C the point (e, a2), and D the point (e, b2).
Area x.XbXc:= area x.xcC - area x,.x¡,B- area ""x.,D - area ""OCB = (e - aXc2 - a2)/2 - (b-
aXb2 - a2)/2 - (e - bXc2 - b2)12 - (e - bXb2 - a2) which is rationa!.
...
A better soluüon by Sawa Pavlov is as follows

Let N be the set of non-negative integers. Put A = N - f(N) (the set ofall n such that we Problem B3
cannot find m with f(m) = n). Put B = f{A).
Let n be an integer greater than or equal to 2. Prove that if k2 + k + n is prime for all
Note that f is injective because if f{n) = f{m), then f{f(n)) = f(f(m)) so m = n. We claim that integers k such that O ::; k::; "<nl3), then ~ + k + n is prime for all integers k such that O::; k
B = f(N) - f{ f(N) ). Obviously B is a subset of f(N) and ifk belongs to B, then it does not ::;n - 2.
belong to f{ f(N) ) since f is injective. Similarly, a member of f( f(N) ) cannot belong to B.

Clearly A and B are disjoint. They have union N - f{ f(N) ) which is {O, 1, 2, ... , 1986}. But
since f is injective they have the same number of elements, which is impossible since {O, 1, Solution
... , 1986} has an odd number of elements.
First observe that if m is relatively prime to b + 1, b + 2, ... , 2b - 1, 2b, then it is not
divisible by any number less than 2b. For if e <= b, then take the largestj :::O such that zic
::;b. Then zi+1c lies in the range b + 1, ... , 2b, so it is relatively prime to m. Hence c is also.
If we also have that (2b + 1i > m, then we can conclude that m must be prime, since if it
Problem B2 were composite it would have a factor::; "m. ..1

..J
..J



• Let o = 3? + h, where o S h < 6r + 3, so that r is the greatest integer less than or equal to

• ..,J(n/3). We also take r ~ l. That excludes the value o = 2, but for o = 2, the result is
vacuous, so nothing is lost. ProblemAl

• Assume that o + k(k+ 1) is prime for k = O, 1, ... , r. We show by induction that N = o + (r +


sXr + s + 1) is prime for s = 1, 2, ... , o - r - 2. By the observation aboye, it is sufficient to
Let o be a positive ioteger and let Al, A2, ... , A2n+1be subsets of a set B. Suppose that:
(i) Each A¡ has exactly 20 elements,


show that (2r + 2s + li > N, and that N is relatively prime to all ofr + s + 1, r + s + 2, ... , (ii) The intersection of every two distioct A¡ contains exactly ooe element, and
2r + 2s. We have (2r + 2s + 1)2= 4? + 8rs + 4s2 + 4r + 4s + l. Since r, s ~ 1, we have 4s + (iii) Every element of B belongs to at least two of the A¡.
i
I > s + 2, 4s2 > S2,and 6rs > 3r. Hence (2r + 2s + I > 4? + 2rs + S2+ 7r + s + 2 = 3? + 6r For which values of o can one assign to every element of B one of the numbers O and I in


+2+(r+sXr+s+ I»=N. such a way that A¡ has O assigned to exactly o of its elements?

Now ifN has a factor which divides 2r - i with i in the range -2s to r - s - 1, then so does N -

• (i + 2s + IX2r- i) = o + (r - i - s - IXr- i - s) which has the form 0+ s'(s'+I) with s' in the
range Oto r + s - l. But o + s'(s' + 1) is prime by induction (or absolutely for s = 1), so the
ooly way it can have a factor in common with 2r - i is if it divides 2r - i. But 2r - i S 2r + 2s
Solutioo

• 520 - 4 < 20 and o + s'(s' + 1) ~ o, so if n + s'(s' + 1) has a factor in common with 2r - i,


then it equals 2r - i = s + r + I + s'. Hence S,2= s - (o - r - 1) < O,which is not possible. So
Answer: o even,

..•
we can conclude that N is relatively prime to all of r + s + 1, ... , 2r + 2s and hence prime. Each ofthe 20 elements of A¡ belongs to at least one other Aj because of (iii). But given
another Aj it canoot contain more than one element of A¡ because of (ii). There are just 20
IMO 1988 other Aj available, so each must contain exactly one element of A¡. Hence we can
strengthen (iii) to every elernent ofB belongs to exactly two ofthe As .
ProblemAl
This shows that the arrangement is essentially unique. We may call the element ofB which

• Consider two coplanar circles of radii R > r with the same center. Let P be a fixed point 00 belongs to A¡ and Aj (ij). Then A¡ contains the 20 elements (i, j) withj not i.
the smaller circle and B a variable point 00 the larger circle. The line BP meets the larger
circle again at C. The perpendicular to BP at P meets the smaller circle again at A (if it is IBI= 1/2 x no. of As x size ofeach A = 0(20+1). Ifthe labeling with Os and Is is possible,

• tangent to the circle at P, then A = P). then if we Iist all the elements io each A, 0(20+ 1) out of the 20(20+ 1) elemeots have value
O. But each elemeot appears twice io this Iist, so 0(20+ 1) must be eveo. Heoce o must be


(i) Fiod the set ofvalues of AB2 + BC2 + CA2. eveo.
(ii) Fiod the locus of the midpoiot of BC.
Next part thanks to Stan Dolan

• SolutioD
Label (ij) O ifj = i-n/2, i-(n/2 - 1), ... ,i-I, i+l, i+2, ... ,i+n/2 (workiog mod 20+1 wheo
oecessary). This clearly has the required property.

• (i) Let M be the midpoiot ofBC. Let PM = x. Let BC meet the small circle agaio at Q. Let
O be the ceoter of the circles. Sioce angle APQ = 90 degrees, AQ is a diameter of the small
My original solution was a pedestrian iMuetion:

• circle, so its leogth is 2r. Heoce Apl = 4r - 4x2. BM2 = R2 - OM2 = R2 - (? - X2).That is
esseotially all we oeed, because we now have: AB2 + AC2 + BC2 = (Apl + (BM - xi) +
(Apl + (BM + xi) + 4BM2 = 2W + 6BM2 + 2x2 = 2(4r - 4x2) + 6(R2 -? + X2)+ 2x2 =
We show by ioductioo that a labeliog is always possible for n eveo. If n = 2, there is
certaioly a labeliog. For example, we may assign Oto (1,2), (1,3), (2,4), (3,5), (4,5). Now
suppose we have a labeling for n. For n + 2, we label (i ,j) O if it was labeled O for o or if it

•.- 6R2 + 2? , which is iodependent ofx.

(ii) Mis the midpoiot ofBC and PQ sioce the circles have a commoo ceoter. Ifwe shrink
the small circle by a factor 2 with P as ceoter, theo Q moves to M, and heoce the locus of M
is the circle diameter OP.
is:
(i, 20+2) or(i, 20+3) for i = 1,2, ... ,0+1
(i, 20+4) or (i, 2n+5) for i = n+2, n+3, ... , 2n+ I
(2n+2, 20+4), (20+3, 20+5), (2n+4,20+5).
For i = 1, 2, ... 0+ 1, A¡ has o elements (i, j) labeled zero with j S 20+ I and also (i, 20+2)


and (i, 20+3), giviog 0+2 io all. For i = 0+2, 0+3, ... ,20+1, A¡ has o elemeots (i,j) labeled



• '13
zero withj ~ 2n+ 1 and also (i, 2n+4) and (i, 2n+5), giving n+2 in all. A2n+2has the n+1 Problem BI
elements (i, 2n+2) with i <= n+1 and also (2n+2, 2n+4), giving n+2 in all. A2n+3has the n+I
elements (i, 2n+3) for i ~ n+ 1 and also (2n+3, 2n+5), giving n+2 in all. A2n+4has the n Show that the set of real numbers x which satisfy the inequality:
elements (i, 2n+4) with n+2 ~ i ~ 2n+ 1 and also (2n+2, 2n+4) and (2n+4, 2n+5), giving n+2
in all. Finally A2n+5has the n elements (i, 2n+5) with n+2 ~ i ~ 2n+ 1 and also (2n+ 3, 2n+5) I/(x - 1) + 2/(x - 2) + 3/(x - 3) + ... + 70/(x - 70) 2: 5/4
and (2n+4, 2n+5), giving n+2 in all.
is a union of disjoint intervals, !he sum of whose lengths is 1988.

ProblemA3
Solutioo
A function fis defined on the positive integers by: ftl) = 1; f(3) = 3; f(2n) = ttn), tt4n + 1)
= 2tt2n + 1) - f(n), and tt 4n + 3) = 3tt2n + 1) - 2ftn) for all positive integers n. Determine Let ttx) = l/(x - 1) + 2/(x - 2) + 3/(x - 3) + ... + 70/(x - 70). For any integer n, n/(x - n) is
the number of positive integers n less than or equal to 1988 for which ttn) = n. strictly monotonically decreasing except at x = n, where it is discontinuous. Hence ftx) is
strictly monotonically decreasing except at x = 1, 2, ... , 70. For n = any of 1, 2, ... , 70, n/(x
- n) tends to plus infinity as x tends to n from aboye, whilst the other terms m1(x - m)
remain bounded. Hence ñx) tends to plus infinity as x tends to n from aboye. Similarly, f(x)
Solutioo tends to minus infinity as x tends to n from below. Thus in each ofthe intervals (n, n+ 1) for
n = 1, ... ,69, ttx) decreases monotonically from plus infinity to minus infinity and hence
Answer: 92. ttx) = 5/4 has a single foot xn. Also ftx) 2: 5/4 for x in (n, x,J and flx) < 5/4 for x in (x¿
n+ 1). Ifx < O, then every term is negative and hence ftx) < 0< 5/4. Finally, as x tends to
fln) is always odd. Ifn = b,..¡br... b2b¡bo in binary and n is odd, so that b,..¡ = bo = 1, then f(n) infinity, every term tends to zero, so ftx) tends to zero. Hence f(x) decreases monotonicaHy
= b,..¡b¡h2...b,bo. Ifn has r+2 binary digits with r> O, then there are i<"'¡Y21numbers with
the central r digits symmetrical, so that ttn) = n (because we can choose the central digit
and those Iying before it arbitarily, the rest are then determined). Also there is one number
from plus infinity to zero over the range [70, infinity). Hence ttx) = 5/4 has a single root X70
in this range and ftx) >= 5/4 for x in (70, X70)and flx) < 5/4 for x > X70.Thus we have
established that flx) 2: 5/4 for x in any ofthe disjoint intervals (1, xi], (2, X2),... ,(70, X70) ...
with 1 digit (1) and one number with two digits (3) satisfying f(n) = 1. So we find a total of
1 + 1 +2+2+4+4+8+8+ 16+ 16=62numbersintherange

11111111111 and 111110 11111, giving another 30, or 92 in total.


1 to 1023withttn)=n.
1988 = 11111000011. So we also have all 32 numbers in the range 1023 to 2047 except for
and ftx) < 5/4 elsewhere.

The totallength ofthese intervals is (x¡ - 1) + ... + (X70- 70) = (x¡ + ... + X70)- (1 + ... + 70).
The x, are the roots ofthe 70th order polynomial obtained from I/(x - 1) + 2/(x - 2) + 3/(x-
...
3) + ... + 70/(x - 70) = 5/4 by multiJIlying both sides by (x - 1) ... (x - 70). The sum ofthe
It remains to prove the assertions aboye. ftn) odd foHows by an easy induction. Next we roots is minus the coefficient of x" divided by the coefficient ofx70. The coefficient ofx70
show that if 2m< 2n+ 1 < 2m+l,then tt2n+ 1) = ttn) + 2m• Again we use induction. It is true is simply k, and the coefficient OfX69 is - (1 + 2 + ... + 70)k - (1 + ... + 70). Hence the sum
for m = 1 (f(3) = ttl) + 2). So suppose it is true for 1, 2, ... , m. Take 4n+1 so that 2m+¡< ofthe roots is (1 + ... + 70XI + k)/k and the totallength ofthe intervals is (1 + ... + 70)/k =
4n+ 1 < 2m+2,then tt 4n+ 1) = 2f(2n+ 1) - f(n) = 2(ftn) + 2m) - ftn) = ftn) + 2m+¡ = ft2n) + 1/270,71 4/5 = 28·71 = 1988.
2m+¡,so it is true for 4n+ 1. Similarly, if 4n+3 satisfies, 2m+¡< 4n+3 < 2m+2,then ft4n+3) =
3ft2n+l) - 2ftn) = f(2n+l) + 2(ftn) + 2m) - 2ftn) = f(2n+l) + 2m+¡,so it is true for 4n+3 and
hence for m+ 1.
Problem5
FinaHy, we prove the formula for ft2n+I). Let 2n+1 = b,..¡br...b2b¡bo with bo = b,..¡ = 1. We
use induction on r. So assume it is tme for smaller values. Say b¡ = = b, = Oand b'+l = 1 ABC is a triangle, right-angled at A, and O is the foot of the altitude from A. The straight
(we may have s = O, so that we have simply b¡ = 1). Then n = b,..¡ b¡ and ttn) = line joining the incenters of the triangles ABO and ACO intersects the sides AB, AC at K,
b,..¡b'+2bS+3
...b,b,+¡ by induction. So f(n) + 2"'¡ = b,..¡0...Ob,..¡bs+2 b,bs+¡,where there are s L respectively. Show that the area ofthe triangle ABC is at least twice the area ofthe
zeros. But we may write this as b,..¡b¡...b,b,+¡...b,b,..¡, since b¡ = = b, = O,and bs+¡= b,..¡ = triangle AKL.
l. But that is the formula for tt2n+ 1), so we have completed the induction.

..J
..1



• SolutioD A slightly neater variation on this is due to Stan Dolan

• The key is to show that AK = AL = AD. We do this indirectly. Take K' on AB and L' on
AC so that AK' = AL' = AD. Let the perpendicular to AB at K' meet the line AO at X. Then
As aboye take a2 + b2 = k(ab + 1), so a, b, and k are all positive integers. Now fixing k take
positive integers A, B such that A2 + B2 = k(AB + 1) (*) and min(A,B) is as small as

•.. the triangles AK'X and ADB are congruent. Let J be the incenter of ADB and let r be the
in-radius of ADB. Then J lies on the angle bisector of angle BAO a distance r from the line
AD. Hence it is also the incenter of AK'X. Hence JK' bisects the right angle AK'X, so
possible. Assume B ::;A. Regarding (*) as a quadratic for A, we see that the other root C
satisfies A + C = kB, AC = B2 - k. The second equation implies that C = B2/A - k/A < B. So
C cannot be a positive integer (or the solution C, B would have min(C,B) < min(A,B». But

.. L AK'J = 45° and so J lies on K'L'. An exactly similar argument shows that 1, the incenter we have (A+IXC+l)= A+C + AC + 1 = B2 + (B-l)k + 1 > O, soC > -1. C = kB - A isan
of AOC, also lies on K'L'. Hence we can identify K and K', and L and L'. integer, so C = O. Hence k = B2.

The area of AKL is AK'AL12 = AD2/2, and the area of ABC is BC'ADI2, so we wish to Note thatjumping straight to the minimal withoia the infinite deseent avoids some of the
show that 2AD::; BC. Let M be the midpoint ofBC. Then AM is the hypoteneuse of AMD, verifieation needed in the infinite deseent.
so AM ~ AD with equality if and only if D = M. Hence 2AD ::;2AM = BC with equality if
and only if AB = AC.

IMO 1989

ProblemAl

..•
Problem B3

Let a and b be positive integers such that ab + 1 divides a2 + b2. Show that (a2 + b2)/(ab + 1) Prove that the set {1, 2, ... , 1989} can be expressed as the disjoint union ofsubsets Al, A2,
... , AII7 in such a way that each A¡ contains 17 elements and the sum ofthe elements in

..
is a perfect square .
each A¡ is the same .

SolutioD
SolutioD

•.. A little experimentation reveals the following solutions: a, a3 giving a2; a3, aS- a giving a2;
and the recursive al = 2, b¡ = 8, an+1= bn, bn+1= 4b. - Ro giving 4. The latter may lead us to:
ifa2 + b2 = k(ab + 1), then take A = b, B = kb - a, and then A2 + B2 = k(AB + 1). Finally,
we may notice that this can be used to go down as well as up.
We construct 116 sets ofthree numbers. Each set sums to 3 x 995 = 2985. The 348
numbers involved form 174 pairs Ir, 1990 - r}. At this point we are essentially done. We
take a 117th set which has one Ir, 1990 - r} pair and 995. The original 1989 numbers
comprise 995 and 994 Ir, 1990 - r} pairs. We have used up 995 and 175 pairs, leavingjust


So starting again suppose that a, b, k is a solution in positive integers to a2 + b2 = k(ab + 1). 819 pairs. We now add 7 pairs to each of our 117 sets, bringing the total of each set up to
lf a = b, then 2a2 = k(a2 + 1). So a2 must divide k. But that implies that a = b = k = 1. Let us 2985 + 7.1990 = 1990 x 17/2.
assume we do not have this trivial solution, so we may take a < b. We also show that a3 > b.

• For (b/a - l/aXab + 1) = b2 + b/a - b - l/a < b2 < a2 + b2. So k > b/a - l/a. But if a3 < b, then lt remains to exhibit the 116 sets. There are many possibilities. We start with:
b/a (ab + 1) > b2 + a2, so k < b/a. But now b > ale and < ale + 1, which is impossible. lt 301,801,1883 and the "complementary" set 1990 - 301 = 1689, 1990 - 801 = 1189, 1990-
follows that k ~ b/a 1883 = 107. We then add one to each ofthe first two numbers to get:

• Now define A = ka - b, B = a. Then we can easily verify that A, B, k also satisfies a2 + b2 =


302,802,1881 and 1688, 1188, 109, and so on:
303, 803, 1879 and 1687, 1187, 111,

•••
k(ab + 1), and B and k are positive integers. Also a < b implies a2 + b2 < ab + b2 < ab + b2 +
1 +b/a=(ab+ IX1 +b/a),andhencek< 1 +b/a,soka-b<a Finally, since k> b/a, ka-b 358,858,1769 and 1632, 1132,221.
~ O. lfka - b > O, then we have another smaller solution, in which case we can repeat the We can immediately see that these triples are all disjoint. So the construction is complete.
process. But we cannot have an infinite sequence of decreasing numbers all greater than
zero, so we must eventually get A = ka - b = O. But now A2 + B2 = k(AB + 1), so k = B2. k

l.
was unchanged during the descent, so k is a perfect square.


ProblemAl

l.

In an acute-angled triangle ABC, the internal bisector of angle A meets the circurncircle ProblemAJ
again at A,. Points B, and C, are defined similarly. Let Ao be the point ofintersection of the
line AA, with the external bisectors of angles B and C. Points Bo and Co are defined Let n and k be positive integers, and let S be a set of n points in the plane such that no three
similarly. Prove that the area of the triangle AoSoCo is twice the area of the hexagon points of S are collinear, and for any point P of S there are at least k points of S equidistant
AC,BA,CB, and at least four times the area of the triangle ABC. from P. Prove that k < 1/2 + ..,J(2n).

Solutlon SolutioD ...


By Marcin Mazur, University of lllinois at Urbana-Champaign

Let I be the point of intersection of AAo, BSo, CCo (the in-center). BIC = 180 - 1/2 ABC -
Three variants on a theme, all kindly supplied by others (1 spent 2 hours failing to solve it).
My favorite first.
...
1/2 BCA = 180 - 1/2 (180 - CAB) = 90 + 1/2 CAB. Hence CA,B = 180 - CAB [BA,CA is
cyclic) = 2(180 - BIC) = 2CAoB. But A,B = A,C, so A, is the center of the circurncircle of By EIi Bachmtasky

..
BCAo. But I lies on this circurncircle (IBAo = ICAo = 90), and hence A,Ao = A,I.
Consider the pairs P, (A, B), where P, A, B are points ofS, and P lies on the perpendicular
Hence area IBA, = area AoBA, and area ICA, = area AoCA,. Hence area IBAoC = 2 area bisector of AB. There are at least n k(k - 1)/2 such pairs, because for each point P, there are
!BA,C. Similarly, area ICSoA = 2 area ICB,A and area IACoB = 2 area IAC,B. Hence area at least k points equidistant from P and hence at least k(k - 1)/2 pairs of points equidistant
AoBoCo= 2 area hexagon AB,CA,BC,. from P.

A neat solution for the rest is as follows by Thomas Jager If k > 1/2 + ..,J(2n), then k(k - 1);:::2n - 1/4> 2(n - 1), and so there are more than n(n - 1)
pairs P, (A, B). But there are only n(n - 1)/2 possible pairs (A, B), so for sorne (Ao, Bo)
Let H be the orthocentre of ABC. Let H, be the reflection ofH in BC, so H, lies on the we must be able lo find at least 3 points P on the perpendicular bisector of AoBo. But these
circumcircle. So area BCH = area BCH, <= area BCA,. Adding to the two similar points are collinear, contradicting the assumption in the question.
inequalities gives area ABC <= area hexagon - area ABC.

Mazur's solution was as follows From an anonymous source

..
CAB = 1800 - CA,B and A,B = A,C, so A,BC = 900 - 1/2 CA,B = 1/2 CAB. Hence the Let the points be PI. P2, ... , PR• Let C¡ he a circle center P, containing at least k points ofS.
perpendicular from A, lo BC has length 1/2 BC tan(CAB/2) and area CA,B = 1/4 BC2 There are at least nk pairs (C¡,Pj), where Pj lies on C, Hence there must a point P Iying on at
tan(CAB/2). least k circles. Take k such circles C u- For each such circle C.. take a subset Sa comprising
exactly k points of S n C•.
Put r = radius of in-circle of ABC, x = cot(CAB/2), y = cot(ABC/2), z = cot(BCAI2). Then
BC = rey + z) and area CA,B = rey + z)2/(4x). AIso area BIC = 1/2 r BC. Similarly for the We now count the points in USa. Apart from P, there are k-1 points in each SaoSo we start
other triangles, so area ABC = area BIC + area CIA + area AIB = r(x + y + z). We have to with I + k(k-I). But this counts sorne points more than once. Each pair (S.. S~)(with a 1: fJ)

...
show that area ABC S area CA,B + area AB,C + area BC,A, or (x + y + z) S (y + z)2/(4x) + has at most one common point apart from P (because distinct circles have at most two

..
(z + xi/(4y) + (x + y)2/(4z). common points). So we deduct 1 for each of the 1/2 k(k - 1) pairs (S., S~), giving 1 + k(k -
1) - 1/2 k(k - 1)(*).
Putting s = x + y + z, this is equivalent to: 4s S (s - xi/x + (s - y)2/y + (s - Z)2/Z,or 9s S
s2(I/x + l/y + lIz), but this isjust the statement that the arithmetic mean oh, y, z is not If Q (1: P) is in exactly r sets S.. then it is counted r times in the second term k(k - 1), and
less than the harmonic mean. subtracted 1/2 r(r - 1) times in the third termo So it is counted 1/2 r(3 - r) times in all, That is
correct for r = O, 1 or 2 and too low for r > 2. So (*) is S I U Sol.Clearly I U SolS n, so n;::: 1
Note in passing that the requirement for ABC to be acute is unnecessary. + k(k - 1)12 = (k - 1/2i/2 + 7/8 > (k - 1/2)2/2. Hence ..,J(2n) + 1/2> k. ....



• Prove that for each positive integer n there exist n consecutive positive integers none of

• By Thomas Jager which is a prime or a prime power.

Define Pi and Si as aboye. Let g(i) be the number of S. containing Pi, and let f(i, j) be ISin

• Sjl.Let h(x) = x(x· ly2. We count the number N ofpairs i, {a, b}, where point i is in S.
and Sb. Solution

• Point i is in g(i) sets Sj, from which we can choose S",Sbin h(g(i)) ways. Hence N = L
h(g(i)). But f(a, b) points are in S. and Sb, so N = L f(a, b). Bu1,since distinct circles
Consider (N!l+2, (N!l+3, ... ,(N!l+N. (N!i+r is divisible by r, but «N!l+ryr = N! (N!/r)
+ 1, which is greater than one, but relatively prime to r since N! (N!/r) is divisible by r. For


intersect in at most 2 points, fla, b):5 2, so L f(a, b):5 h(n) 2. We conclude that 2 h(n) ~ L each r we may take a prime p, dividing r, so (N!l+r is divisible by Pr, but is not a power of
h(g(i)). Pr. Hence it is not a prime or a prime power. Taking N = n+1 gives n consecutive numbers
as required.
h is a convex function, so IIn L h(g(i)) ~ h(l/n L g(i)) = n h(1/n nk) = n h(k). Hence n- 1 ~
h(k), which gives the result, as aboye.
Problem B3

A permutation {x¡ X2,... , Xm}ofthe set {l, 2, ... , 2n} where n is a positive integer is said
to have property P if IXi- xi+ll= n for at least one i in {l, 2, ... , 2n-I}. Show that for each n
Problem Bl there are more permutations with property P than without.

• Let ABCD be a convex quadrilateral such that the sides AB, AD, BC satisfy AB = AD +
BC. There exists a point P inside the quadrilateral at a distance h from the line CD such that

• AP= h + AD and BP= h + BC. Show that:

1"/h ~ I/'¡AD + II'¡BC.


Solution

from Arthur Engel, Problem-Solving Strategies, Springer 1998 [Problem books in

• mathematics series], ISBN 0387982/9/. A rather good training book:

Let A.. be the set of permutations with k and k+n in neighboring positions, and let A be the

• Solution set of permutations with property P, so that A is the union of the Ak.


by Gerhard Woginger, Technical University, Graz Then IAI= Sum, IAkl- Sumk<lIAknA,1+ Sumk<l<m IAknA,nAml- .... But this is an
altemating sequence of monotonically decreasing terms, hence IAI~ Lk IAkl- Sumk<l
Let CAbe the circle center A, radius AD, and Ce the circle center B, radius BC. The circles IAknA'¡.

• touch on AB. Let Cp the the circle center P, radius h. Cp touches CAand Ce and CD. Let t
be the common tangent to CAand Cs whose two points of contact are on the same side of
AB as C and D. Then Cp is confined inside the curvilinear triangle whose sides are
But IAkl= 2 (2n - 1)! (two orders for k, k+n and then (2n - I)! ways of arranging the 2n - 1
items, treating k, k+n as a single item). Similarly, IAtnA,1= 4 (2n - 2)! SOIAI~ (2n - 2)!

• segments of t, CAand Cs. Evidently h attains its maximum value, for given lengths AB,
AD, BC, when Cptouches 1, in which case D must be the point at which t touches CA,and
[n.2(2n -1) - n(n - IY2 4] = 2n2 (2n - 2)! > (2n)!/2.

IMO 1990

•••
C the point at which it touches Cs. Suppose E is the point at which t touches CP.

An~les ADC and BCD are right angles, so CD2= AB2 - (AD - Bci = 4 AD BC. Similarly, ProblemAl
DE = 4 h AD, and CE2 = 4 h BC. But CD = DE + CE, so I/'¡h = lI'¡AD + II'¡BC. This
gives the maximum value of h, so in general we have the inequality stated. Chords AB and CD of a circle intersect at a point E inside the circle. Let M be an interior
point of the segment EB. The tangent at E to the circle through D, E and M intersects the

• Problem B2 lines BC and AC at F and G respectively. Find EFIEG in terms oft = AMlAB.



• f.¡S
consecutive numbers colored black. But this means that at most n - l out ofthe 2n - l
numbers in the sequence can be black. This establishes the result for n = 3m.
Solution Take n = 3m + 1. Abad coloring with n - l black points has the following black points: 1,
By Theo Koupelis, University of Wisconsin, Marathon

LECF = LOCB (same angle)= LDAB (ACBD is cyelic)= LMAD(same angle). Also
LCEF = LEMD (GE tangent to cirele EMD) = LAMD(same angle). So triangles CEF
and AMD are similar.
4,7, ... ,3m - 2 (m points) and 2, 5, 8, ... ,6m - 1 (2m points). As before we add n - 2
repeatedly starting wilh I to get: 1, 3m, 6m -1, 3m - 3, 6m - 4, ... ,3, 3m + 2, 6m + 1, 3m-
1, ... ,2, 3m + 1, 6m, 3m - 2, ... ,1. No two consecutive numbers can be black in abad set,
so a bad set can have at most n - I points.

Finally, take n = 3m + 2. Abad coloring with n - 2 points is 1,2, ... ,n - 2. This time when
we add n - 2 = 3m repeatedly starting wilh 1, we get back to I after including only one-third
.....
...
LCEG = 1800 - LCEF = 1800 - LEMD = LBMD. AIso LECG = LACD{same angle) ofthe numbers: 1, 3m + 1, 6m + 1, 3m - 2, ... ,4, 3m + 4,1. The usual argument shows that
= LABD (BCAD is cyelic) = LMBD (same angle). So triangles CEG and BMD are at most m oflhese 2m + I numbers can be colored black in a bad set. Similarly, we may
similar. add 3m repeatedly starting wilh 2 to get another 2m + I numbers: 2, 3m + 2, 6m + 2, 3m -

Hence EF/CE = MD/AM, EG/CE = MD/BM, and so dividing, EF/EG = BMlAM = (1- t)/t.

ProblemAl
l 3m + 5 2 At most m ofthese can be black in abad set. Similarly at most m oflhe
2~'~' I numbers: 3, 3m + 3, 6m + 3, 3m, ... , 3m + 6,3 can be black. So in total at most 3m
= n - 2 can be black in abad set.
...
ProblemA3
Take n > 3 and consider a set E of 2n-1 distinct points on a cirele. Suppose that exactly k of
lhese points are to be colored black. Such a coloring is "good" if there is at least one pai~ of Determine all integers greater than I such that (2' + I )ln2 is an integer.
black points such that lhe interior of one of the ares between lhem contams exactly n pomts
from E. Find the smallest value of k so that every such coloring of k points of E is good.

Solution ..1
Snlution by Gerhard Woginger, Technical University, Graz
..1
Answer: n for n = O or I (mod 3), n - I for n = 2 (mod 3).

..
Answer: n = 3.

Label the points I to 2n - l. Two points have exactly n points between them if their Since 2" + 1 is odd, n must also be odd. Let p be its smallest prime divisor. Let x be the
difference (mod 2n - 1) is n - 2 or n + 1. We consider separately the three cases n = 3m, 3m smallest positive integer such lhat 2x = -1 (mod p), and let y be the smallest positive integer
+ I and3m+2. such that 2Y = l (mod p). y certainly exists and indeed y < p, since 21'"1= 1 (mod p). x exists
since 2' = -1 (mod p). Write n = ys + r, wilh 0:5 r < y. Then - l = 2' = (2Y)'2' = 2' (mod p),
Let n = 3m. First, we exhibit a bad coloring with n - 1 black points. Take the black points to so x:5 r < y (r cannot be O, since - 1 is not 1 (mod p) ),
be 1,4,7, ... , 6m - 2 (2m points) and 2,5,8, ... , 3m - 4 (m - 1 points). It is easy to check
lhat this is bad. The two points which could pair with r to give n points between are r + 3m Now write n = hx + le, with 0:5 k < x. Then -1 = 2" = (_I)hzk (mod p). Suppose k > O. Then
_ 2 and r + 3m + 1. Considering lhe first ofthese, 1,4,7, ... , 6m - 2 would pair with 3m - 1, if h is odd we contradict the minimality of y, and if h is even we contradict the minimality
3m + 2, 3m + 5, ... , 6m - 1, 3, 6, ... , 3m - 6, none ofwhich are black. Considering the of x. So k = Oand x divides n. But x < p and p is the smallest prime dividing n, so x = 1. ..1
second, they would pair with 3m + 2, 3m + 5, ... ,6m - 1,3, ... ,3m - 3, none ofwhich ~
black. Similarly, 2, 5, 8, ... ,3m - 4 would pair with 3m, 3m + 3, ... ,6m - 3, none of'which
are black. So the set is bad.
Hence 2 = -1 (mod p) and so p = 3.

Now suppose that 3m is the largest power of3 dividing n. We show that m must be 1.
Expand (3 - 1)' + 1 by the binomial theorem, to get (since n is odd): 1 - 1 + n.3 - 1/2 n(n-
...
Now if we start wilh 1 and keep adding 3m - 2, reducing by 6m - 1 when necessary to keep 1) 32 + ... = 3n - (n - 1)12n 32 + .... Evidently 3n is divisible by 3m+l, but not 3m+2.We show
the result in the range 1, ... , 6m - 1, we eventually get back to 1: 1, 3m - 1, 6m - 3, 3m - 4, lhat the remaining terms are all divisible by 3m+2.lt follows that 3m+! is the highest power 3
6m - 6, ... ,2, 3m, 6m - 2, 3m - 3, 6m - 5, ... ,3, 3m + 1, 6m - 1, ... ,4, 3m + 2,1. The dividing 2" + 1. But 2' + 1 is divisible by n2 and hence by 32m,so m must be 1.
sequence ineludes all 6m - 1 numbers. Moreover a bad coloring cannot have any two

..J
.J



• The general tenn is (3ma)Cb 3b, for b::::3. The binomial coefficients are integral, so the tenn Knowing n2k+I.B chooses any integer n2k+2such that n2k+l/n2k+2= p' for sorne prime p and

• is certainly divisible by 3m+2for b::::m+2. We may write the binomial coefficient as (3ma/b) integer r:::: 1.
(3m _ 1)/1 (3m - 2)/2 (3m - 3)/3 ... (3m - (b-I)) / (b - 1). For b not a multiple 00, the first
tenn has the fonn 3mc/d, where 3 does not divide e or d, and the remaining tenns have the Player A wins the game by choosing the number 1990; player B wins by choosing the

•.. fonn c/d, where 3 does not divide e or d. So if b is not a multiple of 3, then the binomial
coefficient is divisible by 3m, since b > 3, this means that the whole tenn is divisible by at
least 3m+3.Similarly, for b a multiple of 3, the whole tenn has the same maximum power of
3 dividing it as 3m 3b/b. But b is at least 3, so 3b/b is divisible by at least 9, and hence the
whole tenn is divisible by at least 3mH
number 1. For which 110does
(a) A have a winning strategy?
(b) B have a winning strategy?
(e) Neither player have a winning strategy?

• We may check that n = 3 is a solution. Ifn > 3, let n = 3 t and let q be the smallest prime
divisor oft. Let w be the smallest positive integer for which 2'" = -1 (mod q), and v the Solution

• smallest positive integer for which 2' = I (mod q). v certainly exists and < q since 2q-1= I
(mod q). 2° = -1 (mod q), so w exists and, as before, w < v. AIso as before, we conclude
that w divides n. But w < q, the smallest prime divisor of n, except 3. So w = I or 3. These
Answer: ifno = 2, 3, 4 or 5 then A loses; ifno:::: 8, then A wins; ifllo = 6 or 7 ,!hen it is a
draw.

• do not work, because then 2 = -1 (mod q) and so q = 3, or 23 = -1 (mod q) and again q =3,
whereas we know!hat q > 3. A's strategy given a number n is as follows:
(1) ifn E [8, 11], pick 60

..• Problem Bl

Construct a function from the set ofpositive rational numbers into itselfsuch that f(x f(y))
= ttx)/y for all x, y.
(2) ifn E [12,16], pick 140
(3) ifn E [17,22], pick 280
(4) ifn E [23,44], pick 504
(5) ifn E [45, 1990], pick 1990
(6) ifn = 1991 = 11.181 (181 is prime), pick 1991
(7)ifn E [11'181 + 1, 1Ir+1181] forsome r>O, pick 1Ir+1181.

Solution
Clearly (5) wins immediately for A. After (4) B has 7.8.9 so must pick 56, 63, 72 or 168,
JI We show firstthat ttl) = 1. Taking x = y = 1, we have ttttl)) = f(I). Hence ttl) = ttttl)) =
which gives A an immediate win by (5). After (3) B must pick 35, 40, 56, 70 or 140, so A
wins by (4) and (5). After (2) B must pick 20, 28, 35 or 70, so A wins by (3) - (5). After (1)


ttl ttf(I)))=ttl)/ttl)= 1.
B must pick 12, 15,20 or 30, so A wins by (2) - (5).
Next we show that ttxy) = f(x)tty). For any y we have I = ttl) = ttl/f(y) tty)) = f(l/tty))/y,
IfB is given II r+1181,then B must pick 181, 11.181, ... , 11'.181 or II r+1,all ofwhich are s


so ifz = l/f(y) then ttz) = y. Hence ttxy) = f(xttz)) = f(x)/z = ttx) tty)·
II '.181. So if A is given a number n in (6) or (7) then after a turn each A is given a number
< n (and >= 11), so after a finite number ofturns A wins.
Finally, ttf(x)) = ttl f(x)) = ttl)/x = l/x.

• We are not required to find all functions, just one. So divide the primes into two infinite
sets S = {PI, P2, ... l and T= (qI. q2, ... l·Define ttp.) = qo, and tt q.) = l/p n- We extend this
If B gets a number less than 6, then he can pick I and win. Hence if A is given 2, he loses,
because he must pick a number less than 5. Now if B gets a number of II or less, he wins


by picking I or 2. Hence if A is given 3, he loses, because he must pick a number less than
definition 10all rationals using f(xy) = f(x) fly): ttPiIPi2..·ljjlljj2...I(PkI...qml...)) =
10. Now if B gets a number of 19 or less, he can win by picking 1, 2 or 3. So if A is given 4
Pml...qU.../(Pjl...qkl...). It is now trivial to verify that ttx tty)) = f(x)/y.
he loses. Now if B is given 29 or less, he can pick 1, 2, 3 or 4 and win. So if A is given 5 he

• Problem B2
loses.

We now have 10consider what happens if A gets 6 or 7. He must pick 30 or more, or B


Given an initial integer no> 1, two players A and B choose integers ni, n2, n3, ... altemately
wins. Ifhe picks 31, 32, 33, 34, 35 or 36, !hen B wins by picking (for example) 1, 1,3,2,5,
according to !he following rules:
4 respectively. So his only hope given 6 is 10 pick 30. Balso wins given any of37, 38, 39,
Knowing na, A chooses any integer n2k+1such lhat na:S n2k+1:s n2k2.


40,41,43,44,45,46,47,48,49 (winning moves, for example, 37, 1; 38, 2; 39, 3; 40, 5;
41,1; 43,1; 44, 4; 45, 5; 46, 3; 47,1; 48, 3]. So A's only hope given 7 is to pick 30 or42.



• 'lb
IfB is faced with 30=2'3'5, then he has a choice of6, lO, 15. We have already established ProblemA!
that lO and 15 willlose, so he must pick 6. Thus 6 is a draw: A must pick 30 or lose, and
then B must pick 6 or lose. Given a triangle ABC, let I be the incenter. The intemal bisectors of angles A, B, C meet
the opposite sides in A', B', C' respectively. Prove that:
IfB is faced with 42=2'3,7, then he has a choice of6, 14 or 21. We have already
established that 14 and 21 lose, so he must pick 6. Thus 7 is also a draw: A must pick 30 or 114< AI·BI·CI/(AA'-BB'-CC'):'ó 8/27.
42, and then B must pick 6.

Solution
Comment
Consider the areas ofthe three triangles ABI, BCI, CAl. Taking base BC we conclude that
I am grateful to Gerhard Woginger and Jean-Pierre Ehrmann for finding errors in my (area ABI + area CAI)/area ABC = Al/AA'. On the other hand, ifr is the radius of the in-
original solution. circle, then area ABI = AB.r/2 and similarly for the other two triangles. Hence Al/AA' =
(CA + AB)/p, where p is the perimeter. Similarly BIIBB' = (AB + BC)/p and CI/CC' = (BC
+ CA)/p. But the arithmetic mean of (CA + AB)/p, (AB + BC)/p and (BC + CA)/p is 2/3.
Hence their product is at most (2/3)3 = 8/27.
Problem B3
LetAB + BC - CA =2z, BC + CA-AB =2x, CA + AB - BC = 2y. Then x, y, zare all
Prove that there exists a convex 1990-gon such that all its angles are equal and the lengths positive and we have AB = Y + z, BC = z + x, CA = x + y. Hence
of the sides are the numbers ¡l, 22, ... , 19902 in sorne order. (AI/AA')(BIIBB')(CI/CC') = (1/2 + y/p)(1I2 + zlp)(I/2 + xlp) > 1/8 + (x+y+z)/(4p) = 1/8 +
1/8 = 1/4.

Solution

By Robin Chapman, Dept 01Maths, Macquarie University, Australia ProblemA2

In the complex plane we can represent the sides as po2WD,where Po is a permutation of (1, 2, Let n > 6 be an integer and let al, a2, ... , llIt be all the positive integers less than n and
... , 1990) and w is a primitive I 990lh root ofunity. relatively prime 10 n. If a2 - al = a, - a2 = ... = llIt - 1lIt.1 > O,prove that n must be either a
prime number or a power of2.
The critical point is that 1990 is a product of more than 2 distinct primes: 1990 = 2·5·199.
So we can write w = -l-a-b, where -1 is primitive 2nd root ofunity, a is a primitive 5lh root
ofunity, and b is a primitive 199th root ofunity.
Solution
Now given one ofthe I 990th roots we may write it as (-liaib\ where O < i < 2, O <j < 5, O
< k < 199 and hence associate it wilh the integer r(ij,k) = 1 + 995i + 199j + k. This is a byanon
bijection onto (1, 2, ... , 1990). We have to show that the sum ofr(ij,ki (-liaibk is zero.
If n is odd, then I and 2 are prime 10n, so all integers < n are prime to n, and hence is
We sum first over i. This gives _9952x sum ofaibk which is zero, and -1990 x sum s(j,k) prime.
aibk, where s(j,k) = 1 + 199j + k. So it is sufficient 10show that the sum of s(j,k) aibk is zero.
We now sum over j. The 1 + k par! of s(j,k) immediately gives zero. The 199j par! gives a Ifn = 4k, then 2k-1 and 2k+1 are prime 10 n, so all odd integers < n are prime to n, and
constant times b\ which gives zero when summed over k. hence n must be a power of 2.

IMOl991 Ifn = 4k+2, then 2k+1 divides n, but 2k+3 and 2k+5 are prime to n. But ifn > 6, then 2k+5
< n, so this cannot be a solution.

..J
_,

ProblemAJ The basic idea is that consecutive numbers are relatively prime.

Let S = {I, 2, 3, ... 280}. Find the smallest integer n such that each n-element subset ofS We construct a labeling as follows. Pick any vertex A and take a path from A along
contains five numbers which are pairwise relatively prime. unlabeled edges. Label the edges consecutively 1, 2, 3, ... as the path is constructed.
Continue the path until it reaches a vertex with no unlabeled edges. Let B be the endpoint
ofthe path. A is now guaranteed to have the gcd (= greatest common divisor) ofits edges 1,
because one of its edges is labeled l. AH the vertices between A and B are guaranteed to
Solution have gcd 1 because they have at least one pair of edges with consecutive numbers. Fínally,
either B has only one edge, in which case its gcd does not matter, or it is a1so one of the
Answer: 217. vertices between A and B, in which case its gcd is 1.

Let A be the subset ofall multiples of2, 3, 5 or 7. Then A has 216 members and every 5- Now take any vertex C with an unlabeled edge and repeat the process. The same argument
subset has 2 members with a comrnon factor. [To show that IAI = 216, let an be the number shows that all the new vertices on the new path have gcd l. The endpoint is fine, because
ofmultiples ofn in S. Then a2 = 140, aJ = 93, as = 56, 116= 46, alO = 28, alS = 18, aJO= 9. either it has only one edge (in which case its gcd does not matter) or it has already got gcd
Hence the number of multiples of 2, 3 or 5 = a2 + aJ + as - 116- alO- alS + aJO= 206. There 1.
are ten additional multiples of7: 7,49,77,91, 119, 133, 161,203,217,259.]
Repeat until all the edges are labeled.
Let P be the set consisting of 1 and all the primes < 280. Define:
Al = {2'41, 3'37, 5'31, 7'29,11'23, 13·19}
A2 = {B7, 3'31, 5'29, 7'23,11'19, 13·17} Problem B2
A3 = {2"31, 3'29, 5,23, 7'19,11'17, 13'13}
BI = {2'29,3'23, 5'19, 7'17, 1I'13} Let ABC be a triangle and X an interior point of ABC. Show that at least one of the angles
B2 = {2'23, 3'19,5'17,7,13, 11'11} XAB, XBC, XCA is less tban or equal to 30°.

Note that these 6 sets are disjoint subsets of S and the members of any one set are relatively


prime in pairs. But P has 60 members, the three As have 6 each, and the two Bs have 5
each, a total of 88. So any subset T of S with 217 elements must have at least 25 elements Solution
in cornmon with their union. But 6·4 = 24 < 25, so T must have at least 5 elements in


common with one of them. Those 5 elements are the required subset of elements relatively By Marcin Mazur, University of IlIinois al Urbana-Champaign
prime in pairs.
Let P, Q, R be the feet ofthe perpendiculars from X to BC, CA, AB respectively. Use A, B,


Problem Bl C to denote the interior angles ofthe triangle (BAC, CBA, ACB). We have PX = BX sin
XBC = CX sin(C - XCA), QX = CX sin XCA = AX sin(A - XAB), RX = AX sin XAB =
Suppose G is a connected graph with k edges. Prove that it is possible to label the edges 1, BX sin(B - XBC). Multiplying: sin(A - XAB) sin(B - XBC) sin(C - XCA) = sin A sin B sin

• 2, ... , k in such a way that at each vertex which belongs to two or more edges, the greatest
common divisor of the integers labeling those edges is l.
C.

Now observe that sin(A - x)lsin x = sin A cot x - cos A is a strictly decreasing function ofx

• [A graph is a set of points, called vertices, together with a set of edges joining certain pairs
of distinct vertices. Each pair of edges belongs to at most one edge. The graph is connected
(over the range O to lt), so ifXAB, XBC and XCA are a11greater than 30, then sin(A - 30)
sin(B - 30°) sin(C - 30°) > sinJ300 = 118.


if for each pair of distinct vertices x, y there is sorne sequence ofvertices x = Yo, v" ... , Vm
= y, such that each pair Vi, Vi+! (O :s i < m) is joined by an edge.] But sin(A - 30°) sin(B - 30°) = (cos(A - B) - cos(A + B - 600»)/2:S (1 - cosCA + B - 60"»)12 =
(1 - sin(C - 30"»)12, since (A - 30°) + (B - 30°) + (C - 30, = 90°. Hence sin(A - 30°) sin(B -


30°) sin(C - 300):s 112 (1 - sin(C - 30°» sin(C - 30°) = 1/2 (1/4 - (sin(C - 30°) - 112)2):s 1/8.
So XAB, XBC, XCA cannot a11be greater tban 30°.
Solution





[An infinite sequence xs, XI, X2,... ofreal numbers is bounded ifthere is a eonstant C sueh
By Jean-Pierre Ehrmann that Ix.1< C for all n.J

P, Q, R as above. Area ABX + area BCX + area CAX = area ABC, so AB·XR + BC·XP +
CA'XQ = 2 area ABC:'5 BC·AP:'5 BC(AX + XP). Henee AB-XRlAX + CA'XQ/AX:'5 BC.
SolutioD
Squaring and using (:A. + 11)22:4 :A.f' we have: BC2 2:4 AB·CA. XR-XQ/AX2. Similarly: CA2
2:4 BC'AB-XP'XRlBX2, and AB 2:4 AB·BC·XQ·XP/CX2. By Marcin Mazur, University of Illinois al Urbana-Champaign

Multiplying these three inequalities together gives: 1 2: 64 (XRlAX)2(XPIBXi(xQ/Cxi, Let t = Il2a. Define e = 1 - tI(1 - t). Sinee a> 1, e > O. Now given any integer n > O, take the
and henee: (XRlAX) (XPIBX) (XQ/CX):'5 1/8, or sin XAB sin XBC sin XCA:'5 118.So not binary expansion n = ¿i b, z', and define x. = lIe ¿bi>Of. For example, taking n = 21 = 24 +
all XAB, XBC, XCA are greater than 30·. 22 + 2°, we have X21= (t4 + t2 + t°)le. We show that for any unequal n, m, Ix. - xmlln - mla2:
l. This solves the problem, sinee the x, are all positive and bounded by CI t" )le = 1/(1 - 2t).

Gerard Gjonej noted that the result follows a/most immediately from the Erdos-Mordell Take k to be the highest power of 2 dividing both n and m. Then In - mi 2: 2k. Also, in the
inequa/ity: XA + XB + XC 2: 2(xp + XQ + XR). [For if all the angles are greater than 30, binary expansions for n and m, the coeffieients of2°, 21, ..• ,2k-1 agree, but the eoeffieients
then XRlXA, XPIXB, XQIXC are a/I greater than sin so: = lI2.J. This result was for 2k are different. Hence e Ix, - xml= tk + ¿i>leYi,where Yi= O, ti or - f. Certainly l:i>leYi>
notoriously hard lo prove - Erdos hawked it around a large number of mathematicians r
- ¿i>k f = l/(1 - t), so e Ix, - xml> tk(1 - tI(1 - t) = et.
Henee Ix, - xmlln - m]' > t 2B = 1.
before Mordell found a proof - bu: the proof now appears fairly innocuous, al least if you

...
do not hove to rediscover it:

Let R¡, QI be the feet ofthe perpendieulars from P to AB, CA respeetively. Similarly, let IMO 1992
P2, R2 be the feet of the perpendieulars from Q to BC, AB, and Q3, P3 the feet of the
perpendieulars from R to CA, BC. Then P2P3is the projeetion of QR onto BC, so P2P3/QR SolutioD
:'51. Similarly, Q3QIIRP:'5I, and RIR2IPQ:'5I. Henee XA + XB + XC 2: XA.P2PYQR +
XB'Q3QIIRP + XC·RIR2!PQ (.) Answer: a = 2, b = 4, e = 8; or a = 3, b = 5, e = 15.
..1
Now BPXR is eyclie, beeause BPX and XRB are both right angles. Henee angle BXR = Let k = 21/3 lfa 2: 5, then k(a - 1) > a [Check: (k(a - 1)3- a3 = a3 - 6a2 + 6a - 2. For a 2: 6, a3
angle BPR = angle RPP3, so triangles XBR and PRP3 are similar. Henee PP3 = PR.XRIXB. 2: 6a2 and 6a > 2, so we only need to check a = 5: 125 - 150 + 30 - 2 = 3.J We know that e >
b> a, so if a > 5, then 2(a - IXb - IXe - 1) > abe> abe - 1. So we must have a= 2, 3 or4.
Similarly, QQI = QP-XP/XC, RR2 = RQ'XQ/XA, and PP2 = PQ'XQ/XC, QQ3 =
QR'XRIXA, RRI = RP·XPIXB. Substituting into (.), we obtain: Suppose abe - I = n(a - IXb - IXe - 1). We eonsider separately the cases n = 1, n = 2 and n
XA + XB + XC 2: XA( PQ/QR XQ/XC + PRlQR XRIXB ) + XB( QRfRP XRIXA + QPIRP 2: 3. Ifn = 1, then a + b + e = ab + be + ca But that is impossible, because a, b, e are all
XP/XC ) + XC( RP!PQ XP/XB + RQ!PQ XQ/XA ). greater than 1 and so a < ah, b < be and e < ca

On the right hand side, the terms involving XP are: XP( QPIRP XB/XC + RP!PQ XCIXB ), Suppose n = 2. Then abe - 1 is even, so all a, b, e are odd. In particular, a = 3. So we have
whieh has the form XP (x + l/x) and henee is at least 2 XP. Similarly for the terms 4(b - IXe - 1) = 3be - 1, and henee be + 5 = 4b + 4e. Ifb >= 9, then be >= ge > 4e + 4b. So
involving XQ and XR. we must have b = 5 or 7. Ifb = 5, then we find e = 15, whieh gives a solution. Ifb = 7, then
we find e = 23/3 whieh is not a solution.

The remaining case is n >= 3. If a = 2, we have n(be - b - e + 1) = 2be - 1, or (n - 2)be + (n


Problem B3 + 1) = nb + ne. But b 2: 3, so (n - 2)be 2: (3n - 6)c 2: 2ne for n 2:6, so we must have n = 3, 4
or 5. Ifn = 3, then be + 4 = 3b + 3e. Ifb >= 6, then be 2:6e > 3b + 3e, so b = 3, 4 or 5.
Given any real number a > 1 eonstruet a bounded infinite sequence )(o, XI, X2,... sueh that Cheeking we find only b = 4 gives a solution: a = 2, b = 4, e = 8. Ifn = 4, then (n - 2)be, nb ..,
Ix. - xmlln - mla2: 1 for every pair of distinet n, m. and ne are all even, but (n + 1) is odd, so there is no solution. If n = 5, then 3be + 6 = 5b +

.J
..J



• 5e. b = 3 gives e = 9/4, whieh is not a solution. b >= 4 gives 3bc > 10c > 5b + 5e, so there SolutioD

• are no solutions.

Ifa= 3, we have2n(bc - b - c + 1) = 3bc - 1, or(2n- 3)bc + (2n + 1)= 2nb + 2nc. But b~
by Gerhard Woginger

• 4, so (2n - 3)bc ~ (8n - 12)c ~ 4nc > 2nc + 2nb. So there are no solutions. Similarly, if a =
4, we have (3n - 4)be + (3n + 1) = 3nb + 3nc. But b ~ 4, so (3n - 4)bc ~ (12n - 16)c > 6nc >
We show that for n = 32 we can find a coloring without a monochrome triangle. Take two
squares R.R2R3R. and B.B2B3B4. Leave the diagonals of each square uncolored, color the
remaining edges ofR red and the remainingedges ofB blue. Color blue all the edges from


3nb + 3ne, so there are no solutions.
the ninth point X to the red square, and red all the edges from X to the blue square. Color
R¡Bj red if i and j have the sarne parity and blue otherwise.

•• ProblemA2

Find all funetions f defined on the set of all real numbers with real values, such that f(x2 +
f(y» = y + f(X)2for all x, y.
Clearly X is not the vertex of a monochrome square, because if XY and XZ are the sarne
color then, YZ is either uncolored or the opposite color. There is no triangle within the red
square or the blue square, and hence no monochrome triangle. It remains to consider
triangles of the form R¡RjBt and B¡BjRt. But if i and j have the sarne parity, then R¡Rj is
uneolored (and similarly B¡Bj), whereas ifthey have opposite parity, then R¡Bt and R¡Bt

• have opposite colors (and similarly B¡Rt and BjRt).

It remains to show that for n = 33 we can always find a monochrome triangle. There are

• SolutioD
three uncolored edges. Take a point on each ofthe uncolored edges. The edges between the
remaining 6 points must all be colored. Take one ofthese, X. At least 3 of the 5 edges to X,


The first step is to establish that f(0) = O. Putting x = y = O, and f(0) = t, we get f(t) = t2_ say XA, XB, XC must be the same color (say red). If AB is also red, then XAB is
Also, f(x2+() = f(x)2, and f{f(x» = x + t2. We now evaluate f(t2+f(li) two ways. First, it is monochrome. Similarly, for BC and CA. But if AB, BC and CA are all blue, then ABC is
f(f(li + f(t» = t + f(f(I»2 = t + (1 + t2)2= 1 + t + 2t2 + t4. Second, it is f(t2 + f(1 + 1» = 1 + I monochrome.
+ f(1)2= 1 + 1+14. So t = O, as required.
J,
It follows immediately that f(f(x» = x, and f(x2) = f(xf Given any y, let z = f(y). Then y =

• f(z), so f(x2 + y) = Z + f{xi = f(y) + f(xi. Now given any positive x, take z so that x =
Then f(x + y) = f(Z2+ y) = f(y) + f(zi = f(y) + f(Z2)= f(x) + f(y). Putting y = -x, we gel O =
r. ProblemBl

L is a tangent to the circle C and M is a point on L. Find Ihe locus of all points P such that


f(0) = f{x + -x) = f(x) + f{-x). Hence f(-x) = - f(x). II follows that f(x + y) = f(x) + f(y) and
f(x - y) = f(x) - f(y) hold for all x, y. there exist points Q and R on L equidistant from M with C the incirc1e of the triangle PQR.

Take any x. Let f(x) = y. Ify > x, then let z = y-x. fez) = f(y - x) = f(y) - f(x) = x - y = -z. If
al y < x, then lel z = x - y and f(z) = f(x - y) = f(x) - f(y) = y-x. In either case we gel some z >
O with fez) = -z < O. But now take w so that ~ = z, then f{z) = f(~) = f(wi >= O. SOIUtiOD

• Contradiction. So we must have f{x) = x.


Answer: Let X be the point where C meets L, let O be the center of C, let XO cut C gain at
Z, and take y on QR so that M be the midpoint ofXY. Let L' be the line YZ. The locus is

• ProblemA3
the open ray from Z along L' on the opposite side lo Y.

main/y by GerhardW6ginger,TechnicaJUniversity;Graz (lfilled in afew details)

• Consider 9 points in space, no 4 coplanar. Each pair of points is joined by a line segrnent
which is eolored either blue or red or left uncolored. Find the smallest value of n such !hal Lel C' be the circle on lhe other side oC QR lo C which also touches Ihe segrnenl QR and


whenever exactly n edges are colored, Ihe sel of colored edges necessarily contains a the Jines PQ and QR. Let C' touch QR at Y'. If we take an expansion (teehnically,
triangle all of whose edges have the same color. homothecy) center P, factor PY'!PZ, then C goes to C', the tangent to C al Z goes to the line
QR, and hence Z goes to Y'. But it is easy to show that QX = RY'.





JI
We foeus on the QORO'. EvidentIy X,Y' are the feet ofthe perpendieulars from O, O' by Gerhard Woginger
respeetively to QR. Also, OQO' = ORO' = 90. SO QY'O' and OXQ are similar, and henee
QY'/Y'O' = OXlXQ. Also RXO and O'Y'R are similar, so RXlXO = O'Y'IY'R. Hence Induetion on the number of different z-eoordinates in S.
QY'XQ = OX'O'Y' = RX·Y'R. Henee QXIRX = QXI(QR - QX) = RY'/(QR - RY') =
RY'/QY'. HeneeQX=RY'. For 1, it is suffíeient to note that S = Sz and ISI:5IS.IISyl (at most IS.I points of S project
onto each ofthe points of Sy).
But QX = RY by construction (M is the midpoint ofXY and QR), so Y = Y'. Henee P lies
on the open rayas elaimed. Conversely, if we take P on this ray, then by the same argument In the general case, take a horizontal (constant z) plane dividing S into two non-empty parts
QX = R Y. But M is the midpoint of XY, so M must also be the midpoint of QR, so the T and U. Clearly, ISI= ITI+ IUI, IS.I = IT.I + ADJ, and ISyl= IT 1 + IUyl.
loeus is the entire (open) rayo By induetion, ISI= ITI+ IUI:5(IT.1 ITAIITJ)I + (lU.IIUyIIUJ)l'l2 But ITzl,IUJ :5ISJ, and for
any positive a, b, e, d we have (a b l + (e dln:5 ( (a + e) (b + d) )In (square!).
Gerhard onJyfound this after Theo Koupelis, University of Wisconsin, Marathon had Hence ISI:5ISin( (IT.I + IU.I ) (ITyl + IUyl) in = (IS.IISyIISzl ) In.
already supplied the following ana/ytic solution.
Problem B3
Take Cartesian eoordinates with origin x, so that M is (a, O) and O is (O, R). Let R be the
point (b, O) (we take a, b >= O). Then Q is the point (2a - b, O), and y is (2a, O). ~et angle For eaeh positive integer n, sen) is defined as the greatest integer sueh that for every
XRO be e. Then tan e = RIb and angle PRX = 2e, so tan PRX = 2 tan e/( 1 - tan e) = positive integer k:5 sen), n2 can be written as the sum ofk positive squares.
2Rb/(b2 - R2). Similarly, tan PQX = 2R(b - 2ay( (b - 2ai - R2).
(a) Prove that Sen) <= n2 - 14 for eaeh n ~ 4.
If P has eoordinates (A, B), then B/(b - A) = tan PRX, and B/(b - 2a + x) = tan PQx. SO we (b) Find an integer n sueh that Sen) = n2 - 14.
have two simultaneous equations for A and B. Solving, and simplifying slightly, we find A (e) Prove that there are infinitely many integers n sueh that Sen) = n2 - 14.
= _2aR2/(b2_ 2ab - R2), B = 2b(b - 2a)RI(b2 - 2ab _ R2). (*) :..1
We may now check that B/(2a - A) = Rla, so P lies on YZ as claimed. So we have shown
that the loeus is a subset ofthe line YZ. But sinee b2 - 2ab - R2 maps the open interval (a + Solution
.,J(a2+ R2), 00) onto the open interval (O, 00), (*) shows that we can obtain any value A in the
open interval (-00,0) by a suitable choice ofb, and hence any point P on the ray (except its (a) Let N = n2. Suppose we eould express N as a sum ofN - 13 squares. Let the number of
endpoint Z) by a suitable choice of R. 4s be a, the number of9s be b and so on. Then we have 13 = 3a + 8b + 15e + .... Henee e,
d, ... must all be zero. But neither 13 nor 8 is a multiple of3, so there are no solutions.
Hence S(n):5 N - 14.

Problem 82 A little experimentation shows that the problem is getting started. Most squares eannot be
expressed as a sum oftwo squares. ForN = 132 = 169, we find: 169 = 9 + 4 + 4 + 152 Is, a
Let S be a finite set of points in three-dimensional space. Let S., Sy, Sz be the sets sum of 155 = N - 14 squares. By grouping four Is into a 4 repeatedly, we obtain all
eonsisting of the orthogonal projeetions of the points of S onto the yz-plane, zx-plane, xy- multiples of3 plus 2 down to 41 (169 = 9 + 40 4s). Then grouping four 4s into a 16 gives
plane respectively. Prove that: us 38, 35, ... , 11 (169 = 10 16s + 9). Grouping four 16s into a 64 gives us 8 and 5. We
obtain the last number eongruenl lo 2 mod 3 by the decomposition: 169 = 122+ 5l
ISI2<= IS.IISyIISzl,where IAIdenotes the number of points in the set A.
For the numbers congruent to 1 mod 3, we start with N - 15 = 154 squares: 169 = 5 4s +
[The orthogonal projeetion of a point onto aplane is the foot of the perpendicular from the 149 Is. Grouping as before gives us all3m + 1 down to 7: 169 = 64 + 64 + 16 + 16 + 4 + 4
point lo the plane.) + 1. Wemayuse 169= 102+82+22+ 12for4.

Formultiples of3, we start with N - 16 = 153 squares: 169 = 9 + 9 + 151 Is. Grouping as
before gives us all multiples of3 down lo 9: 169 = 64 + 64 + 16 + 9 + 9 + 4 + 1 + 1 + 1.
Solution Finally, we may take 169 = 122 + 42 + 32 for 3 and split the 42 to get 169 = 122+ 32 + 22 +

.J
.J



• 22 -1"22+ 22 for 6. That completes the demonstration that we can write 132as a sum of k combination of some or all of ai, ... , lit.Either way, 3t+1is divisible by 3. So considering the

••
coefficients ofx, x2, ... , xr-I gives us that all the a's are multiples of3. Now consider the
positive squares for all k <= S(13) = 132 - 14.
coefficient of xr, which is also zero. It is a sum of terrns which are multiples of 3 plus ± 1, so
We now show how to use the expressions for 132 to derive further N. For any N, the it is not zero. Contradiction. Hence the factorization is not possible.
grouping technique gives us the high k. Simply grouping Is into 4s takes us down: from 9 +
4 + 4 + (N-17) Is to (N-14)14 + 6 < N/2 or below; from 4 + 4 +4 + 4 + 4 + (N-20) Is to (N-


23)14 + 8 < N/2 orbelow; from 9 + 9 + (N-18) Is to (N-21)f4 + 5 < N/2 or below. So we
can certainly get all k in the range (N/2 to N-14) by this approach. Now suppose that we ProblemA2
already have a complete set of expressions for NI and for N2 (where we may have NI = N2).


Consider N3 = N¡N2. Writing N1 = NI( an expression for N2 as a sum ofk squares) gives N1 Let D be a point inside the acute-angled triangle ABC such that LADB = LACB + 90°,
as a sum of 1 thru k2 squares, where k2 = N2 - 14 squares (since NI is a square). Now and AC'BD = AD·BC.
eXfress NI as a sum oftwo squares: nl2 + n{ We have N3 = nl2(a sum ofk2 squares) +

• n2 (a sum ofk squares). This gives N1 as a sum of k¡ + 1 thru 2k2 squares. Continuing in (a) Calculate the ratio AB·CD/(AC·BD).
this way gives N1 as a sum of 1 thru klk2 squares. But k¡ = Ni - 14> 2/3 Ni, so klk2 > N1f2.
So when combined with the top down grouping we get a complete set of expressions for (b) Prove that the tangents at C to the circumcircles of ACD and BCD are perpendicular.

• N1.

This shows that there are infinitely many squares N with a complete set of expressions, for

..• example we may take N = the squares of 13, 132, 131, .... SolutiOD

By Glen Ong, Oracle CorporaJion

JMO 1993

• ProblemAl

• Let f(x) = x' + 5Xn-1+ 3, where n > 1 is an integer. Prove that f(x) canoot be expressed as


the product of two non-constant polynomials with integer coefficients.

• SolutioD


1
Suppose f(x) = (xr + lIr.IXr.1+ ... + aix ± 3Xx' + b.. lx.. + ... + bjx ± 1). We show that all the
a's are divisible by 3 and use that to establish a contradiction.

• First, r and s must be greater than 1. For ifr = 1, then ± 3 is a root, so ifn is even, we would
have O = 3D ± 5.30-1 + 3 = 3 1( 3 ± 5) + 3, which is false since 3 ± 5 = 8 or -2. Similarly if n
B'

..
is odd we would have O = 3 1(±3 + 5) + 3, which is false since ±3 + 5 = 8 or 2. If s = 1,


Take B' so that CB = CB', L BCB' = 90° and B' is on the opposite side of BC to A. It is
then ± 1 is a root and we obtain a contradiction in the same way. easy to check that ADB, ACB' are similar and DAC, BAB' are similar. Hence ABfBD =
AB'fB'C and CD/AC = BB'/AB'. It follows that the ratio given is BB'fB'C which is -.J2.
So r:::;n - 2, and hence the coefficients of x, x2, ... , xr are all zero. Since the coefficient of x
is zero, we have: al ± 3bl = O, so al is divisible by 3. We can now proceed by induction.
Take XD the tangent to the circumcircle of ADC at D, so that XD is in the LADB.
Assume al. ... ,lit are all divisible by 3. Then consider the coefficient OfX1+1.Ifs-I 2: t+ 1,
Similarly, take YD the tangent to the circumcircle BDC at D. Then LADX = LACD,
then lIt+1= linear combination of ai, ... ,lit ± 3bt+1.If s-I < t+ 1, then lIt+1= linear



• V7
LBOY= LBCO,so LAOX+ LBOY= LACB and hence LXDY= LAOB-(LAOX
+ L BOY) = LAOB - L ACB = 90°. In other words the tangents to the circumcircles at O
are perpendicular. Hence, by symmetry (reflecting in the line of centers) the tangents at C
x x x x x x x x x x x x x x x x . x x x . x x x
....
are perpendicular. x x x
. x x . x x x x .
Theo Koupelis, University of Wisconsin, Marathon provided a similar solution (about lO
minutes later!) taking the point B' so that LBOB' = 90°, BO = BU and LB'OA = LACB. x x . x x x x x
OAC, B'AB are similar; and ABC, AB'O are similar.
x x x x x x x x x . x . x x . x
Marcin Mazur, University of lllinois at Urbana-Champaign provided the first solution 1 x x x x x x x x x x x x x x . x x
received (about 10 minutes earlier!) using the generalized Ptolemy's equality (as opposed to
the easier equality), but 1do not know of a slick proof of this, so 1prefer the proof aboye.

x x x
ProblemAJ
x . x
On an infinite chessboard a game is played as follows. At the start n2 pieces are arranged in
. x . x x x x
an n x n block of adjoining squares, one piece on each square. A move in the game is a
jump in a horizontal or vertical direction over an adjacent occupied square to an unoccupied
square immediately beyond. The piece which has beenjumped over is removed. Find those
values of n for which the game can end with only one piece remaining on the board.
The key technique is the following three moves which can be used to wipe out three
adjacent pieces on the border provided there are pieces behind them:
...
Solution
x x x
x x x
x x
x x
x
x x .
x
x
x x x

..._,
We show firstthat the game can end with only one piece if n is not a multiple of 3. Note
first that the result is true for n = 2 or 4.

n=2

x
We can use this technique to reduce (r + 3) x s rectangle to an r x s rectangle. There is a
slight wrinkle forthe last two rows of three:

x x x x
x x x x
x x
x x
x
x
x x
x x
x x
x x
x
x
x
x
..
x x .. x ... x . x x x x x .. x x . x
x x x x x Thus we can reduce a square side 3n+2 to a 2 x (3n+2) rectangle. We now show how to
x wipe out the rectangle. First, we change the 2 x 2 rectangle at one end into a single piece
alongside the (now) 2 x 3n rectangle:
n=4
x x
x x x x x
x x
x x x x x x x x x . x xx xx x.
x x x
xxxx xx x .. xx xx x xx x.xx
Then we use the following technique to shorten the rectangle by 3:
x x x x x x x x x x x x x x x x . x x x . x x x
x x x x . x x . x



• The length of an altitude is twice the area divided by the length of the corresponding side.


x x x x . x x . x Suppose that BC is the longest side ofthe triangle ABC. Then m(ABC) = area ABCIBC. [If
A = B = C, so that BC = O, then the result is trivially true.]
x xx x. xx x x

• That completes the case ofthe square side 3n+2. For the square side 3n+ 1 we can use the
technique for removing 3 x r rectangles to reduce it 10 a 4 x 4 square and then use the
Consider first the case of X inside ABC. Then area ABC = area ABX + area AXC + area
XBC, so m(ABC)/2 = area ABXlBC + area AXCIBC + area XBCIBC. We now claim that


technique aboye for the 4 x 4 rectangle. the longest side of ABX is at most BC, and similarly for AXC and XBC. It then follows at
once that area ABXlBC ::::arca ABXllongest side of ABX = m(ABX)l2 and the result
Finally, we use a parity argument to show that if n is a multiple of 3, then the square side n follows (for points X inside ABC).


cannot be reduced to a single piece. Color the board with 3 colors, red, white and blue:
The claim follows from the following lemma If Y lies between D and E, then FY is less
than the greater than FD and FE. Proof: let H be the foot of the perpendicular from F to DE.

• One of D and E must lie on the opposite side of y 10 H. Suppose it is D. Then FD = FHlcos
R W B R W B R W B
HFD> FHlcos HFY = FY. Retuming to ABCX, let CX meet AB al Y. Consider the three
W B R W B R W B R
sides of ABX. By definition AB ::::BC. By the lemma AX is smaller than the larger of AC

• B R W B R W B R W and A Y, both ofwhich do not exceed BC. Hence AX:::: BC. Similarly BX:::: BC.

••
R W B R W B R W B It remains to consider X outside ABC. Let AX meet AC at O. We show that the sum of the
smallest altitudes of ABY and BCY is at least the sum ofthe smallest altitudes of ABO and
ACO. The result then follows, since we already have the result for X = O. The altitude from
Let suppose that the single piece is on a red square. Let A be the number of moves onto a A in ABX is the same as the altitude from A in ABO. The altitude from X in ABX is
red square, B the number of moves onto a white square and C the number of moves onto a clearly longer than the altitude from O in ABO (Iet the altitudes meet the line AB at Q and
blue square. A move onto a red square increases the number ofpieces on red squares by 1, R respectively, then triangles BOR and BXQ are similar, so XQ = OR·BXlBO > OR).

• reduces the number of pieces on white squares by 1, and reduces the number of pieces on
blue squares by l. Let n = 3m. Then there are initially m pieces on red squares, m on white
Finally, let k be the line through A parallel 10 BX, then the altitude from B in ABX either
crosses k before it meets AX, or crosses AC before it crosses AX. If the former, then it is
longer than the perpendicular from B 10 k, which equals the altitude from A to BO. If the


and m on blue. Thus we have:
latter, then it is longer than the altitude from B to AO. Thus each of the altitudes in ABX is
-A+ B+C =m-I; A-B +C =m; A+ B-C =m. longer than an altitude in ABO, so m(ABX) > m(ABO).

• Solving, we get A = m, B = m - 1/2, C = m - 1/2. But the number of moves of each type
must be integral, so it is not possible 10reduce the number of pieces to one if n is a multiple


of3. Problem B2

Does there exist a function f from the positive integers to the positive integers such that f(l)


= 2, fl:fl:n))= fl:n) + n for all n, and fl:n) < fln+ 1) for all n?
Problem B1

• For three points P, Q, R in the plane define m(PQR) as the minimum length ofthe three
altitudes ofthe triangle PQR (or zero ifthe points are collinear). Prove that for any points Answer

..

A,B,C,X:
Yes: fl:n) = [g*n + Y.], where g = (1 + .JS)l2 = 1.618 ....
m(ABC):::: m(ABX) + m(AXC) + m(XBC) .

Solution

• Solution




This simple and elegant solution is due to Suengchur Pyun There are n > Ilamps Lo, LI, ... , Ln_1in a circle. We use Ln+kto mean Lk. A lamp is at all
times either on or off. InitiaHy they are all on. Perform steps So, SI, ... as follows: at step s;,
Let gen) = [g*n + Yz). Obviously g(l) = 2. AIso g(n+ 1) = gen) + 1 or gen) + 2, so certainly if L;_Iis lit, then switch L; from on to off or vice versa, otherwise do nothing. Show that:
g(n+ 1) > gen).
(a) There is a positive integer M(n) such Ihat after M(n) steps all Ihe lamps are on again;
Consider den) = g* [g*n + Yz] + Yz - ( [g*n + Yz] + n). We show that it is between O and l. It
follows immediately Ihat g(g(n» = gen) + n, as required. (b) If n = 2k, then we can take M(n) = n2 - 1.

(e) Ifn = 2k + 1, then we can take M(n) = n2 - n + 1.

...
Certainly, [g*n + Yz] > g*n - Yz, so den) > 1 - gJ2 > O (the n term has coefficient g2 - g - 1
which is zero). Similarly, [g*n + Yz] S g*n + Yz, so den) S gJ2 < 1, which completes the
proof

1 originally put up the much clumsier result following:

Take n = b.b,_1... bo in Ihe Fibonacci base. Then f(n) = b.b,_1... boO. This satisfies the
Solution

(a) The process cannot terminate, because before the last move a single lamp would have
been on. But the last move could not have turned it off, because the adjacent lamp was off.
...
required conditions.
There are onIy finitely many states (each lamp is on or off and tbe next move can be at one
... be the Fibonacci numbers. We say n = b.b,_1... bo in
Let Uo = 1, UI = 2, ... , Un=Un-I+Un-2, of finitely many lamps), hence the process must repeat. The outcome of each step is
the Fibonacci base ifb, = 1, every other b, = O or 1, no two adjacent b, are non-zero, and n = uniquely determined by the state, so either the process moves around a single large loop, or
b,u, + ... + boUo. For example, 28 = 1001010 because 28 = 21 + 5 + 2. Ihere is an initial sequence of steps as far as state k and Ihen the process goes around a loop
back to k. However, tbe latter is not possible because Ihen state k would have had two
We have to show that every n has a unique expression ofthis type. We show first by different precursors. But a state has only one possible precursor which can be found by
induction tbat it has at least one expression of this type. Clearly tbat is true for n = 1. Take toggling tbe lamp at tbe current position if the previous lamp is on and Ihen moving the
u, to be the largest Fibonacci number S n. Then by induction we have an expression for n - position back one. Hence Ihe process must move around a single large loop, and hence it
u, The leading term cannot be u; for i > r - 2, for then we would have n >= u, + U,_I= Ur+I. must return to the initial state.
SOadding u,. to the expression for n-u, gives us an expression of tbe required type for n,
which completes the induction. (b) Represent a lamp by X when on, by - when not. For 4 lamps the starting situation and
the situation after 4,8, 12, 16 steps is as follows:
We show Ihat u, + U,-2+ u,.. + ... = Ur+l - 1. Again we use induction. It is true for r = 1 and
2. Suppose it is true for r - 1, Ihen Ur+1+ U,_I+ ... = Ur+2- u, + U,_I+ U,_3+ ... = Ur+2- u, + u,-
I = 1Ir+2- l. So it is true for r + l. Hence it is true for all r. Now we can prove Ihat the
xx x x
expression for n is unique. It is for n = l. So assume it is for all numbers < n, but tbat tbe - x - x
expression for n is not unique, so Ihat we have n = u, + more terms = u, + more terms. If r =
s, Ihen the expression for n-u, is not unique. Contradiction. So suppose r > S. But now the x - -x
second expression is at most Us+1- 1 which is less Ihan u, So the expression for n must be
- x
unique and the induction is complete.
x x x -
It remains to show tbat f satisfies the required conditions. Evidently if n = 1 = Uo, then f(n)
= UI= 2, as required. If n = Ual+ ... + uar, then f(n) = Ual+1+ ... + Uar+1and f(f(n» = Ual+2+ ... On its first move lamp n-2 is switched off and then remains off until each lamp has had n-I
+ Uar+2.SO f(n) + n = (Ual + Ual+l)+ ... + (u., + Üar+1)= f(f(n». moves. Hence for each of its first n-I moves lamp n-I is not toggled and it retains its initial
state. After each lamp has had n-I moves, aH of lamps 1 to n-2 are off. FinaHy over Ihe next
n-I moves,lamps 1 to n-2 are tumed on, so Ihat aH the lamps are on. We show by induction
on k that Ihese statements are aH true for n = 2k. By inspection, they are true for k = 2. So
Problem 83 suppose they are true for k and consider 2n = 2k+1lamps. For Ihe first n-I moves of each
lamp Ihe n left-hand and tbe n right-hand lamps are effectively insulated. Lamps n-I and
2n-1 remain on. Lamp 2n-1 being on meanS tbat lamps O to n-2 are in just the same

....
.J
..J



• situation that they would be with a set of only n lamps. Similarly, lamp n-I being on means ProblemAl

• that lamps n to 2n-2 are in the same situation that they would be with a set of only n lamps.
Hence after each lamp has had n-I moves, all the lamps are off except for n-I and 2n-l. In
the next n moves lamps I to n-2 are turned on, lamp n-I is turned off, lamps n to 2n-2
ABC is an isosceles triangle with AB = AC. M is the midpoint of BC and O is the point on
the line AM such that OB is perpendicular to AS. Q is an arbitrary point on BC different


from B and C. E lies on the line AB and F lies on the line AC such that E, Q, F are distinct
remain off, and lamp 2n-1 remains on. For the next n-I moves for each lamp, lamp n-I is
not togg1ed, so it remains off. Hence all of n to 2n-2 also remain off and 2n-1 remains on. and collinear. Prove that OQ is perpendicular to EF ifand only ifQE = QF.


Lamps O to n-2 go through the same sequence as for a set of n lamps. Hence after these n-I
moves for each lamp, all the lamps are off, except for 2n-1. Finally, over the next 2n-1
moves, lamps O to 2n-2 are turned on. This completes the induction. Counting moves, we


see that there are n-I sets of n moves, followed by n-I moves, a total of n2 - 1. Solution

(e) We show by inductionon the number ofmoves that for n = 2t+ Ilamps after each lamp
) = angleBPZ. So triangles QDZ and
DZlPZ. Let AM meet XY at Q'.
. But BZ·DZ = xz· YZ = AZ-CZ, so • has had m moves, for i = 0,1, ... , 2k - 2, lamp i+2 is in the same state as lamp i is after each
lamp has had m moves in a set of n- I = 2t lamps (we refer to this as lamp i in the reduced
case). Lamp O is offand lamp 1 is on. It is easy to see that this is true for m = I (in both

• cases odd numbered lamps are on and even numbered lamps are off). Suppose it is true for
m. Lamp 2 has the same state as lamp O in the reduced case and both toggle since their


predecessor lamps are on. Hence lamps 3 to n - I behave the same as lamps I to n - 3 in the
reduced case. That means that lamp n - I remains off. Hence lamp O does not togg1eon its B
m+1th move and remains off. Hence lamp I does not toggle on its m+ Ith move and


remains on. The induction stops working when lamp n - 2 toggles on its nth move in the
that: reduced case, but it works up to and including m = n - 2. So after n - 2 moves for each lamp
all lamps are off except lamp l. In the next two moves nothing happens, then in the


Assume OQ is perpendicular to EF. Then L EBO = L EQO = 90·, so EBOQ is cyclic.
following n - I moves lamps 2 to n - I and lamp O are turned on. So all the lamps are on
after a total of (n - 2)n + n + 1 = n2 + n + 1 moves. Hence LOEQ = LOBQ. Also LOQF = LOCF = 90·, so OQCF is cyclic. Hence LOFQ
= LOCQ. But LOCQ = LOBQ since ABC is isosceles. Hence LOEQ = LOFQ, so OE

• = OF, so triangles OEQ and OFQ are congruent and QE = QF.

Assume QE = QF. IfOQ is not perpendicular to EF, then take E'F' through Q perpendicular


IMO 1994
.y+z) = x2¡(:J+z).Let the expression to OQ with E' on AB and F' on AC. Then QE' = QF', so triangles QEE' and QFF' are
+ z+x + x+y)E:::::(x + y + z'l, so E::::: ProblemAl congruent. Hence LQEE' = LQFF'. So CA and AB make the same angles with EF and


-ean result to x, y, z gives (x + y + z) hence are parallel. Contradiction. So OQ is perpendicular to EF.
Let m and n be positive integers. Let al, a2, ... , lIm be distinct elements of {1, 2, ... , n} such
that whenever 8; + a¡ ::::n for some i, j (possibly the same) we have 8; + a¡ = 8t for some k.
.s original solution was wrong.

• Prove that:

(al + ... + a..)!rn:::::(n + 1)12.


ProblemAl

Al, ... , A. in the plane, no three


• For any positive integer k, let fl.k)be the number ofelernents in the set {k+l, k+2, ... ,2k}
which have exactly three 1s when written in base 2. Prove that for each positive integer m,
there is at least one k with fl.k) = rn, and determine all rn for which there is exactly one k.


distinct i, j, k,the area of the triangle
Solution

••
Take al < a2< ...< lIm. Take k::::(rn+I)/2. We show that 8t + 1Im-t+l::::: n + 1. Ifnot, then the
k distinct numbers al + 1Im-k+1, a2+ lIm-t+1, ... , 8t + lIm-t+1 are all ::::n and hence equal to sorne Answer
8;. But they are all greater than lIm-k+1, so each i satisfies m-k+2 ::::i ::::m, which is irnpossible
since there are only k-1 available numbers in the range. 2,4, ... ,n(n-l)/2 + 1, ....



5'l..
• 5;
We start by checking small values of n. n = l gives n3 + l = 2, so m = 2 or 3, giving the
solutions(2, 1) and (3, 1). Similarly, n =2 gives rr'+ l =9,s02m-1 = 1,30r9,givingthe
Solution solutions (1, 2), (2, 2), (5, 2). Similarly, n = 3 gives n3 + l = 28, so 3m - 1 = 2, 14, giving
Solution
the solutions (1, 3), (5, 3). So we assume hereafter that n > 3.

.....
To get a feel, we calculate the first few values of f explicitly:
The first point to notice is that if no ai..
Let n3 + I = (mn - I)h. Then we must have h = -1 (mod n). Put h = kn - 1. Then n3 + I =
f(2) = O, f(3) = O possible for any higher integer. Clearly
mkrr' - (m + k)n + 1. Hence n2 = mkn - (m + k). (*) Hence n divides m + k. Ifm + k 2: 3n,
focus on n = 5.
f(4)= f(5) = 1, [7= 111) then since n> 3 we have at least one ofm, k 2: n + 2. But then (mn - IXkn - 1) 2: (n2 + 2n-
f(6) = 2, [7 = 111, II = 1011) IXn-l) = n3 + n2 - 3n + 1 =(n3 + 1)+ n(n- 3» n3 + 1. So we musthave m + k=nor2n.
f(7) = f(8) = f(9) = 3 [11 = 1011, 13 = 1101, 14 = 1110) If the 5 points form a convex pentagon,
f(IO) = 4 [11,13,14,19 = 10011) made up oftwo triangles in two ways,
Consider first m + k = n. We may take m 2: k (provided that we remember that if m is a
f(11) = f(12) = 5 [13, 14, 19,21 = 10101,22 = 10110) gives rl + r3 = r2 + rs, so r4 = rs.
solution, then so is n-m). So (*) gives n = m(n - m) - l. Clearly m = n - 1 is not a solution.
f(13) = 6 {14, 19,21,22,25 = 11001,26 = 11010] Ifm = n - 2, then n = 2(n - 2) - 1, so n = 5. This gives the two solutions (m, n) = (2, 5) and
We show that we cannot have two r's e
(3, 5). If m < n - 2 then n-m 2: 3 and so m(n - m) - I 2: 3m - I 2: 3n12 - 1 > n for n > 3.
pentagon). For suppose r4 = rs. Then A·
We show that f(k+ 1) = f(k) or f(k) + 1. The set for k+1 has the additional elements 2k+ I
on the same side ofthe line AIAl, then
and 2k+2 and it loses the element k+ 1. But the binary expression for 2k+2 is the same as Finally, take m + k = 2n. So (*) gives n + 2 = m(2n - m). Again we may take m 2: k. m = 2n
that for k+ 1 with the addition of a zero at the end, so 2k+2 and k+ 1 have the same number parallel to AIA2. Iflhey are on opposit
- 1 is not a solution (we are assuming n > 3). So 2n - m 2: 2, and hence m(2n - m) 2: 2m 2: 2n
of Is. So if2k+1 has three Is, then f(k+l) = f(k) + 1, otherwise f(k+I) = f(k). Now clearly The same argument can be applied to t
>n+2.
an infinite number of numbers 2k+ 1 have three Is, (a11numbers 2' + 2' + 1 for r > s > O). So of AIA2, AIA3 and AlA3 parallel to ~J
f(k) increases without limit, and since it only moves up in increments of 1, it never skips a We a1so cannot have the midpoint of "
same reason. So we have established a .
number. In other words, given any positive integer m we can find k so that f(k) = m.
In particular, this shows that the 5 point
Problem B2
From the analysis in the last paragraph we can only have a single k with f(k) = m if both
Suppose the convex hull is a quadrilate
2k-1 and 2k+ I have three Is, or in other words ifboth k-I and k have two 15. Evidently this Let S be the set of all real numbers greater than -1. Find a11functions f :S--+S such that f(x
AIAlA3~. As must lie inside one of A·
happens when k-I has the form 2° + l. This determines the k, narnely 2" + 2, but we need to + f(y) + xfty) = y + f(x) + yf(x) for all x and y, and f(x)/x is strictly increasing on each of
we may take it to be the latter, so that A
determine the corresponding m = f(k). It is the number of elements of {2"+3,2"+4, ... , the intervals -1 < x < O and O < X.
2n+1+4}which have three Is. Elements with three 15are either 2"+2'+2' with OS r < s < n, = rl + r3 and a1so = r¡ + rs. So r3 = rs, gr
or 2n+1+3.So there are m= n(n-I)/2 + I of them, As a check, for n = 2, we have k = 22+2 =
The final case is the convex hull a trianl
6, m = 2, and for n = 3, we have k = 23+2 = 10, m = 4, which agrees with the f(6) = 2, f(10)
= 4 found earlier. the other two points divides its area int
Answer
+~+~+n+~=~+~+~+~j
Problem Bl contradiction.
f(x) = -x/(x+ 1).
So Ihe arrangement is not possible for
Determine a11ordered pairs (m, n) ofpositive integers for which (n3 + I)/(mn - 1) is an
integer.
Solution
Problem Bl
I
Suppose f(a) = a Then putting x = y = a in Ihe relation given, we get f(b) = b, where b = 2a
Find the maximum value of Xo for whi,
Answer + a2. If -1 < a < O,Ihen -1 < b < a. But f(a)/a = f(b)lb. Contradiction. Similarly, ifa > O,Ihen
reals wilh Xo = XI99Ssuch Ihat for i = 1,
b > a, but f(a)/a = f(b)lb. Contradiction. So we must have a = O.
(1,2), (1, 3), (2,1), (2, 2), (2, 5), (3,1), (3, 5), (5, 2), (5, 3).
Xi_1+ 2/Xi_1= 2Xi+ l/Xi.
But putting x = y in Ihe relation given we get f(k) = k for k = x + f(x) + xf(x). Hence for
any x we have x + f(x) + xf(x) = O and hence f(x) = -x/(x+ 1).

Solution Finally, it is straightforward to check Ihat f(x) = -x(x+l) satisfies the two conditions.
-..1
Answer



• We are given a positive integer r and a rectangular board divided into 20 x 12 unit squares.

• Problem BJ
The following moves are permitted on the board: one can move from one square to another
only ifthe distance between the centers ofthe two squares is"¡r. The task is to find a
sequence of moves leading between two adjacent comers of the board which lie on the long

• Let p be an odd prime number. How many p-element subsets A of { 1, 2, ... , 2p} are there,
the sum ofwhose elements is divisible by p?
side.


(a) Show that the task cannot be done ifr is divisible by 2 or 3.
(b) Prove that the task is possible for r = 73.
(e) Can the task be done for r = 97?

• Answer

2 + (2pCp - 2)/p, where 2pCp is the binomial coefficient (2p)!I(p!p!).

• Answer

No.

• Solution


Let A be a subset other than {I, 2, ... , p} and {p+ 1, p+ 2, ... , 2p}. Consider the elements of
A in {l , 2, ... , p}. The number r satisfies O < r < p. We can change these elements to Solution
another set of r elements of {I, 2, ... , p} by adding l to each element (and reducing mod p

•..
ifnecessary). We can repeat this process and get p sets in all. For example, ifp = 7 and the (a) Suppose the move is a units in one direction and b in the orthogonal direction. So a2 +
original subset of {l , 2, ... , 7} was {3, 5}, we get: b2 = r. If r is divisible by 2, then a and b are both even or both odd. But that means that we
can only access the black squares or the white squares (assuming the rectangle is colored
{3, 5}, (4, 6), {5, 7}, {6, I}, {7, 2}, {I, 3}, {2, 4} . like a chessboard). The two comers are of opposite color, so the task cannot be done. AH
squares are congruent to O or 1 mod 3, so if r is divisible by 3, then a and b must both be
multiples of3. That means that ifthe starting square has coordinates (0,0), we can only


The sum of the elements in the set is increased by r each time. So, since p is prime, the
sums must form a complete set of residues mod p. In particular, they must all be distinct move to squares ofthe form (3m,3n). The required destination is (19,0) which is not ofthis
and hence all the subsets must be different. form, so the task cannot be done.

• Now consider the sets A which have a given intersection with {p+ 1, ... , n}. Suppose the
elements in this intersection sum to k mod p. The sets can be partitioned into groups of p by
(b) Ifr = 73, then we musí have a = 8, b = 3 (or vice versa). There are 4 types ofmove:


the process described aboye, so that exactly one member of each group will have the sum - A: (x.,y) to (x+8,y+ 3)
k mod p for its elements in {I, 2, ... , p}. In other words, exactly one member of each group S: (x,y) lo (x+3,y+8)
will have the sum of all its elements divisible by p. C: (x,y) to (x+8,y-3)


O: (x,y) to (x+ 3,y-8)
There are 2pCp subsets of {I, 2, ... ,2p} ofsize p. Excluding {I, 2, ... ,p} and {p+I, ... ,
We regard (x.y) to (x-8,y-3) as a negative move oftype A, and so on. Then ifwe have a


2p} leaves (2pCp - 2). We have just shown that (2pCp - 2Yp of these have sum divisible by
p. The two excluded subsets also have sum divisible by p, so there are 2 + (2pCp - 2Yp moves of type A, b of type S and so on, then we require:
subsets in all having sum divisible by p.


8(a + e) + 3(b + d) = 19; 3(a - e) + 8(b - d) = O.

.'
A simple solution is a = 5, b = -1, e = -3, d = 2, so we start by looking for solutions ofthis
IMO 1996 type. After sorne fiddling we find:

ProblemAl (0,0) to (8,3) to (16,6) to (8,9) to (1 1,1) to (19,4) to (11,7) to (19,10) to (16,2) to (8,5) to

• (16,8) to (19,0).




.l
~

(e) Ifr = 97, then we must have a = 9, b = 4. As before, assume we start at (0,0). A good
deal of fiddling around fails to find a solution, so we look for reasons why one is
impossible. Call moves whieh ehange y by 4 "toggle" moves. Consider the central strip y =
ProblemAJ
.....
.1

4,5,6 or 7. Toggle moves must toggle us in and out ofthe strip. Non-toggle moves eannot Let S be the set of non-negative integers. Find all funetions f: S-S such that f(m + f(n» =
be made ifwe are in the strip and keep us out ofit ifwe are out ofit. Toggle moves also
ehange the parity of the x-coordinate, whereas non-toggle moves do not. Now we start and
finish out of the strip, so we need an even number of toggle moves. On the other hand, we
start with even x and end with odd x, so we need an odd number oftoggle moves. Henee
the task is impossible.
f(Qm» + f(n) for all m, n.

Solution
....
~

ProblemA2

Let P be a point inside the triangle ABC such that LAPB - LACB = LAPC - LABe. Let
D, E be the incenters oftriangles APB, APC respectively. Show that AP, BD, CE meet al a
Setting m = n = O,the given relation becomes: f(f(O» = f(f(0» + f(0). Hence f(0) = O.Hence
also f(f(O» = O. Setting m = O,now gives f(f(n» = f(n), so we may write the original
relation as f(m + f(n» = f(m) + Qn).

So f(n) is a fixed point. Let k be the smallest non-zero fixed point. Ifk does not exist, then
f(n) is zero for all n, which is a possible solution. If k does exist, then an easy induction
shows that f(qk) = qk for all non-negative integers q. Now ifn is another fixed point, write
..
...1
point. n = kq + r, with O:s r < k. Then f(n) = f(r + f(kq» = f(r) + f(kq) = kq + f(r). Hence f(r) = r,
so r must be zero. Hence the fixed points are precisely the multiples of k.
."
But f(n) is a fixed point for any n, so f(n) is a multiple ofk for any n. Let us take nr, n2, ... ,
nk_1to be arbitrary non-negative integers and set no = O.Then the most general function
Solution ~
satisfying the conditions we have established so far is:
We need two general results: the angle bisector theorem; and the result about the feet of the
perpendiculars from a general point inside a triangle. The second is not so well-known. Let f(qk + r) = qk + n,k for O:s r < k. ."
P be a general point in the triangle ABC with X, Y, Z the feet ofthe perpendiculars to BC,
CA, AB. Then PA = YZ/sin A and LAPB - LC = LXZY. To prove the first part: AP = We can check that this satisfies the functionaI equation. Let m = ak + r, n = bk + s, with O:s
AY/sin APY = AY/sin AZY (since A YPZ is cyclic) = YZ/sin A (sine rule). To prove the r, s < k. Then f(f(m» = f(m) = ak + n,k, and f(n) = bk + n,k, so f(m + f(n» = ak + bk + n,k + ..1
second part: LXZY= LXZP+ LYZP= LXBP+ LYAP = 90· - LXPB + 90· - LYPA n,k, and f(f(m» + f(n) = ak + bk + n,k + n,k. So this is a solution and hence the most
= 180· - (360· - LAPB - LXPY) = -180· + LAPB + (180· - LC) = LAPB - LC.
general solution.
..1
Problem Bl
_,
The positive integers a, b are such that 15a + 16b and 16a - 15b are both squares of positive
integers. What is the least possible value that can be taken on by the smaller of these two
..1
squares?
..1
So, returning to the problem, LAPB - Le = LXZY and LAPC
XYZ is isosceles: XY = XZ. Hence PC sin e = PB sin B. But Ae
ABIPB = ACIPC. Let the angle bisector BD meet AP at W. Then,
- LB = LXYZ. Hence
sin e = AB sin B, so
by the angle bisector
Answer
...
theorem, ABIPB = AWIWP. Hence AWIWP = ACIPC, so, by the
ew is the bisector of angle ACP, as required.
angle bisector theorem, ...
."j

.1
...1
.J
-1



• Solution

• Put 15a± 16b=m2,16a-15b=nl Then 15m2+ 16n2=48Ia= 13·37a. Thequadratic


residues mod 13 are O,±I, ±3, ±4, so the residues of 15m2 are O,±2, ±5, ±6, and the

• residues of 16n2 are O,±I, ±3, ±4. Hence m and n must both be divisible by 13. Similarly,
the ,\uadratic residues of37 are O,±I, ±3, ±4, ±7, ±9, ±IO, ±ll, ±12, ±16, so the residues of


15m are O,±2, ±5, ±6, ±8, ±13, ±14, ±15, ±17, ±18, and the residues of 16n2 are O ±I ±3
±4, ±7, ±9, ±IO, ±II, ±12, ±16. Hence m and n must both be divisible by 37. Put m =' ,
481m', n = 481n' and we get: a = 481(15m,2 + 16n,2). We also have 481b = 16m2 - 15n2 and

• hence b = 481(16m,2 - 15n,2).The srnallest possible solution would come from putting m' =
n' = I and indeed that gives a solution.
Extend sides BC and FE and take lines perpendicular to them through A and D, thus
forming a rectangle. Then BF is greater than or equal to the side through A and the side
through D. We may find the length of the side through A by taking the projections of BA

• This solution is straightforward, but something of a slog - all the residues have to be
calculated. A more elegant variant is to notice that m4 + n4 = 481(a2 + b2). Now ifm and n
are not divisible by 13 we have m4 + n4 = O (mod 13). Take k so that km = I (mod 13), then
and AF giving AB sin B + AF sin F. Similarly the side through D is CD sin C + DE sin E.
Hence:

• (nk)' = -{mkt = -1 (mod 13). But that is impossible because then (nk)12 = -1 (mod 13), but
Xl2= I (mod 13) for all non-zero residues. Hence m and n are both divisible by 13. The
2BF ::::AB sin B + AF sin F + CD sin C + DE sin E. Similarly:


same argument shows that m and n are both divisible by 37. 2BD:::: BC sin B +CD sin D + AF sin A + EF sin E, and

2FD::::AB sin A + BC sin C + DE sin D + EF sin F.

• Problem B2 Hence 2BF/sin A + 2BD/sin C + 2FD/sin E::::AB(sin Nsin E + sin B/sin A) + BC(sin B/sin
C + sin C/sin E) + CD(sin C/sin A + sin D/sin C) + DE(sin E/sin A + sin D/sin E) + EF(sin

• Let ABCDEF be a convex hexagon such that AB is parallel to DE, BC is parallel to EF, and
CD is parallel to FA. Let RA, Re, RE denote the circumradii oftriangles FAB, BCD, DEF
respectively, and let p denote the perimeter of the hexagon. Prove that:
E/sin C + sin F/sin E) + AF(sin F/sin A + sin Nsin C).


We now use the fact that opposite sides are parallel, which implies that opposite angles are
equal: A = D, B = E, C = F. Each of the factors multiplying the sides in the last expression
now has the form x + l/x which has minimum value 2 when x = 1. Hence 2(BF/sin A +


BD/sin C + FD/sin E) ::::2p and the result is proved.

Problem Bl


Solution
Let p, q, n be three positive integers with p + q < n. Let X{), xi, ... , Xnbe integers such that
The starting point is the formula for the circurnradius R of a triangle ABC: 2R = a/sin A = = Xo = O,and for each I :5 i :5 n, x, - Xi_1= P or -q. Show that there exist indices i < j with


X{)
b/sin B = e/sin C. [Proof: the side a subtends an angle 2A at the center, so a = 2R sin A] (i, j) not (O, n) such that x¡ = Xj-
This gives that 2RA = BF/sin A, 2Re = BD/sin C, 2RE = FD/sin E. It is clearly not true in
general that BF/sin A > BA + AF, although it is true if angle FAB:::: 120°, so we need sorne


Solution
argument that involves the hexagon as a whole.
Let X¡ - Xi_1= P occur r times and x, - Xi-I= -q occur s times. Then r + s = n and pr = qs. If p

•••
and q have a cornmon factor d, the y¡ = x¡/d form a similar set with p/d and q/d. If the result
is true for the y¡ then it must also be true for the x¡ So we can assume that p and q are
relatively prime. Hence p divides s. Let s = kp. Ifk = 1, then p = s and q = r, so p + q = r + s
= n. But we are given p + q < n. Hence k > 1. Let p + q = nIk = h.

Up to this point everything is fairly obvious and the result looks as though it should be

• easy, but I did not find it so. Sorne fiddling around with examples suggested that we seem




.l

to get x, = Xjfor j = i + h. We observe first that X¡+h- X¡must be a multiple ofh. For suppose (e) Intuitively, it is clear that ifthe hypoteneuse runs along the diagonal ofa series ofblack
e differenees are p, and henee h-e are -q. Then X¡+h- x, = ep - (h - e)q = (e - q)h. squares, and we then extend one side, the extra area taken in will be mainly blaek. We need
to make this rigorous. For the diagonal to run along the diagonal of black squares we must
The next step is not obvious. Let d, = X¡+h- x, We know that all d.s are multiples of h. We have n = m. It is easier to work out tbe white area added by extending a side. The white
wish to show that at least one is zero. Now d¡+1- d¡ = (X¡+h+1 - X¡+h)- (X¡+I- x.) = (p or -q) - (p area takes the form of a series of triangles each similar to the new n+ 1 x n triangle. The
or -q) = O, h or -h, So if neither of di nor d¡+1are zero, then either both are positive or both biggest has sides 1 and n/(n+ 1). The next biggest has sides (n-I)/n and (n-I )/(n+ 1), the next
are negative (a jump from positive to negative would require a differenee of at least 2h).
Henee if none of the d¡s are zero, then all of them are positive, or all of them are negative.
But do + dh + ... + dkhis a concertina sum with value x. - Xo = O. So this subset of the drs
cannot all be positive or all negative. Hence at least one d, is zero.
biggest (n-2)/n and (n-2)/(n+ 1) and so on, down to the smallest whieh is I/n by I/(n+ 1).
Henee the additional white area is 1/2 ~n/(n+ 1) + (n-1)2/(n(n+ 1» + (n-2il(n(n+ 1» + ... +
I/(n(n+ 1» ) = 1/(2n(n+1» (n2 + ... + 1 ) = (2n+ 1)/12. Hence the additional black area is n/2
- (2n+ 1)/12 = n/3 - 1/12 and the black exeess in the additional area is n/6 - 1/6. Ifn is even,
...
then nn,n) = O for tbe original area, so for the new triangle nn+ l,n) = (n-I)/6 whieh is
IMO 1997 unbounded.
ProblemAl
ProblemAl
In the plane the points with integer eoordinates are the vertiees of unit squares. The squares
are eolored altemately blaek and white as on a ehessboard. For any pair of positive integers The angle at A is the smallest angle in the triangle ABC. The points B and C divide the
m and n, eonsider a right-angled triangle whose vertiees have integer coordinates and eireumeircle of the triangle into two ares, Let U be an interior point of the are between B
whose legs, oflengths m and n, lie along the edges ofthe squares. Let SI be the total area of and C which does not contain A. The perpendicular bisectors of AB and AC meet the line
the black part ofthe triangle, and S2 be the total area ofthe white parto Let f(m, n) = ISI - AU at V and W, respeetively. The lines BV and CW meet at T. Show that AU = TB + TC.
S21·

(a) Caleulate nm, n) for all positive integers m and n whieh are either both even or both
odd. Solution
(b) Prove that f(m, n)::; max(m, n)/2 for all m, n.
(e) Show that there is no eonstant C sueh that nm, n) < C for all m, n. Extend BV to meet the eirele again at X, and extend CW to meet the eirele again at Y. Then
by symmetry (since the perpendicular biseetors pass through the center ofthe eirele) AU =
....
Solution
BX and AU = CY. Also are AX = are BU, and are A Y = are UC. Hence are XY = are BC
and so angle BYC = angle XBY and hence TY = TB. So AU = CY = CT + TY = CT + TB. ....
(a) Ifm and n are both even, then nm,n) = O. Let M be the midpoint ofthe hypoteneuse.
The critical point is that M is a lattice point. Ifwe rotate the triangle through 180 to give the ProblemA3
other half of the rectangle, we find that its coloring is the same. Hence SI and S2 for the
triangle are each halftheir values for the reetangle. But the values for the rectangle are Let xi, X2,... , x. be real numbers satisfying IXI+ X2+ ... + x.1 = 1 and IxiI::;(n+I)/2 for all i.
equal, so they must also be equal for the triangle and henee nm,n) = O. Show that there exists a permutation y¡ of'x, such that IYI+ 2Y2+ ... + ny.l::; (n+I)/2.

If m and n are both odd, then the midpoint of the hypoteneuse is the center of a square and
we may still find that the coloring of the two halves of the rectangle is the same. This time
SI and S2 differ by one for the rectangle, so nm,n) = 112. Solution

(b) The result is immediate from (a) for m and n ofthe same parity. The argument in (a) Without loss of generality we may assume XI + ... + x. = +1. [Ifnotjust reverse the sign of
fails for m and n with opposite parity, because the two halves ofthe reetangle are every x..] For any given arrangement x, we use sum to mean XI + 2X2+ 3x] + ... + nx¿ Now
oppositely eolored. Let m be the odd side. Then if we extend the side length m by 1 we if we add together the sums for XI, X2,... , Xn and the reverse Xn,Xn_I,... , XI, we get (n+ 1XXI
form a new triangle whieh eontains the original triangle. But it has both sides even and + ... + x.) = n+ 1. So either we are home with the original arrangement or its reverse, or they
hence SI = S2. The area added is a triangle base 1 and height n, so area n/2. The worst case have sums of opposite sign, one greater than (n+ 1)12and one less than -(n+ 1)/2.
would be that all this area was the same color, in whieh case we would get nm,n) = n/2. But
n <= max(m,n), so this establishes the result.

.J



• A transposition changes the sum from ka + (k+ I)b + other terms to kb + (k+ l)a + other

• terms. Hence it changes the sum by la - bl (where a, b are two ofthe Xi)which does not
exceed n+ 1. Now we can get from the original arrangement to its reverse by a sequence of


transpositions. Hence at sorne point in this sequence the sum must fall in the interval [- Problem B2
(n+ 1)/2, (n+ 1)/2) (because to get from a point below it to a point aboye it in a single step
requires ajump ofmore than n+ 1). That point gives us the required permutation. Find all pairs (a, b) of positive integers that satisfy ab2 = bao


• Problem Bl

An n X n matrix whose entries come from the set S = (1, 2, ... , 2n-I) is called silver matrix
Answer

(1,1), (16,2), (27,3).

• if, for each i = 1,2, ... ,n, the ith row and the ith column together contain all elements ofS.
Show that:

• (a) there is no silver matrix for n = 1997;


(b) silver matrices exist for infinitely many values ofn.
SolutioD


Notice first that if we have am = b", then we must have a = ce, b = cf, for sorne e, where
m=fd, n=ed and d is the greatest common divisor ofm and n. [Proof: express a and b as
products of primes in the usual way.)

• Solution

(a) Ifwe list al! the elements in the rows foHowed by all the elements in the columns, then
In this case let d be the reatest common divisor of a and b2, and put a = de, b2 = df. Then
for sorne e, a = ce, b = e . Hence f ce = e clf. We canoot have e = 2f, for then the c's cancel to

• we have listed every element in the array twice, so each number in S must appear an even
number oftimes. But considering the ith row with the ith column, we have also given n
give e = f. Contradiction. Suppose 2f> e, then f= e C2f-e. Hence e = 1 and f= C2f-l. If e = 1,
then f = 1 and we have the solution a = b = 1. If e 2: 2, then C2f-12: 2f> f, so there are no
solutions.


complete copies of S together with an additional copy of the numbers on the diagonal. If n
is odd, then each ofthe 2n-1 numbers appears an odd number oftimes in the n complete
copies, and at most n numbers can have this converted to an even number by an appearance Finally, suppose 2f< e. Then e = f cC-u Hence f= 1 and e = Cc-2.Cc-22: 2e-22: e for e 2: 5, so


on the diagonal. So there are no silver matrices for n odd. In particular, there is no silver we must have e = 3 or 4 (e > 2f= 2). e = 3 gives the solution a = 27, b = 3. e = 4 gives the
matrix for n = 1997. solution a = 16, b = 2.

• (b) Let Aij be an n X n silver matrix with 1s down the main diagonal. Define the 2n x 2n
matrix Bij with 15 down the main diagonal as foHows: Bij = Aij; Bi+nj+a = Aij; Bij +n = 2n +
Aij; Bi+nj= 2n + Aij for i not equal j and B,+n, i = 2n. We show that Bij is silver. Suppose i:5
Problem6

• n. Then the first half of the ith row is the ith row of Aij, and the top half of the ith column is
the ith column of Aij, so between them those two parts comprise the numbers from 1 to 2n -
l. The second half of the ith row is the ith row of Aij with each element increased by 20,
For each positive integer o, let f(n) denote the nurober ofways ofrepresenting n as asuro of
powers of2 with non-negative integer exponents. Representations which differ only in the
ordering oftheir surnmands are considered to be the same. For example, f(4) = 4, because 4

• and the bottom half of the ith column is the ith column of Aij with each element increased
by 2n, so between them they give the numbers from 2n + 1 to 4n - l. The only exception is
can be represented as 4, 2 + 2, 2 + 1 + 1 or 1 + 1 + 1 + l. Prove that for any integer n 2: 3,
2n214 < f(2°) < 202/2.


that Ai+n,i= 2n instead of 2n + Ai,i. We still get 2n + Ai,ibecause it was in the second half of

.'
the ith row (these two parts do not have an element in cornmon). The 2n fills the gap so that
in all we get all the numbers from 1 to 4n - l.
Solution
An exactly similar argument works for i > n. This time the second half of the row and the
second half ofthe column (which overlap by one element) give us the numbers from 1 to

• 2n - 1, and the first halves (which do not overlap) give us 2n to 4n - 1. So Bij is silver.
Hence there are an infinite nurober of sil ver matrices.



The key is to derive a recurrence relation for ftn) [not for f(2"»). Ifn is odd, then the sum In the convex quadrilateral ABCO, the diagonal s AC and BO are perpendicular and the
must have a l. In faet, there is a one-to-one correspondence between sums for n and sums opposite sides AB and OC are not parallel. The point P, where the perpendicular bisectors
for n-l. So: of AB and OC meet, is inside ABCO. Prove that ABCO is cyclic ifand only ifthe triangles
ABP and COP have equal areas.
ft2n+ 1) = f(2n)

Now consider neven. The same argument shows that there is a one-to-one correspondence
between sums for n-I and sums for n which have a l. Sums which do not have a 1 are in SolutioD
one-to-one correspondence with sums for n/2 (just halve each terrn). So:

ft2n) = ft2n - 1) + ftn) = f(2n - 2) + f(n).

The upper limit is now almost immediate. First, the recurrence relations show that f is
monotonic increasing. Now apply the second relation repeatedly to f(20+1)to get:

f(20+1)= f(2o+l _ 2") + f(2° - 20-1+ 1) + ... + f(2" - 1) + f(2D) = f(2") + f(2" - 1 ) + ... + f(2D-1
+ 1) + f(2") (*)

and hence ft2"+I) ~ (28-1+ 1)f(2")

We can now establish the upper limit by induction. It is false for n = 1 and 2, but almost
true for n = 2, in that: ~ = 22212• Now if f(2~:5 2n212, then the inequality just established
shows that f(20+1)< 2"2 < 2(02+2n+1Y:!
= z<0+1)212,
so it is true for n + l. Hence it is true for
all n > 2. Let AC and BO meet at X. Let H, K be the feet ofthe perpendiculars from P to AC, BO
respectively. We wish to express the areas of ABP and COP in terrns ofmore traetable
Applying the same idea to the lower limit does not work. We need something stronger. We triangles. There are essentially two different configurations possible. In the first, we have
may continue (*) inductively to obtain f(20+1)= f(2D) + f(2" - 1) + ... + f(3) + f(2) + f(1) + 1.
area PAB = area ABX + area PAX + area PBX, and area PCO = area COX - area PCX-
(*.) We now use the following lemrna: area POx. So ifthe areas being equal is equivalent to: area ABX - area COX + area PAX +
area PCX + area PBX + area POX = O.ABX and COX are right-angled, so we may write
f(1) + f(2) + ... + f(2r) ~ 2r f(r) their areas as AX' BX/2 and CX-OX/2. We may also put AX = AH - HX = AH - PK, BX =
BK - PH, CX = CH + PK, OX = OK + PH. The other triangles combine in pairs to give
We group the terrns on the Ihs into pairs and claim that f(1) + f(2r) ~ f(2) + f(2r-l) ~ f(3) + area ACP + area BOP = (AC-PH + BO-PK)/2. This leads, after sorne cancellation to
f(2r-2) ~ ... ~ f{r) + f(r+ 1). Ifk is even, then f(k) = ftk+ 1) and Q2r-k) = f{2r+I-k), so f(k) + AH·BK = CH·OK. There is a similar configuration with the roles of AB and CO reversed.
f(2r+ l-k) = f(k+ 1) + f(2r-k). Ifk is odd, then f(k+ 1) = f(k) + f(k+ 1)12)and f(2r+ l-k) =
f{2r-k) + f«2r-k+I)/2), but fis monotone so f{(k+I)/2):5 f(2r+l-k)l2) and hence f(k) + The second configuration is area PAB = area ABX + area PAX - PBX, area PCO = area
f{2r+l-k) ~ f(k+ 1) + f(2r-k), as required. COX + area POX - area PCX. In this case AX = AH + PK, BX = BK - PH, CX = CH - PK,
OX = OK + PH. But we end up with the same result: AH·BK = CH·OK.
Applying the lemma to (•• ) gives ft20+1)> 20+11\20-1).This is sufficient to prove the lower
limit br induction. It is true for n = l. Suppose it is true for n. Then 1\20+1)> 20+12(0-1)2/4
=
Now if ABCO is cyclic, then it follows irnrnediately that P is the center of the circumcircle
2(02-20++40+4)/4>in+I)2/4,so it is true for n+ 1.
and AH = CH, BK = OK. Hence the areas ofPAB and PCO are equal.

..
Conversely, suppose the areas are equal. IfPA > PC, then AH > CH. But since PA = PB

IMO 1998
and PC = PO (by construction), PB > PO, so BK > OK. So AH·BK > CH·OK.
Contradiction. So PA is not greater than PC. Similarly it cannot be less. Hence PA = PC.
...
But that implies PA = PB = PC = PO, so ABCO is cyclic.
ProblemAl



• (2m - I )k, or (2mk - l)/k. Thus if we take these II;S together with those giving k, we get 2mk -


1, which completes the induction.
ProblemAl
Problem Bl

•.. In a competition there are a contestants and b judges, where b ~ 3 is an odd integer. Each
judge rates each contestant as either "pass" or "fail". Suppose k is a number such that for
any two judges their ratings coincide for at most k contestants. Prove k/a ~ (b-I y2b .
Determine all pairs (a, b) ofpositive integers such that ab2 + b + 7 divides a2b + a + b.


Answer(a, b)=(II, 1),(49, l)or(7k2, 7k).
Solution

• Let us count the number N oftriples (judge, judge, contestant) for which the two judges are
distinct and rate the contestant the same. There are b(b-I )/2 pairs of judges in total and each
pair rates at most k contestants the same, so N ::;kb(b-I y2.
Solution

• Now consider a fixed contestant X and count the number of pairs of judges rating X the
same. Suppose x judges pass X, then there are x(x-I y2 pairs who pass X and (b-x'(b-x-I y2
Ifa< b, then b z a+ 1, so ab' + b+ 7 >ab2 + b~ (a+ IXab+ 1)= a2b +a+ ab z a2b+ a+
b. So there can be no solutions with a < b. Assume then that a ~ b.

• who fail X, so a total of(x(x-I) + (b-xXb-x-I)Y2 pairs rate X the same. But (x(x-I) + (b-
xXb-x-I))12 = (2x2 - 2bx + b2 - bY2 = (x - b/2)2 + b2/4 - bl2 ~ b2/4 - b/2 = (b - li/4 - 1/4.
Let k = the integer (a2b + a + b)/(ab2 + b + 7). We have (a/b + IlbXab2 + b + 7) = ab2 + a +
ab + 7a/b + 71b + I > ab2 + a + b. So k < a/b + lib. Now ifb ~ 3, then (b -71b) > O and


But (b - 1)2/4 is an integer (since b is odd), so the nurnber of pairs rating X the same is at hence (a/b - IlbXab2 + b + 7) = ab2 + a - a(b -71b) - I -71b < ab2 + a < ab2 + a + b. Hence
least (b - 1)2/4. Hence N ~ a (b - Ii/4. Putting the two inequalities together gives k/a ~ (b - either b = I or 2 or k > a/b - lib.
1)/2b.


If a/b - IIb < k < a/b + IIb, then a - I < kb < a + 1. Hence a = kb. This gives the solution (a,
b) = (7e, 7k).

• ProblemA3
It remains to consider b = I and 2. Ifb = 1, then a + 8 divides a2 + a + I and hence also a(a
+ 8) - (a2 + a + 1) = 7a - 1, and hence also 7(a + 8) - (7a - 1) = 57. The only factors bigger
than 8 are 19 and 57, so a = II or 49. It is easy to check that (a, b) = (11, 1) and (49, 1) are


For any positive integer n, let den) denote the number of positive divisors of n (including I
and n). Determine all positive integers k such that d(n2) = k den) for sorne n. indeed solutions.


If b = 2, then 4a + 9 divides 2a2 + a + 2, and hence also a(4a + 9) - 2(2a2 + a + 2) = 7a - 4,
and hence a1so 7(4a + 9) - 4(7a - 4) = 79. The only factor greater than 9 is 79, but that gives
a = 35/2 which is not integral. Hence there are no solutions for b = 2.
Solution

• Let n = Plal... Pra,. Then den) = (al + IXa2 + 1) ... (a, + 1), and d(n2) = (2al + IX2a2 + 1) ...
(2a, + 1). So the 11;must be chosen so that (2al + IX2a2 + 1) ... (2a, + 1) = k (al + IXa2 + 1)
A variant on thisfromJohannes Tang Lek Huo is as follows:

• ... (a, + 1). Since all (2a; + 1) are odd, this clearly implies that k must be odd. We show that
conversely, given any odd k, we can find 11;.
We have ab2 + b + 7 divides a(ab2 + b + 7) - b(a2b + a + b) = 7a - b2 . If7a = b2, then b must
be a multiple of 7, so b = 7k for sorne k. Then a = 7e, and it is easy to check that this is a
solution. We cannot have 7a < b2 for then O < b2 - 7a < ab2 < ab2 + b + 7. If 7a > b, then we


• >
We use a form of induction on k. First, it is true for k = I (take n = 1). Second, we show
that if it is true for k, then it is true for 2~ - l. That is sufficient, since any odd number has
the form 2~ - I for sorne smaller odd number k. Take 11;= 2'«2m - I)k - 1) for i = O, 1, ... ,
m-l. Then 211;+ I = i+l(2m - I)k - (2i+l - 1) and 11;+ I = i(2m - I)k - (i -1). So the product
ofthe (211;+ I)'s divided by the product ofthe (11;+ l)'s is 2mc2m- I)k - (2m - 1) divided by
must have 7a - b::; ab2 + b + 7 > ab2, so 7 > b2, so b = I or 2.

We can then continue as aboye .

ProblemBl





Let I be the incenter of the triangle ABC. Let the incircle of ABC touch the sides BC, CA, Thus we have f(f(t) = t and f(r) = f(tl Hence f(sti = f(N) = f(s2f(f(r))) = f(sif(t2) =
AB at K, L, M respectively. The line through B parallel to MK meets the lines LM and LK f(sif(tl So f(st) = f(s) fl,t).
at R and S respectively. Prove that the angle RIS is acute.
Suppose p is a prime and fl,p) = m·n. Then fl,m)f(n) = fl,mn) = f(f(p» = p, so one offl,m),
fl,n) = 1. But iff(m) = 1, then m = f(fl,m» = f(1) = 1. So f(p) is prime. Iff(p) = q, then f(q) =
p.
Solution
Now we may define f arbitarily on the primes subject only to the conditions that each
We show that ru2+ Se - RS2 > O. The result then follows from the cosine rule. f(prime) is prime and that iff(p) = q, then f(q) = p. For suppose that s = Plal...p,a, and that
f(p¡) = q, 1ft has any additional prime factors not included in the q¿ then we may add
BI is perpendicular to MK and hence also to RS. So IR2 = BR2 + Be and IS2 = BI2 + Bs2. additional p's to the ~xp~essi<:,nfor s Sf
tha~ thex, ~re incl~d~ (taking the ~diti~nal :'~o be
Obviously RS = RB + BS, so RS2 = BR2 + BS2 + 2 BR·BS. Hence ru2 + se-
RS2 = 2 Be - zero, So suppose r=q¡"¡ ...q",Thent f(S)-ql
...p,2 ,+a, = S fl,t)2
1 l ...q, r ,andhencef(t f(S)-PI 1 1
2 BR·BS. Consider the triangle BRS. The angles at B and M are 90 - B/2 and 90 - AI2, so
the angle at R is 90 - C/2. Hence BRlBM = cos A/2/cos C/2 (using the sine rule). Similarly,
considering the triangle BKS, BSIBK = cos C/2/cos A/2. So BR·BS = BM·BK = BK2. We want the minimum possible value off(1998). Now 1998 = 2.33.37, so we achieve the
Hence RI2 + Se - RS2 = 2(B12- BK2) = 2 IK2 > O. minimum value by taking fl,2) = 3, fl,3) = 2, f(37) = 5 (and f(37) = 5). This gives fl,1998) =
3.235 = 120.
Problem B3
IMO 1999
Consider all functions ffrom the set ofall positive integers into itselfsatisfying f(t2fl,s» = s
f(ti for al! s and t. Determine the least possible value off(1998). ProblemAl
.J
Answer
Find al! finite sets S of at least three points in the plane such that for all distinct points A, B
in S, the perpendicular bisector of AB is an axis of symmetry for S.
...
120
Solution

by Gerhard Woeginger
Solution
The possible sets are just the regular n-gons (n > 2).
Let fl,1) = k. Then f(kt2) = f(ti and f(f(t» = k~. Also f(kti = 1'f(kt)2 = f(k3r) = f(12fl,fl,kr»)
= k2f(kt2) = k2f(tl Hence f(kt) = k f(t). Let Al, A2, ... ,Ak denote the vertices ofthe convex hull ofS (and take indices mod k as
necessary). We show first that these form a regular k-gon, Ai+l must lie on the
By an easy induction k"f(tn+l) = f(trl. But this implies that k divides f(t). For suppose the perpendicular bisector of A¡ and A¡+2(otherwise its reflection would lie outside the hull).
highest power of a prime p dividing k is a > b, the highest power of p dividing f(t). Then a Hence the sides are all equal. Similarly, A¡+l and Ai+2must be reflections in the
> b(1 + IIn) for some integer n. But then na > (n + I)b, so k" does not divide f(trl. perpendicular bisector of A¡ and Ai+3(otherwise one of the reflections would lie outside the
Contradiction. hull). Hence al! the angles are equal.

Let g(t) = f(t)/k. Then f(t2f(s» = f(t2kg(s» = k f(t2g(s) = k2g(rg(s», whilst s fl,ti = k2s f(t)2. Any axis of symmetry for S must also be an axis of symmetry for the A¡, and hence must
So g(rg(s» = s g(t)2. Hence g is also a function satisfying the conditions which eyidently pass through the center C of the regular k-gon. Suppose X is a point of S in the interior of
has smaller yalues than f (for k > 1). It also satisfies g(1) = 1. Since we want the srnallest k-gon. Then it must lie inside or on some triangle A¡A¡+lC.C must be the circumcenter of
possible yalue off(1998) we may restrict attention to functions fsatisfying f(1) = 1. A¡A¡+lX(since it lies on the three perpendicular bisectors, which must all be axes of
symmetry of S), so X must lie on the circle center C, through A¡ and A¡+l. But all points of



• the triangle AiAi+IX lie strictly inside this circle, except Aand Ai+l, so X cannot be in the Comment. The first so/ution is e/eganl and shows c1early why the inequality is true. The

• interior of the k-gon. second solution is more plodding, bia uses an approach which is more general and can be
applied in many other cases. Al least with hindsight, thefirst soluüon is not as impossib/e
lo find as one might think: A little p/aying around soon uncovers the fact that one can gel C


ProblemA2
= 1/8 with two x, equal and the rest zero, and that this looks lilee the best possib/e. One JUSI
Let n >= 2 by a fixed integer. Find the smallest constant C such that for all non-negative has to malee the jump 10 rep/acing (xl + xl) by L xl. The soluüon is then fairly c/ear. Of


reals Xl, ... , Xn: course, that does not detraet from the contestant's achievement, because 1 and almost
everyone else who has /ooleed al the prob/em fai/ed to malee that jump.

• Determine when equality occurs.


ProblemAl

• Given an n X n square board, with neven. Two distinct squares of the board are said to be


Answer Answer: C = 1/8. Equality ifftwo Xiare equal and the rest zero. adjacent if they share a common side, but a square is not adjacent to itself. Find the
minimum number of squares that can be marked so that every square (marked or not) is
adjacent to at least one marked square.

• Solution

• By a member of the Chinese team at the IMO - does anyone know who?

a; xi)' = CL x? + 2 ¿i<jXiXi;:>:4 CL Xi2)(2 ¿i<j XiXj)= 8 ¿i<j ( XiXj¿ Xk2);:>:8 ¿i<j XiXj(Xi2
+
Answer

Answer: nl2 (nl2 + 1) = n(n + 2)14.

• Xj ).


The second inequality is an equality only ifn - 2 oflhe Xiare zero. So assume that X3= l4 =
... = X. = O. Then for Ihe first inequality to be an equality we require that (X12+ xl) = 2 XIX2 Solution
and hence Ihat XI = X2.However, that is clearly also suffícient for equality.


Let n = 2m. Color alternate squares black and white (Iike a chess board). It is suffícient to
show Ihat m(m+ 1)/2 white squares are necessary and suffícient to deal wilh aH Ihe black
squares.

• A/temative so/ulion by Gerhard Woeginger

Setting XI = X2= 1, Xi= O for i > 2 gives equality with C = 1/8, so, C cannot be smaHer than
This is almost obvious if we look at the diagonals.

• 1/8. Look first at Ihe odd-Iength white diagonals. In every other such diagonal, mark alternate
squares (starting from the border each time, so that r+1 squares are marked in a diagonal


We now use induction on n. For n = 2, the inequality wilh C = 1/8 is equivalent to (XI - X2)' length 2r+ 1). Now each black diagonal is adjacent to a picked white diagonal and hence
;:>:
O, which is true, wilh equality iff XI = X2.SOIhe result is true for n = 2. each black square on it is adjacent to a marked white square. In all 1 + 3 + 5 + ... + m-I + m
+ m-2 + ... + 4 + 2 = 1 + 2 + 3 + ... + m = m(m+ 1)12 white squares are marked. This proves


For n > 2, we may take XI+ ... + X. = 1, and XI ::;X2::;... ::;X•. Now replace XI and X2by O suffíciency.

.'
and XI + X2.The sum on Ihe rhs is unchanged and the sum on Ihe Ihs is increased by (XI +
xd S - (X13+ x/> S - XIX2(XI2 + X22),where S = X3+ l4 + ... + X•. But S is at least 1/3(lhe For necessity consider Ihe alternate odd-Iength black diagonals. Rearranging, Ihese have
critica! case is n = 3, Xi= 1/3),so Ihis is at least XIX2(XI+ X2- XI2- xl). This is strictly lengths 1,3,5, ... ,2m-1. A white square is only adjacent to squares in one oflhese
greater than O unless XI = O (when it equals O), so Ihe result foHows by induction. alternate diagonals and is adjacent to at most 2 squares in il So we need at least 1 + 2 + 3 +
... + m = m(m+I)/2 white squares.





• 51
Problem Bl Problem B2

Find all pairs (n, p) ofpositive integers, such that: pis prime; n:S 2p; and (p - 1)" + 1 is The circles el and e2 lie inside the circle e, and are tangent to it at M and N, respectively.
divisible by np-I el passes through the center ofe2. The common chord of'C¡ and e2, when extended, meets

.....
e at A and B. The lines MA and MB meet el again at E and F. Prove that the line EF is
tangent to Ca.

Answer

(1, p) for any prime p; (2, 2); (3, 3). Solution

Solution by Jean-Pierre Ehrmann

Solution Let 0,01,02 and r, ri, r2 be the centers and radii of'C, el, e2 respectively. Let EF meet the
line 0102 at W, and let 02W = x. We need to prove that x = r2.
by Gerhard Woeginger, Technical University, Graz
Take rectangular coordinates with origin O2, x-axis 0201, and let O have coordinates (a, b).
Answer: (1, p) for any prime p; (2, 2); (3, 3). Notice that O and M do not, in general, lie on 0102. Let AB meet the line 0102 at V.

Evidently (1, p) is a solution for every prime p. Assume n > 1 and take q to be the smallest We observe first that 02V = rl/(2 rl). [For example, let X be a point ofintersection of C¡
prime divisor ofn. We show first that q = p. and e2 and let y be the midpoint of 02X. Then 01 Y02 and XV02 are similar. Hence,
ON/02X= 02Y/020I-]
Let x be the smallest positive integer for which (p - I)X = - 1 (mod q), and y the smallest
positive integer for which (p - l'f = 1 (mod q). Certainly y exists and indeed y < q, since (p An expansion (or, to be technical, a homothecy) center M, factor tlt, takes 01 to O and EF
- I)q-I = 1 (mod q). We know that (p - 1)" = -1 (mod q), so x exists also. Writing n = sy + r, to AB. Hence EF is perpendicular to 0102. Also the distance of'O¡ from EF is rl/r times the
with O :s r < y, we concl ude that (p - 1)' = -1 (mod q), and hence x :s r < y (r cannot be zero, distance ofO from AB, so (r, - x) = rl/r (a - rl/(2 rl» (*).
since 1 is not -1 (mod q) ).
We now need to find a. We can get two equations for a and b by looking at the distances of
Now write n = hx + k with O:S k < x. Then -1 = (p - 1)" = (-I)\p - Il (mod q). h cannot O from 01 and O2. We have:
be even, because then (p - I)k = -1 (mod q), contradicting the minimality ofx. So h is odd (r - rli = (r¡ - ai + b2, and
and hence (p - I)k = 1 (mod q) with O:S k < x < y. This contradicts the minimality ofy (r- rd = a2 + b2
unless k = O, so n = hx. But x < q, so x = 1. So (p - 1) = -1 (mod q). p and q are primes, so q
= p, as claimed. Subtracting to eliminate b, we get a = r//(2 rl) + r - r r2/rl. Substituting back in (*), we
get x = r2, as required.
So P is the smallest prime divisor of n. We are also given that n :s 2p. So either p = n, or p =
2, n = 4. The latter does not work, so we have shown that n = p. Evidently n = p = 2 and n = Alternative solution by Marcin Kuczma, communicated Ame Smeets
p = 3 work. Assume now that p > 3. We show that there are no solutions ofthis type.

Expand (p - 1)" + 1 by the binomial theorem, to get (since (-1)" = -1): 1 + -1 + p2 - 1/2 p(p-
l)p2 + p(p _ l)(p _ 2)/6 p3 _ ...
The terms ofthe form (bin coeff) pi with i >= 3 are obviously divisible by ~3, since the
binomial coefficients are all integral. Hence the sum is p2 + a multiple of p . So the sum is
not divisible by p3. But for p > 3, ~I is divisible by p3, so it cannot divide (p - 1)" + 1, and
there are no more solutions.



• In particular, this is true for x in A. Comparing with (*) we deduce that e = l. So for aIl x in


M
R we must have f{x) = l - x2/2. Finally, it is easy to check that this satisfies the original
relation and hence is the unique solution.

• IM02000

..
~


ProblemAl

AB is tangent to the circles CAMN and NMBD. M lies between C and D on the line CD,
and CD is parallel to AB. The chords NA and CM meet at P; the chords NB and MD meet
at Q. The rays CA and DB meet at E. Prove that PE = QE.

• Let C, and C2 meet at X and Y, and let AN meet C2 again at D. Then AE·AM = AXA y =
Solutioo

• AD·AN, so triangles AED and ANM are similar. Hence LADE = LAMN.

Takethe tangentAPas shown. Then LPAN = LAMN = LADE, soAP and DE are


JI
parallel. The homothecy center M mapping C to C, takes the line AP to the line ED, so ED
is tangent to C, at E. A similar argument show that it is tangent to C2 at D. The homothecy
takes AB to EF, so EF is perpendicular to 0,02 (the line of centers). Hence 02EF is
isosceles. So angle 02EF = angle 02FE = angle DE02 (DE tangent). In other words, 02E

...
bisects angle DEF. But ED is tangent to C2, so EF is also .

• Problem6

Determine all functions f: R -> R such that f{x - f(y) ) = f( f{y) ) + x f{y) + f(x) - l for all x,
Angle EBA = angle BDM (because CD is parallel to AB) = angle ABM (because AB is
tangent at B). So AB bisects EBM. Similarly, BA bisects angle EAM. Hence E is the

• y in R. IR is the reals.] reflection of M in AB. So EM is perpendicular to AB and hence to CD. So it suffices to


show that MP = MQ.

• Solutioo
Let the ray NM meet AB at X. XA is a tangent so XA2 = XM·XN. Similarly, XB is a
tangent, so XB2 = XM·XN. Hence XA = XB. But AB and PQ are parallel, so MP = MQ.

• Solution communicated by Ong Shien Jin ProblemAl

• Let e = f1:0)and A be the image f{R). If a is in A, then it is straigbtforward to find f\a): A, B, C are positive reals with product 1. Prove that (A - 1 + llBXB - 1 + l/CXC - 1 + l/A)
putting a = f(y) and x = a, we get f{a - a) = f1:a)+ a2 + f{a) - 1, so f{a) = (1 + c)/2 - a 12 (*). ::;1.


.'
The next step is to show that A - A = R. Note first that e canoot be zero, for if it were, then
putting y = O,we get: f{x - e) = f{c) + xc + f{x) - 1 (**) and hence f(O) = f{c) = 1.
Contradiction. But (**) also shows that f{x - e) - f{x) = xc + (f{c) - 1). Here x is free to vary Solutioo
over R, so xc + (f{c) - 1) can take any value in R.
An elegant solution due lo Robin Chapman is as follows:


Thus given any x in R, we may find a, b in A such Ihat x = a-b. Hence f1:x)= f{a - b) = f{b)
+ ab + f{a) - 1. So, using (*): f1:x)= e - b2/2 + ab - a2/2 = e - x212.





1I
~

...
(B - I + l/C) = B(I - I/B + I/(BC) ) = B(I + A - IIB). Hence, (A - I + IIBXB - I + l/C) = move the rightmost point arbitrarily far to the right (and hence all the points, since another
..1

....
B(A2 _ (1 - I/Bi):5 B A2 So the square of the product ofall three S B A2 e B2 A e' = 1. N-I moves will move the other points to the right of the rightmost point).
...1
Actually, that is not quite true. The last sentence would not follow ifwe had sorne negative Problem Bl
left hand sides, because then we could not multiply the inequalities. But it is easy to deal
separately with the case where (A - I + IIB), (B - I + I/C), (e - I + l/A) are not all 100 cards are numbered I to lOO(each card different) and placed in 3 boxes (at least one
positive. If one ofthe three terms is negative, then the other two must be positive. For card in each box). How many ways can this be done so that iftwo boxes are selected and a
example, if A - I + IIB < O,then A < 1, so e - 1 + l/A> O,and B > 1, so B - 1 + I/C> O. card is taken from each, then the knowledge of their sum alone is always sufficient to

...
But if one term is negative and two are positive, then their product is negative and hence identify the third box?
less than l.

Few peop/e would manage this under exam conditions, but there are plenty of/onger and
easier rofind solutions! Answer
..1

.._,
12. Place 1, 2, 3 in different boxes (6 possibilities) and then place n in the same box as its
residue mod 3. Or place I and lOOin different boxes and 2 - 99 in the third box (6
ProblemA3 possibilities ). ...1
k is a positive real. N is an integer greater than l. N points are placed on a line, not all
coincident. A move is carried out as foHows. Pick any two points A and B which are not

..
coincident. Suppose that A lies to the right of B. Replace B by another point B' to the right Solution
of A such that AB' = k BA. For what values ofk can we move the points arbitrarily far to
the right by repeated moves? An elegant solutton communicated (in outline) by both Mohd Suhoimi Ram/y and Fokko J
van de Búlt is as follows:

Let H, be the corresponding result that for cards numbered I to n the only solutions are by
Answer residue mod 3, or I and n in separate boxes and 2 to n - I in the third box. It is easy to
check that they are solutions. Hu is the assertion that there are no others, H3 is obviously ...1
k ~ I/(N-l). true (although the two cases coincide). We now use induction on n. So suppose that the
result is true for n and consider the case n + l.
...
...
..
Suppose n + 1 is alone in its box. If 1 is not also alone, then let N be the sum of the largest
Solution cards in each of the boxes not containing n + 1. Since n + 2 :5N :5n + (n - 1) = 2n - 1, we
can achieve the same sum N as from a different pair ofboxes as (n + 1) + (N - n -1).
An elegam soluüon by Gerhard Woeginger is as follows: Contradiction, So 1 must he alone and we have one of the solutions envisaged in Hu+l.

Suppose k < l/(N-I), so that leo = IIk - (N - 1) > O. Let X be the sum ofthe distances ofthe If n + 1 is not alone, then if we remove it, we must have a solution for n. But that solution
points from the rightrnost point If a move does not change the rightmost point, then it cannot be the n, 1, 2 to n - 1 solution. For we can easily check that none of the three boxes
reduces X. If it moves the rightmost point a distance z to the right, then it reduces X by at will then accomodate n + l. So it mus! be the mod 3 solution. We can easily check that in ..1
least z/k - (N-I)z = leo Z. X cannot be reduced below ni!. So the total distance moved by the this case n + 1 must go in the box with matching residue, which makes Ihe (n + 1) solution
rightrnoSI point is al most XoIleo, where Xo is Ihe inilial value of X. the olher solution envisaged by Hn+l. That completes the induction.
..1
eonversely, suppose k ~ I/(N-I), so Ihat kl = (N-I) - IIk ~ O. We always move the leftmost My much more p/odding so/ution (which I was quite p/eased wirh unti/I saw the more
point This has Ihe effect of moving the rightrnost point z > O and increasing X by (N-I)z - e/eganr so/urion above) follows. Ir rook about holf-an-hour and shows the kind of k1udge . ...1
zIk = klz~ O. So X is neverdecreased. Butz ~ k X1(N-I)~ k XoI(N-I) > O. So we can one is Iike/y ro come up wirh under rime pressure in an exam!

~
...
....
....
...1
..J



• W ith a suitable labeling ofthe boxes as A, B, C, there are 4 cases to consider:

• Case 1: A contains 1; B contains 2; C contains 3 Answer


Case 2: A contains 1,2
Case 3: A contains 1, 3; B contains 2 Yes
Case 4: A contains 1; B contains 2,3 .

• •
We show that Cases I and 4 each yieldjust one possible arrangement and Cases 2 and 3
none. Solution

• In Case 1, it is an easy induction that n must be placed in the same box as its residue (in Note that for b odd we have 2ab + 1 = (2' + 1X2a(b-I)- 2a(b-2)+ ... + 1), and so 2' + 1 is a


other words numbers with residue 1 mod 3 go into A, numbers with residue 2 go into B, factor of 2ab + 1. It is sufficient therefore to find m such that (1) m has only a few distinct
and numbers with residue O go into C). For (n + 1) + (n - 2) = n + (n - 1). Hence n + 1 must prime factors, (2) 2m + 1 has a large number of distinct prime factors, (3) m divides 2m + 1.
go in the same box as n - 2 (ifthey were in different boxes, then we would have two pairs For then we can take k, a product of enough distinct primes dividing 2m + 1 (but not m), so

•..
from different pairs ofboxes with the same sum). lt is also clear that this is a possible that km has exactly 2000 factors. Then km still divides 2m + 1 and hence 2km + l.
arrangement. Given the sum oftwo numbers from different boxes, take its residue mod 3. A
residue of O indicates that the third (unused) box was C, a residue of 1 indicates that the The simplest case is where m has only one distinct prime factor p, in other words it is a
third box was A, and a residue of 2 indicates that the third box was B. Note that this unique power of p. But if P is a prime, then p divides 2P - 2, so the only p for which p divides 2P +
arrangement gives 6 ways for the question, because there are 6 ways of arranging 1, 2 and 3 1 is 3. So the questions are whether a¡,= 2m + 1 is (1) divisible by m = 3b and (2) has a large
in the given boxes. number of distinct prime factors.

• In Case 2, let n be the smallest number not in box A. Suppose it is in box B. Let m be the
smallest number in the third box, C. m - I cannot be in C. If it is in A, then m + (n - 1) = (m
- 1) + n. Contradiction (m is in C, n - I is in A, so that pair identifies B as the third box, but
m - 1 is in A and n is in B, identifying C). So m - I must be in B. But (m - 1) + 2 = m + l.
a¡,+1= ah(22m- 2m + 1), where m = 3b But 2m = (ah - 1), so a¡,+1= ah(ah2- 3 a¡,+ 3). Now al =
9, so an easy induction shows that 3h+1divides a¡" which answers (1) affirmatively. Also,
since ah is a factor of as-r, any prime dividing ah also divides ah+l. Put a¡,= 3h+1~. Then ~+1
= bh(32h+lbh2_ 3h+2bh+ 1). Now (32h+I~2 - 3h+2~ + 1~> 1, so it must have sorne prime


Contradiction. So Case 2 is not possible. factor p > l. But p cannot be 3 or divide ~ (since (3 h+I~2 - 3h+2bh+ 1) is a multiple of 3~
plus 1), so bh+1has at least one prime factor p > 3 which does not divide bh. So bh+1has at
In Case 3, let n be the smallest number in box C, so n - 1 must be in A or B. If n - 1 is in A, least h distinct prime factors greater than 3, which answers (2) affirmatively. But that is all

• then (n - 1) + 2 = n + 2. Contradiction (a sum of numbers in A and B equals a sum from C we need. We can take m in the first paragraph aboye to be 32000:(1) m has only one distinct
and A). If n - I is in B, then (n - 1) + 3 = n + 2. Contradiction ( a sum from B and A equals prime factor, (2) 2m+ 1 = 32001b2000has at least 1999 distinct prime factors other than 3, (3)
a sum from C and B). So Case 3 is not possible. m divides 2m + 1. Take k to be a product of 1999 distinct prime factors dividing b2000.Then

• In Case 4, let n be the smallest number in box C. n - 1 cannot be in A, or (n - 1) + 2 = 3 + n


N = km is the required number with exactly 2000 distinct prime factors which divides 2N +
1.


(pair from A, B with same sum as pair from B, C), so n - 1 must be in B. Now n + 1 cannot
be inA (or(n + 1)+2=3+n),orinBorC(orl +(n+ 1)=2+n).Son+ 1 cannotexist
and hence n = 100. Jt is now an easy induction that all of 4, 5, ... 98 must be in B. For given Problem B3


that m is in B, ifm + 1 were in A, we would have 100 + m = 99 + (m + 1). But this
arrangement (1 in A, 2 - 99 in B, 100 in C) is certainly possible: sums 3 - 100 identify C as AIA2A3 is an acute-angled triangle. The foot ofthe altitude from A¡ is K¡ and the incircle
the third box, sum 10I identifies B as the third box, and sums 102-199 identify A as the touches the side opposite A¡ at L¡. The line KIK2 is reflected in the line LIL2. Similarly, the

• third box. Finally, as in Case 1, this unique arrangement corresponds to 6 ways ofarranging line K2K3 is reflected in L2L3and K3KI is reflected in L3LI. Show that the three new Iines
the cards in the given boxes. form a triangle with vertices on the incircle.

.-• Problem B2

Can we find N divisible by just 2000 different primes, so that N divides 2N + 1? [N may be
divisible by a prime power.]

._
parallel, SO K2X and OA¡ are parallel. But OA¡ is perpendicular to L2L3,so K2X is also
A,
perpendicular to L2L3and hence X is the reflection of K2in L2L3.

",

Solutíon

Now the angle K3K2A¡= angle A¡A2A3,because it is 90° - angle K3K2A2= 90° - angle
K3A3A2(A2A3K2K3is cyclic with A2A3a diarneter) = angle A¡A2A3.So the reflection of
K2K3in L2L3is a line through X making an angle A¡A2A3with L2X, in other words, it is
the line through X parallel to A2A3.

Let Mi he the reflection of Lo in AiO. The angle M2XL2= 2 angle OXL2 = 2 angle A¡A20
(since A¡A2 is parallel to L2X) = angle A¡A2A3,which is the angle betwee L2Xand A2A3.
So M2X is parallel to A2A3,in other words, M2 lies on the reflection of K2K3in L2L3.

If follows similarly that M3 lies on the reflection. Similarly, the line M¡M3 is the reflection
ofK¡K3 in L¡L3, and the line M¡M2 is the reflection ofK¡K2 in L¡L2and hence the triangle
formed by the intersections ofthe three reflections is just M¡M2M3.

Let O be the centre of the incircle. Let the line parallel to A¡A2 through L2 meet the line
A20 at X. We will show that X is the reflection of'Kj in L2L3.Let A¡A3 meet the line A20
at B2. Now A2K2is perpendicular to K2B2and OL2 is perpendicular to L2B2,so A2K2B2and IM02001
OL2B2are similar. Hence K2L2/L2B2= A20/OB2. But OA3 is the angle bisector in the
ProblemAl
triangle A2A3B2,so A20/0B2 = A2A3IB2A3.
ABC is acute-angled. O is its circurncenter. X is the foot of the perpendicular from A to
Take B'2 on the line A20 such that L2B2= L2B'2(B'2 is distinct from B2 unless L2B2is
perpendicular to the line). Then angle L2B'2X= angle A3B2A2.Also, since L2X is para1lel to BC. Angle C 2': angle B + 30°. Prove that angle A + angle COX < 90°
A2A¡, angle L2XB'2= angle A3A2B2.So the triangles L2XB'2and A3A2B2are similar.
Hence A2A3IB2A3 = XL2IB2'L2= XL2/B2L2(since B'2L2= B2L2).

Thus we have shown that K2L2/L2B2= XL2IB2L2and hence that K2L2= XL2. L2X is SolutioD
parallel to A2A¡ so angle A2A¡A3= angle A¡L2X = angle L2XK2+ angle L2K2X= 2 angle
L2XK2(isosceles). So angle L2XK2= 1/2 angle A2A¡A3= angle A2A¡O. L2Xand A2A¡ are


• ProblemA3

Integers are placed in each ofthe 441 cells ofa 21 x 21 array. Each row and each column


has at most 6 different integers in it. Prove that sorne integer is in at least 3 rows and at
least 3 columns.

..
.,
• Take D on the circumcircle with AD parallel to BC. Angle CBD = angle BCA, so angle
ABD?: 30°. Hence angle AOD?: 60°. Let Z be the midpoint of AD and y the midpoint of
Solution

Notice first that the result is not true for a 20 x 20 array. Make 20 rectangles each 2 x 10,
labelled 1,2, ... ,20. Divide the 20 x 20 array into four quadrants (each lO x lO). In each of

• BC. Then AZ?: R/2, where R is the radius ofthe circumcircle. But AZ = YX (since AZYX the top left and bottom right quadrants, place 5 rectangles horizontally. In each of the other

'.
is a rectangle). two quadrants, place 5 rectangles vertically. Now each row intersects 5 vertical rectangles
and I horizontal. In other words, it contains just 6 different numbers. Similarly each
Now O eannot coincide with y (otherwise angle A would be 90° and the triangle would not colurnn. But any given number is in either 10 rows and 2 colurnns or vice versa, so no
be aeute-angled). So OX > YX?: R/2. But XC = YC - YX < R - YX :s R/2. So OX> XC. number is in 3 rows and 3 columns. [None ofthis is neeessary for the solution, but it helps


to show what is going on.]
Henee angle COX < angle OCx. Let CE be a diameter of the eircle, so that angle OCX =
angle ECB. But angle ECB = angle EAB and angle EAB + angle BAC = angle EAC = 90°, Returning to the 21 x 21 array, assume that an arrangement is possible with no integer in at
sinee EC is a diameter. Henee angle COX + angle BAC < 90°. least 3 rows and at least 3 columns. Color a eell white if its integer appears in 3 or more
rows and blaek if its integer appears in only l or 2 rows. We eount the white and black
ProblemA2 squares.

a, b, e are positive reals. Let a' = v(a' + 8bc), b' = V(b' + 8ea), e' = vCe' + 8ab). Prove that Eaeh row has 21 eells and at most 6 different integers. 6 x 2 < 21, so every row ineludes an
aja' + b/b' + c/c' >= 1. integer whieh appears 3 or more times and henee in at most 2 rows. Thus at most 5
different integers in the row appear in 3 or more rows. Eaeh sueh integer can appear at most
2 times in the row, so there are at most 5 x 2 = 10 white eells in the row. This is true for
every row, so there are at most 210 white eells in total.
Solution
Similarly, any given eolumn has at most 6 different integers and henee at least one appears

• a: xy)':s L x' L y'.


A not particularly elegant, but fairly easy, solution is to use Cauchy: 3 or more times. So at most 5 different integers appear in 2 rows or less. Eaeh sueh integer
can occupy at most 2 eells in the eolumn, so there are at most 5 x 2 = 10 black eells in the
To get the inequali7 the right way around we need to take x' = aja' [to be precise, we are eolurnn. This is true for every eolumn, so there are at most 210 black eells in total.

• a:
y
takin XI' = aja', x, = b/b', x/ = c/c'.]. Take y' = a a', so that xy = a. Then we get Laja' >=
a) IL aa'. This gives a eontradietion sinee 210 + 210 < 441.

• Evidently we need to apply Cauehy again to deal with L a a'. This time we want L a a' :s
something. The obvious X=a, Y=a' does not work, but ifwe put X=al12, Y=al12a', then we
a: a: a:
Comment. This 100M easy, but (like question 6) 1found it curiously difficult (it took me well
over 2 hours). For a while 1 could not see how to do better than a 12 x 12 array (with 2


have Y a a' :s ail2 a a,')In So we get the required inequality provided that ai12?: rows of 1s, then 2 rows of 2s etc), which was disorienting. Then 1got the argument almost
a: a ~)112 or a:a)3?: L a a". right, but not quite right, which took more time.

Multiplying out, this is equivalent to: 3(ab' + ac' + ba' + be' + ca' + eb')?: 18abe, or a(b - The original question was phrased in terms of 21 boys and 21 girls in a competition with an
e)' + b(c - a)' + e(a - b)'?: O,whieh is c1early true, unknown number of problems. Each boy, girl pair solved at least one problem. Each
competitor solved at most 6problems. One had to show that some problem was solved by at
least 3 boys and at least 3 girls. The recasting in the terms above is almost immediate.
I •

1,.'

'.
I

(,D
Equally, one can easily recast the solution aboye into the competitionformat. Take any boy Answer: 80·.
BI).At least one of the questions he attempts must be attempted by 3 or more girls (because
he attempts at most 6 questions and there are more than 6x2 girls). Hence he attempts at
most 5 questions which are only attempted by less than 3 girls. So at most 5 x 2 = 10 of the
21 pairs (Eo. G) attempt aquestion attempted by less than 3 girls. So at most 210 ofthe 441 Solutioo
pairs pairs (E, G) aitempt such a question. Similarly, at most 210 pairs (E, G) attempt a
question attempted by less than 3 boys. Hence at least 21 pairs (E, G) attempt a question /lo.
attempted by 3 or more girls and 3 or more boys. So there must be at least one such
question.

Note that the arguments aboye generalise immediately to show that in a 4N+ 1by 4N+ 1
array with at most N+ 1different integers in each row and column, there is some integer
that appears in at least 3 rows and 3 columns, but this is not true for a 4N by 4N array.
B e

This is an inelegant solution, but I did get it fast! Without 1055of generality we can take
Problem 81
length AB = l. Take angle ABY = x. Note that we can now solve tbe two triangles AXB
and A YB. In particular, using the sine rule, BX = sin 30·/sin(150·-2x), A y = sin
Let nr, n2, ... , nm be integers, where m is odd. Let x = (XI. ... , xm) denote a permutation of
xlsin(l20·-x), YB = sin 60·/sin(120·-x). So we bave an equation for x.
the integers 1,2, ... , m. Let f{x) = x(n( + X2n2+ ... + xmnm.Show that for sorne distinet
permutations a, b the differenee f{a) - f{b) is a multiple ofm!.
Using the ~ual fo~ula for sin(a + b) etc, and writing s = sin x, e = cos x, we get: 2"';3s2e_
4sc - 2"';3e + 2"';3 e + 6sc - 25 -"';3 = O or -"';3(4e3 - 2e2 - 2e + 1) = 2s(2e2 -3e + 1). This
has a eommon factor 2e - 1. So e = 1/2 or -"';3(2e2 - 1) = 2s(e - 1)(*).
Solutioo
e = 1/2 means x = 60· or angle B = 120·. But in that case the sides opposite A and B are
This is a simple applieation ofthe pigeon bole principIe. pa:alle~ ~d the2triangle is de.rene~te (a fase we assume i.sdisallowed). So squaring (*) and
usmg s - I - e , we get: l6c - 8e - 12e + 8e - I = O. This has another factor 2e - 1.
Dividing that out we get: 8e3 - 6e + I = O. But we remember that 4e3 - 3e = cos 3x, so we
The sum of all m! distinet residues mod m! is not divisible by m! because m! is even (sinee
conclude that eos 3x = -112. That gives x = 40·,80·,160·,200·,280·,320·. But we require
m> 1). [Tbe residues come in pairs a and m! - a, except for m!/2.).
that x < 60· to avoid degeneracy. Henee the angle B = 2x = 80·.
However, the sum of all f{x) as x ranges over all m! permutations is 1/2 (m+ I)! L ru, wbieb
1subsequently found this geometric solution on the ojJicial Wolfram site (Wolfram was one
is divisible by m! (sinee m+ I is even). So at least one residue must oeeur more tban once
ofthe spansors of IMO 2001). 1 cannot say it is much easier, but at least it is geometric.
among the f{x).

/lo.

Problem 82

ABC is a triangle. X lies on BC and AX biseets angle A. y lies on CA and BY bisects


angle B. Angle A is 60·. AB + BX = AY + YB. Find all possible values for angle B.

z
Aoswer

..
.J


••

Extend AB to X' with BX' = BX. Extend A Y to Z with YZ = YB. Then AZ = A Y + YZ = IM02002
AY + YB = AB + BX = AB + BX' = AX'. Angle A = 60°, so AZX' is equilateral.
ProblemAI

••
Use Balso to denote the angle at B. Then angle YBX = B/2. Also angle BXX' + angle
BX'X = B. The triangle is isosceles by construction, so angle BX'X = B/2. Hence angle S is the set of all (h, k) with h, k non-negative integers such that h + k < n. Each element of
XX'Z = 60° - B/2. X lies on the bisector of A and AZ = AX', so XZ = XX'. Hence XZX' = S is colored red or blue, so that if (h, k) is red and h' ~ h, k' ~ k, then (h', k') is also red. A
60° - B/2 also. But angle Z = 60°, so angle YZX = B/2 = angle YBx. type I subset of S has n blue elements with different first member and a type 2 subset of S

.. has n blue elements with different second member. Show that there are the same number of

••
Now YZ = YB, so angle YZB = angle YBZ. Hence angle XZB = angle XBX (they are the type I and type 2 subsets.
difference of pairs of equal angles). If X does not lie on BZ, then we can conclude that XB
=XZ.

In that case, since XZ = XX', we have XB = XX'. But already XB = BX' (by construction), Solutioo
so BXX' is equilateral and hence B/2 = 60°. But then angle B + angle A = 180°, so the


triangle ABC is degenerate (with C at infinity), which we assume is disallowed. Hence X Let a; be the number ofblue members (h, k) in S with h = i, and let b, be the number ofblue
must lie on BZ, which means Z = C and angle B = 2 angle C. Hence angle B = 80, angle C members (h, k) with k = i. It is sufficient to show that bo, bj, ... , bn-I is a rearrangement of
=40.

..•
110, al, ... , an·1(because the number oftype I subsets is the product ofthe a; and the number
of type 2 subsets is the product of the b.),
Problem B3

..
Let C¡be the largest k such that (i, k) is red. If(i, k) is blue for all k then we put e, = -1. Note
K > L > M > N are positive integers such that KM + LN = (K + L - M + NX-K + L + M + that if i < i. then C¡2': Cj,since if (j, C¡) is red, then so is (i, c.), Note also that (i, k) is red for
N). Prove that KL + MN is composite. k ~ C¡,so the sequence Co, cr. ... ,Cn-I completely defines the coloring ofS.

Let S¡ be the set with the sequence Co, CI, ... , C¡,-1, ... ,-1, so that Sn'l = S. We also take S.I
as the set with the sequence -1, -1, ... , -1, so that all its members are blue. We show that the


Solutioo rearrangement result is true for S.I and that if it is true for S¡ then it is true for S¡+I. It is
obvious for S.I, because both Il¡ and b, are n, n·l, ... , 2, l. So suppose it is true for S¡ (where
Note first that KL+MN > KM+LN > KN+LM, because (KL+MN) - (KM+LN) = (K - NXL i < n-I). The only difference between the a¡ for S¡ and for S¡+Iis that ~I = n-i-I for S¡ and

• - M) > O and (KM+LN) - (KN+LM) = (K - LXM - N) > O.

Multiplying out and rearranging, the relation in the question gives K 2 - KM + M2 = L 2 + LN


(n-i-I)-(C¡+I+ 1) for S¡+I.In other words, the number n-i-I is replaced by the number n-i-e-2,
where e = C¡+I·The difference in the bj is that I is deducted from each ofbo, bl, ... , be. But


these numbers are just n-i-I, n-i-l , n-i-2, ... , n-i-e-l. So the effect of deducting I from each
+ N2. Hence (KM + LNXL2 + LN + N2) = KM(L2 + LN + N2) + LN(K2 _ KM + M2) = is to replace n-i-I by n-i-c-Z, which is the same change as was made to the a¡. So the
KML2 + KMN2 + K2LN + LM2N = (KL + MNXKN + LM). In other words (KM + LN) rearrangement result also holds for S¡+I.Hence it holds for S.


divides (KL + MNXKN + LM).

Now suppose KL + MN is prime. Since it greater than KM + LN, it can have no common


factors with KM + LN. Hence KM + LN must divide the smaller integer KN + LM.
Contradiction. ProblemAl

• Comment. This looks easy, but in faet Jfound it curiously diffieult. It is easy to go around in BC is a diameter of a circle center O. A is any point on the circle with angle AOC > 60°. EF

•.
eirc1esgetting nowhere. Either J am getting older, or this is harder than it looks! is the chord which is the perpendicular bisector of AO. D is the midpoint of the minor are
AB. The line through O paraJlel to AD meets AC at J. Show that J is the incenter oftriangle
Note that it is not hard tofind K, L, M, N satisfying the condition in the question. For CEF.
example 11, 9, 5, l .

••
• {,/
SolutioD

A
Obviously m > n. Take polynomials ,\(x). r(x) with integer coefficients and with degree r(x)
< n such that xm+ X - 1 = q(x) (x" + x - 1) + r(x). Then x" + X2- 1 divides r(x) for infinitely
many positive integers x. But for sufficiently large x, x· + X2- 1 > r(x) since r(x) has
smaller degree. So r(x) must be zero. So xm+ X - 1 factorises as q(x)(x· + X2- 1). where
..•
q(x) = xm-. + am_'_IXm-'-1
+ ... + 110.

At this point 1 use an elegant approach provided by Jean-Pierre Ehrmann

We have (x" + x _ 1) = xm-n(x·+ X2-1) + (1 - XXXm-n+1 + xm-D- 1), so (x' + X2-1) must
8 . o ¡e
divide (Xm-n+1+ xm-n_ 1). So, in particular, m ~ 2n-1. Also (x" + X2- 1) must divide (Xm-n+1
+
xm-D_ 1) _ xm-2n+\x' + X2_ 1) = xm-._ xm-2n+3 + xm-2n+1- 1 (*). ,_'
\ ....
,..
.
"<.""'
..- .......
_--~
./i
........
,.' At this point there are several ways to go. The neatest is Bil! Dubuque's:

(*) can be written as Xm-2.+3(X·-3- 1) + (Xm-(2n-1)- 1) which is < Ofor all x in (0,1) unless n-
F is equidistant from A and O. But OF = OA, so OFA is equilateral and hence angle AOF = 3 =Oandm-(2n-I)=0. So untess n=ü.m=S, ít ís has no roots intü, 1). Butx'+x2-1
60°. Since angle AOC > 60°. F lies between A and C. Hence the ray CJ lies between CE (which divides it) has at least one becaause it is -1 at x = O and + 1 at x = 1. So we must
andCF. have n = 3, m = 5. lt is easy to check that in this case we have an identity.

o is the midpoint ofthe are AB, so angle DOB = Yz angle AOB = angle ACB. Hence DO is Two alternativesfollow. Jean-Pierre Ehrmann continued:
parallel to AC. But OJ is parallel to AO. so AJOO is a parallelograrn. Hence AJ = OO. So
AJ = AE = AF. so J lies on the opposite side ofEF to A and hence on the same side as C. If m = 2n- 1, (*) is Xn-I- X2.If n = 3, this is O and indeed we find m = 5, n = 3 gives an
So J must lie inside the triangle CEF. identity. Ifn > 3. then it is X2(X·-3- 1). But this has no roots in the interval _2'
(O 1), whereas x"
2
+ x - 1 has at least one (because it is -1 at x = Oand +1 at x = 1). so x" + x - l cannot be a
Also, since EF is the perpendicular bisector of AO, we have AE = AF = OE. so A is the factor.
center of the circle through E. F and J. Hence angle EFJ = Yz angle EAJ. But angle EAJ =
angle EAC (same angle) = angle EFC. Hence J lies on the bisectorofangle EFC. 2
If m> 2n-I, then (*) has four terms and factorises as (x - 1Xxm-n-I+ x".... + ... + Xm-2n+3 +
Xm-2.+ Xm-2n-1 + ... + 1). Again, this has no roots in the interval (O. 1). whereas x· + X2- 1
Since EF is perpendicular to AO, A is the midpoint ofthe are EF. Hence angle ACE = has at least one, so x· + X2- 1 cannot be a factor.
angle ACF, so J lies on the bisector of angle ECF. Hence J is the incenter.
Francois Lo Jacomo, having got to X' + :r! -J divides X""'+/ + X"'" - J and looking at the
Many thanks to Dirk Lauriefor pointing out that the original version ofthis solutionfailed case m -n + J > n, continues:
to show the relevance of angle AOC > 6(/'. According to the official marking scheme, one
apparently lost a mark for failing lo show J lies inside CEF. x·+ X2-1 has aroot rsuch thatO <r< 1 (because it is -1 atx=O and +1 atx = 1). So ~= 1
-~. It mustalso bea root of xf'+ x-I, so 1- r=~~~=(I-~f Hence (1- ~i -(1- r)
= (1 - r) r (1 - r -~) ~ O. so 1 - r - ~ ~ O. Hence r" = 1 - ~ ~ r. which is impossible.

ProblemAJ Many thanks to Carlos Guslavo Moreira for polienlly explaining why the brote force
approach of calculaling the coefficients of q(x), slarling al lhe low end, isfoil of pitfalls.
Find all pairs of integers m > 2, n > 2 such that there are infinitely many positive integers k After several failed atlempts, 1 have given up on il!
for which (k· + ~ - 1) divides (km+ k - 1).

Problem Bl
SolutioD

Answer: m = 5. n = 3.



• The positive divisors ofthe integer n > 1 are di < d2< ... < dk, so that di = 1, dk = n. Let d = From (0) aboye, we have fl:x2)= fl:xi ::::O,so fl:x)is alwar non-negative forr,'sitive x and

• dld2 + d2d3+ ... + dk_Idk.Show that d <·n2and find all n for which d divides n2. hence for all x. Putting u = y, v = x, we get ( fl:x)+ fl:y» = f(x2 +r), so f\x + y2J= fl:x)2
+ 2fl:x)fl:y)+ fl:yi:::: fl:xi = fl:x2).For any u> v > O,we may put u = X2+ y , v = x and
hence fl:u)::::fl:v). In other words, f is an increasing function.

••. (
SolutioD

dk+l-m<= n/m. So d < n2(1/(1.2) + 1/(2.3) + 1/(3.4) + ... ). The inequality is certainly strict
because d has only finitely many terms. But 1/(1.2) + 1/(2.3) + 1/(3.4) + ... = (1/1 - 112) +
So for any x we may take a sequence of rationals r. all less than x we converge to x and
another sequence of rationals S. all greater than x which converge to x. Then r/ = fl:rn)~
fl:x)~ fl:8o)= 802for all x and hence fl:x)= X2.

• (1/2 - 1/3) + (1/3 - 1/4) + ... = 1. So d < n2_


Problem Bl


Obviously d divides n2for n prime. Suppose n is composite. Let p be the smallest prime
dividing n. Then d > n2/p.But the smallest divisor of n2apart from 1 is p, so if d divides n2,
then d ~ n2/p. So d cannot divide n2for n composite. 2 circles of radius 1 are drawn in the plane so that no line meets more than two of the
circles. Their centers are 01, 02, ... , On- Show that L¡<j1/0¡Oj ~ (n-I)lrI4.


• Problem B2 SolutioD

• Find all real-valued functions f on the reals such that (flx) + fl:y» (f(u) + f(v» = f(xu - yv) +
fl:xv+ yu) for all x, y, u, v.


SolutioD

Answer: there are three possible functions: (1) fl:x)= Ofor all x; (2) fl:x)= 1/2 for all x; or

• (3) fl:x)= X2.

Put x = y = O,u = v, then 4 feO)f(u) = 2 fl:0).So either fl:u)= 1/2 for all u, or feO)= O.fl:u)=
Denote the circle center O¡ by C¡. The tangents from 01 to C¡ contain an angle 2x where sin
x = 11010,. So 2x > 2/010,. These double sectors cannot overlap, so L 2/010, < n. Adding

• 1/2 for all u is certainly a solution. So assume fl:0)= o.

Putting y = v = O,fl:x)fl:u)= fl:xu)(O). In particular, taking x = u = 1, fl:1)2= fl:1).So fl:1)=


the equations derived frorn O2, 03, ... we get 4 L O¡Oj< nn, so L O¡Oj< nlt/4, which is not
quite good enough.

• Oor l. Suppose fl:1)= O.Putting x = y = 1, v = O,we get O= 2fl:u), so flx) = Oor all x. That
is certainly a solution. So assume fl:1)= 1.

• Putting x = O,u = v = 1 we get 2 fl:y)= f(y) + fl:-y),so fl:-y)= fl:y). So we need only
r

••.
consider fl:x)for x positive. We show next that fl:r)= for r rational. The first step is to
show that fl:n)= n2for n an integer. We use induction on n. It is true for n = Oand l.
Suppose it is true for n-l and n. Then putting x = n, y = u = v = 1, we get 2f(n) + 2 = fl:n-I)
+ fl:n+I), so fl:n+l) = 2n2+ 2 - (n-li = (n+ li and it is true for n+l. Now (0) implies that
fl:n)fl:mln)= fl:m),so fl:mln)= m2/n2for integers m, n So we have established fl:r)= for r
' all rational r.

•• There are two key observations. The first is that it is better to consider the angle O,OIOj
than the angle between the tangents to a single circle. It is not hard lo show that this angle

••
• G¿
must exceed both 2/010¡ and 2/010j- For consider the two comrnon tangents to CI and C¡ Having found XI, X2,... , Xkthere are k·IOI·loo forbidden values for Xk+1ofthe form X¡+ 11m
which intersect at the midpoint ofOIO¡. The angle between the center line and one ofthe - a" with m and n unequal and another k forbidden values with m = n. Since 99·10 1·100 +
tangents is at least 2/010¡. No part ofthe cirele Cj can cross this line, so its center O¡ cannot 99 = 106 - 1, we can successively choose 100 distinct x;
cross the line parallel to the tangent through 01. In other word, angle 0¡010¡ is at least
2/010¡. A similar argument establishes it is at least 2/010¡. Gerhard Woeginger sent me a similar solution

Now consider the convex hull of the n points O¡. m ~ n of these points form the convex hull ProblemA2
and the angles in the convex m-gon sum to (m-2)lt. That is the second key observation.
That gains us not one but two amounts x/4. However, we lose one back. Suppose 01 is a Find all pairs (m, n) of positive integers such that m2/(2mn2 - n3 + 1) is a positive integer.
vertex ofthe convex hull and that its angle is 91. Suppose for convenience that the rays
0102, OIÜJ, ... , OIOn occur in that order with O2 and O. adjacent vertices to 01 in the
convex hull. We have that the n-2 angles between adjacent rays surn to 91. So we have L
2/010¡ < 91, where the sum is taken over only n-2 ofthe i, not all n-1. But we can choose Answer
which i to drop, because of our freedom to choose either distance for each angle. So we

..
drop the longest distance 010¡. [lfOIÜt is the longest, then we work outwards from that (m, n) = (2k, 1), (k, 2k) or (8k4 - k, 2k)
rayo Angle Üt.IOIÜt > 2/0IÜt.¡, and angle ÜtOIÜt+1 > 2/0IÜt+1 and so on.] ." )'

We now sum over all the vertices in the convex hull. For any centers O¡ inside the hull we
use the 2J 2/0¡0¡ < It which we established in the first paragraph, where the suro has all n-I Solution
terms. Thus we get L¡j 2/0¡Oj < (n-2)n, where for vertices i for which O¡ is a vertex ofthe
convex hull the surn is only over n-2 values of j and exeludes 2/0¡0 .... ¡ where O.... Thanks lo Li Yi
¡denotes the furthest center from O¡.
The denominator is 2mn2 - n3 + 1 = n2(2m - n) + 1, so 2m >= n > O. If n = 1, then m must be
Now for Oi a vertex ofthe convex hull we have that the sum over all j, L 2/0¡Oj, is the sum even, in other words, we have the solution (m, n) = (2k, 1).
~' over all but j = max i plus at most l/(n-2) ~'. In other words we must increase the sum by
at most a factor (n-I )/(n-2) to inelude the missing termo For O¡ not a vertex ofthe hull, So assume n > l. Put h = m2/(2mn2 - n3 + 1). Then we have a quadratic equation for m,
obviously no increase is needed. Thus the full sum L¡j 2/0;0j < (n-I)lt. Hence L¡<j I/O¡Oj < namely m2 _ 2hn2m + (n3 - I)h = O.This has solutions hn2 +- N, where N is the positive
(n-I)lt!4 as required. square root of h2n4 - hn' + h. Since n > 1, h ~ 1, N is certainly real. But the sum and product
of the roots are both positive, so both roots must be positive. The sum is an integer, so ir
one root is a positive integer, then so is the other.

IM02oo3 The larger root hn2 + N is greater than hn2, so the smaller root < h(n3 - 1Y(hn2) < n. But
ProblemAl note that if2m - n > O,then since h > O, we must have the denominator (2m - n)n2 + 1
smaller than the nuroerator and hence m > n. So for the smaller root we cannot have 2m - n
S is the set (1, 2, 3, ... , 10000001. Show that for any subset A of S with 101 elements we > O.But 2m - n must be non-negative (since h is positive), so 2m - n = O for the smaller
can find 100 distinct elements X¡of S, such that the sets x, + A are all pairwise disjoint. root. Hence hn2 - N = nI2. Now N2 = (hn2 - n/2i = h2n4 - hn3 + h, so h = n2/4. Thus n must
[Note that x, + A is the set (a + x, la is in Al ]. be even. Put n = 2k and we get the solutions (ro, n) = (k, 2k) and (8k4 - k, 2k).

We have shown that any solution must be of one of the three forros given, but it is trivial to
check that they are all indeed solutions.
Solution

Thanks lo Li Yi
ProblemAl

...
.J
.J
~

••
• A convex hexagon has the property that for any pair of opposite sides the distance between Hence HJ/AB = HKIIG = (HK - HJ)/(IG - AB) = JKI(AB + ED) = Y,"3. Similarly, each of

•., their midpoints is Y, "3 times the sum of their lengths. Show!hat all the hexagon's angles
are equal.
the medians of the triangle HAB is Y, "3 times lhe corresponding side. We will show that
this implies it is equilateral. The required result then follows immediately.

• Suppose a triangle has side lengths a, b, e and the length of the median to the midfoint of
side length e is m. Then applying the cosine rule twice we get m2 = a2/2 + b212 - e /4. So if

.~ Solution m2 = JI. c2, it follows that a + b2 = 2c2. Similarly, b2 + c2 = 2a2. Subtracting, a = c. Similarly

JI. Thanks to Li Yi
for tbe other pairs of sides.

An alternative (and rather more e/egant) so/ution sent my some anonymous contestants al
We use bold to denote vectors, so AB means the vector from A to B. We take sorne the IMO is as follows
arbitrary origin and write the vector OA as A for short. Note that the vector to the midpoint

• of AB is (A + B)/2, so the vector from the midpoint ofDE to the midpoint of AB is (A + B


- D - E)/2. So the starting point is lA + B - D - El 2: "3 ( lA - BI + ID - El ) and two similar
H


equations. The key is to notice that by the triangle inequality we have lA - BI + ID - El 2: lA
- B - D + El with equality iffthe opposite sides AB and DE are paraIlel. Thus we get IDA +
EBI2: "3 IDA - EBI. Note that DA and EB are diagonals. Squaring, we get DA2 + 2 DA.EB


+ EB2 2: 3(DA2 - 2 DA.EB + EB\ or DA 2 + EB2 ~ 4 DA.EB. Similarly, we get EB2 + FC2
~ 4 EB.FC and FC2 + AD2 ~ 4 FC.AD = - 4 FC.DA. Adding the three equations gives
2(DA - EB + Fci ~ O. So it must be zero, and hence DA - EB + FC = O and opposite sides


of the hexagon are parallel.

Note that DA - EB + FC = A - D - B + E + C - F = BA + DC + FE. So BA + DC + FE =


O. In other words, the three vectors can form a triangle. Let the diagonals AD and BE meet at P. We show that angle APB <= 60°. Suppose angle
APB > 60°. Take X and y inside the hexagon so that ABX and DEY are equilateral (as


shown). Then since angle APB > angle AXB, P lies inside the circurncircle of ABX (which
we take to have center O, radius r), Similarly, it lies inside the circumcircle of DEY (which
we take to have center O' radius r'), so these circles must meet and hence 00' < r + r'. Now


"3 (AB + DE)12 = MN (where M, N are the midpoints of AB, DE) ~ MO + 00' + O'N <
r/2 + (r + r') + r'/2 = (3/2)(r + r') = "3 (AB + DE)/2. Contradiction.

• The sarne argument applies to any two long diagonals. Hence the angles must all be 60°.
Also we must have MP ~ MX with equality iff P = X, and similarly NP ~ NY with equality
iff P = Y. So MN ~ MP + PN ~ MX + NY = "3 (AB + DE)/2 = MN. Hence we have

• equality and so P = X = Y.


Since EF is parallel to BC, ifwe translate EF along the vector ED we get CG, an extension
of Be. Similarly, if we translate AB along the vector BC we get an extension of ED. Since
n

...
BA, DC and form a triangle, AB must translate to OO. Thus HAB and COO are

• congruent. Similarly, if we take AF and DE to intersect at 1, the triangle FIE is a1so


congruent (and similarly oriented) to HAB and COO. Take J, K as the midpoints of AB,
ED. HIG and HAB are equiangular and hence similar. lE = 00 and K is the midpoint of
ED, so K is a1so the midpoint of IG. Hence HJ is paraIlel to HK, so H, J, K are collinear .

-
¡.



•11
(~

B
_----"!I.

"
F - - --
--7"---·:". _
I Q ",
e

APRO is cyclic with diameter AD (because angle APD = angle ARO = 90°. Suppose its
Hence angle APB = 60°. Suppose AD and CF meet at Q. The same argument shows that center is O and its radius r. Angle PAR = Y:.angle POR, so PR = 2r sin Y:.POR= AD sin
angle AQF = 60°. So the hexagon angle at A is angle APB + angle AQF = 120°. Similarly PAR. Similarly, RQ = CD sin RCQ. (Note that it makes no difference ifR, Pare on the
for the other angles. same or opposite sides ofthe line AD.) But sin PAR = sin BAC, sin RCQ = sin ACB, so
applying the sine rule to the triangle ABC, sin RCQ/sin PAR = ABIBC. Thus we have
Finally, note that the only possible configuration is: AD/CD = (PRlRQ) (AB/BC). Suppose the angle bisectors of B, D meet AD at X, Y. Then
we have ABIBC = AXlCX and AD/CD = A Y/CY. Hence (A Y/CY)/(AXlCX) = PRlRQ. So
PR = RQ iff X = Y, which is the required result.

Note that ABCD does not need to be cyclic! Exercise: does it need to be convex?

Problem B2

The ratio ABIBC is arbitrary, but the figure is symmetrical under rotations through 120°. Given n > 2 and reals x¡ <= X2<= ... <= Xn, show that O::ij [x, - xii )2::; (2/3) (n2 - 1) ¿ij (x, -
That follows immediately from either of the two solutions above. xii. Show that we have equality iffthe sequence is an arithmetic progression.

Problem B1 Solution

ABCD is cyclic. The feet ofthe perpendicular from D to the lines AB, BC, CA are P, Q, R Thanks to Li Yi
respectively. Show that the angle bisectors of ABC and CDA meet on the line AC iffRP =
Notice first that if we restrict the sums to i < j, then they are halved. The Ihs sum is squared
RQ.
and the rhs sum is not, so the the desired inequality with sums restricted to i <j has (1/3) on
the rhs instead of(2/3).

Consider the sum of all lx, - xii with i <jo xi occurs in (n-I) terms with a negative signo X2
Solutlon
occurs in one term with a positive sign and (n-2) terms with a negative sign, and so on. So
Thanks to Li Yi we get -(n-I)xl - (n-3)x2 - (n-Sjx, - ... + (n-I)x, = ¿ (2i-I-n)x;.
,.
'a

We can now apply Cauchy-Schwartz. The square ofthis sum isjust L x? L (2i-I-nl. other words, nP = 1 mod q. But nP = p mod q, so p = 1 mod q. Contradiction (we showed
aboye that q does not divide p - 1).
Looking at the other side ofthe desired inequality, we see immediately that it is n L X¡2- (L


x¡l We would like to gel rid of the second termo but that is easy because if we add h lo
every X¡the sums in the desired inequality are unaffected (since they use only differences of
X¡),so we can choose h so that L X¡is zero. Thus we are home if we can show that L (2i-l-
ni:s n(n2- 1)/3. That is easy: lbs = 4 L i2- 4(n+l) L i + n(n+ li = (2/3)o(n+ IX2n+ 1)-
2n(n+ 1) + n(n+ li = (1/3)n(n+IX2(2n+l) - 6 + 3(n+ 1» = (1/3)n(n2 - 1) = rhs. That
establishes the required inequality.

. We have equality iffwe have equality at the Cauchy-Schwartz step and hence iff x, is
proportional to (2i-I-n). That implies that X¡+I- X¡is constant. So equality implies tbat the
sequence is an AP. But ifthe sequence is an AP with difference d (so X¡+I= Xi+ d) and we
take XI = -(d/2Xn-I). then we get x¡ = (dI2X2i-I-n) and L Xi= O, so we have equality.

• Problem B3

• Show that for each prime P. there exists a prime q such that nP- p is not divisible by q for
any positive integer n.



Solution

Thanks lo Li Yi

• If P = 2, then we can take q = 3. since squares cannot be 2 mod 3. So suppose p is odd.

• Consider N = 1 + P + p2+ ... + pp-I.There are p terms. Since p is odd, that means an odd
number of odd terms, so N is odd. AIso N = P + 1 mod p2. which is not 1 mod p2. so N must
have a prime factor q which is not 1 mod p2.We will show that q has the required property.

• Since N = 1 mod p, p does not divide N, so q cannot be p. Ifp = 1 mod q, then N = 1 + 1 +


... + 1 = P mod q. Since N = Omod q, that implies q divides p. Contradiction. So q does not

• divide p - 1.

•.-
Now suppose nP = p mod q (.). We havejust shown tbat n cannot be 1 mod q. We have
also shown that q is not P. so n cannot be a multiple of q. So assume n is not Oor 1 mod q.
Take the pth power of both sides of (.). Since (p - I)N = pp- 1. we have pP = 1 mod q. So n
to the power of p2is 1 mod q. But nq-I = 1 mod q (the well-known Fermat little theorem) .
So if d = gcd(q-I, p2).then nd = 1 mod q. We chose q so that q-I is not divisible by p2, so d
must be 1 or p. But we are assuming n is not 1 mod q, so d cannot be 1. So it must be p. In



(,1
11
l.
I~

¡.

••
••.:


•-,.
-,_

•..
d.•



;·.1í, ".

You might also like